You are on page 1of 347

See discussions, stats, and author profiles for this publication at: https://www.researchgate.

net/publication/341868697

Riyaziyyatdan olimpiada məsələləri. 8 - 11 -ci sinif şagirdləri üçün vəsait

Book · December 2012

CITATIONS READS

0 4,299

2 authors, including:

Ilyas Ravan Hasanov


State Oil Company of Azerbaijan Republic
510 PUBLICATIONS 46 CITATIONS

SEE PROFILE

All content following this page was uploaded by Ilyas Ravan Hasanov on 15 November 2021.

The user has requested enhancement of the downloaded file.


Nadir Nəsibov, İlyas Həsənov

RİYAZİYYATDAN
OLİMPİADA
MƏSƏLƏLƏRİ
8 - 11 -ci sinif şagirdləri üçün vəsait

Bakı-2012
Elmi redaktoru:
N.B. Kərimov, fizika-riyaziyyat elmləri doktoru, professor.

Rəy verənlər:
M.İ. Aranlı, Respublikanın əməkdar müəllimi
N.Y. Mehrabov, 271 №-li tam orta məktəbin riyaziyyat
müəllimi

N.N. Nəsibov, İ.R. Həsənov. Riyaziyyatdan olimpiada


məsələləri. (VIII - XI sinif şagirdləri üçün vəsait). – Ləman
Nəşriyyat Poliqrafiya, Bakı, 2012, 346 səh.

Vəsait, riyziyyata xüsusi maraq göstərən yuxarı sinif şagird-


lərinin olimpiadalara hazırlaşması üçün nəzərdə tutulmuşdur. Bu
vəsaitə, müxtəlif olimpiadalarda təklif olunmuş bir çox maraqlı
məsələ, misallar və onların həlli, həmçinin müəlliflərin tərtib
etdikləri məsələ və misallar daxil edilmişdir.
Vəsait 8-11-ci sinif şagirdləri, riyaziyyatdan fakültətiv və
dərnək məşğələləri aparan müəllimlər və ümumiyyətlə, riyaziyyatla
ciddi maraqlananlar üçün nəzərdə tutulmuşdur.

N 170203000 − 134  N.N. Nəsibov, İ.R. Həsənov, 2012.


2012

2
MƏSƏLƏLƏR
1. 245, 336, 427, 518 ədədlərini artan sırada yazın.

2. İsbat edin ki, 2100 + 3100 < 4100 .

3. Hansı ədəd böyükdür: 21000 yoxsa, 3750 ?

4. Hansı ədəd böyükdür: 3303 yoxsa, 2454 ?

5. İsbat edin ki, 127 23 < 51318 .

6. İsbat edin ki, 3111 < 1714 .

7. İsbat edin ki, 21995 > 5854 .

8. İsbat edin ki, 21995 > 5856 .

9. 29, 38, 47, 56 ədədlərindən hansı ən böyükdür?

10. Hansı ədəd böyükdür: 510 + 610 yoxsa, 710 ?

11. Hansı ədəd böyükdür: 4825 yoxsa, 34417 ?

12. İsbat edin ki, 3


4 − 3 10 + 3 25 > 3 6 − 3 9 + 3 15 .

13. İsbat edin ki, 665 < 956 .

14. İsbat edin ki, 29200 ⋅ 2151 < 5279 ⋅ 3300 .

3
15. Natural a, b, c ədədləri üçün ab + bc + ac ≤ 3abc
olduğunu isbat edin.

16. Üçrəqəmli ədədi onun rəqəmləri cəminə böldükdə alınan


nisbətin ən böyük qiymətini tapın.

17. İsbat edin ki, 200! < 100200 .

18. Natural ədədin kvadratı 1999 -la qurtara bilərmi?

19. 2005 -rəqəmli 111 11 ədədi tam ədədin kvadratı ola


bilərmi?

20. 5 müxtəlif sadə ədədin hasilinin neçə natural böləni var?

21. 1999n ədədinin onluq yazılışında bütün rəqəmlərdən eyni


sayda ola bilərmi?

22. İsbat edin ki, 20032 + 2001 ⋅ 2002 ⋅ 2004 ⋅ 2005 ədədi
ifadəsi natural ədədin kvadratına bərabərdir.

23. 9 kağız vərəqi verilir. Bunlardan bir neçəsini və ya


hamısını 9 hissəyə böldülər. Alınmış hissələrin bir
neçəsini yenidən 9 hissəyə böldülər. Bu qayda ilə
müəyyən addımdan sonra 1001 sayda kağız parçası
alına bilərmi?

24. Natural ədədin kvadratı 2005 -lə qurtara bilərmi?


25. İsbat edin ki, istənilən natural n ədədi üçün 7 n ⋅ 23n − 32 n
ifadəsi 47-yə bölünür.

4
26. 5n + 1 şəklində olan ədəd 5k − 1 şəklində olan ədədə
bölünə bilərmi (burada n, k ∈ N )?

27. İsbat edin ki hər bir tam n ədədi üçün 7 n + 2 + 82 n +1 cəmi


57-yə bölünür.

28. İsbat edin ki, hər bir tam n ədədi üçün n(n 2 + 5) ədədi
6-ya bölünür.

29. İsbat edin ki, 2233 + 3322 cəmi 97-yə bölünür.

30. Üçrəqəmli abc ədədi 37-yə bölünür. İsbat edin ki,


bca + cab cəmi də 37-yə bölünür.

31. İsbat edin ki, 13 + 23 +  + 993 cəmi 100 -ə bölünür

32. İsbat edin ki, 13 + 23 + 33 + ⋅ ⋅ ⋅ + 93 cəmi 10-a bölünür.

33. p və q -nün hansı qiymətlərində x 2 + px + q =0


tənliyinin kökləri p və q olar?

34. p -nin hansı tam qiymətlərində x 2 + px + p =


0 tənliyinin
tam həlləri vardır?

35. Əmsalları cəmi 19 olan tam əmsallı çevrilmiş kvadrat


tənliyin kökləri natural ədədlər olarsa, bu tənliyi qurun.
36. x 2 + px + q + 1 = 0 tənliyinin kökləri natural ədədlər
olarsa, p + q cəminin mürəkkəb ədəd olduğunu isbat
2 2

edin.
5
37. Tam əmsallı kvadrat tənliyin diskriminantı 2002 -yə
bərabər ola bilərmi?

38. İsbat edin ki, x 2 + y 2 =


1999 tənliyinin tam həlli yoxdur.

39. Tənliyin tam həllərini tapın: x + xy + y =


100 .

40. Tənliyin tam həllərini tapın: x + 2 xy + y =


999 .

41. 19m + 98n = 1999k bərabərliyini ödəyən natural m, n, k


ədədləri varmı?

42. 2 x + 2 y + 2 z =
97 bərabərliyini ödəyən x, y, z tam
ədədlərini tapın.

1 1 1 1
43. + + =tənliyini ödəyən sadə ədədləri tapın.
xy yz zx 5

44. x 2 − y 2 = 2002 tənliyinin natural həlləri varmı?

45. Elə sadə x, y, z ədədləri tapın ki, xyz= 1997 ⋅ ( x + y + z )


bərabərliyi ödənsin.

46. x y + 1 =z tənliyini ödəyən sadə ədədləri tapın.

47. İsbat edin ki, 19 x + 99 y =19 ⋅ 99 tənliyinin natural həlli


yoxdur.

6
1 1 1
48. p sadə ədəd olduqda + = tənliyinin tam həllərini
x y p
tapın.

y 3 tənliyinin müsbət tam həllərini tapın.


49. x3 + 1331 =

50. xy ⋅ x =zzz bərabərliyini ödəyən x, y , z rəqəmlərini


tapın.

51. Kökü 2 + 3 olan tam əmsallı tənliyi tapın.

52. İsbat edin ki, p 3 − q 3 =


2004 tənliyinin natural həlləri
yoxdur.

53. Tənliyi həll edin:


( x 2 + 3 x + 2)( x 2 + 7 x + 12) + ( x 2 + 5 x − 6) =
0.

54. Tənliyi həll edin: x3 − x − 2 =.


0

1 2
55. Tənliyi həll edin: x 2 + = .
x 2
x − 2x + 2
2

56. Tənliyin natural həllərini tapın: x 2 + 2 y =


2 xy .

57. Tənliyin tam həllərini tapın: x 2 + y 2 =.


74
58. Tənliyin natural həllərini tapın: x 2 y + yz 2 =xz + y .

59. Tənliyi həll edin: 2 x 2 + 6 xy + 10 y 2 − 2 x + 6 y + 10 =


0.

7
60. | x | ⋅ | y | ⋅ | z |=
12 tənliyinin tam həllər sayını tapın.

1 x 1
61. + + 1 tənliyinin bütün tam həllərini tapın.
=
x y xy

62. f ( x) = 2 x funksiyasını cüt və tək funksiyanın cəmi


şəklində göstərin.

63. Həqiqi x və y ədədləri üçün x + y + z =5 və


3 olduqda z ədədinin ən böyük həqiqi
xy + yz + xz =
qiymətini tapın.

64. Hansı natural m, n ədədləri üçün m 4 + 4n 4 ədədi


sadədir?

65. 11
 1 − 22
 2 =
33
3 olduğunu isbat edin.
2n n n

66. Natural n ədədi üçün 16n + 10n − 1 ifadəsinin 25 -ə


bölündüyünü isbat edin.

67. Ardıcıl 4 natural ədədin hasili bir ədədin kubu ola


bilərmi?

68. İfadəni sadələşdirin:


b c d
+ + .
a (a + b) (a + b)(a + b + c) (a + b + c)(a + b + c + d )

69. Bərabərsizliyi isbat edin: 3(1 + a 2 + a 4 ) ≥ (1 + a + a 2 ) 2 .

8
70. Müsbət a, b, c ədədləri üçün
(a + b + c)3 − (a 3 + b3 + c3 ) > (a + b)(b + c)(a + c)
bərabərsizliyini isbat edin.

71. Bərabərsizliyi isbat edin (a, b, c, d > 0):


(a 2 + a + 1)(b 2 + b + 1)(c 2 + c + 1)(d 2 + d + 1)
≥ 81
abcd

72. abc ədədi 27-yə bölünürsə, bca və cab ədədləri də 27-


yə bölünür. Bunu isbat edin.

73. İsbat edin ki,


10010 + 9910 +  + 5110 − 5010 − 4910 −  − 310 − 210 − 1
cəmi 101-ə bölünür.

74. İsbat edin ki, 1110 − 1 ədədi 100 -ə bölünür.

75. İsbat edin ki, istənilən natural n ≥ 2 ədədi üçün


3 4 n+2 1
+ + + < .
1!+ 2!+ 3! 2!+ 3!+ 4! n !+ (n + 1)!+ (n + 2)! 2

76. 12003 + 22003 + 32003 +  + 20012003 + 20022003 cəminin 91-ə


qalıqsız bölündüyünü isbat edin.

77. İfadənin qiymətini tapın:


80 ⋅ (819 + 818 + 817 +  + 812 + 82) + 1 .

78. a 2 + b 2 = 1, c 2 + d 2 = 1 və ac + bd =
0 olduqda ab + cd
ifadəsinin qiymətini tapın.

9
79. Tam x, y, z ədədləri üçün xy + yz + zx = 1 ödənir. İsbat
edin ki, (1 + x )(1 + y )(1 + z ) ədədi tam ədədin
2 2 2

kvadratıdır.

80. İsbat edin ki, a + b + c cəmi 6 -ya bölünürsə, a 3 + b3 + c3


cəmi də 6 -ya bölünür ( a, b, c tam ədədlərdir).

81. Əgər a + b + c = 3abc olduğunu


0 olarsa, a 3 + b3 + c3 =
isbat edin.

82. Əgər 3
0 olarsa, (a + b + c)3 =
a+3b+3c= 27 abc
olduğunu isbat edin.

a+b a 2 + b2
83. Bərabərsizliyi isbat edin: ≤ .
2 2

84. n > 1 olduqda n5 + n 4 + 1 ədədinin mürəkkəb ədəd


olduğunu isbat edin.

85. İsbat edin ki, ∀n ∈ N üçün n5 − 5n3 + 4n ədədi 120 -yə


bölünür.
a 2 b2 c2
86. İxtiyari a, b, c > 0 ədədləri üçün + + ≥ a+b+c
b c a
bərabərsizliyini isbat edin.

87. İsbat edin ki, 1 + 2 +  + 2009 < 1.


2! 3! 2010!
88. Bərabərsizliyi isbat edin:

10
1 1 1 1 1 1 2
− + − + − + < .
2 3 4 5 999 1000 5

89. Tam əmsallı bx 2 + cx + a və cx 2 + ax + b kvadrat


üçhədliləri, x = 13 olduqda eyni qiymətlər alır.
a + b + c cəmi 65 ola bilərmi ?

90. Müsbət a1 , a2 , a3 , , an ədədlərinin hasili 1-ə bərabərdir.


İsbat edin ki, (1 + 2a1 )(1 + 2a2 ) ⋅ ⋅ (1 + 2an ) ≥ 3n .

91. Hansı böyükdür: 10100 yoxsa, 10199 ?

92. (Bernulli bərabərsizliyi) Həqiqi x > −1 və n ∈ N üçün


(1 + x) n ≥ 1 + nx bərabərsizliyini isbat edin.

93. İsbat edin ki, istənilən natural n ədədi üçün


1 ⋅1!+ 2 ⋅ 2!+  + n ⋅ n ! = (n + 1)!− 1 .

94. Düzbucaqlı üçbucaqda a və b katetlər, c hipotenuz


olduqda, hər bir natural n ≥ 2 ədədi üçün a n + b n ≤ c n
olduğunu isbat edin.

95. İsbat edin ki, sin 2 n α + cos 2 n α ≤ 1, (n ∈ N ) .

96. İsbat edin ki, istənilən natural n ədədi üçün


an =n3 + 3n 2 + 5n ədədi 3-ə bölünür.

97. İsbat edin ki, hər bir natural n ≥ 4 üçün n ! > 2n doğrudur.

11
98. İsbat edin ki, hər bir natural n ≥ 3 ədədi üçün
n n +1 ≥ (n + 1) n doğrudur.

99. n ≥ 5 olduqda 2n > n 2 olduğunu isbat edin.

100. Bərabərsizliyi isbat edin: a 2 + b 2 + c 2 ≥ ab + bc + ac .

x2 + y 2
101. x > y və xy = 1 olduqda ≥ 2 2 olduğunu
x− y
isbat edin.

1 olduqda − 2 ≤ x + y ≤ 2 olduğunu isbat


102. x 2 + y 2 =
edin.

103. Bərabərsizliyi isbat edin:

a 2b 2 + b 2 c 2 + a 2 c 2 ≥ abc(a + b + c) .

a2 + a + 2
104. Bərabərsizliyi isbat edin: ≥ 2.
a2 + a + 1

105. Bərabərsizliyi isbat edin:


1 1 1 99
2
+ 2 + + 2
< .
2 3 100 100

106. Bərabərsizliyi isbat edin:

1 3 5 9999
⋅ ⋅ ⋅ ⋅ < 0, 01 .
2 4 6 10000
107. Bərabərsizliyi isbat edin:
12
1 1 1 1
+ + + ≥ (n ∈ N )
n +1 n + 2 2n 2

108. 200! ədədinin sonunda neçə sıfır var?

109. Natural a və b ədədləri üçün


ƏBOB (a, b) + ƏKOB (a, b)= a ⋅ b tənliyini həll edin

110. Müqayisə edin: 51101 və 101! .

111. S= cos1 + cos 3 + cos 5 +  + cos1001 cəmini tapın.

112. Hansı böyükdür: 999 + 1001 yoxsa, 2 1000 ?

1 1 1 n
113. İsbat edin ki, + + + = .
1⋅ 3 3 ⋅ 5 (2n − 1)(2n + 1) 2n + 1

 n(n + 1) 
2

114. İsbat edin ki, 1 + 2 + 3 +  + n =


3 3 3

3
 .
 2 

115. İsbat edin ki,


n(2n − 1)(2n + 1)
12 + 32 + 52 +  + (2n − 1) 2 = .
3
1 1 1
116. Natural n > 1 ədədi üçün + + + > n
1 2 n
bərabərsizliyini isbat edin.
n(n + 1)(n + 2)
117. 1 ⋅ 2 + 2 ⋅ 3 + 3 ⋅ 4 +  + n(n + 1) =
3
bərabərliyinin doğruluğunu isbat edin.

13
118. İsbat edin ki,

1000 − 101 ⋅ 99 + 98 − 99 ⋅ 97 + 96 − 97 ⋅ 95 +  +
+ 2 − 3 ⋅1 < 41.

119. Bərabərsizliyi isbat edin: e x > 1 + x, x > 0 .

120. Bərabərsizliyi isbat edin:


(a b c )
a b c 2
≥ a b + cb c + a c a + b , (a, b, c > 0) .

121. İstənilən a>1, b>1, c>1 olduqda


log b a log c b log a c 4,5
+ + ≥
a +b b+c c+a a+b+c
bərabərsizliyini isbat edin.
122. f ( x ) funksiyası istənilən a və b ədədləri üçün
 a + 2b  f (a ) + 2 f (b )
f = şərtini ödəyir.
 3  3
Əgər f (1) = 1 və f (4) = 7 olarsa, f (2011) -i tapın.

123. Hansı böyükdür: 1000 1000 yoxsa, 1001 999 ?


124. İsbat edin ki, 00 < x < 900 olduqda

 1   1 
1 +  ⋅ 1 +  > 5 doğrudur.
 sin x   cos x 

125. Bərabərsizliyi isbat edin:


(a 2
)( ) (
+ b 2 a 4 + b 4 ≥ a 3 + b3 . )2

14
126. α , β , γ üçbucağın bucaqları olarsa,

( sin α + sin β + sin γ ) > 9sin α ⋅ sin β ⋅ sin γ


2

olduğunu isbat edin.

1
127. x + y + z = 1 və x, y, z ≥ − olarsa,
4
4 x + 1 + 4 y + 1 + 4 z + 1 < 5 olduğunu isbat edin.

128. a + b + c = 1 olarsa, a 2 + b 2 + c 2 ifadəsinin ən kiçik


qiymətini tapın.

129. Bərabərsizliyi isbat edin: n


a − n b ≤ n a − b ( a > b ≥ 0) .

130. Bərabərsizliyi isbat edin: ln (1 + x ) ≤ x ( x ≥ 0 ) .

a 3 + b3  a + b 
3

131. Bərabərsizliyi isbat edin: ≥  , (a, b > 0 ) .


2  2 

132. Bərabərsizliyi isbat edin:


α + β sin α + sin β
sin ≥ , α , β ∈ [0;π ] .
2 2

133. Bərabərsizliyi isbat edin: 99 (2 ln10 − ln 99) < 1 .

134. Bərabərsizliyi isbat edin:

15
n n +1
 1  1 
1 +  ≤ 1 +  ,(n ∈ N ) .
 n   n +1 

135.. Bərabərsizliyi isbat edin:


x1 + x2 +  + xn ≤ x1 + x2 +  + xn .

136. İsbat edin ki, hər bir natural n ≥ 2 ədədi üçün


1 1 1 1 n −1
2
+ 2 + 2 ++ 2 < .
2 3 4 n n

137. Eyniliyi isbat edin:


sin 2n α
cos α ⋅ cos 2α ⋅ cos 4α ⋅ ⋅ cos 2n −1α = .
2n sin α
138. İsbat edin ki, istənilən n ∈ N üçün
π
2 + 2 + 2 +  + 2 =2 ⋅ cos n +1 .
  2
n

139. Bərabərsizliyi isbat edin: a 4 + b 4 + c 4 ≥ abc(a + b + c ).

x3
140. Bərabərsizliyi isbat edin: sin x ≥ x − , x ≥ 0.
6

141. Bərabərsizliyi isbat edin: ln 2 n > ln(n − 1)ln(n + 1), n > 2.

π ln cos a − ln cos b
142. 0 < a < b < olarsa, tga < < tgb
2 b−a
olduğunu isbat edin.

16
( x + y + z )6
143. Müsbət x, y, z ədədləri üçün ifadəsinin ən
xy 2 z 3
kiçik qiymətini tapın.

144. x 2 ≤ [ 2 x ] ⋅ {2 x} bərabərsizliyinin bütün həqiqi həllərini


tapın. ([a ] və {a} uyğun olaraq, ədədin tam və kəsr hissəsidir).

145. Hesablayın: tg 200 ⋅ tg 400 ⋅ tg 800 .

146. f ( x + y ) + f ( x − y ) = 2 x 2 + 2 y 2 bərabərliyini ödəyən


bütün f : R → R funksiyalarını tapın.

147. f ( x + y ) − f ( x − y ) =
4 xy bərabərliyini ödəyən bütün
f : R → R funksiyalarını tapın.

148. İstənilən a > 0, b > 0, c > 0 ədədləri üçün


a 2 + b2 b2 + c2 c2 + a 2
+ + ≥ 3 bərabərsizliyini isbat edin.
c 2 + ab a 2 + bc b 2 + ca

sin( x − α ) cos( x − α )
149. Məlumdur ki, = m, = n,
sin( x − β ) cos( x − β )
 π
x ∈  0,  , m, n > 0. cos(α − β ) -nı tapın.
 2
150. İsbat edin ki, ∀x ≥ 1 üçün
x 2003 ≥ 2003 ( x1002 − x1001 ) + 1 bərabərsizliyi doğrudur.

151. İstənilən x, y ∈ R üçün

17
f ( x 2 + y ) + f ( f ( x=
) − y ) 2 f ( f ( x)) + 2 y 2 bərabərliyini
ödəyən bütün f : R → R funksiyalarını tapın.

152. A =1 + 2 ⋅ 2 + 3 ⋅ 22 + 4 ⋅ 23 +  + 2005 ⋅ 22004 ifadəsini


hesablayın və A -nın 2004 -ə bölünməsindən alınan
qalığı tapın.

153. Tənliklər sistemini həll edin:


sin x + 2sin( x + y + z ) =0,

sin y + 3sin( x + y + z ) =0,
sin z + 4sin( x + y + z ) =
 0.

154. Müsbət a, b, c, d ədədləri üçün abcd = 1 . İsbat edin ki,


a 2 + b 2 + c 2 + d 2 + ab + ac + ad + bc + bd + cd ≥ 10
bərabərsizliyi doğrudur.

 x 2 − xy + y 2 = 19,
155. 
 x − xy + y = 7
tənliklər sistemini ödəyən ( x; y ) tam ədədlər cütünü
tapın.

 n +1 
n

156. n > 1 olduqda n ! <   olduğunu isbat edin.


 2 

an +1 + 1
157. an ardıcıllığı istənilən natural n üçün an + 2 =
an
münasibətini ödəyir. Əgər=
a33 3,=
a 44 4 olarsa, a2007 -
ni tapın.

18
158. Əgər a 2 + b 2 =
1 şərti ödənirsə, a 3b − b3 a ifadəsinin ən
böyük qiymətini tapın.

159. Dövrü T = 10 olan f ( x) funksiyası tək funksiyadır.


1,5 olarsa, f (2007) − ni tapın.
Əgər f (−7) =

160. f ( x) funksiyası ∀x ∈ R üçün f ( x + 2) + f ( x) =


0
bərabərliyini ödəyir. İsbat edin ki, f ( x), dövrü 4
olan dövri funksiyadır.

161. İstənilən müsbət x və y üçün f ( x) funksiyası


xy ) f ( x) + f ( y ) bərabərliyini ödəyir. Əgər
f (=
 1 
f (2011) = 1 olarsa, f   -i tapın.
 2011 

162. Bütün x -lər üçün f ( x + 1) + f (2 x) = ( x + 1) 2


münasibətini ödəyən xətti f ( x) funksiyası varmı?

163. f ( x) funksiyası bütün rasional x və y ədədləri üçün


f ( x + y )= f ( x) + f ( y ) bərabərliyini ödəyir.
 2
f (10) = −π olarsa, f  −  -ni tapın.
 7

164. f ( x) = x( x − 1)( x − 2) ⋅ ⋅ ( x − 2011)( x − 2012) funksiyası


verilmişdir. f ′(0) - ı tapın.

165. Tənliyin neçə həqiqi kökü var:


x 2011 + 2011 ⋅ x =2011?

19
166. 3m − 3n =2106 tənliyini ödəyən bütün natural
(m, n) həllər cütünü tapın.

167. Tənliyi həll edin: 2 cos 2 ( x=


2010
) 2010 x + 2010− x.

168. Tənliyi həll edin: sin 2011 x + cos 2010 x =


1.

x
169. Tənliyi həll edin: sin ⋅ cos 2 x =
1.
2

170. Tənliyi həll edin: sin 2011 x − cos 2010 x =


1.

πx 1 + x2
171. Tənliyi həll edin: sin = .
2 2x

172. Tənliyi həll edin: sin x + sin 2 x + sin 3x +  + sin nx =


n.

173. Tənliyi həll edin: 2 ⋅ log 6 ( x+4 x = )


log 4 x .

174. İstənilən x, y üçün f ( x) ⋅ g ( y ) = x + y + 1 bərabərliyini


ödəyən f və g funksiyası varmı?

175. f ( x) funksiyası hər hansı x üçün f ( x) + f (− x) =


sin x
tənliyini ödəyir. Bu funksiyanı və onun daha geniş təyin
oblastını tapın.

176. İstənilən x, y ∈ R üçün


f ( x ) ⋅ g ( y=
) sin( x + y ) + sin( x − y ) bərabərliyini ödəyən
f və g funksiyalarını tapın.

20
177. Tənliyi həll edin: f ( x) + x ⋅ f (1 − x) =1 + x .

178. İstənilən x, y ∈ R üçün


f ( x + y 2 + 2 y + 1) = y 4 + 4 y 3 + 2 xy 2 + 5 y 2 + 4 xy +
+2 y + x 2 + x + 1
tənliyini ödəyən bütün f : R → R funksiyalarını tapın.

 x   x −1 
179. Tənliyi həll edin: f  = 2 ⋅ f .
 x −1   x 

1
180. Tənliyi həll edin: a ⋅ f ( x) + f   =
ax .
x

181. Funksional tənliyi həll edin: 2 f ( x) + f (1 − x) =.


x2

182. Tənliyi həll edin: y 4 +2x 4 +1 = 4 x 2 y .

183. a -nın hansı qiymətində x 4 + ax 3 + 15 x 2 − 18 x + 9


ifadəsi x dəyişənindən asılı kvadrat üçhədlidir.

184. Tənliyin natural həllərini tapın: 2xy = x 2 + 2y .

185. Tənliyi həll edin: x 2 + xy + y 2 − 2x + 2y + 4 = 0 .

186. xyz = x ⋅ yz bərabərliyini ödəyən x, y, z rəqəmləri


varmı?

21
 x+y+z = 16,
187.  sistemini ödəyən sadə ədədləri tapın.
 xy+yz+zx = 61

 xy + z =97
188.  sisteminin tam həllərini tapın.
 x + yz =98

 x 2 =y − 1,

189.  y 2 =z − 1, sisteminin tam həllərini tapın.
z 2 =x − 1.

190. Müxtəlif a, b, c rəqəmləri üçün abc + ab + a = bcb


ödəndikdə a + b + c cəmini tapın.

2 2 2
191. ab + cd = ba bərabərliyini ödəyən ikirəqəmli
ab, ba, cd ədədlərinin cəmini tapın.

192. Tənliyin natural həllərini tapın: n! = 20n 2 .

a c
193. a, b, c, d ∈ R və 2 ≤ a ≤ b ≤ c ≤ d ≤ 98 olmaqla +
b d
ifadəsinin ala biləcəyi ən kiçik qiyməti tapın.

194. 33a +34b +35c =37d bərabərliyini ödəyən natural a, b, c, d


ədədləri üçün a+b+c+d cəminin ala biləcəyi ən kiçik
qiyməti tapın.

195. Rəqəmləri cəminin 4-cü qüvvətinə bərabər olan 4-


rəqəmli ədədi tapın.

22
196. a, b, c ədədləri x 3 − 3x 2 + 2x − 1 =0 tənliyinin kökləri
(həqiqi və ya kompleks) olduğuna görə a 2 +b 2 +c 2
cəmini tapın.

197. x, y, z müsbət həqiqi ədədlər və x 2 +y 2 +z 2 =2 olduqda


xy yz zx
+ + ifadəsinin ən kiçik qiymətini tapın.
z x y

198. a, b, c ədədləri x 3 - x - 1 = 0 tənliyinin kökləridirsə,


1 1 1
+ + cəmini tapın.
a+1 b+1 c+1

199. Tənliyi həll edin: 2 x3 +8 = x 2 - x + 6.

200. Hansı sadə p ədədi üçün p 2 - 6 və p 2 + 6 ədədləri


sadədirlər?

201. Hansı sadə p ədədi üçün 2p - 1 və 2p +1 ədədləri


sadədir?

202. Heç biri tam kvadrat olmayan ardıcıl 1000 natural ədəd
tapın.

203. 1997 -yə böldükdə qalıqda 97, 1998 -ə böldükdə isə


qalıqda 98 verən ən kiçik natural ədədi tapın.

204. 3 -rəqəmli ədədi onun rəqəmləri cəminə böldükdə,


qismətdə 13, qalıqda isə 15 alınır. Bütün belə 3 -rəqəmli
ədədləri tapın.

23
205. 3333400 ədədindəki rəqəmlərin yerini dəyişdirməklə,
bir-birindən fərqli neçə 7-rəqəmli ədəd yazmaq olar?

206. Tənliyin müsbət rasional həllərini tapın:


x y z
+ + = 2,345 .
y z x

207. Tənliyin natural həllərini tapın: x 3 - 8y3 =19.

208. 5n − 1 şəklində olan ədəd 4n − 1 ədədinə bölünə


bilərmi?

209. 11 tam ədədin cəmi 1000 -lə qurtarır. İsbat edin ki,
onların hasili 2011 -lə qurtara bilməz.

210. Məlumdur ki, a + b + c + d = 6 -dır.


ab + ac + ad + bc + bd + cd = 18 ola bilərmi?

211. 19 x − y z = 1995 tənliyini ödəyən bütün (x, y, z) sadə


ədəd üçlüklərini tapın.

212. x1 və x2 ədədləri [x 2 ] = [6 − x ] − {x − 111} tənliyinin


kökləri olduqda x13 + x23 cəmini tapın ([a] və {a} uyğun
olaraq, ədədin tam və kəsr hissəsidir).

213. a 1 , a 2 , ... , a 100 tam ədədlərinin cəmi 101101 olarsa,


a13 + a23 + ... + a 3100 cəmini 6-ya böldükdə alınan qalığı
tapın.
214. İsbat edin ki, 2110 −1 fərqi 2200 -ə bölünür.

24
215. Tənliyi həll edin:
(x + 2010)(x + 2011)(x + 2012) =
= ( x + 2011)( x + 2012)( x + 2013).

1 2
216. Tənliyin natural həllərini tapın: 2n − 5
=3 − .
n n

217. x − y + z = x − y + z bərabərliyi ödəndikdə


(x − y )( y − z )(z − x ) ifadəsi hansı qiymət alır?
x+ y x− y x2 + y2 x2 − y2
218. + = 3 olduğunu bilərək, 2 +
x− y x+ y x − y2 x2 + y2
ifadəsinin qiymətini tapın.

219. x7 + 5 x − 3 = 0 tənliyinin 7 kökü olduğu məlumdursa,


onların 7 -ci qüvvətlərinin cəmini tapın.

220. Mənfi olmayan a, b, c tam ədədləri üçün


28 a +30 b +31 c = 365 -dir. İsbat edin ki, a+b+c = 12.

221. Natural n ədədinin hansı ən kiçik qiymətində n! ədədi


999 -a bölünür?

222. x, y, z tam ədədləri üçün x y + y z + z x = 1 olduqda,


( )( )( )
isbat edin ki, 1 + х 2 1 + y 2 1 + z 2 ədədi natural ədədin
kvadratıdır.

223. Natural a, b, c, d ədədləri üçün ab = cd olduqda,


a + b + c + d ədədi sadə ola bilərmi?

25
224. Vuruqlara ayırın:
bc(a + d )(b − c ) − ac(b + d )(a − c ) + ab(c + d )(a − b ).

225. a + b + c = 0 olduqda a 3 + b3 + c 3 = 3 abc olduğunu


isbat edin.

226. Tənliyin tam həllərini tapın: 19 x3 − 84 y2 = 1984.

227. Cədvəldən istifadə etmədən


log 4 5 + log5 6 + log 6 7 + log 7 8 + log8 4 > 5 olduğunu
isbat edin.

228. Bərabərsizliyi isbat edin:


2006
 2007 
1 ⋅ 2 ⋅ 3 ⋅  ⋅ 2006 <   .
 2 

229. a, b, c ∈ R ədədləri üçün a 2 + b 2 + c 2 = 1 və


a 3 + b3 + c 3 = 1 münasibətləri ödəndikdə a + b + c
ifadəsinin qiymətini tapın.

230. Birrəqəmli ədədin sağ tərəfinə 5 yazıb alınan ədədi 5


dəfə artırsaq, ikirəqəmli bir ədədin kvadratı alınar. Bu
ədədləri tapın.

231. x 2 − 3 x + 1 = 0 tənliyinin kökləri x1 və x2 olduqda,


2 ifadəsinin qiymətini tapın.
4 x +4 x
1

232. x 5 + x 4 + 3 x 3 + x 2 + x − 1 ifadəsini vuruqlara ayırın.

26
233. x 3 + x + 350 ifadəsinin (x+7) -yə bölündüyünü isbat
edin.

( )
234. a ≥ 0, b ≥ 0 olduqda 3 a 4 + b 4 + 101> 24 ab olduğunu
isbat edin.

235. x 44 + x33 + x 22 + x11 + 1 çoxhədlisinin x 4 + x3 + x 2 +


+ x + 1 çoxhədlisinə bölündüyünü isbat edin.

236. Tənliyi həll edin: 6 ⋅ xx ⋅ xy =xxyy .

a 2 + (a − c ) a−c
2
237. a 2 + b 2 = ( a + b − c ) olarsa, =
2

b + (b − c ) b−c
2 2

olduğunu isbat edin.

238. x 4 + x 2 + 1 üçhədlisini vuruqlara ayırın.

239. 210 + 213 + 2 n ifadəsi n -in hansı natural qiymətlərində


tam kvadratdır?

240. Hansı sadə p və q ədədləri üçün (p+1)q ifadəsi tam


kvadrat olar?

241. Müsbət a, b, c ədədləri üçün a+b+c = 1 olarsa,


ab bc ca 3
+ + ≤
c + ab a + bc b + ca 2
bərabərsizliyini isbat edin.

242. Həqiqi a, b, c ədədləri üçün


27
a−c b−a c−b
+ + = 1 olduqda
b+c c+a a+b
a+b b+c c+a
+ + ifadəsinin ədədi qiymətini tapın.
b+c c+a a+b

243. Neçə ( p, q ) sadə ədədlər cütü üçün p q + q p ədədi


sadədir?

244. a, b, c, d tam ədədləri a 2 − b 2 = c 2 − d 2 = 2004


bərabərliyini ödəyir. İsbat edin ki,
2(a + b )(с + d )(ac + bd − 2004)
ədədi tam ədədin kvadratıdır.

245. x y− x− y = 102 bərabərliyini ödəyən mənfi olmayan


m, n tam ədədlərini tapın.
246. 16913 + 16256 ədədinin 85-ə bölündüyünü isbat edin.

247. İsbat edin ki, istənilən müsbət x və y ədədi üçün


x y 1
+ 2 ≤ bərabərsizliyi doğrudur.
x +y x +y
4 2 4
xy

248. a = 21000 − 1 və b = 21001 + 1 ədədinin ən böyük ortaq


bölənini tapın.

249. Elə natural (a, b) ədədlər cütünü tapın ki,


ab
ƏKOB(a, b) − ƏBOB(a, b) = bərabərliyi ödənsin.
5

28
250. Elə natural n ədədi tapın ki, n 4 − 22n 2 − 46 ədədi n + 5
ədədinə tam bölünsün.

251. 56 ilə qurtaran, rəqəmləri cəmi 56 olan və 56 -ya


bölünən ən kiçik natural ədədi tapın.

252. İsbat edin ki, x 2011 y = xy 2011 + 2011 tənliyinin tam


ədədlər çoxluğunda həlli yoxdur.

253. abc = 2 olduqda a 4 + b 4 + c 2 ≥ 4 olduğunu isbat


edin.

254. Ədədi silsilə əmələ gətirən 2007 sayda müxtəlif natural


ədədin hasili tam kvadrat olarsa, bu ədədləri tapın.

255. x 2 + 98 x 4 y 4 + y 8 üçhədlisini tam əmsallı iki çoxhədlinin


hasili şəklində göstərin.

256. 332− 232 ədədinin 100 -dən kiçik olan 4 sadə vuruğunu
tapın.

257. n∈N olduqda 5n + 6 və 8n + 7 şəklində olan tam


ədədlərin bütün ƏBOB -nu tapın.

258. Məlumdur ki, 2 x 2 + mx + 2 − n = 0 tənliyinin hər iki


kökü sıfırdan fərqli tam ədədlərdir. İsbat edin ki,
m2 + n2
ədədi mürəkkəb ədəddir.
4
259. İsbat edin ki, əgər sin x + cos 2x və sin 2 x + cos x
ədədləri sıfırdan fərqli rasional ədədlərdirsə, sin x və
cos x ədədləri də rasionaldırlar.
29
260. 7-yə bölünən bir ədədi 2-yə, 3-ə, 4-ə, 5-ə və 6-ya
böldükdə hər dəfə qalıqda 1 alınır. Bu şərti ödəyən ən
kiçik ədədi tapın.

( )3
261. a + b = a b olarsa, a 3 + b3 − a 3b3 + 27 a 6b 6 = 0
olduğunu isbat edin.

262. a, b, c, d həndəsi silsilənin ardıcıl hədləri olduqda


(a 2
)( )
+ b 2 + c 2 b 2 + c 2 + d 2 = (ab + bc + cd )
2

olduğunu isbat edin.

263. x, y, z cüt-cüt bərabər olmayan tam ədədlər olduqda


(x − y )7 + ( y − z )7 + (z − x )7 ifadəsinin
7( x − y )( y − z )( z − x ) ifadəsinə bölündüyünü isbat edin.

x− y
264. ifadəsinin ən böyük qiymətini tapın.
x2 + y 2

265. Tənliklər sisteminin tam həllərini tapın:


 x 2 − y 2 − z 2 =1,

 y + z − x = 3.

266. x və y-in hansı qiymətlərində xxyy ədədi natural ədədin


kvadratı olar?
x5 + x3 + 6
267. f ( x ) = rasional funksiyasının (0; +∞)
x
intervalında ən kiçik qiymətini tapın.

30
268. Tənliyi həll edin:
(x 2
) (
3
) (
3
)3
+ 3x − 4 + 2 x 2 − 5 x + 3 = 3x 2 − 2 x − 1 .

269. a, b, c, d ∈ N üçün a 2 + b 2 = c 2 + d 2 olarsa, a+b+c+d


cəminin mürəkkəb ədəd olduğunu isbat edin.

270. İkirəqəmli ədədin onluq rəqəminin kvadratı ilə təklik


rəqəminin kubunun cəmi onun özünə bərabərdir. Həmin
ədədi tapın.

271. 3 -rəqəmli ədəd 7 rəqəmi ilə başlayır. 7 rəqəmini


axırıncı yerə keçirməklə, başqa 3 -rəqəmli ədəd aldılar.
Alınmış ədəd əvvəlkindən 117 vahid kiçik olarsa,
əvvəlki ədədi tapın.

272. Elə bir irrasional α və β ədədləri varmı ki, αβ rasional


ədəd olsun. Əgər varsa, nümunə göstərin.

273. Tənliyi həll edin:


x − 49 x − 50 49 50
+ = + .
50 49 x − 50 x − 49

274. İkirəqəmli ədədin təklik rəqəminin kvadratı ilə onluq


rəqəminin cəmi ədədin özünə bərabərdir. Bu ədədi tapın.

275. Heç biri 50-yə bölünməyən 50 ədəd vardır. Bu ədədlər


içərisində ən azı iki ədədin 50-yə bölünməsindən eyni
qalıq alındığını isbat edin.

276. İsbat edin ki, üç ardıcıl ədədin cəmi tək ədəd olarsa,
onların hasili 24-ə bölünür.

31
277. 9-a bölünən elə ən kiçik ədəd tapın ki, o ədədi 2-yə, 4-ə,
5-ə, 7-yə və 8-ə böldükdə qalıqda 1 alınsın.

278. x 6 − 6 x 5 + ax 4 + bx 3 + cx 2 + dx + 1= 0 tənliyinin bütün


kökləri müsbətdirsə, a, b, c, d -ni tapın.

279. a, b, c ∈ R üçün a+b+c = 2 və a 2 + b 2 + с 2 = 2


olduqda cmax = ?

280. a ≥ 0, b ≥ 0, c ≥ 0 üçün a+b+c =1 olduqda


1− a 1− b 1− c 3
+ + ≥ olduğunu isbat edin.
1+ a 1+ b 1+ c 2

4
281. Tənliyi həll edin: 2 x 2 − 16 + x2 − 9 = .
x−4

282. Əgər a+b+c+d = 8 olarsa, a+ b+ c+ d ≤ 6


olduğunu isbat edin.

283. a, b, c, d ∈ R üçün a 2 + b 2 = 1 və c 2 + d 2 =1 olduqda


ac + bd ≤ 1 olduğunu isbat edin.

284. Həqiqi a, b, c, d, e ədədləri üçün a+b+c+d+e = 8 və


a 2 + b 2 + c 2 + d 2 + e 2 = 16 olduqda e -nin ən böyük
qiymətini tapın.

285. Həqiqi x1 , x2 ,  , xn ədədləri üçün


(x1 + x2 +  + xn )2 ≤ n ⋅ (x12 + x22 + + xn2 ) olduğunu isbat
edin.

32
 n +1
n

286. ∀n∈ N üçün n! ≤   olduğunu isbat edin.


 2 

287. Müsbət a1 , a2 ,  , an ədədləri üçün


 1
(a1 + a2 +  + an ) ⋅  1 +
1
+  +  ≥ n 2 olduğunu
 a1 a2 an 
isbat edin.

288. İfadənin qiymətini tapın:


 2011 
2
2011
1 + 2011 + 2
 + .
 2012  2012
289. Tənliyin tam həllərini tapın:
x 4 + 2 x3 + 2 x 2 + 2 x + 1 = y 2 .

290. 117 + 217 + 317 + +100017 ifadəsinin 1001-ə


bölündüyünü isbat edin.

291. Bərabərsizliyi həll edin: [x ]⋅ {x} < x − 1 .

292. 2011 ⋅ 2111 + 2500 ədədi sadədirmi?

( )
293. Tənliyi həll edin: x 2 − 16 ⋅ ( x − 3) + 9 x 2 = 0.
2

x : y : z : t = 1 : 2 : 3 : 4
294.  olduqda x, y, z və t -ni tapın.
9 x + 7 y + 3z + 2t = 200

295. log 3 x + log 3 y + log 3 z = 3 olduqda x+y+z cəminin ala


biləcəyi ən kiçik qiyməti tapın.

33
296. Bir 1-dən, iki 2-dən, üç 3-dən və ilə axır, doqquz 9-dan
ibarət olan ədəd tam kub ola bilərmi?

297. Ardıcıl 2002 natural ədədin cəmi 2002-yə bölünə


bilərmi?

298. Tənliyi həll edin: 32 x − y + 6 + 5 x + y − 3 = 2.

299. Müsbət a, b, c həqiqi ədədləri üçün a 2 + b 2 + c 2 = 3


1 1 1
olduqda + + ≥ 1 bərabərsizliyini
1 + 2ab 1 + 2bc 1 + 2ca
isbat edin.
300. Düzbucaqlı üçbucağın tərəfləri ədədi silsilə əmələ
gətirir. Isbat edin ki, bu silsilənin fərqi daxilə çəkilmiş
çevrənin radiusuna bərabərdir.

301. İsbat edin ki, 11122 2 (10 vahid və 10 iki) ədədi iki
ardıcıl tam ədədin hasilidir.

302. Vuruqlara ayırın:


a(b + c ) + b(c + a ) + c(a + b ) − 4abc.
2 2 2

303. 9 ( x − 2 ) =194 y bərabərliyindən x və y rəqəmlərini


3

tapın.

304. 1, 7, 19,37, 61, 91, … ədədlər ardıcıllığı üçün


a) Hn ümumi həddinin düsturunu tapın
b) İsbat edin ki, H1 + H 2 + H 3 +  + H n = n3 .

34
305. Hasili, natural ədədin 2008-ci qüvvəti olan 5 müxtəlif
natural ədəd, ədədi silsilə əmələ gətirə bilərmi?

306. x, y, z ∈[1;2] olduqda


x2 y2 z2 x+ y+z
+ + ≥
yz + 2 xz + 2 xy + 2 2
bərabərsizliyini isbat edin.

307. İsbat edin ki, müsbət a, b, c ədədləri üçün


a 2 + b2 b2 + c2 a 2 + c2
+ + ≥ 3.
c 2 + ab a 2 + bc b 2 + ac

x
308. İstənilən x, y ∈R + ədədləri üçün y 2 f ( x ) = f  
 y
bərabərliyini ödəyən bütün f : R → R funksiyalarını
+ +

tapın.

309. İsbat edin ki, 2011! < 10062011.

310. Tənliyi həll edin: 10arccos x + lg x = x − 2 .

311. İkirəqəmli ədədlə onun rəqəmlərinin tərs sırada


yazılışından alınan ədədi topladıqda tam kvadrat alınır.
Həmin ədədləri tapın.

312. Tənliyin həqiqi həllərini tapın:


5 ( x 2 + y 2 + 1) = 6 x + 8 y .

35
313. a, b və c ədədləri x 3 + 3x 2 − 7 x + 1 = 0 tənliyinin
kökləridirsə, A = a 2 + b 2 + c 2 ifadəsinin qiymətini tapın.

314. ai > 0 üçün a1 ⋅ a2  an = 1 olduqda


(1 + a1 ) ⋅ (2 + a2 )(n + an ) ≥ 2n n! olduğunu isbat edin.

315. 29 + 299 cəminin 100-ə bölündüyünü isbat edin.

316. Tənliklər sisteminin tam həllini tapın:


 x + y = 2,

 xy − z = 1 .
2

317. İsbat edin ki, 11111 (27 vahid) ədədi 27-yə bölünür.

318. Elə ikirəqəmli ab və cd ədədləri varmı ki,


ab ⋅ cd = abcd ödənsin?

319. İsbat edin ki, istənilən natural n üçün n3 + 3n 2 + 6n + 8


ədədi mürəkkəb ədəddir.

320. a + b + c = −5, ab + ac + bc = 3 və abc = 9 olarsa,


ab ac bc
+ + ifadəsinin qiymətini tapın.
c b a

321. 58 + 28 + 20000 ifadəsinin qiymətini tapın.

x4 + x2 + 1 1 1
322. = 4 olarsa, + =?
x +1
3
x x2

36
323. abc = a ! + b! + c ! tənliyini həll edin.

324. x 5 − 2 x 2 + 3 = 0 tənliyinin tam həllərini tapın.

325. sin x = [x ] tənliyini həll edin.

326. a, b, c üçbucağın tərəfləri olduqda


a b c
+ + ≤2
1 + b + c 1 + c + a 1 + a + b2
2 2

bərabərsizliyini isbat edin.

327. p1 və p2 ax 2 + bx + c = 0 tənliyinin iki müxtəlif kökü,


q1 və q2 isə cx 2 + bx + a = 0 tənliyinin iki müxtəlif
köküdür. Əgər p1 , q1 , p2 , q2 ədədi silsilə əmələ gətirirsə,
isbat edin ki, a + c = 0.

328. Tənliyi həll edin:


x 2 + x − 1 + 1 + x − x 2 = x 2 − x + 2.

a ⋅b⋅c
329. Üçbucaqda ≥ 4 r 2 olduğunu isbat edin. Burada
a+b+c
a, b, c tərəflər, r isə daxilə çəkilmiş çevrənin
radiusudur.

330. Üçbucaqda a 2 + b 2 + c 2 ≥18 Rr olduğunu isbat edin.


Burada a, b, c tərəflər, R və r uyğun olaraq, xaricə və
daxilə çəkilmiş çevrələrin radiuslarıdır.

37
331. a(b + c ) + b(a + c ) + c(a + b ) − 3abc ifadəsini
2 2 2

vuruqlara ayırın.

332. P ( x ) = x 3 + 6 x 2 + 5 x + a çoxhədlisinin kökləri ədədi


silsilə əmələ gətirirsə, P(x) çoxhədlisini və onun
köklərini tapın.

1 2 2011
333. İsbat edin ki, + ++ < 1.
2! 3! 2012 !

334. 21000 ədədi m -rəqəmli, 51000 ədədi isə n -rəqəmli


ədəddirsə, m + n cəmini tapın.

1
335. Müsbət x ədədi üçün x 2 + = 7 -dir. İsbat edin ki,
x2
1
x5 + ədədi tamdır və həmin ədədi tapın.
x5

336. İfadənin qiymətini tapın:


1!⋅ 3 − 2!⋅ 4 + 3!⋅ 5 − 4!⋅ 6 +  − 2000!⋅ 2002 + 2001! .

337. İsbat edin ki, tg 11° < 0,2.

1
338. İsbat edin ki, sin 10o > .
6

339. x -in istənilən qiymətində

x6 + 2 x5 + 2 x 4 + 2 x3 + 2 x 2 + 2 x + 1 ≥ 0
bərabərsizliyinin doğru olduğunu isbat edin.

38
340. n -in hansı natural qiymətində n5 + n + 1 və n11 + n + 1
ədədləri sadə ədəddir?

341. a və b ədədlərinin hansı qiymətlərində


x 4 + ax 3 + bx 2 − 8 x + 4 çoxhədlisi tam kvadratdır?

342. y = sin100 x + cos100 x funksiyasının ən kiçik qiymətini


tapın.

343. Tənliyi həll edin: [x ] ⋅ {x}= 2005x.


( [x] və {x} uyğun olaraq, ədədin tam və kəsr hissəsidir).
344. n -in bütün elə natural qiymətlərini tapın ki, həmin
qiymətlərdə n8 + n + 1 ifadəsinin qiyməti sadə ədəd
olsun.

345. n -in hansı natural qiymətlərində 32 n +1 − 22 n +1 − 6n


ədədi mürəkkəb ədəddir?

346. Yazılışında yalnız bir dəfə iştirak etməklə, 1-dən 9-a


qədər rəqəmlərdən təşkil olunmuş bütün natural ədədlər
cəminin 999999999 -a bölündüyünü isbat edin.

347. Vuruqlara ayırın: x 3 ( y − z ) + y 3 ( z − x ) + z 3 ( x − y ).

348. a, b, c ədədləri ədədi silsilə əmələ gətirdikdə

a 2 (b + c ) + b 2 (a + c ) + c 2 (a + b ) = (a + b + c )
2 3

9
olduğunu isbat edin.

39
2 6
349. sin α + sin β = və cos α + cos β = olduqda
2 2
sin (α + β ) − nın qiymətini tapın.

350. Bütün x və y üçün f ( x − y ) = f ( x ) ⋅ f ( y ) -dir.


f ( x ) ≠ 0 olduqda f (3) -ü tapın.

351. Hər bir (a, b) ədəd cütü üçün f ( x ) funksiyası

b 2 ⋅ f (a ) = a 2 ⋅ f (b ) bərabərliyini ödəyir. f (2 ) ≠ 0

f ( 5 ) − f (1)
olmaqla ifadəsinin qiymətini tapın.
f ( 2)

352. cos10 x − sin10 x = 1 tənliyinin [0; π] parçasındakı


həllərini tapın.

353. ABC üçbucağında ab =2


=
cos A bc 2
cos B ca 2 cos C
olarsa, bu üçbucağın bərabəryanlı olduğunu isbat edin.

354. Tənliyi həll edin: 8 cos x ⋅ cos 4 x ⋅ cos 5 x = 1.

355. Tənliyi həll edin:


cos 2 x + cos 2 y = cos x ⋅ cos y + cos x + cos y − 1.

356. Üçbucaqda tgA − tgB − tgC = tgA ⋅ tgB ⋅ tgC münasibəti


ödənirsə, bu üçbucağın düzbucaqlı üçbucaq olduğunu
isbat edin.

40
α β γ
357. α + β + γ = 0 və a + b + c = 0 və =0 + +
a b c
olduqda, α a 2 + β b 2 + γ c 2 = 0 olduğunu isbat edin.

358. x, y ∈ R üçün f ( x ) ⋅ f ( y ) = f ( x − y ) şərtini ödəyən və


eyniliklə sıfıra bərabər olmayan f ( x ) funksiyasını tapın.

359. İxtiyari x, y ∈ R üçün f ( x ) + f ( y ) = f ( x − y ) + 2 xy


şərtini ödəyən bütün f funksiyalarını tapın.

360. x 3 − 3x 2 − x + a = 0 tənliyinin kökləri ədədi silsilə


əmələ gətirsə, həmin kökləri və a -nı tapın.
1 + f (x )
361. İsbat edin ki, f (x + a ) = funksiyası dövri
1 − f (x )
funksiyadır və onun dövrünü tapın.

362. tg 210o + tg 2 50o + tg 2 70o = 9 olduğunu isbat edin.

363. sin α + sin β = a, cos α + cos β = b olarsa,


α β 4a
tg + tg = 2 olduğunu isbat edin.
2 2 a + b 2 + 2b

364. α + β + γ =
π olduqda
α β β γ γ α
tg ⋅ tg + tg ⋅ tg + tg ⋅ tg = 1 olduğunu isbat edin.
2 2 2 2 2 2

365. İstənilən həqiqi x və y ədədləri üçün


f ( x y ) = f ( x ) ⋅ cos y funksional tənliyini həll edin.

41
366. f (2 x + 1) = 4 x 2 + 14 x + 7 funksional tənliyini həll edin.

x + y + z = 6
367. İsbat edin ki, koordinatları 
 xy + yz + zx = 11
tənliklər sistemini ödəyən nöqtələrin həndəsi yeri
çevrədir. Həmin çevrənin radiusunu tapın.

 1
368. x ≠ 0 olduqda f 1 +  = x 2 − 1 tənliyini ödəyən f ( x )
 x
funksiyasını tapın.

369. İsbat edin ki, əgər a, b, c parçaları üçbucaq əmələ


gətirirsə, onda a , b , c parçaları da üçbucaq əmələ
gətirir.

370. Bərabərsizliyi isbat edin:


n n >1 ⋅ 3 ⋅ 5 ⋅ 7(2n − 1).

371. Bərabərsizliyi isbat edin:


( )
2
sin n 2 x + sin n x − cos n x ≤1.

372. tg x + ctg x = 6 y − y 2 − 7 tənliyinin bütün həllərini


tapın.

sin 1o sin 3o
373. Hansı böyükdür: yoxsa , ?
sin 2o sin 4o

374. Dörd dərəcəli P(x) çoxhədlisində P (1=) P ( −1) və

42
P ( 2=
) P ( −2 ) şərtləri ödənir. İstənilən x üçün
P ( x=) P ( − x ) olduğunu isbat edin.

375. Uzunluqları həndəsi silsilə əmələ gətirən üç parça


verilmişdir. Bu silsilənin ortaq vuruğunun hansı
qiymətlərində həmin parçalardan üçbucaq qurmaq olar?

376. cos 2 α + cos 2 β + cos 2 γ = 1 olduqda ABC üçbucağının


düzbucaqlı üçbucaq olduğunu isbat edin.

2
377. sin x + cos x = 2 olduqda sin 7 x + cos7 x =
8
olduğunu göstərin.
1 1
378. ( x − 1) ⋅ f ( x ) + f   = tənliyini ödəyən f (x)
 x  x −1
funksiyasını tapın.

379. x 3 − x + 1 = 0 tənliyinin köklərinin 16 -cı dərəcədən


qüvvətləri cəmini tapın.

380. f ( x + y ) − f ( x − y ) = 2 f ( y ) ⋅ cos x funksional tənliyini


həll edin (burada ∀x, y ∈ R) .

381. f (sin x ) + f (cos x ) = 3 olduqda f (x) funksiyasını tapın.

382. Məlumdur ki,


sin x + sin y + sin z cos x + cos y + cos z
= = a.
sin ( x + y + z ) cos( x + y + z )

43
İsbat edin ki, cos( x + y ) + cos( y + z ) + cos(z + x ) = a.

383. x 2 − 3x + 1 = 0 tənliyinin kökləri x1 və x2 olduqda


2n x1 + 2 n x2 > 2 olduğunu isbat edin.

384. İfadənin ən böyük qiymətini tapın:


sin x ⋅ sin y ⋅ sin z + cos x ⋅ cos y ⋅ cos z .

x2 + x
385. Tənliyi həll edin: 2 cos 2 = 3x + 3− x.
6

386. İsbat edin ki, 2010 + 2008 ⋅ 2009 ⋅ 2011 ⋅ 2012 ədədi
natural ədədin kvadratıdır.

( )
387. log5 1 + x > log16 x bərabərsizliyini həll edin.

388. a -nın hansı qiymətində a 2 x 2 − a ⋅ tg (cos x ) + 1 = 0


tənliyinin yeganə kökü var?

389. Kəsilməz f : R → R funksiyası üçün f ( f ( x )) ⋅ f ( x) = 1


bərabərliyi ödənir. Məlumdur ki, f (1000) = 999 -dir.
f (500) -ü tapın.

390. x ⋅ f ( y ) + y ⋅ f (x ) = (x + y ) f (x ) ⋅ f ( y ) eyniliyini ödəyən


bütün f : R → R funksiyalarını tapın.

391. Tənliyi həll edin:


f ( x + y ) + f ( x − y ) − f ( x ) − x 3 − 6 xy ⋅ 3 f ( y ) =
0.

44
392. f : R → R + üçün f (x + y ) = ( f (x )) tənliyini həll edin.
y

393. Tənliyi həll edin: f (xy ) = ( f (x )) .


y

394. Tənliyi həll edin: f ( x + y ) = f ( x ) + y .

395. İsbat edin ki, x <1 olduqda


(1 − x )n + (1 + x )n ≤ 2n , (n∈ N ) bərabərsizliyi doğrudur.
1 1 1 2n − 1
396. + ++ ≤ bərabərsizliyini isbat edin.
1! 2! n! n

397. İsbat edin ki, natural n ədədi üçün n! ≥ 2n −1


bərabərsizliyi doğrudur.

398. İsbat edin ki, 3


3+ 3 3 + 3
3 − 3 3 < 23 3 .

399. Tənliklər sistemini həll edin:


 x y = 1,

 x + y + cos z = 2 .
2

400. Tənliyi həll edin:


[x] + [2 x] + [4 x] + [8 x] + [16 x] + [32 x] = 12345.
401. Tənliyi həll edin:
[x] + [2 x] +  + [nx] = 1, (n ≥ 2, n∈ N ).

45
402. Tənliyin bütün həllərini tapın: x 2 − 8 [x ] + 7 = 0.

403. Tənliyi həll edin: { ( x + 1) } =


3
x .3

1 + sin x − cos x
404. Göstərin ki, f ( x ) = funksiyası
1 + sin x + cos x
 π π
 − ,  aralığında tək funksiyadır.
 2 2

405. Tənliyin natural həllərini tapın:


[3 1] + [3 2 ] +  + [3 x 3 − 1] = 400.

 x + 2y
 x + x 2 + y 2 = 2,

406. Tənliklər sistemini həll edin: 
 y + 2 x − y = 0.
 x2 + y2

407. Tənliklər sistemini həll edin:


 3
sin x + sin y + sin z = 2 ,

cos x + cos y + cos z = 3 3 .
 2

408. İsbat edin ki, x istənilən müsbət ədəd olduqda е х ≥ х е


doğrudur.

46
409. Tam əmsallı elə çoxhədli tapın ki, α = 3 2 + 3 3 ədədi
onun kökü olsun.

1 + x2 + x −1
410. İsbat edin ki, f ( x ) = funksiyası bütün
1 + x2 + x + 1
həqiqi ədədlər çoxluğunda təyin olunmuşdur və tək
funksiyadır.

1
411. Tənliyi həll edin: x 3 − x 2 − x = .
3

412. Məlumdur ki, ax 5 + bx 4 + c = 0 tənliyinin 3 müxtəlif


kökü var. İsbat edin ki, cx5 + bx + a = 0 tənliyinin də 3
müxtəlif kökü var.

413.
(a − b )(b − c )(c − a ) = 12 olduqda
(a + b )(b + c )(c + a ) 2012
a b c
+ + ifadəsinin qiymətini tapın.
a+b b+c c+a

414. sin 7 x -i sin x və cos x -lə ifadə edin.

415. Tənliyi həll edin: logsin x cos x + log cos x sin x =


2.

416. Tənliyi həll edin:


(1 + tg1 )⋅ (1 + tg 2 )⋅  ⋅ (1 + tg 44 )⋅ (1 + tg 45 ) = 2 .
o o o o x

417. Tənliyi həll edin:


sin x sin x + cos x cos x = sin x + cos x .

47
418. Tənliyi həll edin: sin x + cos x = tgx + ctgx .

419. Tənliyin natural həllərini tapın:


xy yz xz
+ + = x + y + z.
z x y

420. Tənliyi həll edin: x


x4
= 4 , ( x > 0 ).

421. Tənliyi həll edin:


(1 + x + x )(1 + x +  + x ) = (1 + x +  + x ) .
2 10 6 2

( )(
422. İsbat edin ki, x + x 2 + 1 y + y 2 + 1 =1 olduqda )
x + y = 0 olur.
423. İfadənin ən böyük qiymətini tapın:
x 1 − y2 + y 1 − x2 .
3
 
424.  999
 99  ədədinin rəqəmləri cəmini tapın.


 2002 

425. İsbat edin ki, p, q, r və s ədədi silsiləsinin hədləri


həndəsi silsilə əmələ gətirərsə, onda p − q, q − r və
r − s ədədləri də həndəsi silsilə əmələ gətirər.

426. İsbat edin:


2 3 4 2 3 4 2 3 4 41
+ 2 + 3 + 4 + 5 − 6 + 7 + 8 + 9 + =
7 7 7 7 7 7 7 7 7 114

48
427. İsbat edin: a 2 + b 2 + c 2 ≤| a | + | b | + | c | .

428. İsbat edin ki, a > b > 0 olarsa, x > ab üçün


x+a x+b
>
x2 + a2 x2 + b2 .

429. Cəmi tapın: S = a12 − a22 + a32 − a42 +  + a22n −1 − a22n ,


burada a1 , a2 , a3 , , a2 n −1 , a2 n − silsilə fərqi d olan ədədi
silsilənin hədləridir.

430. İsbat edin ki, x 2 + y 2 ≤ 2 olarsa, | x + y |≤ 2 .

431. Bərabərliyi isbat edin:


1 2 4 2n 1 2n +1
+ 2 + 4 +  + 2n = − 2n+1
a +1 a +1 a +1 a +1 a −1 a −1

n
432. İsbat edin ki, n ∈ N , n > 2 və a = üçün
n−2
x n − ax n −1 + ax + 1 çoxhədlisi ( x − 1) 2 ifadəsinə bölünür.

433. Bərabərsizliyi isbat edin:


x 2 y 4 + 2( x 2 + 2) y 2 + 4 xy + x 2 ≥ 4 xy 2

434. İsbat edin ki, istənilən n və p natural ədədləri üçün


aşağıdakı bərabərsizlik doğrudur:

49
1 1 1 1 1
− < + + + <
n + 1 n + p + 1 (n + 1) (n + 2)
2 2
(n + p) 2
1 1
< −
n n+ p .

435. İsbat edin ki, əgər n və k tək natural ədədləridirsə,


onda 1k + 2k + 3k +  + (n − 1) k cəmi n − ə bölünür.

436. İsbat edin ki, a > 0, b > 0, c > 0 olarsa,


3 1 1 1
< + + bərabərsizliyi doğrudur.
a+b+c a+b a+c b+c

437. İsbat edin ki, 600! ədədi 799 -a bölünmür.

438. İsbat edin ki, x3 + y 3 + z 3 + axyz çoxhədlisi, ancaq


a = −3 olduqda x + y + z cəminə bölünür.

439. İsbat edin ki, a ≥ 0, b ≥ 0 olarsa,


1 + a + b 1 + a + b + ab
≥ bərabərsizliyi doğrudur.
2 2+a+b

440. İsbat edin ki, a > b > 0 və n vahiddən böyük natural


ədəd olarsa, n a − b > n a − n b bərabərsizliyi doğrudur.

2n + 3
441. İsbat edin ki, n -in heç bir qiymətində ifadəsi
5n + 7
ixtisar edilməyəndir.

442. Bərabərsizliyi isbat edin:

50
a 2b 2 + b 2 c 2 + c 2 a 2 ≥ abc(a + b + c) .

443. İsbat edin ki, f ( x, y ) çoxhədlisi x və y -ə görə


simmetrik olarsa və x − y ifadəsinə bölünərsə,
onda f ( x, y ) həm də ( x − y ) 2 ifadəsinə bölünər.

444. İsbat edin ki, x 2 − 3 y 2 =


17 tənliyinin tam həlləri yoxdur.

445. İsbat edin ki, a − nın istənilən natural qiymətində a 3 − a


ifadəsi 3-ə bölünür.
446. İsbat edin ki, tam ədədin kvadratının 3-ə bölünməsindən
alınan qalıq ya 0, ya da 1-dir (yəni 2 ola bilməz).

447. a 3 + 2a cəmi a -nın istənilən tam qiymətində 3-ə


bölünürmü?

448. İsbat edin ki, a, b, c -nin tam qiymətlərində a + b + c


cəmi 3-ə bölünürsə, onda a 3 + b3 + c3 cəmi də 3-ə
bölünər.

449. İsbat edin ki, a ədədi 3-ə bölünürsə, 5a 2 + 1 ədədi 3-ə


bölünür.

450. İsbat edin ki, a 2 + b 2 cəmi a və b -nin tam


qiymətlərində 3-ə bölünərsə, onda a və b ədədlərindən
hər biri 3-ə bölünər.

451. İsbat edin ki, a -nın istənilən tam qiymətində a 5 − a


ifadəsi 5-ə bölünür.

51
452. İsbat edin ki, a-nın istənilən tam qiymətində a 4 ədədinin
5-ə bölünməsindən alınan qalıq ya 0, ya da 1-dir.

453. İsbat edin ki, a -nın istənilən tam qiymətində


9a 5 − 5a 3 − 4a ifadəsi 15-ə bölünür.

454. x və y -in elə bütün tam qiymətlərini tapın ki,


x 2 − 2 xy + 2 y 2 və x 2 + 2 xy + 2 y 2 ifadələrindən ən azı
biri 5-ə bölünsün.

455. İsbat edin ki, istənilən natural n ədədi üçün elə natural
a ədədi tapmaq olar ki, a (n + 1) − (n 2 + n + 1) ifadəsi n3 -
na bölünər.

456. Şəhər olimpiadasında 2 əkiz qardaş iştirak edirdi.


Onlardan digər qardaşlarının olub olmaması barəsində
soruşduqda deyirlər: “Bizim daha bir qardaşımız var.
Onun yaşını göstərən ikirəqəmli ədədin rəqəmləri
eynidir. Üçümüzün yaşları cəmi, təklik rəqəmi onluq
rəqəmindən iki dəfə böyük olan ikirəqəmli ədəddir”.
Qardaşların yaşını tapın.

457. a, b, c həqiqi ədədləri üçün a > b > c . İsbat edin ki,


a 2b + b 2 c + c 2 a > b 2 a + a 2 c + c 2b

1
458. Əmsalları p + q =2001 şərtini ödəyən y = x 2 + px + q
2
kvadrat üçhədlərin qrafiklərinin eyni bir nöqtədən
keçirdiyini isbat edin.

459. İfadənin qiymətini tapın:

52
1! ⋅ 3 − 2! ⋅ 4 + 3! ⋅ 5 − 4! ⋅ 6 +  − 2000! ⋅ 2002 + 2001!

460. İsbat edin ki, cəmi 201 olan, hasili isə 201-ə bölünən iki
natural ədəd yoxdur.

cos 2 x + 3
461. cos x ≠ 0 olarsa, ≥ 4 bərabərsizliyini
cos x
isbat edin.

462. Natural ədədlərin ardıcıl olaraq yazılmasından ibarət


olan 1234567891011...9899100 ədədinin 36-ya bölünüb
bölünmədiyini müəyyən edin.

463. Tənliklər sistemini həll edin:


( x + y )( x + z ) =
x

( y + z )( y + x) =2y
( z + x)( z + y ) =
 3z

464. a.b və c -nin müəyyən qiymətləri üçün


x+a x +b + x = c tənliyin sonsuz sayda kökü ola
bilərmi (bütün ədədlər həqiqidir),

1
465. Bərabərsizliyi həll edin: x –= 1 − 1 > 1 − .
x

466. Bərabərsizliyi isbat edin:

1 52 1
< lnn <
52 51 51 .

53
2 101
467. və llnn ədədlərini müqayisə edin.
201 100

468. Bərabərsizliyi isbat edin:


1 1
< arcsin 0,8 − arcsin 0, 6 <
4 3

469. Bərabərsizlikləri isbat edin:


7 1
a) sin 200 < ; b) sin 200 >
20 3

470. İsbat edin ki, x ≥ 1 olduqda e x ≥ ex


xe .

e
e
1 1

π e
471. Müqayisə edin: e və  
π  .

472. İsbat edin ki, ədədi silsilədə kənar hədlərdən eyni


məsafədə duran hədlərin hasili, kənar hədlərin
hasilindən kiçik deyildir.

473. Ədədi silsilə əmələ gətirən elə ortaq natural ədəd tapın
ki, bunların ortaq böləni olmasın və ən böyüyünün kubu
qalan üç həddin kubları cəminə bərabər olsun.

474. Funksiyanın ən kiçik qiymətini tapın:


f ( x) =| x − 2 | + | x − 3 | + | x 2 − 4 | − | x 2 − 9 | .

54
475. Hər bir həqiqi a ədədi üçün
sin x + cos( x + a ) + cos(a − x) =
2 tənliyinin bütün həqiqi
həllərini tapın.

476. İsbat edin ki, =


f ( x) sin ax + cos x dövri funksiyadırsa,
onda a rasional ədəddir.

477. Bərabərsizliyi isbat edin x və y -in hansı


qiymətlərində bərabərliyə çevrildiyini göstərin:
( x + y )( x + y + 2sin x) + 2 ≥ 2 cos 2 x

478. Tənliyi həll edin:


10sin 2 x 2 + 4 cos 2 x + 3sin 2 x − 5 =0.

479. A = 0,99
 9 ədədini vergüldən sonra 101 onluq

100

işarəyə qədər hesablayın.

480 . Tənliyin həlli olmadığı isbat edin:


π
sin 
2


(
x  cos π x − 2 =
1 )
481. 1715 -315 fərqi 4-ə bölünürmü?

482. İsbat edin ki, n -nin istənilən natural qiymətində


2 ⋅ 7 n + 1 ədədi 3-ə bölünür.

483. İsbat edin ki, 2n + 2 ⋅ 3n + 5n − 4 cəmi n -in istənilən


natural qiymətində 25-ə bölünür.

55
484. İsbat edin ki, n -in istənilən natural qiymətlərində
11n + 2 + 122 n +1 cəmi 133 ədədinə bölünür.

485. Cəmi 37-yə bölünən iki üçrəqəmli ədəd ard-arda


yazılmışdır. Alınmış altırəqəmli ədədin 37-yə
bölündüyünü isbat edin.

486. İsbat edin ki, 111-dən 999-a qədər bütün üçrəqəmli


ədədləri istənilən ardıcıllıqla ard-arda yazsaq, alınan
çoxrəqəmli ədəd 37-yə bölünər.

487. İsbat edin ki, əgər 6a + 11b cəmi (a, b tam ədədlərdir)
31-ə bölünürsə, a + 7b cəmi də 31-ə bölünər.

488. 456 ədədinin sonuna hansı üç rəqəmi yazmaq lazımdır


ki, alınan 6 -rəqəmli ədəd 504-ə bölünər. Məsələnin
şərtini ödəyən bütün mümkün həlləri tapın.

489. İsbat edin ki, a -nın istənilən tək qiymətində a 2 − 1 fərqi


8 -ə bölünür.

490. İsbat edin ki, n -nın istənilən tam qiymətində


n5 − 5n3 + 4n çoxhədlisi 120-yə bölünür.

2k 2 + k − 8
491. Bütün elə tam k ədədlərini tapın ki,
k −1
kəsrlərinin qiyməti tam olsun.

492. x dəyişəninin bütün elə həqiqi qiymətlərini tapın ki,


x3 + 2 x − 3
ifadəsi tam qiymətlər alsın.
x2 + 1

56
493. İsbat edin ki, istənilən on ardıcıl natural ədəddən elə
üçünü, məsələn a, b və c ədədlərini seçmək olar ki,
a (b + c) hasili 100 -ə bölünsün.

494. İsbat edin ki, istənilən üç ardıcıl tam ədədin kubları


cəmi 3-ə və 9-a bölünür.
495. İsbat edin ki, əgər a 2 + b 2 , ( a, b ∈ N ) cəmi 11-ə bölünər-
sə, onda a və b ədədlərindən hər biri 11-ə bölünər.

496. İsbat edin ki, n -in heç bir qiymətində n 2 + 3n + 4 cəmi


49 -a bölünmür.

497. İsbat edin ki, m, n, p, q - natural ədədlər, m ≠ p və


(mn + pq ) (m − p ) olarsa, (mq + np ) (m − p ) olar.

498. İsbat edin ki, əgər p ədədi 2 -dən böyük sadə ədəd
1 1 1
olarsa, onda S =1 + + +  + cəminin əmələ
2 3 p −1
gətirdiyi kəsrin surəti p ədədinə bölünür.

499. Rəqəmləri fərqli olan elə bir ikirəqəmli ab ədədi varmı


ki, ba -ya bölünsün?

0, 0 < a < 1,


1
ax

500. İsbat edin: lim = = , a 1,
x →∞ 1 + a x 2

1, a > 1

57
−1, 0 < a < 1,
a x − a− x 
501. İsbat edin: lim x = =0, a 1
x →∞ a + a x
1, a > 1

xm −1 m
502. İsbat edin: lim n = ,
x →1 x − 1 n
burada m və n natural ədədlərdir.

n
503. İsbat edin: lim = 0.
n →∞ 2 n

504. İsbat edin ki, n > 2 olduqda lim n n = 1 .


n →∞

x − a + x−a 1
505. İsbat edin: lim = .
x→a
x −a
2 2
2a

1− x − 3
506. İsbat edin: lim = −2 .
x →−8 2+ 3 x

n2 + n
3
507. İsbat edin: lim = 0.
n →∞ n − 2

508. İsbat edin: lim


x→
( 3
)
( x + 1) 2 − 3 ( x − 1) 2 =
0.

x →∞
 3
509. İsbat edin: lim  x 2

( 
x3 + 1 − x3 − 1)  =

1. )
58
a+b
510. İsbat edin: lim
x →∞
( )
( x + a )( x + b) − x = .
2

3
1 + x2 −1 1
511. İsbat edin: lim = .
x →0 x2 3

( 1
512. İsbat edin: lim x( x 2 + 1 − x) = .
x →∞ 2
)
1 − cos(1 − cos x) 1
513. İsbat edin: lim =
x →0 x4 8

514. Bərabərsizliyi isbat edin:


 1   1  π
1 +  ⋅ 1 +  ≥ 3 + 2 2, burada 0 < α < 2 .
 sin α   cos α 

tgx − sin x 1
515. İsbat edin: lim = .
x →0 x3 2

sin 2 x − sin 2 α sin 2α


516. İsbat edin: lim = .
x →α x2 − α 2α

π 
517. İsbat edin: lim  − x  ⋅ tgx
x→  2
π
2

518. İsbat edin ki, istənilən natural n ≥ 2 ədədi üçün


4n (2n)!
<
n + 1 (n !) 2

59
tg 2 x 2
519. İsbat edin: lim = .
x → 0 sin 5 x 5

520. Tənliyi həll edin:


sin 2 x + sin 2 x ⋅ sin 4 x + sin 3x ⋅ sin 9 x +  + sin nx sin(n 2 x) =
1.

521. Tam əmsallı x 2 + px + q üçhədlində p + q = 30 olarsa,


kökləri tam ədəd olan neçə belə üçhədli var?

522. İsbat edin ki, istənilən müsbət x ədədi üçün aşağıdakı


bərabərsizlik doğrudur: 5 x + 13x > 23 x+14

523. İsbat edin ki, istənilən n ≥ 3 natural ədədi üçün


2 n > 2n + 1 .

524. Tənliyi həll edin: sin x + cos x =


0.

π π π
525. İsbat edin ki, 0 ≤ x ≤ , <α < olarsa,
2 6 3
 π sin x   π cos x 
tg   + tg   > 1 bərabərsizliyi doğrudur.
 4sin α   4 cos α 

cos(a + x) − cos(a − x)
526. İsbat edin: lim = −2sin α .
x →0 x

 π
sin  x − 
527. İsbat edin: lim  3
=
3
.
π 1 − 2 cos x 3
x→
3

60
2sin 2 x + sin x − 1
528. İsbat edin: lim = −3 .
x→
π 2sin 2 x − 3sin x + 1
6

2
529. İsbat edin ki, tg10 x = tənliyinin
(1 − tg 5 x) cos 5 x
2

həlli yoxdur.
530. Bərabərsizliyi həll edin:

x + 2 x −1 + x − 2 x −1 ≥ 2 .

x2 − 4 16
531. İsbat edin: lim = − .
x→2 πx π
cos
4

sin 3 x 3
532. İsbat edin: lim = − .
x →π sin 2 x 2

533. Tənliklər sisteminin tam həllərini tapın:


x + y + z =2,
 3
x + y + 3 =
3 3
2.

534. Tənliyin tam həllərini tapın:


1 1 1
arctg + arctg =. arctg
x y 2

535. Tənliyin natural həllərini tapın:


xy 2 − xy − y 2 + y =
94 .

61
536. Tənliyin tam həllini tapın: x+ y =98 .

537. İnteqralı hesablayın: ∫ ( x − a )(b − x)dx , (b > a ) .

538. İnteqralı hesablayın: ∫ a 2 + x 2 dx .

dx
539. İnteqralı hesablayın: ∫ sin 3
x
.

dx
540. İnteqralı hesablayın: ∫ sin x − sin a .
dx
541. İnteqralı hesablayın: ∫ (1 − x) 1 − x2
.

x
542. İnteqralı hesablayın: ∫ 2a − x
dx .

dx
543. İnteqralı tapın: ∫x 4x2 −1
.

544. İnteqralı hesablayın: ∫ ctg


6
xdx .

545. 2sin 2 α + 3tgα ⋅ ctgα ifadəsinin ən böyük və ən kiçik


qiymətini tapın.

546. Funksiyanın ən böyük qiymətini tapın:

62
y=
10 cos 2 x − 6sin x cos x + 2sin 2 x.

 15π   4
547. Hesablayın: arccos  sin  + cos  2 arcsin  .
 7   5

548. İfadəni hesablayın:


 3π π  7π 5π π
 cos + cos  sin ⋅ sin ⋅ sin .
 5 5 18 18 18

π cos α ⋅ cos13α
549. α = olarsa, ifadəsini hesablayın.
17 cos 3α + cos 5α

550. Funksiyanın təyin oblastını tapın:


1
f ( x) = .
−6sin 2 2 x − 2sin 2 x cos 2 x + 8 − 3

551. f ( x) = 2 x − 3, g ( f ( x)) = 4 x 2 − 8 x + 7 olarsa, g ′( x) -i


tapın.
1
) ( x + 1) x olarsa, f ′(1) − i tapın.
552. f ( x=

553. f ( x) =x5 − 2 x 4 + x3 − 2 x 2 + 4 x − 4 olarsa, ( f −1 )′(4)


törəməsini hesablayın.
π
554. f ( x) = ( x 2 )sin x olarsa, f ′   törəməsini hesablayın.
2

555. ∀×∈ Z + , f (0)= 0, f ( x=


) x 2 + f ( x − 1) olarsa,
f ′(1) + f (0) cəmini hesablayın.

63
556. f ( x) = ( x 2 + 1) ⋅ g (2 x + 1), g (5) = 6, g `(5) = 3 olarsa,
f ′(2) törəməsini hesablayın.

557. Bərabərsizliyi isbat edin: e3 > 3e .

558. Bərabərsizliyi isbat edin: sin 2 > sin 2 .


x 1
559. F ( x) =∫ (t + 2) 2 dt olarsa, f ′(4) törəməsini tapın.
0

x2
lnt
560. F ( x) = ∫ dt olarsa, F ′(e) törəməsini tapın.
x 2 + ln 2t
π
6
561. Müəyyən inteqralı hesablayın: ∫ sec x dx
0

562. İsbat edin ki, n nəfər adam dəyirmi masa ətrafında


(n − 1)! üsulla əyləşə bilər.

563. Neçə üsulla n oğlan və n qız dəyirmi masa ətrafında


əyləşə bilər, bu şərtlə ki, ixtiyari 2 qız (2 oğlan) yanaşı
oturmasın.

564. İsbat edin ki, n elementdən ibarət çoxluğun 2n sayda


alt çoxluğu var.

565. Tənliyi həll edin: sin 5 x − 2 cos 2 x =


3.

566. Tənliyi həll edin: sin 8 x − cos5 x =


1.

567. Tənliyi həll edin: 3sin x + 4 cos 3 x cos x + 2sin 5 x =


7.

64
568. Tənliyi həll edin: 4 cos 2 x − 4 cos 2 3 x cos x + cos 2 3 x =
0.

x
π.
569. Tənliyi həll edin: arcsin + 2 arcsin x =
2
570. Tənliyi həll edin: ( x + 1)( x + 2 )( x + 3)( x + 4 ) =
8.

571. Tənliyi həll edin: 3x + 4 x =.


7x

572. Tənliyi həll edin: 1+ x =


−1 .

573. Tənliyi həll edin: x3 =


+1 2 3 2x −1 .

574. Tənliklər sistemini həll edin:


 x3 + 2 x 2 + 2 x =
y
 3
 y + 2 y + 2x =
2
z
 z3 + 2z 2 + 2x = x

x 2 x + 2− x .
575. Tənliyi həll edin: 2 cos =

576. Tənliyi həll edin: x 1 + x + 3 − x= 2 1 + x 2 .

577. Tənliyi həll edin:


lg (cos x − 0,5) + lg (sin x − 0,3) + 1 =0.

578. Tənliyi həll edin: 8 x(2 x 2 − 1)(8 x 4 − 8 x 2 + 1) =.


0

579. n -in elə bütün natural qiymətlərini tapın ki, n 2 + 3n


ifadəsi tam kvadrat olsun.
65
580. İsbat edin ki, 2n + 4k (n, k ∈ N ) ədədləri arasında sonsuz
sayda tam kvadrat vardır.

581. İsbat edin ki, 2a, a + b və c tam ədədlər olarsa,


f ( x) = ax 2 + bx + c kvadrat üçhədlisi x -in istənilən tam
qiymətində yalnız tam qiymətlər alır.

582. 1 + 2 + 3 +  + n cəmindən neçə toplanan götürmək


lazımdır ki, rəqəmləri eyni olan üçrəqəmli ədəd alınsın?

583. Bütün elə k və n natural ədədlərini tapın ki,


7 k + 15n − 1
ifadəsi natural olsun.
3k + 4n

584. Müqayisə edin: sin10 və sin 11.

585. İsbat edin ki, aşağıdakı funksiyalar dövri funksiya


deyildir: a) y = sin x ; b) = y cos x + cos 2 x ;
c) y = cos x .
2

586. Tənliyi həll edin: 3cos x + 4sin x =


5sin 5 x .

587. Tənliklər sistemini həll edin:


5sin 2 x tgy = 12,

5sin 2 y tgx = 6

588. Tənliyin bütün tam həllərini tapın:

66
π
( 
cos  3 x − 9 x 2 + 160 x + 800  =
8 
1. )
589. Qabarıq n bucağının heç bir üç diaqonalı bir nöqtədə
kəsişmir. Diaqonalların kəsişmə nöqtələrinin sayını
tapın.

590. Şəhərdə n sayda işıqfor vardır. Onlardan hər biri


qırmızı, sarı və ya yaşıl rəngdə ola bilər. Bütün
işıqforları neçə üsulla yandırmaq olar?

591. İsbat edin ki, tənliyin həlli yoxdur:


1
sin10 x ⋅ cos12 x = .
1000
592. İsbat edin ki, əgər A, B, C üçbucağın bucaqları olarsa,
A B C
onda cos 2 , cos 2 , cos 2 ədədləri hər hansı
2 2 2
üçbucağın tərəfləri olar.

593. İsbat edin ki, əgər üçbucağın tərəflərinin uzunluğu


cos 50 , cos 350 , cos 500 olarsa, onda üçbucaq itibucaqlı
olar.
π π π
594. İsbat edin ki, 0 < α < , 0<β < , 0<γ < və
2 2 2
α β γ
α + β +γ =
π olarsa, tg 2 + tg 2 + tg 2 ≥ 1.
2 2 2

ϕ
595. İsbat edin ki, 0 < ϕ < π olarsa, 1 + ctgϕ ≤ ctg .
2

67
596. İsbat edin ki, α , β , γ üçbucağın daxili bucaqları olarsa,
3
cos 2 α + cos 2 β + cos 2 γ ≥ .
4

597. İsbat edin ki, α , β , γ üçbucağın daxili bucaqları və ya


kor bucaq olarsa, onda tgα tg β < 1 .
1 1 π
598. İsbat edin ki, 4arctg − arctg = .
5 239 4

599. İsbat edin ki, tənliyin həlli yoxdur:


 π  π
tg  x 2 +  tg  x 2 −  =1.
 6  3
600. İsbat edin ki, sin x və cos x ifadələrinin rasional olması
x
üçün tg ifadəsinin rasional olması zəruri və kafidir.
2
π
601. Əgər arctg x + arctg y + arctg z = olarsa,
2
1 olduğunu isbat edin.
xy + yz + zx =

π olarsa,
602. Əgər arctg x + arctg y + arctg z =
x+ y+z = xyz olduğunu isbat edin.

603. İfadənin qiymətini tapın: arctg1 + arctg 2 + arctg 3 .

604. İsbat edin ki, ctg 700 + 4 cos 700 =


3.

605. Tənliyi həll edin: 8 x3 − 6 x − 1 =0 .

68
606. Hər bir x, y ∈ R üçün f ( x) ≤ x və
f ( x + y ) ≤ f ( x) + f ( y ) şərtini ödəyən
f : R → R funksiyasını tapın.

607. Tənliyi həll edin: x 2 − 10 [ x ] + 9 =0.

608. Tənliyi həll edin: 19 [ x ] − 96 { x} =


0.
([ x ] və { x} uyğun olaraq, ədədin tam və kəsr hissəsidir)

609. Tənliklər sistemini həll edin:


 x + [ y ] + { z} =
1,1

 y + [ z ] + { x} =
2, 2

 z + [ x ] + { y} =
3,3

610. Tənliyin köklərinin sayını tapın:


19 ⋅ [ x ] + 98 ⋅ { x} =
1998 .

 x3 + 3x 2 + 2 x + 4 
611. Tənliyi həll edin:  = x + 1 .
 6 

612. Tənliyi həll edin: [tg x ] = 2 cos 2 x .

613. İsbat edin ki, x5 − y 5 =


1997 tənliyinin tam həlli
yoxdur.

69
614. İsbat edin ki, ( x − y) + ( y − z) + ( z − x )3 =
3 3
30
tənliyinin tam həlləri yoxdur.

615. Tənliyi həll edin: tg [ x ] ⋅ tg { x} =


1.

616. Tənliyi həll edin: x3 − [ x ] =


3.

617. Tənliklər sistemini həll edin:


 x 2 =y 3 − 3 y 2 + 2 y,
 2
 y =x − 3 x + 2 x
3 2

618. x 4 − 10 x 2 + a =0 tənliyinin kökləri ədədi silsilə əmələ


gətirirsə, a -nı tapın.

619. y = x 2 parabolası ilə ( x − 3) + ( y − 3) =


2 2
1 çevrəsinin
nöqtələri arasındakı ən qısa məsafəni tapın.

620. İsbat edin ki,


sin 2 α ⋅ cos 2 β + sin 2 β ⋅ cos 2 γ + sin 2 γ ⋅ cos 2 α ≤ 1 .

π
621. 0 ≤ α ≤ olduqda cos ( sin α ) > sin ( cos α ) bərabər-
2
sizliyini isbat edin.

622. Tənliyi həll edin:


sin 2 x + sin 2 y = 2sin x ⋅ sin y + sin x + sin y − 1 .

70
m və sin (α − β ) =
623. İsbat edin ki, sin (α + β ) = n
olduqda 1 − cos 2α ⋅ cos 2 β =
m 2 + n 2 doğrudur.

3x − 1
3
624. Tənliyi həll edin: ∫ 3x += dx log 3 ( 2 x − 7 ) + x − 6 .
−3
1
1
1
625. İnteqralı hesablayın: ∫ (x
0
2
+ 5 x + 6) 2
dx.

ln 2
626. İnteqralı hesablayın: ∫0
e x − 1 dx .

1 e
627. İnteqralı hesablayın: ∫ e dx + ∫ ln xdx .
2
x

0 1

∫ ln ( x + )
3
628. İnteqralı hesablayın: 1 + x 2 dx.
−3

xb − x a
1
629. İnteqralı hesablayın: ∫0 ln x dx ( a, b > 0 ) .

sin 5 x  π
630. f ( x ) = funksiyasının  0,  aralığındakı
 2
2
cos x
ibtidai funksiyalarından birini tapın.
1
x
631. Bərabərsizliyi isbat edin: ∫ cos xdx < l n 2.
0

71
632. İsbat edin ki, x -in həqiqi qiymətlərində
f ( x ) = a0 + a1 cos x + a2 cos 2 x + ... + an cos nx
funksiyası müsbət olduqda a0 > 0 − dır.

1
633. Bərabərsizliyi isbat edin: ∫0
xe x dx < e − 1 .

634. Əgər sin ( x + y ) − cos ( x − y ) + 2 =0 olarsa,


sin ( x + y ) + cos ( x − y ) =
0 olduğunu isbat edin.

635. Üçbucağın daxili bucaqlarının sinusları rasional


ədəddirsə, isbat edin ki, həmin bucaqların kosinusları da
rasionaldır.

636. Tərəfləri ədədi silsilə əmələ gətirən düzbucaqlı


üçbucağın iti bucaqlarını tapın.

637. Tərəfləri həndəsi silsilə əmələ gətirən düzbucaqlı


üçbucağın iti bucaqlarını tapın.

638. Üçbucaqda cos 2 A + cos 2 B + cos 2 C =


1 ödəndikdə, isbat
edin ki, həmin üçbucaq düzbucaqlı üçbucaqdır.

639. İsbat edin ki,


3
sin x ⋅ cos y + sin y ⋅ cos z + sin z ⋅ cos x ≤ .
2

27
640. İsbat edin ki, sin 2 x ⋅ cos 6 x ≤ .
256

72
641. Bərabərsizliyi həll edin:
tg 2 x1 + ctg 2 x1 + tg 2 x2 + ctg 2 x2 + ... + tg 2 xn + ctg 2 xn ≤ 2n ,
( n ∈ N ).
642. Tənliklər sistemini həll edin:
 2x + y
 x − x2 − y 2 =
1,


y + x + 2y = −1
 x2 − y 2

xy yz zx
643. Tənliyin tam həllərini tapın: + + = 3.
z x y

644. Hansı ədəd böyükdür: 99! yoxsa, 5099 ?

645. İsbat edin ki, 2 − 2 + 2 + 2 + 2 + 2 < 2 2.

646. Tənliyi həll edin: cos 4 x + sin10 x =


1.

647. İsbat edin ki,=


sin x 2sin 490 ⋅ cos 90 ola bilməz.

648. Tənliyi həll edin: x 2 + 2 x ⋅ sin( xy ) + 1 =0.

649. Tənliyi həll edin: sin 5 x + cos5 x =


2 − sin 4 x.

x2 + 1
650. Tənliyi həll edin: = sin ( xy ) .
2x

73
651. Tənliyi həll edin: sin 4 x + cos 4 y +=
2 4sin x ⋅ cos y .

652. Tənliyi həll edin: tg 10 x + ctg 10 x =


2.

653. İstənilən x, y ∈ N üçün f (1) = 1 şərtini və


f ( x + y=
) f ( x ) + f ( y ) + xy tənliyini ödəyən bütün
f : N → N ardıcıllıqlarını tapın.

sin 2 A + sin 2 B + sin 2 C


654. İsbat edin ki, üçbucaqda =2
cos 2 A + cos B + cos 2 C
münasibəti ödəndikdə həmin üçbucaq düzbucaqlı
üçbucaqdır.

655. Tənliyi həll edin:


1
cos x ⋅ cox 2 x ⋅ cos 4 x...cos 2n x = .
2n +1

656. Tənliyi həll edin: tgx + tg 2 x + tg 3 x =


0.

657. İsbat edin ki, 0 < α < π , 0 < β < π olduqda

ctgα + ctg β α +β
≥ ctg .
2 2

(
658. Tənliyi həll edin: log 2 1 + x = )
log 3 x.

659. Tənliyi həll edin: 2 log 3 ctgx = log 2 cos x .

660. Tənliyi həll edin: 3x + 4 x + 5 x =


6x .

74
661. Tənliklər sistemini həll edin:
 x3 + y 3 − z 3 − xyz + 11 =
0,
 3
 x − y + z − xyz − 21 =
3 3
0,
− x3 + y 3 + z 3 − xyz − 3 =0.

662. Tənliyi həll edin: 6 x3 − x 2 − 20 x + 12 =


0.

663. Tənliyi həll edin: x9 − 2002 x 3 + 2001 =


0.

664. Tənliyi həll edin:

a 2 x − b 2 − c 2 + b 2 x − a 2 − c 2 + c 2 x − a 2 − b 2 = a 3 + b3 + c 3 ,

burada a > 0, b > 0, c > 0.

2 2
 x   x 
665. Tənliyi həll edin:   +  =
2.
 x−2  x+2

666. Tənliyi həll edin:


 1  3 1   2001 1 
 x +  +  x + 3  + ... +  x + 2001  =2002 .
 x  x   x 

667. Tənliyi həll edin: x 4 − 8 x 3 + 17 x 2 − 8 x + 1 =0.

668. Tənliyi həll edin: x3 − ( )


2 + 1 x 2 + 2 =.
0

(x − 6x − 9) = x ( x2 − 4x − 9) .
2 2
669. Tənliyi həll edin:

75
670. Bütün ədəd oxunda təyin olunmuş və hər bir x ∈ R üçün
1+ f ( x)
f (x + π ) = bərabərliyini ödəyən f funksiya-
1− f ( x)
sının dövri olduğunu göstərin.

76
HƏLLƏR

1. 245=25⋅9=329, 336=34⋅9=819,
427=43⋅9=649, 518=52⋅9=259
Bunları müqayisə etsək: 518 < 245 < 427 < 336 alarıq.

2. Aydındır ki, 2100 + 3100 < 3100 + 3100 = 2 ·3100 .


Digər tərəfdən
3 97 97 97
2 · 3100 3 3 27  3  27  3 
= 2·   ·   = 2· ·  = ·   < 1.
4 100
4 4 64  4  32  4 
Buradan 2 ·3100 < 4100 . Deməli, 2100 + 3100 < 4100 doğrudur.

4⋅250 3⋅250
3. 2=
1000
2= 16250 və =
3750 3= =
27 250
27 250 , burada
16250 < 27 250 olduğundan 21000 < 3750 olur.

4. 3303 = 3 ⋅ 32⋅151 =
3 ⋅ 3302 = 3 ⋅ 9151 ,
2454 =⋅ 2 23⋅151 =⋅
2 2453 =⋅ 2 8151 , burada
3 ⋅ 9151 > 2 ⋅ 8151 olduğundan 3303 > 2454 olur.

127 23 < 12823 = ( 27 ) = 2161 


23

 ⇒ 2 < 2 olduğundan
161 162
5.
51318 > 51218 = ( 29 ) = 2162 
18

127 23 < 51318 .

255, 
3111 < 3211 =
6. 56 
⇒ 3111 < 255 < 256 < 1714 ⇒ 311 < 1714 .
17 > 16 =
14 14
2 

77
7. 128 > 125 və ya 27 > 53 ; 1995 =
7 ⋅ 285 nəzərə alsaq:
21995 =27⋅285 > 53⋅285 =5855 > 5854 .
Deməli, 21995 > 5854 .

21995
8. 21995 > 5856 ⇔ > 5 olduğunu isbat edək:
5855
285
21995  27 
285 285
 128   3 
=  =  = 1 +  >
5855  53   125   125 
3 3
> 1 + 285 ⋅ > 1 + 250 ⋅ = 7 > 5.
125 125
21995
Deməli, > 5 ⇔ 21995 > 2856 .
5855
Qeyd: Burada Bernulli bərabərsizliyindən istifadə edilmişdir:
(1 + x )n > 1 + n x, (x > 0, n > 0).

9. 1) 29 və 38 -i müqayisə edək.
38 = 94 > 84 = 212 > 29 ⇒ 38 > 29 ;
2) 38 və 47 -ni müqayisə edək:
7 7
 4  1 1 10 4
  = 1 +  > 1 + 7 ⋅ = >3⇒  >
 3  3 3 3 3
>3⇔ 4 >3 ;
7 8

3) 47 və 56 -nı müqayisə edək:


6 2 2 6
 5   125   128  5
  =   <  ==
22 4 ⇒   <
 4   64   64  4
<4⇔4 >57 6

78
Aldıqlarımızı nəzərə alsaq: 47 > 38 > 29 və 47 > 56 .
Deməli, verilmiş ədədlərin ən böyüyü 47 -dir.

10. Qeyd edək ki, 53 + 63 =341 < 343 = 73, buradan


3 3
5 6
  +   <1,
7 7
10 3 10 3
5 5 6 6
  <   və   <   olduğundan
7 7 7 7
10 10
5 6
  +   <1⇒ 5 + 6 < 7 .
10 10 10

7 7

11. Gücləndirmə üsulu tətbiq etsək:

4825 < 4925 < ( 7 2 ) = 750 < 751 = ( 73 ) = 34317 < 34417 .
25 17

Deməli, 4825 < 34417.

( ) ( ) ( )
2 2 2
12. Aydındır ki, 3
5−33 + 3
3−3 2 + 3
2−35 > 0,

onda mötərizələri açsaq:


3
22 + 3 32 + 3 52 > 3 2 ⋅ 3 + 3 2 ⋅ 5 + 3 3 ⋅ 5 və buradan
3
4 − 3 10 + 3 25 > 3 6 − 3 9 + 3 15 alarıq.

13. 23 < 32 ⇔ (23 ) 22 < (32 ) 22 ⇒ 266 < 344.


Gücləndirmə tətbiq etsək: 265 < 347 və 265 ⋅ 365 < 347 ⋅ 365 ,
buradan 665 < 3112 ⇔ 665 < 956 .

79
14. Verilmiş ədədlərin nisbətini tapaq:

29200 ⋅ 2151
=
( (29= ⋅ 2) ) ⋅ 5
2 100 21
3364100 ⋅1257
=
5279 ⋅ 3300 ( (5 ⋅ 3) ) ⋅ 2
3 100 49 3375100 ⋅1287
.
100 7
 3364   125 
=   ⋅  < 1.
 3375   128 

Deməli, 29200 ⋅ 2151 < 5279 ⋅ 3300 .

ab ≤ abc,

15. + bc ≤ abc, ⇒ ab + bc + ac ≤ 3abc.
ac ≤ abc.

16. Üçrəqəmli ədəd xyz olsun. Onda şərtə görə

100 x + 10 y + z 100 x + 100 y + 100 z − 90 y − 99 z


= =
x+ y+z x+ y+z
90 y + 99 z
=100 − ≤ 100.
x+ y+z

Burada y = 0 və z = 0 olduqda çıxılanın ən kiçik qiyməti

0 olur, onda fərqin ən böyük qiyməti 100 olar.


Deməli, həmin nisbətin ən böyük qiyməti 100 -dür.

200!
17. nisbətini aşağıdakı kimi qiymətləndirək:
100200

80
 1 199   2 198   3 197   99 101 
 ⋅ ⋅ ⋅ ⋅ ⋅  ⋅ ⋅  ⋅ ×
 100 100   100 100   100 100   100 100 
100 200  100 − 99 100 + 99   100 − 98 100 + 98 
× ⋅ = ⋅ ⋅ ⋅ ×
100 100  100 100   100 100 
 100 − 97 100 + 97   100 − 1 100 + 1 
× ⋅  ⋅ ⋅  ⋅  ⋅1 ⋅ 2 =
 100 100   100 100 
1002 − 992 1002 − 992 1002 − 97 1002 − 12
= ⋅ ⋅ ⋅  ⋅ ⋅1 ⋅ 2 < 1
1002 1002 1002 1002
Belə ki, buradakı bütün kəsrlər 1-dən kiçikdir. “2” vuruğunu
isə I kəsrə vursaq: yəni,
1002 − 992 1 199 199
2
⋅ 2= ⋅ ⋅ 2= < 1 alırıq.
100 100 100 5000
Deməli, 200! < 100200.

18. Tutaq ki, a 2 = 1999 , onda =


a 2k − 1 və 4-ə bölünmə
əlamətinə görə a = 2
4m + 3, (m, k ∈ N ) .
Onda
(2k − 1) 2 = 4m + 3,
4k 2 − 4k + 1= 4m + 3,
4(k 2 − k − m) =2,
2(k 2 − k − m) =
1
Burada sol tərəf cüt, sağ tərəf isə tək ədəd olduğundan
ziddiyyət alırıq. Deməli, kvadratı 1999-la qurtaran natural
ədəd yoxdur.

19. I üsul: Tutaq ki, a 2 = 111


 
11 , onda
2005

81
a 2k − 1 (tək ədəd) və 4 -ə bölünmə əlamətinə görə
=
a2 = 4m + 3, (m, k ∈ N ) , onda
(2k − 1) 2 = 4m + 3,
4k 2 − 4k + 1= 4m + 3,
4(k 2 − k − m) =2
2(k 2 − k − m) =
1
Burada sol tərəf cüt, sağ tərəf isə tək olduğundan
ziddiyyət aldıq. Deməli, ola bilməz.

II üsul: Tutaq ki, 111


11 = a 2 , onda a ədədinin təklik

2005

rəqəmi ya 1, ya da 9-dur.
I halda= a 10k + 1 , II halda isə=
a 10n + 9 .
Əgər=
a 10k + 1 , onda =
a 2 100k 2 + 20k=
+ 1 111
 
11 ,
2005

buradan 100k + 20k =


2
1110 və 10k + 2k =
 11
 1, 2

2004 2004

bu isə ola bilməz (bir tərəf cüt, o biri tərəf isə təkdir).
Əgər=
a 10n + 9, onda a=
2
100n 2 + 180n + 81
= 1111

2005

buradan 10n + 18n + 8 =


2
11
1, bu da ola bilməz, çünki
2004

sol tərəf cüt, sağ tərəf isə təkdir.

20. Sadə ədədlər: a1 , a2 , a3 , a4 , a5 və onların hasili


A = a1 ⋅ a2 ⋅ a3 ⋅ a4 ⋅ a5 olsun. Onda A -nın müsbət
bölənlərinin sayı: (1+1) ⋅ (1+1) ⋅ (1+1) ⋅ (1+1) = 25 = 32 olar.

82
21. Tutaq ki, ola bilər: x dənə 1, x dənə 2, x dənə 3, ... ,
x dənə 9 olsun. Onda həmin ədədin rəqəmləri cəmi:
x + 2 x + 3 x + . . . + 9 x = 45 x  9,
lakin 1999n ədədi 9 -a bölünmür. Deməli, ola bilməz.

22. 2003 = n qəbul edək. Onda


n 2 + (n − 2)(n − 1) ⋅ (n + 1)(n + 2) = n 2 + (n 2 − 1) ⋅ (n 2 − 4) =
= n 2 + n 4 − 5n 2 + 4 = n 4 − 4n 2 + 4 = (n 2 − 2) 2
alırıq . Buradan aydındır ki, verilən ifadə (20032 − 2) 2
ədədinə bərabərdir.

23. Tutaq ki, I addımda k sayda vərəqi böldülər və


9 - k sayda bütöv vərəq qaldı. Onda
9k + (9 – k) = 8k + 9 = 8(k + 1) + 1 = 8p1 + 1
vərəq alınar.
II addımda m vərəq bölsək: 8k + 9 – m vərəq qalmış olar və
9m + (8k + 9 – m) = 8 (k + m+ 1) + 1 = 8p2 + 1
hər addımdan sonra vərəqlərin sayı 8p + 1 olacaqdır.
1001 = 8 ⋅ 125 + 1 olduğundan verilmiş şərt daxilində
1001 sayda vərəq alına bilər.

24. Əgər ədədin kvadratı 5-lə qurtarırsa, onda həmin ədəd


5 -ə bölünür. 5 -ə bölünən ədədin kvadratı 25 -ə bölünməlidir,
lakin 2005 -lə qurtaran ədəd 25 -ə bölünmür.
Deməli, natural ədədin kvadratı 2005 -lə qurtara bilməz.

25. Verilmiş ifadəni aşağıdakı kimi çevirək:

83
7 n ⋅ 23n − 32 n = 7 n (23 ) n − (32 ) n = 7 n ⋅ 8n − 9n = 56n − 9n =
= (56 − 9) ⋅ (56n −1 + 56n − 2 ⋅ 9 +  + 56 ⋅ 9n − 2 + 9n −1 )=
=47 ⋅ A, ( A ∈ N ).
Deməli, verilmiş ifadə 47 -yə bölünür.

26. n > 1 olduqda 5n + 1 ədədi 26 ilə qurtarır, n = 1 olduqda


isə 51 + 1 =6 . Deməli, 5n + 1 ədədi 4-ə bölünmür.
k > 1 olduqda 5k − 1 ədədi 24 -lə qurtarır; k = 1 olduqda
isə 51 − 1 =4 . Deməli, 5k − 1 ədədi 4-ə bölünür.
Beləliklə, aldıq ki, 5n + 1 şəklində əlan ədəd 5k − 1
şəklində olan ədədə bölünə bilməz.

27. 7 n + 2 + 82 n +=
1
7 n + 2 + 82 n +1 + 7 n ⋅ 8 − 7 n ⋅ =
8
= 7 n (7 2 + 8) + 8(82 n − 7 n ) = 7 n ⋅ 57 + 8(64n − 7 n ) .
Burada 64n − 7 n fərqi 64 – 7 = 57 -yə bölünür, onda
7 n + 2 + 82 n +1 = 57 ⋅ 7 n + 57 k = 57 ⋅ (7 n + k ) 57, (k ∈ Z ) .

28. n(n 2 + 5)
= n(n 2 − 1 + 6)
= n(n 2 − 1) + 6=
n
= (n − 1) ⋅ n(n + 1) + 6n.
Üç ardıcıl tam ədədin hasili 6-ya bölündüyündən
(n − 1)n(n +=
1) 6k , (k ∈ Z ), onda
n(n 2 + 5) = 6k + 6n = 6(k + n) 6 .

29. 2233 + 3322 = 1133 ⋅ 233 + 1122 ⋅ 322 = 1122 (1111 ⋅ (23 )11 +
+(32 )11 ) = 112 ⋅ (8811 + 911 ) = 112 ⋅ (88 + 9) ⋅ A =
= 121 ⋅ 97 ⋅ A 97
84
Deməli, 2233 + 3322 cəmi 97 -yə bölünür.

30. Şərtə görə


= abc 37 m, (m ∈ N ). Onda
abc + bca + cab= 111(a + b += c)
= 37 ⋅ 3(a + b + c) = 37 n, (n ∈ N ).
Buradan bca + cab = 37n − abc = 37n − 37m = 37(n − m) 37.
Aydındır ki, bca + cab cəmi 37-yə bölünür.

31. 12 + 23 +  + 993 = (13 + 993 ) + ( 23 + 983 ) +  +

+ ( 493 + 513 ) + 50=


3
100a1 + 100a2 +  +
+100a49 + 125000
= 100(a1 + a2 +  + a49 + 1250)100

32. 13 + 23 + 33 +  93 = (13 + 93 ) + (23 + 83 ) +

+(33 + 73 ) + (43 + 63 ) + 53 =
= 10a + 10b + 10c + 10d + 125= 10(a + b + c + d + 12) +
+=
5 10 A + 5
Deməli, verilmiş cəmi 10-a böldükdə qalıqda 5 alınır.

− p və x1 ⋅ x2 =
33. Viyet teoreminə görə x1 + x2 = q , onda
x1 = p və x2 = q nəzərə alsaq:
q = −2 p;
 p + q =− p,  p = 1  p = 0,
 ⇔  p = 1 ⇔  və ya 
 pq = q, q = 0 q = −2 q = 0.


34. Verilmiş tənliyin tam həlləri x1 və x2 olsun.

85
Onda Viyet teoreminə görə x1 + x2 =
− p və x1 ⋅ x2 =p.
Buradan
x1 + x2 + x1 x2 =0 ⇔ x1 + x2 + x1 x2 + 1 =1 ⇔ ( x1 + 1) ⋅ ( x2 + 1) =1
alarıq. Şərtə görə x1 + 1 və x2 + 1 tam ədədlərdir. Onda
x +1 = 1
1)  1 ⇒ x1 = x2 = 0; bu halda p = 0;
 x2 + 1 =1

 x + 1 =−1
2)  1 ⇒ x1 =x2 =−2, bu halda isə p = 4 alırıq.
 x2 + 1 =−1

35. Axtarılan tənlik x 2 + px + q =,


0 onun kökləri isə x1 və
x2 olsun. Onda Viyet teoreminə görə
 x1 + x2 = −p
 ⇒ x1 x2 − ( x1 + x2 ) =p + q
 x1 ⋅ x2 =q
x1 x2 − x1 − x2 + 1 = p + q + 1,
( x1 − 1) ⋅ ( x2 − 1) = p + q + 1
alarıq. Şərtə görə p + q + 1 =19, onda ( x1 − 1) ⋅ ( x2 − 1) =
19
olar. 19 sadə ədəd olduğundan ( x1 − 1) ⋅ ( x2 − 1) =1 ⋅19 .
x1 < x2 qəbul etsək:
x1 − 1 =1 x2 − 1 =19
və alarıq.
x1 = 2 x2 = 20
Bunları nəzərə alsaq: p = 40 və axtarılan tənlik isə
−22, q =
x 2 − 22 x + 40 =
0 olar.

86
36. Verilmiş tənliyin kökləri x1 , x2 olsun. Onda Viyet
teoreminə görə
 x1 + x2 =− p,  p 2 =x12 + x22 + 2 x1 x2 ,
 ⇒  2 ⇒
 x1 x2= q + 1 q = x1 x2 − 2 x1 x2 + 1
2 2

⇒ p 2 + q 2 = x12 + x22 + x12 x22 + 1 ⇔ p 2 + q 2 =


=( x12 + 1)( x22 + 1)
Şərtə görə x1 , x2 ∈ N olduğundan ( x12 + 1) və (x 2
2 + 1)
vuruqlarının hər biri 2 -dən kiçik olmayan natural ədədlərdir.
Deməli, p 2 + q 2 cəmi mürəkkəb ədəddir.

37. ax 2 + bx + c = = b 2 − 4ac .
0 tənliyinin diskriminantı: D
Onda şərtə görə b 2 − 4ac = 2010 alarıq.
=
1) b 2k (k ∈ Z ) olduqda 4k 2 − 4ac =
2010 buradan
1005 . Bu bərabərliyin sol tərəfi cüt, sağ tərəfi
2(k 2 − ac) =
isə təkdir. Deməli, bu halda tam həll yoxdur.
2) b = 2k − 1(k ∈ Z ) olduqda 4k 2 − 4k + 1 − 4ac =2010,
4(k 2 − k − ac) =2001, burada sol tərəf cüt, sağ tərəf isə
təkdir. Bu halda da tam həll yoxdur. Deməli, tam əmsallı
kvadrat tənliyin diskriminantı 2010 ola bilməz.

38. Üç hala baxaq: 1) x və y hər biri cüt ədəd olduqda


x 2 + y 2 = (2n) 2 + (2k ) 2 = 4(n 2 + k 2 ) cüt ədəddir;
2) x və y hər biri tək ədəd olduqda
x 2 + y 2 = (2n + 1) 2 + (2k + 1) 2 = 4n 2 + 4k 2 + 4n + 4k + 2

87
cəmi də cüt ədəddir;
3) x və y -in biri tək, o biri cüt ədəd olduqda verilmiş
tənlikdən
(2n) 2 + (2k + 1) 2 =
1999,
4n 2 + 4k 2 + 4k + 1 =1999,
2n 2 + 2k 2 + 2k =
999 alariq
Burada sol tərəf cüt, sağ tərəf isə təkdir. Deməli, verilmiş
tənliyin tam həlli yoxdur.

39. Tənliyin hər tərəfinə 1 əlavə etsək:


x + xy + y + 1 =101,
( x + 1) ⋅ ( y + 1) =
101 alarıq.
101 sadə ədəddir. 101 = 1⋅ 101 və ya 101 = (-1) ⋅ (-101)
olduğunu nəzərə alsaq, aşağıdakı sistemləri alarıq:
=  x + 1 1, =  x 0,
1)  ⇔
=  y + 1 101,
=  y 100;
=  x + 1 101,
=  x 100,
2)  ⇔
=  y + 1 1, =  y 0;
 x + 1 =−1  x =−2,
3)  ⇔
 y + 1 =−101,  y =−102;
 x + 1 =−101,  x =−102,
4)  ⇔
 y + 1 =−1,  y =−2.
40. Tənliyin hər iki tərəfini 2-yə vurub, hər tərəfə 1 əlavə
edək: 4 xy + 2 x + 2 y =1 =1999,
(2 x + 1)(2 y + 1) =
1999 olar.

88
1999 sadə ədəd olduğundan aşağıdakı sistemləri alarıq:
= 2 x + 1 1 =  x 0,
1)  ⇔
= 2 y + 1 1999
=  y 999;
= 2 x + 1 1999,
=  x 999,
2)  ⇔
= 2 y + 1 1, =  y 0;
2 x + 1 =−1,  x =−1,
3)  ⇔
2 y + 1 =−1999,  y =−1000;
2 x + 1 =−1999,  x =−1000,
4)  ⇔
2 y + 1 =−1,  x =−1.

98n 1999k − 19m şəklində yazaq.


41. Verilən bərabərliyi=
Burada 199k və 19m ədədlərini 3-ə böldükdə
hər iki halda qalıqda 1 alınır, ona görə də 1999k − 19m
fərqi 3 -ə bölünür. Lakin 98n ədədi 3-ə bölünmür.
Deməli, verilmiş şərti ödəyən natural ədədlər yoxdur.

42. Verilmiş bərabərliyin sağ tərəfi tək ədəd olduğundan sol


tərəfdəki toplananlardan biri tək ədədə bərabər olmalıdır.
2 z = 1 olsun, onda z = 0 və 2 x + 2 y + 1 =97, buradan
2x + 2 y =
96.
Hər tərəfi 25 -ə bölsək: 2 x −5 + 2 y −5 =
3.
Eyni qayda ilə 2 y−5 = 1 , buradan y − 5 = 0 ⇔ y = 5 .
Onda 2 x −5 + 1 = 3 ⇔ 2 x −5 = 2 ⇔ x − 5 = 1 ⇔ x = 6 .
Cavab: 6; 5; 0.

89
x+ y+z 1
43. Verilmiş tənliyi = şəklində yazaq. Buradan
xyz 5
xyz= 5( x + y + z ) . Aydındır ki, xyz 5, onda x, y, z sadə
ədədlər olduğundan x = 5 və ya y = 5 və ya z = 5 .
Tutaq ki, z = 5, onda xy ⋅ 5= 5( x + y + 5),
xy = x + y + 5,
xy − x − y =5,
xy − x − y + 1 =6
( x − 1) ⋅ ( y − 1) =6 alarıq.
 x − 1 =2, x −1 = 1,
Buradan  və ya 
 y − 1 =3,  y − 1 =6;
 x = 3,  x = 2,
 (bu şərti ödəmir) 
y = 4  y = 7.
Deməli, verilmiş tənliyi ödəyən sadə ədədlər 2; 5 və 7-dir.

44. Verilmiş tənliyi ( x − y )( x + y ) =


2002 şəklində yazaq.
2002 = 2⋅7 ⋅11⋅13 olduğundan bu tənliyin sol tərəfində
vuruqlardan biri cüt, o biri isə təkdir.
x − y = 2k ,
Tutaq ki,  Onda buradan yalnız
 x + y = 2m − 1, k , m ∈ N
kəsr həllər alınar. Deməli, verilmiş tənliyin natural həlləri
yoxdur.

45. Aydındır ki, xyz hasili 1997-yə bölünür.


Onda x, y, z ədədləri sadə olduğundan

90
x = 1997 və ya y = 1997 və ya z = 1997 olar.
Tutaq ki, z = 1997, onda verilmiş bərabərlikdən
xy ⋅1997
= 1997 ⋅ ( x + y + 1997),
xy = x + y + 1997,
xy − x − y + 1 =1998,
1998 alarıq.
( x − 1) ⋅ ( y − 1) =
1998= 2 ⋅ 999 olduğundan axırıncı tənlikdə vuruqlardan
biri cüt, o biri isə tək ədəddir.
Tutaq ki, x − 1= 2m − 1 ⇒ x= 2m.
Yeganə cüt sadə ədəd 2 olduğundan x = 2 alarıq.
Onda y − 1 = 1998 və y = 1999 (sadədir).
Deməli, verilmiş şərti ödəyən ədədlər 2; 1997 və 1999 -dur.
x, y, z -in yerini dəyişdikdə bərabərlik dəyişmədiyindən
aşağıdakı 6 həllər üçlüyünü alarıq: (2;1997;1999), (2;1999;
1997), (1997; 2; 1999), (1997; 1999;2), (1999; 1997; 2),
(1999; 2; 1997).

46. x və z müxtəlif cütlüyə malik olduğundan bunların


kiçiyi, yəni x = 2 -dir. Əgər y təkdirsə, onda 2 y +1 cəmi
3-ə bölünür. Lakin 23 + 1 = 9 > 3 . Deməli, y cüt sadə
ədəddir, onda y = 2 və z = 5 olur.
Cavab: {( 2; 2;5)} .
47. Tutaq ki, verilən tənliyin natural həlli var. Onda
19 ⋅ 99 − 19 x =
99 y,
19(99 − x) =
99 y
91
19 və 99 qarşılıqlı sadə olduğundan y 19 ⇔ y =
19n .
=
Oxşar qayda ilə, x 99m, (m, n ∈ N ); Bunları verilmiş
tənlikdə nəzərə alsaq:
19 ⋅ 99m + 99 ⋅19n = 19 ⋅ 99, buradan m + n =1.
Bu bərabərliyi isə ödəyən natural m və n ədədləri yoxdur.
Deməli, verilmiş tənliyin natural həlli yoxdur.

48. Aydındır ki, x ≠ 0, y ≠ 0 . Onda verilmiş tənliyi kəsrdən


= px + py,
azad etsək: xy xy − px − py + p 2 =
p2 ,
p 2 alarıq.
( x − p )( y − p ) =
p sadə ədəd olduğundan p 2 ədədinin 6 tam böləni var:
± 1; ±p; ±p2 . Bunları nəzərə alsaq, verilmiş tənliyin
( p + 1; p + p 2 ), ( p − 1; p − p 2 ), (2 p; 2 p ),
( p + p 2 ; p + 1), ( p − p 2 ; p − 1)
kimi 5 tam həlli alınar.

49. Verilmiş tənliyi y 3 − x 3 =


113 şəklində yazaq. Buradan
( y − x)( y 2 + yx + x 2 ) =
113 ,
113 alarıq.
( y − x)(( y − x) 2 + 3 xy ) =
11 ədədi sadə olduğundan 113 ədədinin bir böləni ( y − x) -dir.
Onda ( y − x) -in mümkün qiymətləri 113 , 112 , 11 və 1 olar.
I hal: y − x =
113 olduqda, ( y − x) 2 + 3 xy =
1 bərabərliyi
mümkün deyil, çünki ( y − x) 2 + 3 xy > ( y − x) 2 > 1 -dir.
112 olduqda ( y − x) 2 + 3 xy =
II hal: y − x = 11 , bu

92
mümkün deyil, çünki ( y − x) 2 + 3 xy > ( y − x) 2 > 11 .
III hal: y − x =
11 olduqda
( y − x) 2 + 3xy =112 ⇒ 112 + 3xy =112 ⇒ 3xy = 0,
bu da mümkün deyil, çünki x və y müsbət tam ədədlərdir.
IV hal: y − x =
1 olduqda
( y − x) 2 + 3 xy =113 ⇒ 1 + 3 xy =113 ⇒ 3 xy =1330 .
Bu da mümkün deyil, çünki 1330 ədədi 3 -ə bölünmür.
Deməli, verilmiş tənliyin tam həlləri yoxdur.

50. Şərtə görə, xy ⋅ x =


111z; xy ⋅ x = 37 ⋅ 3 z ⇒ xy  37 .
Onda xy ədədi 37, ya da 74 olmalıdır. Yoxladıqda
görünür ki, xy = 74 şərti ödəmir, çünki 74 ⋅ 7= 518 ≠ 111z .
xy = 37 şərti ödəyir, çünki, 37 ⋅ 3 = 111 ⋅ 1.
=
Deməli, x 3,=y 7 və z = 1 olur.

51. =
x 2 + 3 olsun. Onda x 2 = 5 + 2 6,

( )
2
x2 − 5 =2 6 , ( x 2 − 5) 2 =2 6 ,

x 4 − 10 x 2 + 25 =
24,
x 4 − 10 x 2 + 1 =0

52. Verilmiş tənliyi aşağıdakı kimi çevirək:


Sadə vuruqlara ayırsaq: 2004 = 2 ⋅ 2 ⋅ 3 ⋅ 167. Onda
p −=
q 2, p −= q 4, p −= q 12 ola bilər. Yoxlama aparsaq,
görərik ki, bu halların heç birində natural həllər alınmır.

93
53. Vuruqlara ayırsaq:

( x + 1)( x + 2)( x + 3)( x + 4) + ( x 2 + 5 x − 6) =


0 olur. Buradan

(( x + 1)( x + 4)) ⋅ (( x + 2)( x + 3)) + ( x 2 + 5 x − 6) =


0,

( x 2 + 5 x + 4)( x 2 + 5 x + 6) + ( x 2 + 5 x − 6) =
0.

t əvəz edək. Onda (t + 4)(t + 6) + (t − 6) =


x2 + 5x = 0,

0 , t1 = −9 və t2 = −2 alarıq
t 2 + 11t + 18 =
1) x 2 + 5 x + 9 =0 2) x 2 + 5 x + 2 =0;
−5 ± 17
D = 25 − 36 < 0 x1,2 = olur.
2
Bu halda həll yoxdur.

54. Tutaq ki, x = y 2 . Onda verilmiş tənlik


2 y3 2 − y 2 − 2 =
0,
2 y 3 − y − 1 =0 və ya ( y − 1)(2 y 2 + 2 y + 1) =
0
şəklinə düşər. Buradan y = 1 alarıq. Onda x = 2 olur.

55. Verilmiş tənliyi aşağıdakı kimi yazaq:


1 2
x2 + 2 = 2 , ( x ≠ 0)
x ( x − 1) + 1
1 2
x2 + ≥ 2, ≤ 2 olduğundan
x 2
( x − 1) 2 + 1
2

94
 2 1
x + 2= 2,
 ( x − 1) = x = ±1,
2 2
x 0
 ⇒  2 ⇒ ⇒x=
1.
2  − + =  x = 1
 =2 
x 2 x 1 0
 x 2 − 2 x + 2

56. 2 xy − 2 y =
x2 ,
2 y ( x − 1) =x2 ,
x2 −1 + 1 1
2y = = x +1+ .
x −1 x +1
1
2 y ∈ N olduğundan ∈ N ⇒ x −1 = 1 ⇔ x = 2 .
x −1
Onda 2 y = 2 + 1 + 1, y = 2 alarıq. Cavab: (2 ; 2).

57. Əvvəlcə tənliyin natural həllərini tapaq. Tutaq ki, x ≥ y


olmaqla, ( x, y ) cütü tənliyin həllidir. Onda x 2 ≤ 74 ≤ 2 x 2 ,
buradan 6 < x < 9. Aydındır ki, x = 7 və ya x = 8 . Lakin
tənliyi , yalnız x = 7 ödəyir. Onda y = 5 olur. Beləliklə,
tənliyin tam həlləri: (7;5), (5;7), (5;-7), (-7; 5), (7;-5), (-5; 7),
(-7; -5), (-5; -7) cütləri olur.

58. Tənliyi xz + y ( x 2 + z 2 − 1) (*) şəklində yazaq. Aydındır


ki, xz ≥ 1 , onda x 2 + z 2 ≥ 2 xz ≥ xz + 1 ⇒ x 2 + z 2 − 1 ≥ xz
Bərabərlik halı yalnız x= z= 1 olduqda mümkündür.
(*) -dan y = 1 . Onda yeganə həll:= x 1,= y 1,= z 1 olur.

59. Aşağıdakı çevirmələri aparsaq:

95
( x 2 − 6 xy + 9 y 2 ) + ( x 2 − 2 x + 1) + ( y 2 + 6 y + 9) =
0,

( x + 3 y ) 2 + ( x − 1) 2 + ( y + 3) 2 =
0.

x + 3y = 0,  x =−3 y,
 
Buradan  x − 1= 0, ⇒  x= 1,
 y + 3 =0  y =−3.
 
Lakin x = 1 və y = −3 qiymətləri x = −3 y bərabərliyini
ödəmir, yəni 1 = (-3) ⋅ (-3) = 9 alınır ki, bu isə ziddiyyətdir.
Deməli, həll yoxdur.

60. Əvvəlcə müsbət tam ədədlərdən ibarət üçlüklərin sayını


tapaq. Həmin üçlüklər 12-nin vuruqlarından ibarətdir.
Yerdəyişmələri də nəzərə alsaq, müsbət həllər 18 üçlükdən
ibarət olacaqdır.
x 12 1 1 6 6 2 2 1 1 4 4 3 3 1 1 3 2 2
y 1 12 1 2 1 6 1 6 2 3 1 4 1 4 3 2 3 2
z 1 1 12 1 2 1 6 2 6 1 3 1 4 3 4 2 2 3
Digər tərəfdən mənfi tam ədədləri də nəzərə alsaq. Müsbət
üçlüklər sayı cüt olduğundan, bütün tam həllər sayı:
18 ⋅ 2 ⋅ 2 ⋅ 2 =144 olar.
61. Verilmiş tənliyin hər tərəfini xy hasilinə vursaq:
y + x 2 + 1 = xy ⇒ xy − y = x 2 + 1 ⇒
x2 + 1 2
⇒y= = x +1+ .
x −1 x −1
x və y müsbət tam ədədlər olduğundan ( x − 1) yalnız 1 və
2 olar, yəni x = 2 və ya x = 3, y = 5 olur.

96
Onda ( x; y ) = {(2 : 5), (3,5)} alarıq.

2 x 2 x 2− x 2− x 22 + 2− x 2 x − 2− x
62. f ( x) = 2 = + +
x
− = + .
2 2 2 2 2 2
2 x + 2− x 2 x − 2− x
Yoxlasaq, f1 = cüt, f 2 = tək olduğunu
2 2
görərik.

63. I şərtin hər tərəflərini kvadrata yüksəltsək:


x 2 + y 2 + z 2 + 2( xy + yz + zx) =
25
II şərti burada nəzərə alsaq: x 2 + y 2 + z 2 + 2 ⋅ 3 =25,
x2 + y 2 + z 2 =
19 olur.
Onda x 2 + y 2 =19 − z 2 və x + y = 5 − z nəzərə alıb, Koşi -
Bunyakovski bərabərsizliyini tətbiq etsək:
(12 + 12 ) ⋅ ( x 2 + y 2 ) ≥ ( x + y ) 2 ,
2(19 − z 2 ) ≥ (5 − z ) 2 ,
38 − 2 z 2 ≥ 25 − 10 z + z 2 ,
3 z 2 − 10 z − 13 ≤ 0,
13 13
buradan z1 =
−1, z2 =və −1 ≤ z ≤ , onda zmax = 13 olur.
2 3 3

64. Çevirmə aparsaq:

m 4 + 4n 4 =m 4 + 4m 2 n 2 + 4n 4 − 4m 2 n 2 =(m 2 + 2n 2 ) 2 −
−(2mn) 2 = (m 2 − 2mn + 2n ) ⋅ (m 2 + 2mn + 2n 2 )
2

97
Əgər m 4 + 4n 4 sadədirsə, 1 və özündən başqa vuruğu
yoxdur. Axırıncı hasildə kiçik vuruq olduğundan
1 olmalıdır. Buradan
m 2 − 2mn + 2n 2 =
m 2 − 2mn + 2n 2 = (m 2 − 2mn + n 2 ) + n 2 = (m − n)2 + n 2 = 1 olur.
Bu isə yalnız m= n= 1 olduqda mümkündür.

65.
 
11
 1=− 22
 2 11
1111 −=
2 ⋅  11
 1 11
 100
0 +
2n n n n  n  n n

 
+ 11
 1 − 2  11
 =
1 11
 100
0 − 11
=1
n  n  n n n
2
  
2

= 11
 1 ⋅ (10 − 1)
= 11
10
1 ⋅ 99
 =
9 3 ⋅  11
 1=  33

2
3  .
n n n  n   n 

Deməli, 11
 1 − 22
 2= 33 32= 33
 3.
2n n n n

66. 1) n = 1 olduqda 16-10 ⋅ 1 – 1 =25, yəni n = 1 üçün təklif


doğrudur.
2) n = k üçün f (k ) = 16k + 10k − 1, k > 1 ifadəsinin 25-ə
bölündüyünü qəbul edib, n= k + 1 üçün təklifin doğruluğunu
isbat edək:
f (k + 1) = 16k +1 + 10(k + 1) − 1 = 16 ⋅16k + 160k − 150k + 25 −
−16 = 16(16k + 10k − 1) − 150k + 25 = 16 ⋅ f (k ) − 25(6k − 1) =
= 16 ⋅ 25 A − 25(6k − 1) = 25(16 ⋅ A − 6k − 1) 25

98
Deməli, verilmiş təklif n= k + 1 üçün də doğrudur.

67. n(n + 1)(n + 2)(n + 3)= n((n + 1)3 + (n + 1)(3n + 5)) .


Əgər verilmiş hasil tam kub olarsa,
n 2 A3 olmalıdır.
(n + 1)3 + (n + 1)(3n + 5) =
Lakin (n + 1)3 + (n + 1)(3n + 5) cəmi n -ə bölünmür.
Deməli, həmin hasil tam kub ola bilməz.
68. Hər bir toplananı aşağıdakı kimi yazaq:
b ( a + b) − a a+b a 1 1
= = − = − ;
a ( a + b) a ( a + b) a ( a + b) a ( a + b) a a + b
c 1 1
= − ;
(a + b)(a + b + c) a + b a + b + c
d 1 1
= − ,
(a + b + c)(a + b + c + d ) (a + b + c) (a + b + c + d )
1 1 1 1 1 1
− + − + − =
a a + b a + b a + b + c a + b + c ab + c + d
1 1 b+c+d
= − =
a a + b + c + d a(a + b + c + d )

69. Fərqi qiymətləndirək:


3(1 + a 2 + a 4 ) − (1 + a + a 2 ) 2 =+
3 3a 2 + 3a 4 − 1 − a 2 −
−a 4 − 2a − 2a 2 − 2a 3 = (a 4 − 2a 3 + a 2 ) + (a 4 − 2a 2 + 1) +
+(a 2 − 2a + 1) = (a 2 − a ) 2 + (a 2 − 1) 2 + (a − 1) 2 ≥ 0
a -nın istənilən qiymətində bu toplananların heç biri mənfi
deyildir. Deməli, 3(1 + a 2 + a 4 ) ≥ (1 + a + a 2 ) 2 doğrudur.

99
70. Bərabərsizliyin sol tərəfini çevirək:

(a + b + c)3 − a 3 − b3 − c3 = (a + b)3 + 3(a + b) 2 ⋅ c +


+3(a + b) ⋅ c 2 + c3 − a 3 − b3 = (a + b)3 + 3(a + b) 2 ⋅ c +
+3(a + b) ⋅ c 2 − (a 3 + b3 ) = (a + b)(a 2 + 2ab + b 2 +
+3ac + 3bc + 3c 2 − a 2 + ab − b 2 ) =
=(a + b)(3ab + 3ac + 3bc + 3c 2 ) =
=3(a + b)(b + c)(a + c) > (a + b)(b + c)(a + c).
Buradan (a + b + c)3 − (a 3 + b3 + c3 ) > (a + b)(b + c)(a + c)
alınır.

71. Bərabərsizliyin sol tərəfini aşağıdakı kimi çevirək:


a 2 + a + 1 b2 + b + 1 c2 + c + 1 d 2 + d + 1
⋅ ⋅ ⋅ =
a b c d
 1  1  1  1
=  a +1+  ⋅ b +1+  ⋅ c +1+  ⋅ d +1+ 
 a  b  c  d
İki müsbət qarşılıqlı tərs ədədin cəmi 2 -dən kiçik
1 1 1 1
olmadığından a + ≥ 2, b + ≥ 2, c + ≥ 2, d + ≥ 2
a b c d
bu bərabərsizlikləri axırıncı ifadədə nəzərə alsaq:
a 2 + a + 1 b2 + a + 1 c2 + c + 1 d 2 + d + 1
⋅ ⋅ ⋅ ≥
a b c d
≥ (2 + 1) ⋅ (2 + 1) ⋅ (2 + 1) ⋅ (2 + 1) =
81 olur.
Bununla da isbat tamam oldu.

72. Şərtə görə acb


= 100a + 10b +=
c 27 k , (k ∈ N ) . Onda

100
bca = 100b + 10c + a = 10(100a + 10b + c) − 999a = 10 ⋅ abc −
−999a = 10 ⋅ 27 k − 999a = 27(10k − 37 a ) =27 m
= 100c + 10a + =
cab b 10(100b + 10c + a) − 999=
b
=⋅
10 bca − 999b =⋅
10 27m − 999b =
27(10m − 37b) 27.

73. Verilən cəmi


(10010 − 1) + (9910 − 210 ) +  + (5210 − 4910 ) + (5110 − 5010 ) kimi
yazıb, n -in cüt qiymətlərində a n − b n fərqinin (a + b) cəminə
bölündüyünü nəzərə alsaq, mötərizələrdəki fərqlərin hər biri
101-ə bölünür. Deməli, verilən cəm 101-ə bölünür.

74. Görünür ki,


1110 −=
1 1215 −=
1 (1215 − 215 ) + (215 − 1)
= (121 − 21) ⋅ A +
+(215 − 1)
= 100 A + (215 − 1).
Digər tərəfdən 215 − 1= (21 − 1) ⋅ (214 + 213 + 212 + 21 + 1).
214 , 213 , 212 , 21 və 1 toplananlarının hər biri 1 -lə
qurtardiğindan, onların cəmi isə 5 qurtarır. Onda
215 − 1 = 20 ⋅ 5 ⋅ B = 100 B.
Deməli, 1110 −
= 1 100 A + 100=
B 100( A + B)100.

75. Verilmiş bərabərsizliyin sol tərəfini aşağıdakı kimi


çevirək:
 1 1   1 1 
 − + −  + +
 (1 + 1)! (1 + 2)!   (2 + 1)! (2 + 2)! 
 1 1  1 1 1
+ − = − <
 (n + 1)! (n + 2)!  2 (n + 2)! 2
101
alarıq.

76. İsbat edək ki, cəm 1001-ə bölünür. 10012003 və 20022003


toplananları 1001-ə bölünür. Qalan toplananları aşağıdakı
kimi cüt-cüt qruplaşdıraq:
(1 + 2001 ) + ( 2
2003 2003 2003
+ 20002003 ) +  +
+ (1000 + 1002 )
2003 2003

Aydındır ki, hər bir mötərizə 2002-yə bölünür. Onda 1001-ə


bölünür. 1001=91⋅ 11 olduğundan verilmiş cəm 91-ə də
bölünür.

77. 81 = a olsun. Onda


(a − 1)(a 9 + a8 + a 7 +  + a 2 + a + 1) + =
1 (a10 + a 9 +
+ a8 +  + a 2 + a ) − (a 9 + a8 + a 7 +  + a 2 + a + 1) +
+1= a10= 8110= (34 )10= 340.
Verilmiş ifadə 340 –a bərabərdir.

78. ac + bd =
0 bərabərliyinin hər tərəfini ad + bc ifadəsinə
vuraq:
(ac + bd )(ad + bc) =0,
a 2 cd + abc 2 + abd 2 + b 2 cd =
0,
ab(c 2 + d 2 ) + cd (a 2 + b 2 ) =
0
1 və c 2 + d 2 =
a 2 + b2 = 1 şərtini nəzərə alsaq, ab + cd =
0
alarıq.

79. Aydındır ki, 1 + x 2 = xy + yz + zx + x 2 = ( x + y )( x + z ).

102
Analoji qayda ilə 1 + y 2 = ( y + x)( y + z ) və
1 + z 2 = ( z + x)( z + y ) .
Tərəf-tərəfə vursaq:
(1 + x 2 )(1 + y 2 )(1 + z 2 ) =( x + y ) 2 ( y + z ) 2 ( z + x) 2 alarıq.

80. Qeyd edək ki, a 3 − a = (a − 1) ⋅ a ⋅ (a + 1) . Üç ardıcıl tam


ədədin hasili 6-ya bölündüyündən a 3 − a fərqi 6-ya bölünür.
Onda (a 3 − a ) + (b3 − b) + (c 3 − c) =
6m , buradan
a 3 + b3 + c 3 = 6m + (a + b + c). Şərtə görə a + b + c =6n
olduğundan a 3 + b3 + c3 = 6m + 6n = 6(m + n) .
Deməli, a 3 + b3 + c3 cəmi 6 -ya bölünür.

81. a + b + c =0 olduğundan (a + b + c)(a 2 + b 2 + c 2 ) =.


0
Buradan a 3 + b3 + c3 + ab(a + b) + ac(a + c) + bc(b + c) =
0.
Digər tərəfdən a + b =−c, a + c =−b, b + c =−a nəzərə
0 , a 3 + b3 + c 3 =
alsaq: a 3 + b3 + c3 − abc − abc − abc = 3abc
olar.

82. Şərtə görə 3


a+3b=
− 3 c . Bu bərabərliyin hər tərəfini
kuba yüksəltsək: a + 3 3 a 2b + 3 3 ab 2 + b =−c, buradan
a + b + c =−3 3 ab ( 3
)
a + 3 b =3 3 ab ⋅ 3 c =3 3 abc və

(3 =
abc )
3
(a + =
b + c )3 3
27 abc alarıq.
Tələb olunan isbat alındı.

103
83 Məlum 2ab ≤ a 2 + b 2 bərabərsizliyindən istifadə etsək:
a 2 + b 2 + 2ab ≤ 2a 2 + 2b 2 və ya
a 2 + b 2 + 2ab a 2 + b 2  a+b a +b
2 2 2

≤ ⇒  ≤ ⇒
4 2  2  2
a+b a 2 + b2 a+b a 2 + b2
⇒ ≤ ⇒ ≤
2 2 2 2
Bərabərlik halı a= b ≥ 0 olduqda alınır.

84. İfadəni aşağıdakı kimi çevirək:


n5 + n 4 + 1 = (n5 + n 4 + n3 ) + (n 2 + n + 1) − (n3 + n 2 + n) =
= n3 (n 2 + n + 1) + (n 2 + n + 1) − n(n 2 + n +=
1)
= (n 2 + n + 1) ⋅ (n3 − n − 1).
n > 1 olduqda, vuruqların hər biri vahiddən böyük tam
ədəddir. Deməli, n5 + n 4 + 1 ədədi mürəkkəb ədəddir.
n = 1 olduqda, II vuruq vahidə bərabərdir.
Onda n5 + n 4 + 1 = 1 + 1 + 1 = 3 sadə ədəd olur.

85. Çevirmə aparsaq:


n5 − 5n3 + 4n= n(n 4 − 5n 2 + 4)= n(n 2 − 4)(n 2 − 1)=
= n(n − 2)(n + 2)(n − 1)(n + 1) =(n − 2)(n − 1)n(n + 1)(n + 2)
Aldığımız ifadə beş ardıcıl natural ədədin hasilidir. Ardıcıl
beş ədədin hasili isə 120-yə bölünür (məs: 1⋅2⋅3⋅4⋅5= 120).

86. Ədədi və həndəsi orta bərabərsizliyini tətbiq etsək:


a2 a2 b2 b2
+b ≥ 2 ⋅b =2a, +c ≥ 2 ⋅c =2b,
b b c c

104
c2 c2
+a≥2 ⋅a =2c alarıq.
a a
Bu bərabərsizlikləri tərəf-tərəfə toplasaq:
 a2   b2   c2 
 + b +
  + c  +  + a  ≥ 2(a + b + c), buradan
 b   c  a 
 a 2 b2 c2 
 + +  + (a + b + c) ≥ 2(a + b + c) ,
 b c a
a 2 b2 c2
+ + ≥ a + b + c olur.
b c a

1 2 2009  2 1   3 1 
87. + + + =  −  +  −  + +
2! 3! 2010!  2! 2!   3! 3! 
 2010 1   1 1 1
+ −  = 1 −  +  −  +  +
 2010! 2010!   2!   2! 3! 
 1 1  1
+ − = 1− < 1.
 2009! 2010!  2010!
Tələb olunan isbatı aldıq.

88. Çevirmə aparıb, qiymətləndirmə tətbiq etsək:


1 1 1 1 1 1 1 1 1 1 1
− + − + + = −  −  −  −  − −
2 3 4 5 999 1000 2  3 4   5 6 
 1 1  1 1 1 1 1 1
− −  = − − − − < − −
 999 1000  2 12 30 999000 2 12
1 23 24 2
− = < = .
30 60 60 5

105
89. Hər bir üçhədlidə x = 13 nəzərə alsaq:
169b + 13c + =
a 169c + 13a + b,
168b − 156c − 12a =
0,
14b − 13c − a =0, buradan=
a 14b − 13c.
Onda, a + b + c= (14b − 13c) + b + c= 15b − 12c= 3(5b − 4c) 3,
lakin 65 ədədi 3-ə bölünmür. Deməli, a + b + c cəmi 65 -ə
bərabər ola bilməz.

90. Koşi bərabərsizliyini tətbiq etsək:


2
1 + 2ak =1 + ak ≥ 3 1 ⋅ ak ⋅ ak =3a
3 3
k

Onda aydındır ki,


2 2 2
(1 + 2a1 )(1 + 2a2 ) ⋅ ⋅ (1 + 2an ) ≥ 3a13 ⋅ 3a23 ⋅ ⋅ 3an3 =
2
= 3n (a1 ⋅ a2 ⋅ ⋅ an )=
3 3n.
 
1

Beləliklə, isbat tamam oldu.

91. Nisbəti qiymətləndirək:


100 100
10199 100100  101  1  1  1
= =  ⋅ =1 +  ⋅ <
100100
100 ⋅101  100 
100
101  100  101
3
< < 1.
101
α
 1
Qeyd: Burada 2 ≤ 1 +  < 3 faktından istifadə edilmişdir.
 α

106
1 x, təklif doğrudur.
92. 1) n = 1 olduqda (1 + x)1 =+
2) n = k üçün (1 + x) k ≥ 1 + kx doğru olduğunu qəbul
edib, n= k + 1 üçün təklifin doğruluğunu isbat edək. Onda
1 + x > 0 şərtini nəzərə alsaq:
(1 + x) k +1 =+
(1 x)(1 + x) k ≥ (1 + x)(1 + k x) =
1+
+ x + kx + kx 2 ≥ 1 + (k + 1) x olur.
İsbat etdik ki, Bernulli bərabərsizliyi n= k + 1 üçün də
doğrudur.

93. 1) n = 1 olduqda, S1 = 1 ⋅1! = (1 + 1)! − 1 = 2 − 1 = 1, yəni


təklif doğrudur.
2) n = k olduqda S k = (k + 1)! − 1 qəbul edək və n= k + 1
üçün doğruluğu yoxlayaq:
S k +1 = S k +(k + 1)(k + 1)! = (k + 1)! − 1 + (k + 1)(k + 1)! =
= (k + 1)! (1 + k + 1) − 1 = (k + 1)! (k + 2) − 1 = (k + 2)! − 1
Deməli, eynilik istənilən n ∈ N üçün doğrudur.

94. 1) n = 2 olduqda bərabərlik halı alınır: a 2 + b 2 =


c2
(Pifaqor teoremidir). 2) Tutaq ki, n = k üçün təklif doğrudur:
a k + bk ≤ ck .
n= k + 1 üçün təklifin doğruluğunu isbat edək:
a k +1 + b k +1 = a k a + b k b < a k c + b k c = ( a k + b k ) c ≤ c k c = c k +1
aldıq. Deməli, verilmiş təklif hər bir natural n ≥ 2 ədədi üçün
doğrudur.

107
95. 1) n = 1 olduqda sin 2 α + cos 2 α =
1 doğrudur.
2) Tutaq ki, n = k olduqda sin 2 k α + cos 2 k α ≤ 1 doğrudur.
Onda n= k + 1 üçün
sin 2 k + 2 α + cos 2 k + 2 α =sin 2 k ⋅ sin 2 α + cos 2 k α ⋅ cos 2 α ≤
≤ sin 2 k α ⋅1 + cos 2 k=
α ⋅1 sin 2 k α + cos 2 k α ≤ 1
Belə ki, sin 2 α ≤ 1 və cos 2 α ≤ 1 -dir.
Ona görə də sin 2 n α ≤ 1, ∀n ∈ N üçün doğrudur.

96. 1) n = 1 olduqda, a1 = 13 + 3 ⋅12 + 5 ⋅1 = 9 3, yəni, təklif


doğrudur.
2) n = k üçün ak = k 3 + 3k 2 + 5k = 3m, (m ∈ N ) qəbul edək
və n= k + 1 üçün təklifin doğruluğunu göstərək:
ak +1 = (k + 1)3 + 3(k + 1) 2 + 5(k + 1) = k 3 + 3k 2 + 3k +
+1 + 3k 2 + 6k + 3 + 5k + 5 = (k 3 + 3k 2 + 5k ) + 3k 2 +
+9k + 9 = 3m + 3(k 2 + 3k + 3).
Toplananların hər biri 3-ə bölündüyündən cəm də 3-ə
bölünür. İsbat tamam oldu.

97 1) n = 4 olduqda, 4! = 24 > 24 = 16 təklif doğrudur.


2) n= k > 4 üçün k ! > 2k qəbul edib, n= k + 1 üçün təklifi
isbat edək: (k + 1)! = (k + 1)k ! > (k + 1)2k > 2 ⋅ 2k = 2k +1 .
Deməli, verilmiş bərabərsizlik istənilən natural n ≥ 4 üçün
doğrudur.

98 1) n = 1 olduqda, 34 ≥ 43 , doğrudur.

108
2) n = k üçün k k +1 ≥ (k + 1) k olduğunu qəbul edək və
göstərək ki, bərabərsizlik n= k + 1 üçün də doğrudur:
(k + 1) k + 2 k +1 (k + 1) k + 2
1) k + 2
(k += ⋅k ≥ ⋅ (k=
+ 1) k
k k +1 k k +1
k +1 k +1
(k + 1) 2 k + 2  (k + 1) 2   1
= k +1
=   = k + 2+  >
k  k   k
k +1 k +1
> (k + 2) = ((k + 1) + 1) .
Deməli, verilmiş bərabərsizlik istənilən natural n ≥ 2 üçün
doğrudur.

99. n = 5 olduqda 25 > 52 və n = 6 olduqda 26 > 62


bərabərsizlikləri doğrudur.
n= k > 6 olduqda bərabərsizliyin doğruluğunu qəbul edək.
Yəni, qəbul edək ki, 2k > k 2 . Göstərək ki, n= k + 1 olduqda
da bərabərsizlik doğrudur. Yəni, 2k +1 > (k + 1) 2 olduğunu
göstərməliyik. 2k +1 =2 ⋅ 2k > 2k 2 . Digər tərəfdən k ≥ 5
olduqda 2k 2 > (k + 1) 2 . Deməli, 2k +1 > (k + 1) 2 .

100. Bərabərsizliyin hər bir tərəfini 2-yə vursaq,


2a 2 + 2b 2 + 2c 2 ≥ 2ab + 2bc + 2ac,
a 2 − 2ab + b 2 + b 2 − 2bc + c 2 + a 2 − 2ac + c 2 ≥ 0,
(a − b) 2 + (b − c) 2 + (a − c) 2 ≥ 0
Axırıncı bərabərsizliyin sol tərəfi mənfi olmayan 3 ədədin
cəmi olduğundan, həmin bərabərsizlik a, b, c -nin istənilən
qiymətində ödənir. Bərabərlik halı isə yalnız a= b= c
olduqda doğrudur.

109
101. Bərabərsizliyin sol tərəfini aşağıdakı kimi çevirək:
x 2 + y 2 ( x − y ) 2 + 2 xy 2 ⋅1  x− y 2 
= = x− y+ = 2  +  .
x− y x− y x− y  2 x− y
İki müsbət qarşılıqlı tərs ədədin cəmi 2-dən kiçik olmadığına
x− y 2  x− y 2 
görə + ≥ 2 və buradan 2  +  ≥ 2 2 .
2 x− y  2 x− y
x2 + y 2
Onda ≥ 2 2 alarıq.
x− y

102. Məlum x 2 + y 2 ≥ 2 xy bərabərsizliyinin hər tərəfinə x 2 y 2


ifadəsini əlavə etsək: 2( x 2 + y 2 ) ≥ x 2 + 2 xy + y 2 alarıq. Onda
1 olduğundan 2 ≥ ( x + y ) 2 və buradan
x2 + y 2 =
− 2 ≤ x + y ≤ 2 olur.

103. Bərabərsizliyi isbat etmək üçün


a 2b 2 + b 2 c 2 + a 2 c 2 − abc(a + b + c) ≥ 0
olduğunu göstərək. Sol tərəfi aşağıdakı kimi çevirsək:
a 2b 2 + b 2 c 2 + a 2 c 2 − abc(a + b + c) =
1 2 2 1
= (a b − 2a 2bc + a 2 c 2 ) + (a 2b 2 − 2ab 2 c + b 2 c 2 ) +
2 2
1 1 2 1
+ (b 2 c 2 − 2abc 2 + a 2 c 2= ) a (b − c) 2 + b 2 (a − c) 2 +
2 2 2
1
+ c 2 ( a − b) 2 ≥ 0
2
Bərabərlik halı yalnız a= b= c olduqda alınır.

110
104. Bərabərsizliyin sol tərəfini
a2 + a + 2 a2 + a + 1 1 1
= + = a2 + a + 1 +
a2 + a + 1 a2 + a + 1 a2 + a + 1 a2 + a + 1
şəklində yazaq. İki müsbət qarşılıqlı tərs ədədin cəmi 2-dən
1
kiçik olmadığından a 2 + a + 1 + ≥ 2 alarıq.
a2 + a + 1

1 1 1 1 1 1
105. Aydındır ki, < , 2< , , < ⋅
2 1⋅ 2 3 2 ⋅ 3
2
100 99 ⋅100
2

Bu bərabərsizlikləri tərəf-tərəfə toplasaq:


1 1 1 1 1 1
+ 2 + + < + + + =
2
2 3 100 1 ⋅ 2 2 ⋅ 3
2
99 ⋅100
1 1 1 1 1 1 99
=1− + − + + − =
1− = .
2 2 3 99 100 100 100
1 1 1 99
Buradan göründüyü kimi 2 + 2 +  + 2
< .
2 3 100 100

1 3 5 9999
106. A = ⋅ ⋅ ⋅ ⋅ işarə etsək,
2 4 6 10000
12 32 52 99992
A2 = ⋅ ⋅ ⋅  ⋅ <
22 42 62 100002
12 32 52 99992
< 2 ⋅ 2 ⋅ 2 ⋅ ⋅ =
2 −1 4 −1 6 −1 100002 − 1
12 32 52 99992 1
= ⋅ ⋅ ⋅ ⋅ =
1⋅ 3 3 ⋅ 5 5 ⋅ 7 9999 ⋅10001 10000
1 1
olar. Buradan A < < .
10001 100

111
1 1 1 1 1 1 1 1
107. + +  + > + +  + =n ⋅ =
n +1 n + 2 2n 2n 2n 2n 2n 2

108. Hər hansı ədədin sonundakı sıfırlar 2 və 5 vuruqlarının


vasitəsi ilə alındığını, (2 ⋅ 5 = 10) nəzərə alaq. 200! ədədində
2 vuruqlarının sayı, 5 vuruqlarının sayından çox olduğuna
görə sondakı sıfırların sayını tapmaq üçün 5 vuruqlarının
sayını tapmaq kifayətdir. Onda
 200   200   200 
k=  + 2 + 3 = 40 + 8 + 1= 49 alarıq.
 5   5   5 
Deməli, verilən ədəd 49 sıfırla qurtarır.

109. ƏBOB (a, b) = d olsun, onda


= a b=
1d , b b1d (belə ki,
ƏBOB (a1b1 ) = 1) və ƏKOB (a, b) = a1b1d olar. Bunları
verilmiş tənlikdə nəzərə alsaq: d + a1b1d =a1d ⋅ b1d , buradan
1 + a1b1 =
a1b1d və a1b1 (d − 1) =
1.
Bu axırıncı tənliyi natural ədədlər çoxluğunda həll etsək:
a=
1 b=
1 1 və d = 2 alarıq. Deməli, a= b= 2 olur.

110. Qeyd edək ki,


101! =1⋅ 2 ⋅ 3 ⋅ ⋅101 =(1⋅101) ⋅ (2 ⋅100) ⋅ ⋅ (50 ⋅ 52) ⋅ 51 .
Hər bir mötərizədəki hasil: (51 − k )(51 + k ) = 512 − k 2 < 512 .
Onda 101! < (512 )50 ⋅ 51 =
51101 olur.

111. Hər tərəfi 2sin1 ifadəsinə vuraq:


2S ⋅ sin1 = 2sin1⋅ cos1 + 2sin1⋅ cos 3 + 2sin1⋅ cos 5 +  +
+2sin1⋅ cos1001.

112
Hasili cəmə çevirmə düsturlarını tətbiq etsək:
2sin1 ⋅ cos1 = sin 2 − sin 0,
2sin1 ⋅ cos 3 = sin 4 − sin 2,
............................................
2sin1 ⋅ cos1001 = sin1002 − sin1000 alarıq.
Tərəf-tərəfə toplasaq: 2 S ⋅ sin1 = sin1002, buradan
sin1002
S= olur.
2sin1

112. 1000 = n olsun, onda 999 =


n − 1, 1001 =
n +1.
Tutaq ki, n − 1 + n + 1 < 2 n -dir. Hər tərəfi kvadrata
yüksəltsək: n − 1 + 2 n 2 − 1 + n + 1 < 4n, 2 n 2 − 1 < 2n ,

n 2 − 1 < n doğru bərabərsizliyini alarıq.


Deməli, 2 1000 > 999 + 1001.

1 1 1
113. 1) n = 1 üçün= = doğrudur.
1 ⋅ 3 2 ⋅1 + 1 3
1 1 1 k
2) n = k üçün + + + = qəbul
1⋅ 3 3 ⋅ 5 (2k − 1)(2k + 1) 2k + 1
edək və n= k + 1 üçün təklifin doğruluğunu göstərək.
Onda
1 1 1 1
+ + + + =
1⋅ 3 3 ⋅ 5 (2k − 1)(2k + 1) (2k + 1)(2k + 3)
k 1 2k 2 + 3k + 1
= + = =
2k + 1 (2k + 1)(2k + 3) (2k + 1)(2k + 3)

113
(2k + 1)(k + 1) k +1
= =
(2k + 1)(2k + 3) 2(k + 1) + 1

Deməli, verilmiş bərabərlik doğrudur.


 1⋅ 2 
2

114. 1) n = 1 olduqda=
1  13 1, doğrudur.
 ⇒=
3

 2 
 k (k + 1) 
2

2) n = k üçün 1 + 2 + 3 +  + k =
3 3 3

3
 qəbul edib,
 2 
n= k + 1 üçün doğruluğu isbat edək:
 k (k + 1) 
2

1 + 2 + 3 + + k +=
3 3 3
(k + 1) 
3 3
 +=
(k + 1)3
 2 
(k + 1) 2 (k 2 + 4k + 4)  (k + 1)(k + 2) 
2

=  
3  2 
Deməli, təklif doğrudur.

1 ⋅ (2 ⋅1 − 1)(2 ⋅1 + 1) 1 ⋅ 3
115. 1) n = 1 üçün =
12 = = 1, təklif
3 3
doğrudur.
k (2k − 1)(2k + 1)
2) n = k üçün 12 + 32 + 52 +  + (2k − 1) 2 =
3
qəbul edək və n= k + 1 üçün doğruluğu yoxlayaq:
k (2k − 1)(2k + 1)
12 + 32 + 52 +  + (2k − 1) 2=
+ (2k + 1) 2 +
3
k (2k − 1)(2k + 1) − 3(2k + 1)
=+(2k + 1) 2 =
3
( 2k1) ( 2k 5k + 3) (k + 1)(2k + 1)(2k + 3)
2

=
3 3
114
Deməli, təklif n= k + 1 üçün də doğrudur.

1 1 1
116. 1) n = 2 olduqda, 1 + > + =2 , yəni,
2 2 2
bərabərsizlik doğrudur.
1 1 1
2) Tutaq ki, n = k üçün + + + > k.
1 2 k
Onda, n= k + 1 üçün
1 1 1 1 1
+ + + + > k+ =
1 2 k k +1 k +1
k (k + 1) + 1 k 2 +1 k +1
= > = = k +1
k +1 k +1 k +1
alarıq. Beləliklə, isbat tamam oldu.

1⋅ 2 ⋅ 3
117. 1) n = 1 olduqda 1=
⋅2 = 2 olur.
3
Tutaq ki, n = k üçün bərabərlik doğrudur.
k (k + 1)(k + 2)
Yəni, 1 ⋅ 2 + 2 ⋅ 3 + 3 ⋅ 4 +  + k (k + 1) = .
3
İsbat edək ki, n= k + 1 üçün
1 ⋅ 2 + 2 ⋅ 3 + 3 ⋅ 4 +  + k (k + 1) + (k + 1)(k + 2) =
(k + 1)(k + 2)(k + 3) .
=
3
1 ⋅ 2 + 2 ⋅ 3 + 3 ⋅ 4 +  + k (k + 1) + (k + 1)(k + 2) =
k (k + 1)(k + 2) (k + 1)(k + 2)(k + 3)
= + (k =
+ 1)(k + 2)
3 3
Deməli, verilmiş bərabərlik n= k + 1 üçün də doğrudur.
115
101 − 99
118. 100 − 101 ⋅ 99 =
2
99 − 97
98 − 99 ⋅ 97 =
2
......................
3 −1
2 − 3 ⋅1 =
2
Bu bərabərsizlikləri toplasaq,
101 − 1
101 − 101 ⋅ 99 +  + 2 − 3 ⋅1= = 51 − 101 <
2
< 51 − 100 =41.

119. Tutaq ki b ixtiyari müsbət ədəddir. [0; b] aralığında


f ( x ) = e x funksiyasını nəzərdən keçirək. Bu funksiya
Laqranj teoreminin bütün şərtlərini ödədiyi üçün elə
c ∈ (0; b ) nöqtəsi var ki, f (b ) − f (a ) = f `(c )(b − a )
bərabərliyi, yeni eb − eo = ec (b − 0 ) , eb −1 = bec ödənir.
c > 0 olduğu üçün ec > 1. Onda eb −1 = bec bərabərliyindən
alınır ki, eb − 1 > b . Bu bərabərsizlik ixtiyari b > 0 ədədi
üçün ödənildiyindən alınır ki , e x >1+ x , x >0.

120. Bərabərsizlik dəyişənlərə nəzərən simmetrik olduğuna


ğörə, ümumiliyi pozmadan, a ≥ b ≥ c qəbul edək.
Onda a, b, c > 0 olduğundan ln a ≥ ln b ≥ ln c . Buradan
a ln a + b ln b + c ln c ≥ b ln a + c ln b + a ln c

116
və a ln a + b ln b + c ln c ≥ c ln a + a ln b + b ln c .
Bu bərabərsizlikləri tərəf-tərəfə toplasaq,
2(a ln a + b ln b + c ln c ) ≥ (b + c )ln a + (c + a )ln b + (a + b )ln c
və ya 2 ln a abb c c ≥ ln a b + cb c + a c a + b ,
(a b c )
a b c 2
≥ a b + cb c + a c a + b olur.
121. Şərtə görə log b a , log c b , log a c ədədləri müsbətdir,

onda logb a ⋅ log c b ⋅ log a c = 1 . Ədədi orta və həndəsi orta

haqqında bərabərsizliyi tətbiq etsək:

logb a log c b log a c logb a log c b log a c


+ + ≥ 3 · · =
a+b b+c c+a a+b b+c c+a

1 9 4,5
=3 ≥ = .
(a + c)(b + c)(a + c) a + b + b + c + c + a a + b + c

122. Əvvəlcə a = 4 , b = 1 ; sonra a = 1 , b = 4


nəzərə alsaq: f (1) = 1 ,
f (4) + 2 f (1) 7 + 2 ⋅ 1
f (2) = = = 3,
3 3
f (1) + 2 f (4) 1 + 2 ⋅ 7
f (3) = = = 5,
3 3
f (4 ) = 7.

Buradan f (n ) = 2n − 1, (n ∈ N ) olduğunu fərz etmək olar.


Riyazi induksiya üsulu tətbiq etməklə, aldığımız fərziyyəni
isbat edək: 1) n = 1, 2, 3, 4 üçün təklif doğrudur.

117
2) n = k üçün f (k ) = 2k − 1 olduğunu qəbul edib, n = k + 1
üçün fərziyyəmizin doğruluğunu, yəni, f (k + 1) = 2(k + 1) − 1
olduğunu göstərək. Tutaq ki, a = k − 1 , b = k + 1. Onda,
f (k − 2 ) + 2 f (k + 1) 3 f (k ) − f (k − 2)
f (k ) = ⇒ f (k + 1) = =
3 2
3(2k − 1) − (2(k − 2) − 1) 6k − 3 − 2k + 5
= = =
2 2
4k + 2
= = 2k + 1 = 2(k + 1) − 1 .
2
Deməli, fərziyyəmiz n = k + 1 üçün də doğrudur. Onda,
f (2011) = 2 ⋅ 2011 − 1 = 4021 olur.

123. Nisbəti qiymətləndirək:


1000
1001999 1  1  3
= 1 +  < < 1;
10001000
1001  1000  1001
n
 1
Burada 2< 1 +  < 3, n > 1 nəzərə alınmışdır.
 n
Deməli, 1001999 < 10001000 .

124. Qeyd edək ki, 00 < x < 900 olduqda sin x > 0 və cos x > 0 .
Koşi bərabərsizliyini tətbiq etsək:
1 1 1 1
1+ ≥2 və 1 + ≥2 .
sin x sin x cos x cos x
Bunları tərəf-tərəfə vursaq:

118
 1   1  1 4 2
1 +  ⋅ 1 + ≥4 = alarıq. Digər
 sin x   cos x  sin x ⋅ cos x sin 2 x
4 2
tərəfdən sin 2 x ≤ 1 və 4 2 > 5 olduğundan > 5 -dir.
sin 2x
Beləliklə, isbat tamam oldu.


( )
125. a = (a + b ), b = a 2 ; b 2 işarə edib,
( )
 
a , b = a ⋅ a 2 + b ⋅ b 2 = a 3 + b3 ,
 
( )
 
a = a 2 + b 2 , b = a 4 + b 4 olduğunu a , b ≤ a ⋅ b
 

x1 x2 + y1 y2 ≤ x1 + y1 ⋅ x2 + y2
2 2 2 2
və ya
bərabərsizliyində nəzərə alsaq, verilmiş bərabərsizliyi
alarıq. Bərabərlik halı a = b olduqda alınır.

126. Ədədi orta və həndəsi orta haqqında Koşi


bərabərsizliyini tətbiq etsək,
sin α + sin β + sin γ ≥ 3 ⋅ 3 sin α + sin β + sin γ olar.
Bu bərabərsizliyin hər iki tərəfini kvadrata yüksəldək:
(sin α + sin β + sin γ )2 ≥ 9 ⋅ 3 (sin α ⋅ sin β ⋅ sin γ )2 .
sin α ≤ 1, sin β ≤ 1, sin γ ≤ 1 olduğu üçün
(sin α ⋅ sin β ⋅ sin γ )2 > (sin α ⋅ sin β ⋅ sin γ )3 .
Onda (sin α + sin β + sin γ ) ≥ 9 ⋅ 3 (sin α ⋅ sin β ⋅ sin γ ) >
2 2

> 9 ⋅ 3 (sin α ⋅ sin β ⋅ sin γ ) = 9 ⋅ sin α ⋅ sin β ⋅ sin γ .


3

119
127. 4 x + 1 + 4 y + 1 + 4 z + 1 ifadəsini
(4 x + 1) ⋅ 1 + (4 y + 1) ⋅ 1 + (4 z + 1) ⋅ 1 şəklində yazıb, ədədi
orta və həndəsi orta haqqında bərabərsizliyi tətbiq etsək:
(4 x + 1) ⋅ 1 + (4 y + 1) ⋅ 1 + (4 z + 1) ⋅ 1 <
<
(4 x + 1) + 1 + (4 y + 1) + 1 + (4 z + 1) + 1 =
2 2 2
4 ( x + y + z ) + 6 4 ⋅1 + 6
= = = 5 olduğundan,
2 2
(4 x + 1) ⋅ 1 + (4 y + 1) ⋅ 1 + (4 z + ) ⋅ 1 < 5 alarıq.

128. 1 = a + b + c = a ⋅ 1 + b ⋅ 1 + c ⋅ 1 ≤

≤ a 2 + b 2 + c 2 ⋅ 12 + 12 + 12 , a 2 + b2 + c2 ⋅ 3 ≥ 1.
Bu bərabərsizliyin hər iki tərəfini kvadrata yüksəltsək,
1
a 2 + b 2 + c 2 ≥ alarıq.
3
1
Beləliklə, a 2 + b 2 + c 2 ifadəsinin ən kiçik qiyməti olur.
3
1
Bərabərlik halı yalnız a = b = c = olduqda mümkündür.
3

129. f ( x ) = n x − n x − b (x ∈ [b;+∞ )) götürsək,

1 1 
f ′( x ) =  n −
1 <0.
n  x n −1 n ( x − b )n −1 
 
Deməli, [b;+∞ ) aralığında funksiya azalandır.

120
Onda f ( x ) ≤ f (b ) ⇔ n x − n b − n x − b ≤ 0 .
x = a yazsaq, verilmiş bərabərsizliyi alarıq.

130. x ∈ [0;+∞ ) olduqda f ( x ) = x − ln (1 + x )


funksiyasının artması və ya azalmasını araşdıraq.
Funksiya (0;+∞ ) aralığında kəsilməzdir və aralıqda

f ′( x ) = 1 −
1 x
= > 0.
1+ x 1+ x
Yəni, funksiya (0;+∞ ) aralığında artandır. Ona görə də
ixtiyari x ∈ [0;+∞ ) üçün f ( x ) ≥ f (0 ) .
Yəni, x − ln (1 + x ) ≥ 0 − ln (1 + 0 ), x − ln (1 + x ) ≥ 0 .

131. (0;+∞ ) aralığında f ( x ) = x 3 funksiyasının çökük və ya


qabarıq olmasını araşdıraq. x ∈ (0;+∞ ) olduqda
f ′′(x ) = 6 x > 0 olduğu üçün bu aralıqda f (x ) = x 3 funksiyası
çökündür. Onda çökük funksiyanın tərəfinə əsasən
f (a ) + f (b )  a + b  , yəni
a,b ∈ (0;+∞ ) olduqda ≥ f 
2  2 
a 3 + b3  a + b 
3

≥  olur.
2  2 
Bu isə isbatı tələb olunan bərabərsizlikdir.

132. α + β -dan heç olmazsa biri 0 -a və ya π -yə bərabər


olarsa, verilmiş bərabərsizlikdə bərabərlik halı alınır. Ona
görə də α , β ∈ (0;π ) halını nəzərdən keçirək. (0;π )
aralığında f ( x ) = sin x funksiyasının qabarıq və ya çökük

121
olmasını araşdıraq. x ∈ (0;π ) olduqda f ′′(x ) = − sin x < 0
olduğu üçün bu aralıqda f ( x ) = − sin x funksiyası
qabarıqdır. Onda qabarıq funksiyanın tərifinə əsasən
 a + b  f (a ) + f (b ) α + β sin α + sin β
f ≥ , yəni sin ≥ olur.
 2  2 2 2

133. Laqranj bərabərliyini y = ln x funksiyası üçün yazaq:


1
ln b − ln a =(b − a ), a ≤ c ≤ b; ln b − ln a ≤ b − a .
c a
Burada a = 99, b = 100 götürsək, verlmiş bərabərsizlik
alınar.

134. Koşi bərabərsizliyini (n + 1) sayda olan


1 1 1
1; 1 + ; 1 + ;  ; 1 + ədədlərinə tətbiq edək:
n
 n
 n
n

n
 1  1  1  1
n +1 1 ⋅  1 +  = n +1  1 +  ⋅  1 +  ⋅  ⋅  1 +  <
 n  n  n  n
 n

 1  1
1 +  +  + 1 + 
1+  n   n = n+2.
<
n +1 n +1
Buradan isə alırıq ki,
n +1 n +1
 1 n+ 2
n n

və ya 1 +  ≤ 1 +
1 1 
1 +  ≤    .
 n   n +1   n   n +1

122
135. n = 1 olduqda yoxlayaq: x1 ≤ x1 alınır ki, bu isə
bərabərlik halı üçün doğrudur.
n = 2 olduqda x1 + x2 ≤ x1 + x2 olduğunu yoxlayaq.
Bilirik ki, − x1 ≤ x1 ≤ x1 , − x2 ≤ x2 ≤ x2 .
Bu bərabərsizlikləri hədbəbəd toplasaq,
− ( x1 + x2 ) ≤ x1 + x2 ≤ x1 + x2 ⇔ x1 + x2 ≤ x1 + x2 .
n = k olduqda bərabərsizliyin doğruluğunu qəbul edək. Yəni
qəbul edək ki, x1 + x2 +  + xk ≤ x1 + x2 +  + xk .
Göstərək ki, n = k + 1 olduqda da bərabərsizlik doğrudur.
Yəni x1 + x2 +  + xk +1 ≤ x1 + x2 +  + xk +1 . Doğrudan da
(x1 + x2 +  + xk ) + xk +1 ≤ x1 + x2 +  + xk + xk + 1 ≤
≤ x1 + x2 +  + xk + xk +1 .

1 2 −1 1 1
136. 1) n = 2 olduqda 2
< ⇒ < , yəni,
2 2 4 2
bərabərsizlik doğrudur. 2) Tutaq ki, n = k üçün
1 k −1
S (k ) = 2 + 2 + 2 +  + 2 <
1 1 1
doğrudur.
2 3 4 k k
Onda n = k + 1 üçün təklifin doğruluğunu isbat edək:

S (k + 1) = 2 + 2 + 2 +  + 2 + = S (k ) +
1 1 1 1 1 1
<
2 3 4 k (k + 1)2
(k + 1)2
<
k −1
+
1
=
2
( )
(k − 1)(k + 1) + k = k 2 − 1 (k + 1) + k =
k (k + 1)2 k (k + 1)
2
k (k + 1)
2

123
k3 + k2 − k −1+ k k3 + k2 −1 k3 + k2 k
= = < = =
k (k + 1) k (k + 1)
2
k (k + 1)
2
k +1

=
(k + 1) − 1
k +1
aldıq. Deməli, verilmiş bərabərsizlik istənilən natural
n ≥ 2 üçün doğrudur.

sin 2α 2 sin α cos α


137. 1) n = 1 olduqda cos α = = = cos α ,
2 sin α 2 sin α
yəni, təklif doğrudur. 2) Tutaq ki, n = k üçün
sin 2k α
cos α ⋅ cos 2α ⋅ cos 4α ⋅  ⋅ cos 2k −1α = doğrudur.
2k sin α
n = k + 1 üçün eyniliyi isbat edək:
cos α ⋅ cos 2α ⋅ cos 4α ⋅  ⋅ cos 2k −1α ⋅ cos 2k α =
sin 2k α 1 2 sin 2k α ⋅ cos 2k α sin 2k +1α
= ⋅ cos 2 k
α = ⋅ = k +1 .
2k sin α 2 2k sin α 2 sin α
Deməli, eynilik doğrudur.

π
138. 1) n = 1 olduqda, 2 = 2 cos , təklif doğrudur.
4
π
2) Tutaq ki, n = k üçün 2 + 2 + 2 +  + 2 = 2 cos k +1
  2
k

doğrudur. İsbat edək ki, n = k + 1 üçün də


π
2 + 2 + 2 +  + 2 = 2 cos k + 2 olur.
  2
k +1

124
2 + 2 + 2 +  + 2 = 2 + 2
+
 +
 2 =

 
k +1 k

π  π  π
= 2 + 2 cos k +1
= 21 + cos k +1  = 2 ⋅ 2 cos 2 k + 2 =
2  2  2
π
= 2 cos
.
2k + 2
Deməli, verilmiş bərabərlik istənilən natural n üçün
doğrudur.

139. a 4 , b 4 və c 4 ədədlərinə iki-iki toplanan olmaqla


ədəbi orta və həndəsi orta haqqında bərbərsizliyi tətbiq
a 4 +b 4 a4 + c4 b4 + c4
edək: ≥ a 2b 2 , ≥ a 2c 2 , ≥ b 2c 2 .
2 2 2
Bu bərabərsizlikləri tərəf-tərəfə toplasaq,
a 4 + b 4 + c 4 + ≥ a 2b 2 + a 2c 2 + b 2c 2 alarıq.
İndi isə a 2b 2 , a 2c 2 və b 2c 2 ədədlərini iki-iki toplayıb,
Koşi bərbərsizliyi tətbiq edək:
a 2b 2 + a 2 c 2
≥ a 2b 2 ⋅ a 2c 2 = a 2 bc ≥ a 2bc,
2
a 2b 2 + b 2 c 2
≥ a 2b 2 ⋅ b 2c 2 = b 2 ac ≥ b 2 ac,
2
a c + b 2c 2
2 2
≥ a 2c 2 ⋅ b 2c 2 = c 2 ab ≥ c 2 ab.
2
Bu bərabərsizlikləri də tərəf-tərəfə toplasaq,
a 4 + b 4 + c 4 ≥ abc(a + b + c ) alarıq.

125
Burada bərabərlik halı yalnız a = b = c olduqda alınır.

x3
140. (0;+∞ ) aralığında f ( x ) = sin x − x +
funksiyasını
6
nəzərdən keçirək. Bu funksiya həmin aralıqda kəsilməzdir
və f (0) = 0. f ′( x ) = cos x + 1 + 0,5 x 2 .
(0;+∞ ) aralığında f ′(x ) -in işarəsini müəyyən etmək üçün
g ( x ) = cos x − 0 + 0,5 x 2 funksiyasını nəzərdən keçirək.
g ′( x ) = − sin x + x, (0;+∞ ) aralığında g ( x ) -in işarəsini
müəyyən etmək üçün h( x ) = − sin x + x funksiyasını
nəzərdən keçirək. h′(x ) = − cos x + 1 ≥ 0 olduğu üçün (0; +∞)
aralığında h( x) funksiyası artandır. Ona görə
h( x) > h(0), sin < x. Onda x ∈ (0; +∞) olduqda g ′(x ) > 0 .
Deməli, (0; +∞) aralığında g ( x) funksiyası artandır.
Ona görə g ( x) > g (0), cos x − 1 + 0,5 x 2 > cos 0 − 1 + 0,5 ⋅ 02
və ya cos x − 1 + 0,5 x 2 > 0 . Deməli, x ∈ (0; +∞) olduqda
f ′( x ) > 0 . Bu isə onu göstərir ki, (0; ±∞) aralığında f ( x)
x3
funksiyası artandır. Ona görə də f ( x) > f (0), sin − x + > 0.
6
x3
Yalnız x = 0 olduqda sin x − x + =0 olur.
6

n n n(n + 1)
141. Verilmiş bərabərsizliyi >
n(n − 1) n n

126
n x
şəklində yazıb, (2; +∞) aralığında f ( x) =
n( x − 1)
funksiyasının monotonluğunu araşdıraq.
n( x − 1) nx x −1
− xn − n( x − 1)
= f `( x) = x x − 1 x
n 2 ( x − 1) x( x − 1)n 2 ( x − 1)
x −1
x ∈ (2; +∞) olduqda xn < 0, n( x − 1) > 0 və
x
x( x − 1)n 2 ( x − 1) > 0 olduğu üçün f `( x) > 0.
n x
Deməli, (2 + ∞) aralığında f ( x) = funksiyası
n( x − 1)
azalandır. Ona görə də n > 2 olduqda f (n) > f (n + 1) .
n n n(n + 1)
Yəni, > və ya n 2 n > n(n − 1)n(n + 1)n > 2
n(n − 1) n n

142. Bərabərsizliyi (b − a)tga < n cos a − n cos b < (b − a)tgb


 π
şəklində yazaq.  0;  aralığında y = tgx funksiyası müsbət,
 2
kəsilməz və artan funksiya olduğu üçün bərabərsizliyi bu
funksiyaya tətbiq edə bilərik.
f=
( x) tgx=
, n 1,= , x1 b olduğunu həmin
x0 a=
bərabərsizlikdə nəzərə alsaq,
b
(b − a )tga < ∫ tgxdx < (b − a )tgb olur.
a
b
−d (cos x)
b
∫ tgxdx =
∫ =
−n cos x |ba =
n cos a − n cos b .
a a cos x

127
n cos a − n cos b
Nəhayət, alırıq ki, tga < < tgb .
b−a

143. Ədədi orta və həndəsi orta haqqında bərabərsizliyə


əsasən
y y z z z
x+ + + + + 2 3
2 2 3 3 3 ≥ 6 x  y   z  olur.
   
6  2 3
y z
Bərabərsizlik halı yalnız x= = olduqda mümkündür.
2 3
x+ y+z xy 2 z 3 ( x + y + z ) 6 66
Buradan ≥ 6 və ya ≥ =
432 .
6 108 xy 2 z 3 108
Deməli, verilmiş ifadənin ən kiçik qiyməti 432-dir.

144. Aydındır ki,


x 2 ≤ [2 x] ⋅ {2 x} ⇔ ([2 x] + {2 x}) ≤ 4 ⋅ [2 x] ⋅ {2 x} ⇔
2

⇔ ([2 x] − {2 x}) ≤ 0 ⇔ [2 x] − {2 x} = 0 ⇔ [2 x] =
2

= {2 x} ⇔ x= 0

145.
2sin 200 ⋅ sin 400 ⋅ sin 800 = (cos 200 − cos 600 ) ⋅ sin 800 =
1
= cos 200 ⋅ sin 800 − sin 800
2
1 1 3
cos 200 ⋅ sin 800 = (sin1000 + sin 600 )= sin 800 + ,
2 2 4

128
3
onda sin 200 ⋅ sin 400 ⋅ sin 800 = . Digər tərəfdən
8
cos 200 ⋅ cos 400 ⋅ cos800 =
8sin 200 ⋅ cos 200 ⋅ cos 400 ⋅ cos800 sin1600 1
= = =
8 ⋅ sin 200 8sin 200 8
3 1
Onda tg 200 ⋅ tg 400 ⋅ tg 800 = : = 3.
8 8

146. y = 0 olduqda f ( x + 0) + f ( x − 0)= 2 x 2 + 2 ⋅ 02 və


buradan 2 f ( x) =2 x 2 ⇔ f ( x) =x 2 .

147. 4 xy = ( x + y ) 2 − ( x − y ) 2 eyniliyini nəzərə alsaq:


f ( x + y ) − f ( x − y ) = ( x + y ) 2 − ( x − y ) 2 alarıq. x + y =
u və
x− y =v ilə işarə etsək: f (u ) − f (v) =u 2 − v 2 olar.
) u 2 + f (0); indi u -nu x -lə əvəz etsək:
v = 0 olduqda f (u=
f ( x=
) x 2 + a=
, a f=
(0) const

148. Koşi-Bunyakovski bərabərsizliyinə görə:


c 2 + ab ≤ c 2 + a 2 ⋅ c 2 + b 2 ,

a 2 + bc ≤ a 2 + b 2 ⋅ a 2 + c 2 ,

b 2 + ca ≤ b 2 + c 2 ⋅ b 2 + a 2 .
Ədədi və həndəsi orta haqqında Koşi bərabərsizliyini
nəzərə almaqla, buradan

129
a 2 + b2 b2 + c2 c2 + a 2 a 2 + b2
+ + ≤ +
c 2 + ab a 2 + bc b 2 + ca c2 + b2 ⋅ a 2 + b2
b2 + c2 c2 + a2
+ + ≥
a 2 + b2 ⋅ a 2 + c2 b2 + c2 ⋅ b2 + a 2
a 2 + b2 b2 + c2 c2 + a2
≥ 3⋅ ⋅ ⋅ =
c2 + a 2 ⋅ c2 + b2 a 2 + b2 ⋅ a 2 + c2 b2 + c2 ⋅ b2 + a 2
=3

149. Şərtə görə sin x(cos α − m =


⋅ cos β ) cos x(sin α − m sin β ),
sin x(sin α − n sin=
β ) cos x(n cos β − cos α ).
Buradan
sin 2 α − (m + n) sin α ⋅ sin β + mn sin 2 β =
− cos 2 α +
+(m + n) cos α ⋅ cos β − mn cos 2 β ,
(m + n) cos(α − β=) (sin 2 α + cos 2 α ) +
+ mn(sin 2 β + cos 2 β ),
1 + mn
(m + n) cos(α − β ) =+
1 mn, cos(α − β ) = alarıq.
m+n

150. x = 1 olduqda bərabərlik halı alınır. x > 1 olduqda


x 2003 − 1
verilən bərabərsizlik ≥ 2003 x1001 ilə eynigüclüdür.
x −1
Koşi bərabərsizliyinə görə
x 2003 − 1
1 + x + x 2 +  + x 2002 ≥ 20032003 x 2003⋅1001 =2003x1001 .
x −1
Bununla da isbat tamam oldu.

130
151. y = f ( x) və y = − x 2 əvəzləmələri ilə ∀x ∈ R üçün
| f ( x) |= x 2 taparıq. Buradan xüsusi halda f (0) = 0 alarıq.
x = 0 olduqda ∀y ∈ R üçün f ( y ) + f (− y ) =
2 y2 .
Belə ki, f ( y ) ≤ y 2 və f (− y ) ≤ y 2 , onda axırıncı eynilikdən
∀y ∈ R üçün f ( y ) = y 2 alarıq. Yoxlama aparsaq, axtarılan
funksiyanın f=
( x) x 2 , x ∈ R olduğunu görərik.

152. f ( x) =1 + x + x 2 +  + x n , n = 2005 funksiyasına baxaq.


x n +1 − 1
Qeyd edək ki, A = f `(2) və f ( x) = ( x ≠) .
x −1
(n + 1) x n ( x − 1) − x n +1 + 1
f `( x) = , onda
( x − 1) 2
(n + 1) ⋅ 2n (2 − 1) − 2n +1 + 1
A= = (n − 1)2n + 1 = 2004 ⋅ 22005 + 1 .
(2 − 1)
Deməli, A ədədini 2004-ə böldükdə qalıqda 1 alınar.

153. Sistemin I və II tənliyini toplayıb, buradan III tənliyi


x+ y x+z y+z
çıxsaq: sin ⋅ cos ⋅ cos = 0 tənliyini alarıq.
2 2 2
=
Onda x π= k , y π= m, z π n, k , m, n ∈ Z .

154. Məlum x 2 + y 2 ≥ 2 xy bərabərsizliyini tətbiq etsək:


(a 2 + b 2 ) + (c 2 + d 2 ) ≥ 2(ab + cd ) ≥ 4 abcd = 4 1 = 4 .
2 , bc + ad ≥ 2 abcd =
Digər tərəfdən av + cd ≥ 2 abcd = 2,
ac + bd ≥ 2 abcd =
2.

131
Bunları verilmiş bərabərsizlikdə nəzərə alsaq:
a 2 + b 2 + c 2 + d 2 + ab + ac + ad + bc + bd + cd ≥
≥ 4+2+2+2 = 10
Bununla da tələb olunan isbatı aldıq.

155. Çevirmə aparsaq:


( x + y ) 2 =19 + 3 xy,
 alarıq.
 x + y = 7 + xy
Buradan ( x + y ) 2 − 3( x + y ) + 2 =,
0
1 və ( x + y ) =
( x + y) = 2.
x + y =1 x =−2;
Onda 1)  ⇔
 xy =−6 y =3;
x + y = 2,
2)  buradan isə tam həll alınmır.
 xy = −5
Deməli, ( x; y ) ∈ {−2;3), (3; −2)} olur.

156. Koşi bərabərsizliyinə görə


 1+ 4
 1⋅ n < 2 ,

 2 ⋅ (n − 1) ≤ 1 + n ,
 2

............................
 1+ n
 (n ⋅1) ⋅ 2 ≤
 2
 1+ n
 n ⋅1 <
 2

132
bu bərabərsizlikləri hədbəhəd vursaq,
 1+ n   1+ n 
n n

(n !) < 
2
 və ya n ! <   olar.
 2   2 

157.= , a2 b qəbul edib, verilmiş ardıcıllığın ilk bir


a1 a=
neçə həddini hesablayaq:
b +1 a + b +1 a +1
= a3 =
, a4 =, a5 =, a6 a=
, a7 b .
a ab b
Sonrakı hədlər isə 5 addımdan bir təkrarlanacaqdır.
=
Beləliklə, a33 a=
3 , a44 a4 və a2007 = a2 olar. Onda
b +1
 a = 3, 2
 və buradan a = və b = 1 alarıq.
 a + b +1 = 4 3
 ab
Deməli, a2007= a=
2 1 olur.

=
158. =
a cos ϕ , b sin ϕ ödəyən ϕ bucağı vardır, çünki
sin 2 α + cos 2 α =
1 eyniliyi ödənir. Onda
a 3b − b3 a= ab(a 2 − b 2 )= cos ϕ . sin ϕ (cos 2 ϕ − sin 2 ϕ )=
1 1 1
= sin 2ϕ ⋅ cos 2ϕ = sin 4ϕ ≤ ,
2 4 4
1 1
çünki, | sin 4ϕ |≤ 1 buradan a 3b − b3a ≤ və ƏBQ = olur.
4 4

159. Əvvəlcə f ( x) in dörülüyündən istifadə edək:


f (2007)= f (7 + 10 ⋅ 200)= f (7)

133
f ( x) tək funksiya olduğundan f (− x) =− f ( x), buradan
f (7) =− f (−7) =−1,5 alarıq.

160. Tutaq ki, x + 2 = t , onda x = t − 2 . Yəni dəyişəni


tənlikdə nəzərə alsaq: f (tT ) + (t − 2) = 0 və burada t
dəyişənin yerinə istənilən başqa hərf ola bilər. Onda t -ni
x -la əvəz etsək: f ( x) + f (k − 2) = 0 . Bu axırıncı tənliyi
verilmiş tənlikdə nəzərə alıb, çevirmə aparsaq:
f ( x + 2) = f ( x − 2) alarıq.
Tutaq ki, x − 2 =u, onda x + 2 = u + 4 və f ( x + 2) = f ( x − 2)
münasibəti f (u + 4) = f (u ) şəklinə düşər. Burada, u ↔ x
nəzərə alsaq: f ( x + 4) = f ( x) . Bu isə f ( x) -in, dövrü 4 olan
dövri funksiya olması deməkdir.

161. x= y= 1 olduqda f (1 ⋅1)= f (1) + f (1) ⇔ f (1)= 0


Digər tərəfdən istənilən x > 0 üçün
 1 1 1
f (1) =f  x ⋅  ⇔ 0 =f ( x) + f   ⇔ f ( x) =− f  
 x x x

Onda x = 2011 olduqda f (2011) = − f 


1 
 buradan da
 2011 
 1 
f  = −1 alarıq.
 2011 

) kx + b şəklində olduğundan
162. Xətti funksiya f ( x=
f ( x + 1)= k ( x + 1) + b və f (2 x) =k ⋅ 2 x + b .
Bunları verilmiş tənlikdə nəzərə alsaq:

134
k ( x + 1) + b + 2kx + b = (k + 1) 2 , buradan
3kx + (k + 2b) = x 2 + 2 x + 1 alarıq.
Burada sol tərəf xətti, sağ tərəf isə kvadratik funksiyadır.
Birdərəcəli çoxhədli, ikidərəcəli çoxhədliyə bütün x -lər
üçün bərabər ola bilməz.
Deməli, verilmiş şərti ödəyən xətti funksiya yoxdur.

163. x= y= 0 olduqda f (0 + 0)= f (0) + f (0), buradan


f (0) = 0 olur. Digər tərəfdən
f (0) = f ( x − x) ⇔ 0 = f ( x + f (− x)), onda
0 =f ( x) + f (− x) ⇔ f (− x) =− f ( x) .
 2 2
Deməli, f  −  =
−f  .
 7 7
 2 2  2 2
= f  34 ⋅ + =
f (1)  f  34 ⋅  + f  = 
 7 7  7 7
 2 2 2  2 2 2
= f  33 ⋅ +  + f   = f  33 ⋅  + f   + f  =
 7 7 7  7 7 7
2 2 2 2
=  = f   + f   +  + f   = 35 ⋅ f  
7
 7
  7 7
35defe

π π
 2  f (10) və buradan f  −  =
2
Beləliklə, f   = = −
7 35 35  7  35
alarıq.

164. f ( x)= x ⋅ g ( x) qəbul edək. Onda f `( x=


) g ( x) + x ⋅ g `( x)
olar. Buradan f `(0) = g (0) alarıq.

135
Beləliklə, f `(0) = (0 − 1)(0 − 2) (0 − 2011) ⋅ (0 − 2012) = 2012! .

165. x1 tənliyin kökü olsun. Onda f ( x1 ) . Göstərək ki,


tənliyin başqa kökü yoxdur. Əksini fərz edək. Tutaq ki,
x2 də tənliyin həqiqi köküdür, yəni, x1 ≠ x2 üçün
f=
( x2 ) f=
( x1 ) 0 olur. f ( x=
) x 2011 + 2011 ⋅ x − 2011
funksiyasına baxaq.
) 2011 ⋅ x 2010 + 2011= 2011( x 2010 + 1) > 0 olduğundan,
f `( x=
f ( x) funksiyası artandır. Onda x1 < x2 üçün f ( x1 ) < f ( x2 ) .
Bu isə f= ( x1 ) 0 şərtinə ziddir. Deməli, tənliyin
( x2 ) f=
yalnız bir həqiqi kökü var.

166. Aydındır ki, m > n. Ona görə m= n + p , ( p ∈ N ) qəbul


edək. Onda 3n + p − 3n =
2106.
2106 =2 ⋅ 34 ⋅13 olduğundan 3n (3 p − 1) =2 ⋅ 34 ⋅13 alarıq.

3 p − 1 cüt ədəd olduğundan 3n = 34 ⇒ n = 4 .

Onda 3 p − 1= 26 ⇒ p = 3, buradan m = 4 + 3 = 7 olur.


Deməli, axtarılan natural həll (7, 4) cütüdür.

167. 2 cos 2 ( x=
2010
) 2010 x + 2010− x .

İstənilən x ∈ R üçün 2 cos 2 ( x 2010 ) ≤ 2 və 2010 x + 2010− x ≥ 2


(iki müsbət və qarşılıqlı tərs ədədin cəmi 2-dən kiçik olmadığına
görə). Onda sol və sağ tərəfin bir-birinə bərabər olması üçün

2 cos 2 ( x 2010 ) = 2, 2010 x + 2010− x = 2 olmalıdır.

136
İkinci tənliyin yeganə x = 0 kökü var. Bu da birinci tənliyi
ödədiyindən, verilmiş tənliyin həlli x = 0 olur.

168. sin 2011 ≤ sin 2 x və cos 2010 x ≤ cos 2 x, x ∈ R . Buradan


sin 2011 x + cos 2010 x ≤ 1 alarıq.
Bərbaərlik halı sin 2011 x = sin 2 x və cos 2010 x = cos 2 x olduqda
ödənildiyindən, sin x = 0 və ya sin x = 1, buradan
π
=x π k, k ∈ z x= + 2π n, n ∈ z .
2
Hər iki cavab II şərti ödəyir.

2
169. sin ≤ 1 və cos 2 x ≤ 1 olduğundan
x
 x
sin = 1 π + 4π , n ∈ z
x =
1)  2 ⇔
cos 2 x = 1=x π k, k ∈ z

 x  x =−π + 4π n, n ∈ z ,
sin = −1 
2)  2 ⇔ π
cos 2 x = −1  x =− + π k, k ∈ z
2
π + 4π n, n ∈ z
1) və 2) -dən çıxır ki, x =

170. sin 2011 x = 1 + cos 2010 x, buradan

sin x ≤ 1, sin x = sin x =


2011 2011
1 1,
 ⇔  ⇔  ⇔
1 + cos ≥ 1 cos 2010 x = cos x = 0
2010
0
π
⇔ sin x =1 ⇔ x = + 2π n, n ∈ z
2

137
1 + x2
171. x ≠ 0, x > 0 olduqda ≥ 1,
2x
1 + x2
x < 0 olduqda isə ≤ −1 .
2x
πx
Digər tərəfdən ∀x ∈ R üçün −1 ≤ sin ≤ 1 olduğundan
2
 πx  πx
 sin = 1 sin 2 = −1
2
1)  ⇒x=
1; 2)  ⇒x=−1,
 1 + x 2
 1 + x 2
=1 = −1
 2  2
buradan x = ±1 alarıq.

172. Qeyd edək ki, istənilən k üçün sin kx ≤ 1 . Onda


verilmiş tənlikdəki n toplananın cəminin n -ə bərabər olması
üçün hər bir toplanan yalnız 1-ə bərabər olmalıdır.
Yəni,=sin x 1,= sin 2 x 1,= sin nx 1 olmalıdır.
sin 3 x 1, ,=
π
Birinci tənlikdən x = + 2π k , k ∈ z. Bu qiymətlər ikinci
2
π 
tənlikdə istənilən k üçün sin 2  + 2π k  = 0 verir ki, onda
2 
toplananların cəmi n -ə bərabər ola bilmir. Deməli, verilən
tənliyin həlli yoxdur.

1
log 4 x = t əvəz edək, onda x = 16 , x = 4t , x = 2t
t 4
173.
2
qiymətlərini tənlikdə nəzərə alsaq:

138
t t
 2 1
log 6 ( 4t + 2t ) =t ⇔ 4t + 2t =6t ⇔   +   =1 .
 3 3
Buradan t = 1 yeganə kökdür. Onda x = 16t = 161 ⇒ x = 16 olur.

174. Tutaq ki, şərti ödəyən elə f və g funksiyası var. Onda


x− y =0 olduqda f (0) ⋅ g (0) =
1;
x = 0 olduqda f (0) ⋅ g ( y ) =
y + 1;
y = 0 olduqda f ( x) ⋅ g (0) =
x +1
Axırıncı 2 tənliyi tərəf-tərəfə vursaq:
f (0) ⋅ g ( y ) ⋅ f ( x) ⋅ g (0) =( y + 1) ⋅ ( x + 1),
1 nəzərə alsaq: f ( x) ⋅ g ( y ) = ( x + 1) ⋅ ( y + 1) .
burada, f (0) ⋅ g (0) =
Onda şərtə görə x + y + 1 = ( x + 1)( y + 1) alarıq.
Aldığımız bərabərlik isə eynilik deyildir. Ziddiyyət aldıq.
Deməli, elə f və g funksiyası yoxdur.

175. Verilmiş tənlikdə x -i (− x) -lə əvəz edək:


f (− x) + f ( x) = sin(− x) .
Onda sin x =sin(− x) ⇔ sin x =− sin x ⇒ sin x =0.
Buradan= x π k , k ∈ z alarıq. Beləliklə, daha geniş təyin
oblastı f üçün {π k | k ∈ z} çoxluğudur.
f funksiyasının özünü tapmaq üçün verilmiş tənlikdə
x = π k yazaq: f (π k ) + f (−π k ) =
0 ⇒ f (−π k ) =
− f (π k ).
Axırıncı aldığımız tənlikdən görünür ki, f ( x) funksiyası
π k şəklində bütün nöqtələr üçün tək funksiyadır. Beləliklə,
D( f ) {π k | k ∈ Z } və f ( x) istənilən tək funksiyadır.
=

139
176. Verilmiş tənlikdə y = 0 yazaq: f ( x) ⋅ g (0) =
2sin x .
Bu bərabərlikdən çıxır ki, g (0) ≠ 0 .Tutaq ki,
2
g=
(0) a, (a ≠ 0). Onda f ( x) ⋅ a = 2sin x ⇔ f ( x) = ⋅ sin x
a
2
g ( y ) funksiyasını tapmaq üçün f ( x) = sin x verilmiş
a
tənlikdə nəzərə alsaq:
2
sin x ⋅ g ( y ) =
2sin x ⋅ cos y ⇒ g ( y ) =
a ⋅ cos y . Yoxlama
a
2
göstərir ki, f ( x) = sin x və g ( y )= a ⋅ cos y, istənilən
a
a ≠ 0 üçün tənliyi ödəyir.

177. Verilən tənlikdə x -i (1 − x) -lə əvəz edək:


f (1 − x) + (1 − x) f ( x) = 2 − x, bu tənliyin hər tərəfini x -ə
vurub, f ( x) + x ⋅ f (1 − x) =1 + x tənliyindən çıxsaq;
x ⋅ f (1 − x) + x(1 − x) ⋅ f ( x) = 2 x − x 2 ,
f ( x) + x ⋅ f (1 − x) − xf (1 − x) − x(1 − x) f ( x) =1 − x + x 2
f ( x) − x(1 − x) ⋅ f ( x) = x 2 − x + 1,
x2 − x + 1
f ( x) ⋅ (1 − x + x 2 ) = x 2 − x + 1, f ( x) = =
1 ⇒ f ( x) =
1
x2 − x + 1

178. y 2 + 2 y + 1 =0 olduqda tənliyin sol tərəfi f ( x) -ə


bərabər olur. Onda y 2 + 2 y + 1 =0 ⇒ y =−1 və bunu verilən
tənlikdə nəzərə alsaq: f ( x) = x 2 − x + 1 olur.

140
x 1
179. = t olsun, onda verilmiş tənlik f (t ) = 2 f  
x −1 t 
1
şəklinə düşər. Burada t -ni ilə əvəz etsək:
t
1 1
f   = 2 f (t ) alarıq. f (t )= 2 ⋅ f   tənliyinin hər tərəfini
t  t 
1
(-2) -yə vurub, f (t ) = 2 f   ilə toplasaq:
t 
−3 f (t ) =(
0 ⇒ t) = 0 . Beləliklə, f ( x) = 0 aldıq.

1
180. Verilmiş tənlikdə x -i -lə əvəz edək:
x
1 a
a ⋅ f   + f ( x) = .
x x
1
a ⋅ f ( x) + f   = ax tənliyinin hər tərəfini a -ya vurub,
x
1 a
a ⋅ f   + f ( x) = tənliyindən çıxsaq:
x x
a
a 2 ⋅ f ( x) + f ( x) =− a 2 x alarıq.
x
a − a2 x2 a − a2 x2
Buradan f ( x) ⋅ (1 − a 2 ) = və ya f ( x) = ,
x x(1 − a 2 )
a2 x2 − a
(a ≠ 1), f ( x) = alarıq.
x(a 2 − 1)

181. Tənlikdə x -i (1 − x) -lə əvəz edək.

141
2 f (1 − x) + f ( x) = (1 − x) 2 , hər tərəfi (-2) -yə vurub,
2 f (1 − x) + f ( x) = (1 − x) 2 tənliyi ilə toplasaq:
1 2 1
−3 f ( x) = − x 2 − 2 x + 1 buradan f ( x) = x 2 + x − alarıq.
3 3 3

182. Verilmiş tənliyi aşağıdakı kimi yazsaq:

y 4 − 2 y 2 + 1 + 2 x 4 − 4 x 2 y + 2 y 2 = 0,
(y 2
) ( )2
− 1 + 2 x 2 − y = 0.
Buradan
 y 2 − 1 = 0,  y 2 = 1,  y = ±1,  x = ±1,
 2 ⇒  2 ⇒  2 ⇒ 
 x − y = 0;  x = y;  x = ±1;  y = 1.
Cavab: (1; 1), (−1; 1).

183. x 4 + ax3 + 15 x 2 − 18 x + 9 = x 2 + px + q olsun. Onda


x 4 + ax3 + 15 x 2 − 18 x + 9 = x 4 + p 2 x 2 + q 2 + 2 px3 + 2qx 2 + 2 pqx,
x 4 + ax3 + 15 x 2 − 18 x + 9 = x 4 + 2 px3 + ( p 2 + 2q ) x 2 + 2 pqx + q 2 .
Qeyri müəyyən əmsallar metoduna görə:

 a = 2 p,  a = 2 p,
 
= p + 2q,  p 2 + 2q =
15
2
15,
 ⇒ ⇒
−18 = 2 pq,  pq = −9,
9 = q 2 . q = ±3.

142
 a = 2 p,

  a = 2 p,
 p 2 + 2q =⇒ 
15  p = −3,
⇒ q = 3 ⇒ a =2(−3) =−6
 
 p = 3

q = ±3.

olar.

184. Verilmiş tənliyi aşağıdakı şəkildə çevirək:


2 xy − 2 y − x 2 = 0,
( )
2 y ( x − 1) − x 2 − 1 =1,
(x − 1)(2 y − x − 1)=1.
Natural ədədlər çoxluğunda 1=1⋅1 olduğunu nəzərə alsaq:
 x − 1 = 1,  x = 2,  x = 2,
 ⇔  ⇒ 
2 y − x − 1 = 1; 2 y = 2;  y = 2.

185. Verilmiş tənliyin hər tərəfini 2-yə vursaq:


2 x 2 + 2 xy + 2 y 2 − 4 x + 4 y + 8 = 0,
(x 2
) ( ) ( )
+ 2 xy + y 2 + x 2 − 4 x + 4 + y 2 + 2 y + 4 = 0,
(x + y ) + (x − 2) + ( y + 2)
2 2 2
= 0.
 x + y = 0,
 x = 2
Buradan  x − 2 = 0, ⇔ 
 y + 2 = 0;  y = − 2.

143
186. xyz = 100 x +10 y + z ≥100 x, digər tərəfdən x ⋅ yz ≤ 99x,
onda xyz ≥ 100 x > 99 x ≥ x ⋅ yz və buradan xyz > x ⋅ yz.
Deməli, verilmiş bərabərliyi ödəyən rəqəmlər yoxdur.

187. Şərtə görə x, y, z toplananlarından heç olmazsa, biri


cütdür. Onda, 2 yeganə cüt sadə ədəd olduğundan toplanan-
lardan biri 2 -yə bərabərdir. Tutaq ki, x = 2 -dir.
Onda y + z = 16 − x = 16 − 2 = 14 olur. Bunları sistemin II
tənliyində nəzərə alsaq:
2 y + yz + 2 z = 61,
yz + 2( y + z ) = 61,
yz = 61 − 2 ⋅14,
yz = 33
Şərtə görə y = 3, z = 11 və ya y = 11, z = 3.
Deməli, axtarılan sadə ədədlər 2, 3, 11 -dir.

188. Sistemin II tənliyindən I tənliyi çıxsaq:


(x − z ) ⋅ (1 − y )=1 alarıq.
Tam ədədlər çoxluğunda 1 = 11 və ya 1 = (−1)⋅(−1). Onda
 x − z =1,  x = z + 1,
1)  ⇔ 
1 − y =1;  y = 0.
Bunları verilmiş sistemdə nəzərə alsaq: z=97 və x=98 olar.
2) x − z = −1 və 1 − y = −1
olduqda z = x + 1 və y = 2. Bunları I tənlikdə nəzərə alsaq:
2 x + x + 1= 97 ⇔ 3 x= 96 ⇔ x= 32.
Onda z = 33 olar.
Cavab: (98; 0; 97), (32; 2; 33).
144
189. Sistemin tənlikləri tərəf-tərəfə toplasaq:
(x 2 − x + 1) + (y 2 − y + 1) + (z 2 − z + 1) = 0.
Toplananların hər biri müsbət olduğundan bu cəm müsbət
ədəddir, lakin sağ tərəf sıfırdır. Deməli, verilmiş sistemin tam
həlləri yoxdur.

190. abc + ab + a = bcb bərabərliyindən


111 ⋅ a + 11 ⋅ b + c = 101b + 10c, 111a = 90d + 9c, 37 a = 3(10b + c )
üçün a = 3; 6; 9 ola bilir. Buradan verilən şərti yalnız a=6
ödəyir ki, onda b=7 və c=4 olur.
Deməli, a+b+c = 6+7+4 = 17-dir.

191. Şərtə görə


2 2 2
(
cd = ba − ab = ba − ab ba + ab ;)( )
cd = 9(b − a ) ⋅ 11(b + a ).
2

Sol tərəf tam kvadrat olduğundan


b − a = 1, b = 6
 ⇒ alarıq.
b + a = 11 a = 5
2
Onda cd = 9 ⋅ 1⋅ 11 ⋅11 və cd = 33 olur.
Buradan ab + ba + cd = 56 + 65 + 33 = 154 alarıq.

192. Yoxlasaq: n ≠ 1; 2; 3; 4; 5. n = 6 olduqda


6!= 20 ⋅ 6 2 ⇔ 720 = 720, doğrudur.
n = 7 olduqda 7! > 20 ⋅ 7 2 aşkardır. Göstərək ki,
n > 7 üçün 7! > 20 ⋅ n 2 doğrudur.

145
Tutaq ki, n = k>7 üçün k!> 20 ⋅ k 2 ödənir. İsbat edək ki,
n = k + 1 üçün (k + 1)!> 20(k + 1) doğrudur.
2

(k + 1)!= k!(k + 1) > 20k 2 (k + 1) > (20(k + 1)) ⋅ (k + 1) = 20(k + 1)2 .


Deməli, n ≥ 7 olduqda verilmiş bərabərlik mümkün deyil.
Onda n=6 tənliyin yeganə həlli olur.

193. a ≥ 2 ≥ 2 və c ≥ c . Buradan
b b c d 98
a c 2 c 2 c 1 2
+ ≥ + ≥2 ⋅ =2 = alarıq.
b d c 98 c 98 49 7

Doğrudan da (a, b, c, d ) = (2;14;14; 98) olduqda verilən ifadə


2
-yə
7
2
bərabər olur. Deməli, həmin ifadənin ən kiçik qiyməti -dir.
7

194. Verilmiş bərabərliyin hər tərəfini 37d-yə bölsək:


33a −7 d + 34 b−7 d + 35c−7 d = 1,
1 1 1
7 d −3 a
+ 7 d − 4 b + 7 d −5 c = 1
3 3 3
1 1 1
alarıq. Buradan + + = 1 olacağı aşkardır.
3 3 3
Onda 7d − 3a = 7d − 4b = 7d − 5c = 1 və
7d = 3a + 1 = 4b + 1 = 5c + 1 alarıq ki, 7d=60 k+1 şəklində
olur. Buradan şərtə görə k=5 və d=43 olar. Onda a=100,
b=75, c=60 və min (a + b + c + d ) = 100 + 75 + 60 + 43 = 278
alarıq.

146
195. Axtarılan ədəd abcd olsun. Onda
(a + b + c + d )4 = abcd olur.
Əgər a+b+c+d = 10 olarsa, (a + b + c + d ) , 5-rəqəmli ədəd
4

olur. Əgər a+b+c+d = 5 olarsa, bu halda da (a + b + c + d )


4

3-rəqəmli ədəd olacaqdır. Deməli, a+b+c+d cəmi 6, 7, 8, 9


ola bilər. Bunlara görə,
6 4 = 1269, şərti ödəmir;
7 4 = 2401, şərti ödəyir, çünki 7 = 2+4+0+1;
84 = 4096, şərti ödəmir;
9 4 = 6561, şərti ödəmir.
Deməli, axtarılan ədəd 2401-dir.

196. Viyet teoreminə görə


a + b + c = 3

ab + bc + ac = 2
abc =1

(a + b + c )2 = a 2 + b 2 + c 2 + 2(ab + bc + ac )
eyniliyini nəzərə alsaq:
a 2 + b 2 + c 2 = (a + b + c ) − 2(ab + ba + ca ) =
2

= 32 − 2 ⋅ 2 = 5 olur.

xy yz zx
197. + + = A olsun. Onda
z x y

147
x2 y2 y2 z 2 z 2 x2
A = 2 + 2 + 2 + 2x2 + 2 y2 + 2z 2 =
2

z x y
1  x2 y 2 y 2 z 2  1  y 2 z 2 z 2 x2  1  x2 y 2 z 2 x2 
= 4 +  2 + 2  +  2 + 2  +  2 + 2 
2 z x  2 x y  2 z y 
alarıq. Hər bir mötərizəyə ədədi və həndəsi orta haqqında
bərabərsizliyi tətbiq etsək:
A2 ≥ 4 + y 4 + z 4 + x 4 = 4 + y 2 + z 2 + x 2 = 4 + 2  6.
2
Buradan A ≥ 6 . Doğrudan da x = y = z = üçün A = 6
3
olduğundan A-nın ən kiçik qiyməti 6 olur.

198. a, b, c ədədləri verilmiş tənliyin kökləri olduğundan


a +1, b +1, c +1 ədədləri
(x − 1)3 − (x − 1) − 1 = 0, x 3 − 3x 2 + 2 x − 1 = 0
1 1 1
tənliyinin kökləridir. Onda, , , ədədləri
a +1 b +1 c +1
1 − 3 x + 2 x 2 − x 3 = 0, yəni, x 3 − 2 x 2 + 3x − 1 = 0 tənliyinin
kökləridir. Viyet teoreminə görə

= − (− 2 ) = 2 olur.
1 1 1
+ +
a +1 b +1 c +1

199. Verilən tənliyi


2 ( x + 2) (x 2 − 2x + 4) = (x 2 − 2x + 4) + ( x + 2)

şəklində yazaq. Buradan (x 2


− 2 x + 4 − x + 2 = 0.)
2

Onda

148
x 2 − 2 x + 4 = x + 2,
x 2 − 3x + 2 = 0,
x1 = 1, x2 = 2 alarıq.

200. Aydındır ki, p ≠ 2, p ≠ 3; p = 5 olduqda isə şərt ödənir.


p > 5 üçün p = 5k ± 1 və p = 5k ± 2 hallarına baxmaq kifayətdir:
(
1) p = 5k ± 1 olduqda p 2 − 6 = 5 5k 2 ± 2k − 1  5 ; )
2) p = 5k ± 2 olduqda p 2 + 6 = 5 (5k 2
± 4k + 2 )  5 .
Buradan alırıq ki, p > 5 olduqda verilmiş ədədlər sadə deyil.
Deməli, məsələnin şərti yalnız p = 5 olduqda ödənir.

201. p = 5 olduqda şərt ödənir. p > 2 olduqda 2 p ± 1 > 3,


( )( )
belə ki, 2 p − 1 2 p + 1 = 4 p − 1 = (3 + 1) − 1 = (3n + 1) − 1 = 3n.
p

Buradan alırıq ki, 2 p − 1 və ya 2 p + 1 ədədlərindən biri 3-ə


bölünür. Deməli, p > 2 halında şərt ödənmir, yalnız p = 2
olduqda verilən ədədlərin hər ikisi sadədir.

202. Məsələn, 5002+1, 5002+2, ..., 5002+1000 şəklində


ədədlərə baxaq. Bu ədədlərin hər biri 2 ardıcıl kvadratın
arasında yerləşir:
500 2 < 500 2 + 1 <  < 500 2 + 1000 < 500 2 + 2 ⋅ 500 + 1 = 5012.

203. Axtarılan ən kiçik natural ədəd a olsun. Onda şərtə görə


a = 1997 m +97 və ya a = 1998 n+98 (m, n ∈ N).
Buradan 1997 m + 87 = 1998 n + 98,
1997 m − 1997 n = n+1,
1997 (m−n) = n+1.

149
Sağ tərəfin 1997-yə bölünməsi üçün n+1=1997, buradan isə
n = 1996 və m = 1997 alarıq. Onda
a =1998 ⋅ 1996 + 98 = (1997 + 1)(1997 − 1) + 98 = 1997 2 + 97
Deməli, həmin ədəd 19972 + 97 -dir.

204. Axtarılan ədəd xyz olsun. Onda şərtə görə


100 x + 10 y + z = 13( x + y + z ) + 15,
87 x = 3 y + 12 z + 15,
29 x = y + 4 z + 5 alarıq.
Digər tərəfdən y + 4z + 5 ≤ 9 + 4 ⋅ 9 + 5 = 50, onda 29 x ≤ 50,
buradan şərtə görə x = 1 olur. Bunu nəzərə alsaq:
29 ⋅ 1 = y + 4 z + 5 və ya y + 4 z = 24.
Buradan görünür ki, y rəqəmi 4-ə bölünür. Yəni, y = 0; 4; 8 -dir.
Onda (y; z) = {(0;6), (4; 5), (8;4)} və şərti ödəyən ədədlər isə
106, 145 və 184 olur.
7!
205. Bütün fərqli 7-rəqəmli düzülüşlərin sayı:
4!⋅2!
olacaqdır. Bu düzülüşlərdə soldan birinci rəqəmli “0”
6!
olanlarının sayı isə olur. Onda şərti ödəyən ədədlər sayı:
4!
7! 6!
− = 75 olar.
4!⋅2! 4!
a+b+c 3
206. Ədədi orta və həndəsi orta arasındakı ≥ abc
3
bərabərsizliyinə görə

150
x y z x y z
+ + ≥ 3 3 ⋅ ⋅ = 3 > 2,345.
y z x y x x
Deməli, verilmiş tənliyin həlli yoxdur.

207. Vuruqlara ayırsaq:


(x − 2 y )(x 2 + 2 xy + 4 y 2 ) = 1 ⋅ 19 .
Natural ədədlər çoxluğunda
x − 2 y < x < x 2 + 2 xy + 4 y 2 olduğundan
x − 2 y = 1 x = 2 y + 1
 2 ⇔  ⇔
 x + 2 xy + 4 y = 19 (2 y + 1) + 2 y (2 y + 1) + 4 y =19
2 2 2

x = 2 y + 1
⇔
y = 1
və y = −1,5 y∈ N olduğundan y = 1 və x = 3 olur.

208. N cüt olduqda 4n ədədi 6 ilə qurtarır. Onda 4n−1 ədədi


5-ə bölünür. Lakin 5n−1 ədədi 5-ə bölünmür.
n tək olduqda 4n−1 ədədi 3-ə bölünür, lakin 5n−1 ədədi 3-ə
bölünmür. Deməli, 5n−1 şəklində ədəd 4n−1 ədədinə bölünmür.

209. Həmin ədədlərin hamısı tək olsaydı, onların cəmi tək


olardı, lakin 1000-cütdür. Deməli, həmin ədədlərdən heç
olmazsa, biri cütdür. Onda onların hasili tək ədəd ola bilməz,
yəni, 2011 -lə qurtara bilməz.

210. Tutaq ki, ab+ac+ad+bc+bd+cd = 18 ola bilər. Onda


şərtə görə (a+b+c+d)2 = 36. Çevirmə aparsaq:

151
(a + b + c + d ( ))2 = ((a + b ) + (c + d ))2 =
= (a + b ) + 2(a + b )(c + d ) + (c + d ) = a 2 + 2ab + b 2 + 2ac +
2 2

+ 2ad + 2bc + 2bd + c 2 + 2cd + d 2 = a 2 + b 2 + c 2 + d 2 +


+ 2 (ab + ac + ad + bc + bd + cd ).
Ədədi qiymətləri yerinə yazsaq:
36 = a 2 + b 2 + с 2 + d 2 + 2 ⋅ 18,
a 2 + b2 + c2 + d 2 = 0 ⇔ a = b = c = d = 0
Bu isə ziddiyyətdir. Deməli, verilən cəm 18 ola bilməz.

211. Tənliyi yz = 19 x − 1995 =19( x − 105) şəklində yazaq.


y, z sadə ədəd olduğundan, onlardan biri 19, digəri isə
(x − 105) -dir. Tutaq ki, y = 19, z = x−105. Onda ya x ya da
z cütdür. Yeganə cüt sadə ədəd 2 olduğundan z = 2, x = 107
olur. z = 19, y = x − 105 olduqda isə y = 2 və x =107 alarıq.
Deməli, (x; y; z ) = {(107; 2;19)} olur.

212. Verilmiş tənliyi {x − 111} = [ 6 − x ] − [x 2 ] şəklində


yazsaq, {x − 111} tam ədəd, yəni sıfır olması aşkar görünür.
Buna görə də x2+x−6=0 tənliyini alarıq. Buradan
x1 = −3, x2 = 2 və x13 + x 32 = ( − 3) 3 + 23 = − 27 + 8 = − 19 .

213. Hər hansı tam x ədədi üçün x3 − x = x(x − 1)(x + 1) və üç


ardıcıl tam ədədin hasili 6-ya bölündüyündən x3 = x (mod 6).
Buna görə də

152
a13 + a23 +  + a100
3
≡ a1 + a2 +  + a100 ≡
alarıq.
≡101101 ≡ 5101 ≡ (− 1) ≡ − 1 ≡ 5(mod 6)
101

214. 2110 − 1 = (215 − 1)(215 + 1) = ( 21 − 1) (214 + 213 + 212 + 21 + 1) ×


(
× (21 + 1) ⋅ 214 − 213 + 212 − 21`+1 = )
( 4 3 2
)(
= 20 ⋅ 22 ⋅ 21 + 21 + 21 + 21 + 1 21 −21 + 212 − 21 + 1 .
4 3
)
Qeyd edək ki, 214 + 213 + 212 + 21 + 1 cəmi 5-ə bölünür, onda
bu cəm 5-lə qurtarır.
Deməli, alınmış hasil 22⋅20⋅5 = 2200 -ə bölünür.

215. Sağ tərəfi sol tərəfə keçirib, çevirmə aparsaq:


3( x + 2011) ⋅ ( x + 2012) = 0 və buradan
x1 = −2011, x2 = −2012 alarıq.

216. Verilmiş tənliyi 2n6 − 3n5 + 2n4 = 1 şəklində yazaq.


Sol tərəf n-ə bölünür. Sağ tərəf isə yalnız n = 1 olduqda
n -ə bölünür.

217. x = a, y = b, z = c, x − y + z = d qəbul edək.


Onda
a − b + c = d , a − b = d − c (I )
 2 və ya  2
a − b = d
2 2
a − b = d − c
2 2 2
(II )
sistemini alarıq.
Əgər a−b = 0, onda x− y = 0. Əks halda II tənliyi I tənliyə
bölsək: a + b = d + c. Buradan I tənliyi çıxsaq: b = c,

153
onda y = z alarıq. Deməli, istənilən halda
(x − y)(y − z)( z − x) = 0 olur.
x+y x−y 2x 2 + 2 y 2
218. Qeyd edək ki, + = .
x−y x+y x 2 − y2
x 2 + y2 3
Onda 2 = və verilən ifadənin qiyməti:
x −y
2
2
x 2 + y2 x 2 − y2 3 2 13
2 + 2 2 = + = olur.
x −y x +y
2
2 3 6

219. Tənliyin kökləri x1 , x2 ,  , x7 olsun. Bunları tənlikdə


yerinə yazıb, tərəf-tərəfə toplasaq:
x17 + x27 +  + x77 = − 5( x1 + x2 +  + x7 ) + 3 ⋅ 7 alarıq.
Viyet teoreminə görə x1 + x2 +  + x7 = 0 olduğundan
x17 + x27 +  + x77 = 21 olur.

220. Tutaq ki, a+b+c ≤ 11, onda 28a+30b+31c ≤


≤ 31a+31b+31c ≤ 31⋅11 = 341 < 365.
Tutaq ki, a+b+c ≥13, onda 28a+30b+31c ≥ 28⋅13 = 364; bu
hal isə a =13, b = c = 0 olduqda mümkündür, bu da
ziddiyyətdir. Qalan bütün hallarda isə 28a+30b+31c ≥ 366.
Deməli, yalnız a+b+c = 12 halı mümkündür.

221. Qeyd edək ki, 999 = 33⋅37.


37 sadə ədəd olduğundan n! hasilində 37 vuruğu hökmən
iştirak etməlidir. Onda məsələnin şərtinə görə n = 37 olur.

154
222. Aydındır ki,
1+x2 = xy+yz+zx+x2 = y(x+z)+x(z+x) = (x+y)(x+z).
Analoji olaraq, 1 + y 2 = xy + yz + zx + y 2 = ( x + y )( y + z );
1 + z 2 = xy + yz + zx + z 2 = ( x + z )( y + z ).
Beləliklə,
(1 + x )(1 + y )(1 + z ) = ( (x + y )( y + z )(x + z ) ) .
2 2 2 2

ab ac + c 2 + ab + bc
223. a + b + c + d = a + b + c + = =
c c
c(a + c ) + b(a + c ) (a + c )(b + c )
= = ∈N .
c c
Buradan a+c > c və b+c > c. Aydındır ki, axırıncı kəsri
ixtisar etdikdə 1-dən fərqli, iki vuruğun hasili alınar.
Deməli, a+b+c+d ədədi mürəkkəbdir.

224. Qeyd edək ki, (b − c ) + (a − b ) = a − c. Onda


bc(a + d )(b − c ) − ac(b + d )(b − c ) − ac(b + d )(a − b ) +
+ ab(c + d )(a − b ) =
= (b − c )(bc(a + d ) − ac(b + d ) + (a − b )(ab(c + d ) − ac(b + d )) =
= c(b − c )(ab + bd − ab − ad ) + a(a − b )(bc + bd − bc − cd ) =
= cd (b − c )(b − a ) + (a − b )ad (b − c ) = (b − c )(a − b )(ad − cd ) =
= d (b − c )(a − b )(a − c ).

225. a+b+c=0 şərtindən olduqda a + b = − c. Hər tərəfi kuba


yüksəltsək: (a + b ) = −c 3 ⇒ (a + b ) + c 3 = 0.
3 3

Buradan a 3 + b3 + c 3 = −3ab(a + b ) = 3abc.

155
Deməli, a 3 + b3 + c 3 = 3 abc olur.

226. Verilmiş tənliyi aşağıdakı kimi çevirək:


19 x 3 − 84 y 2 =1900 + 84,
19 x 3 − 1900 = 84 y 2 + 84,
(( ) ) (
19 x 3 − 2 − 98 = 84 y 2 + 1 . )
Bu axırıncı bərabərlikdə sağ tərəf 7-yə bölünür, sol tərəf isə
bölünmür. Ona görə ki, x 3 − 2 fərqi 7-yə bölünmür. Kubu 2
rəqəmi ilə qurtaran ədəd yoxdur.
Deməli, verilmiş tənliyin tam həlli yoxdur.

227. Ədədi orta və həndəsi orta haqqında teoremə görə


log 4 5 + log 5 6 + log 6 7 + log 7 8 + log8 4
>
5
> 5 log 4 5 ⋅ log 5 6 ⋅ log 6 7 ⋅ log 7 8 ⋅ log8 4 = 5 log 4 4 = 1.
Beləliklə, log 4 5 + log 5 6 + log 6 7 + log 7 8 + log 8 4 > 5.

x+ y
2

228. Məlum xy ≤   bərabərsizliyindən istifadə edək:


 2 
 1 + 2006   2007  
2 2

1. 2006 <   =   ,
 2   2  
2 
 1 + 2005 
2
 2007  
2. 2005 <   =   ,
 2   ⇒ (1 ⋅ 2 ⋅ 3 2006 ) <
2
 2 
  , 

 2006 + 1
2
  2007 
2 
2006 ⋅1 <   =  
 2   2  

156
2006
  2007  2   2007 
2006

<   ⇒ 1 ⋅ 2 ⋅ 3 ⋅  2006 < 


  2    .
   2 

229. a 2 + b 2 + c 2 = 1 şərtindən görünür ki, − 1 ≤ a, b, c ≤ 1.


Onda a 2 ≥ a 3 , b 2 ≥ b3 , c 2 ≥ c 3 .
Lakin a 2 + b 2 + c 2 = a 3 + b3 + c 3 olduğundan alınır.
Deməli, a, b, c ədədləri ya 1, ya da 0 olmalıdır. Digər
tərəfdən a 2 + b 2 + c 2 = 1 olduğundan a, b, c ədədlərindən
ixtiyari ikisi 0 olmalıdır. Onda, a + b + c = 1 olur.

230. Axtarılan birrəqəmli ədədi x qəbul edək, onda onun sağ


tərəfinə 5 yazsaq, 10x+5 ədədini alarıq. Bu ədədi 5 dəfə
artırsaq, 5(10x+5) olar. Bu ədədin təkliklər rəqəminin 5
olduğu aşkardır. Deməli, kvadratı buna bərabər olan
ikirəqəmli ədədin də təkliklər rəqəmi 5 olacaq, həmin ədədin
onluqlar rəqəmini y qəbul etsək, onda bu ədəd 10y+5
şəklində olar. Şərtə görə 5(10 x + 5) = (10 y + 5) olmalıdır. Bu
2

tənliyi aşağıdakı kimi çevirib həll edək:


25(2 x + 1) = 25(2 y + 1) və ya 2 x + 1 = (2 y + 1) olar.
2 2

Buradan x=
(2 y + 1) − 1
2
(1). Şərtə görə 0 ≤ y ≤ 9 və
2
0 ≤ x ≤ 9 olmalıdır. Bu şərtləri (1) -də nəzərə alsaq, y = 1,
x = 4. Beləliklə, axtarılan ədəd 4 və 15 olur.

231. Viyet teoreminə görə x1 + x2 = 3 və x1 ⋅ x2 = 1.


Onda ( )2
x1 + x2 = x1 + x2 + 2 x1 x2 = 3 + 2 = 5,

157
x1 + x 2 = 5.
( 4 x1 + 4 x2 ) 2 = x1 + x1 + 2 4 x1 ⋅ x2 = 5 + 2,

buradan isə 4 x1 + 4 x2 = 2 + 5 olur.

232. Çoxhədli üzərində aşağıdakı çevirməni aparaq:


x5 + x 4 + 3x3 + x 2 + x − 1 = x5 + x 4 + 2 x3 + x3 + x 2 + x − 2 + 1 =
( ) ( ) (
= x5 + x 4 + x3 + 2 x3 − 2 + x 2 + x + 1 = )
( ) (
= x x + x + 1 + 2( x − 1) x + x + 1 + x + x + 1 =
3 2 2
) ( 2
)
( )( ) (
= x 2 + x + 1 x3 + 2 x − 2 + 1 = x 2 + x + 1 x3 + 2 x − 1 . )( )
233. Verilən üçhədlini aşağıdakı şəkildə göstərək:
x 3 + x + 350 = (( x + 7 ) − 7 ) + ( x + 350) =
3

= ( x + 7 ) − 3( x + 7 ) ⋅ 7 + 3( x + 7 ) ⋅ 7 2 + x + 350 =
3 2

= ( x + 7 ) − 21( x + 7 ) + 147( x + 7 ) − 343 + x + 350 =


3 2

= ( x + 7 ) − 21( x + 7 ) + 147( x + 7 ) + ( x + 7 ).
3 2

Burada toplananlardan hər biri (x+7) -yə bölündüyündən


x 3 + x + 350 ifadəsi (x+7) -yə bölünür.

234. Ədədi orta və həndəsi orta haqqında teoremə əsasən:


a 4 + b 4 + 17 = a 4 + b 4 + 16 + 1 > 44 16a 4b 4 = 8ab,
a 4 + b 4 + 2 = a 4 + b 4 + 1 + 1 > 4 a 4b 4 = 4ab,
a 4 + b 4 + 82 = a 4 + b 4 + 81 + 1 > 44 81a 4b 4 = 12 ab.
Bu bərabərsizlikləri tərəf-tərəfə toplasaq:
( )
3 a 4 + b 4 + 101 > 24 ab alarıq.

158
235. x 44 + x 33 + x 22 + x11 + 1 = A, x 4 + x 3 + x 2 + x + 1 = B
qəbul edək. A - B fərqini hesablayaq:
A − B = x 44 − x 4 + x 33 − x 3 + x 22 − x 2 + x11 − x =
( ) ( ) ( ) (
= x 4 x 40 − 1 + x 3 x 30 − 1 + x 2 x 20 − 1 + x x10 − 1 . )
Burada hər bir toplanan (x 5 − 1) -ə bölünür.
(
Onda A − B = x 5 − 1 ⋅ X . )
( )
x 5 − 1= x 4 + x 3 + x 2 + x + 1 ( x − 1) = B( x − 1) olduğundan
A − B = B( x − 1) ⋅ X ⇒ A = B + B( x − 1) ⋅ X ⇒ A : B.

236. Tənlik üzərində çevirmə aparsaq:


6 ⋅11x ⋅ (10 x + y ) = 1100 + 11 y,
6 x(10 x + y ) =100 x + y,
60 x 2 + 6 xy = 100 x + y,
y (6 x − 1) = 20 x(5 − x ).
x > 0, y > 0 və x, y rəqəm olduğundan 5−3x > 0, buradan
isə x = 1 alarıq. Onda y = 8 olur ki, 6⋅11⋅18 = 1188 ödənir.

237. Şərtə görə a 2 = (a − c ) + 2b(a − c );


2

b 2 = (b − c ) + 2a(b − c ).
2

Onda
a 2 + (a − c ) 2(a − c ) + 2b(a − c ) (a − с )(a − c + b ) a − c
2 2
= = = .
b 2 + (b − c ) 2(b − c ) + 2a(b − c ) (b − c )(b − c + a ) b − c
2 2

238. Verilmiş üçhədliyə x2 əlavə edib-çıxsaq:

159
( )
x4 + x2 + 1 = x4 + 2x2 + 1 − x2 = x2 + 1 − x2 =
2

( )(
= x2 − x + 1 x2 + x + 1 )
alarıq.

239. (a + b) 2 = a 2 + 2ab + b 2 eyniliyini nəzərə alsaq,

210 + 213 + 2 n = (25 ) 2 + 2·25·27 + 2 n və buradan

2n = (27 ) 2 , n = 14.

Əgər 2 n = 2ab qəbul etsək, onda 213 ədədi tam kvadrat


olmadığından, bu halda həll ala bilmirik. Deməli, verilmiş
ifadə n = 14 olduqda tam kvadrat olur.

240. Qeyd edək ki, q = 2 olduqda ixtiyarı p sadə ədədi üçün


(p+1)2 tam kvadratdır. Əgər q > 2 olarsa, onda bu ifadə tək
ədəd olar.
Beləliklə, ixtiyari müsbət tam k ədədi üçün q = 2 +1 olur.
( p + 1)q = ( p + 1)2 k +1 = ( p + 1)( p + 1)2 k
ədədi, (p+1) ədədi tam
kvadrat olduqda tam kvadratdır. Onda
p + 1 = n 2 ⇒ p = (n − 1)(n + 1).
Lakin p sadə ədəd olduğundan sonuncu bərabərlik yalnız
n = 2 olduqda mümkündür. Buradan p = 3 olur. Deməli,
istənilən sadə q ədədi üçün 4q tam kvadratdır.

241. Ədədi orta və həndəsi orta haqqında bərabərsizlikdən


istifadə edib, hər bir toplanan üçün qiymətləndirmə aparaq:

160
ab ab ab
= = ≤
c + ab 1 − a − b + ab (1 − a )(1 − b )
1 a b  1 a b 
≤  + =  +  alarıq.
2 1− b 1− a  2  a + c b + c 
Analoji qayda ilə
bc 1 b c  ca 1 c a 
≤  + , ≤  +  olur.
a + bc 2  a + b a + c  b + ca 2  b + c a + b 
Aldığımız bərabərsizlikləri tərəf-tərəfə toplasaq:
ab bc ca 1 a b b
+ + ≤  + + +
c + ab a + bc b + ca 2  a + c b + c a + b
c c a  1a+c b+c a+b 3
+ + + =  + + =
a +c b +c a +b 2a +c b +c a +b 2
alarıq. Bununla da isbat tamam oldu.

242. Verilən bərabərliyin hər tərəfinə 3 əlavə edib,


а−с b−a c−b
+ 1+ +1+ + 1 = 4 şəklində yazsaq:
b+c c+a a+b
a−c+b+c b−a +c+a c−b+a+b
+ + = 4,
b+c c+a a+b
a+b b+c c+a
buradan + + =4 alarıq.
b+c c+a a+b

243. p q + q p cəminin sadə ədəd olması üçün p və ya q


ədədlərindən biri 2 olmalıdır. Əks halda iki tək ədədin tək
qüvvətləri cəmi cüt olacaq. Bu halda pq+qp ədədi sadə ədəd
olmur. p = 2 və q = 3 olduqda 23 + 32 = 17 sadə ədəddir.

161
p və q ədədləri simmetrik yerləşdiyindən ( p; q ) = (2;3) və
ya (p; q) = (3; 2) olur. p = 2 və q > 3 olduqda verilən cəm
sadə ədəd olarsa, hər sadə ədədi 3-ə böldükdə qalıqda 1 və
ya (−1) alındığından, (− 1) + (± 1) = − 1 + 1= 0 (mod 3) .
q 2

Bu halda verilən cəm 3-ə bölünür. Deməli, p q + q p ədədi


yalnız (p; q) = (2; 3) və ya (p; q) = (3; 2) olduqda sadədir,
yəni, 2 halda.

244. Çevirmə aparsaq:


(
2(a + b )(c + d )(ac + bd − 2004) = (a + b )(c + d ) (b + d ) − (a − c ) =
2 2
)
= (a + b )(c + d )(b + d − a + c )(b + d + a − c ) =
( )(
= (a + b )(c + d ) + b 2 − a 2 ⋅ (a + b )(c + d ) + d 2 − c 2 = )
= ((a + b )(c + d ) − 2004) , tam kvadratdır.
2

245. Verilmiş tənliyi ( x − 1)( y + 1) =101 şəklində yazaq.


101 sadə ədəd olduğundan
 x − 1 = 101,  x − 1 = 1,
 və ya 
y +1 = 1  y + 1 = 101.
 x = 102 x = 2
  alarıq.
y = 0  y =100
Deməli, ( x; y ) = {(102;0 ), (2;100 )}.

246. 169 = 85⋅2−1 və 162 = 85⋅3+1 olduğundan


16913 + 16256 = (85 ⋅ 2 − 1) + (85 ⋅ 3 + 1) =
13 128

= K ⋅ 85 + (− 1) + 1 = K ⋅ 85 olur.
13 128

162
Deməli, verilmiş cəm 85-ə bölünür.

247. Qeyd edək ki, x 4 + y 2 ≥ 2 x 2 y və x 2 + y 4 ≥ 2 xy 2 .


Bunları nəzərə alsaq:
x y x y xy + xy 2 xy 1
+ 2 ≤ 2 + = = 2 2= olur.
x + y x + y 2 x y 2 xy
4 2 4 2 2 2
2x y 2 x y xy
Bərabərlik halı x = y = 1 olduqda alınır.

248. Qeyd edək ki, b − 2a = 3 verilən ədədlərin ortaq


bölənidir. 3-ə böldükdə 2-nin cüt qüvvətləri qalıqda 1, tək
qüvvətləri isə qalıqda 2 verdiyini nəzərə alsaq, əmin olarıq
ki, verilən ədədlərin hər biri 3-ə bölünür.
Onda ƏBOB(a, b) = 3 olur.

249. ƏKOB(a, b) = x, ƏBOB(a, b) = y olsun. Onda ab = xy


xy
olar və verilmiş bərabərlik x − y = şəklinə düşər.
5
Çevirmə aparsaq: (5 + x )(5 − y ) = 25 alarıq. x, y ∈ N
olduğundan (5 + x ) ∈ N və (5 − y) ∈ N. 5+x və 5−y ədədləri
25 -in natural böləni olduğundan x = 20 və y=4 olur.

n 4 − 22n 2 − 46 3 29
250. = n − 5n 2 + 3n − 15 + . n ∈ N və 29
n+5 n+5
29
sadə ədəd olduğundan ∈ N olması üçün n +5 = 29,
n+5
buradan n = 24 alarıq.

251. Axtarılan ədədi 100⋅A+56 şəklində yazaq. Həmin ədəd


56-ya bölündüyündən 100A hasili də 56-ya bölünməlidir.
163
Onda A ədədi 56-ya, həm də 14-ə bölünməlidir. Yəni, A
cütdür və 7-yə bölünür. Onda A-nın rəqəmləri cəmi:
56−(5+6) = 45 olar. Rəqəmləri cəmi 45 olan ən kiçik cüt
ədəd 199998 -dir. Bu isə 7-yə bölünmür. Bu şərti ödəyən
sonrakı ədədlər 28998 və 298998-dir.
Yoxlasaq, 298998 ədədinin 7 -yə bölündüyünü görərik.
Deməli, axtarılan ədəd 29899856 -dır.

( )
252. Verilmiş tənliyi xy x 2010 − y 2010 = 2011 şəklində yazaq.
x və ya y heç olmazsa, biri cüt olduqda sol tərəf cüt, sağ tərəf
isə təkdir. Bu halda verilmiş tənliyin həlli yoxdur.
( )
x və y hər ikisi təkdirsə, x 2010 − y 2010 fərqi cüt olacaqdır.
Bu halda da verilmiş tənliyin həlli yoxdur.

253. Məlum bərabərsizliyə görə a 4 + b 4 ≥ 2a 2b 2 və buradan


a 4 + b 4 + c 2 ≥ 2a 2b 2 + c 2 . Digər tərəfdən
2a 2b 2 + c 2 ≥ 2 2a 2b 2c 2 = 2 2 abc = 2 2 ⋅ 2 = 4.
Onda a 4 + b 4 + c 2 ≥ 4 alarıq.

254. 1, 2, 3, ..., 2006, 2007 ədədi silsiləsini götürək və


1⋅2⋅3⋅...⋅2006⋅2007 =2007! = K bərabərliyinin hər tərəfini
K2007 -yə vuraq:
(1 ⋅ K)( 2 ⋅ K)( 3 ⋅ K)  ( 2007 ⋅ K) = K ⋅ K 2007 = K 2008 = (K1004 )2 .
Aşkardır ki, 2007 sayda müxtəlif natural ədəddən ibarət olan
1⋅K, 2K, 3K, ... , 2006K, 2007K ardıcıllığı ədədi silsilədir
və onların hasili tam kvadratdır: (2007!)1004 -ün kvadratıdır.

164
255. x8 + 98 x 4 y 4 + y 8 = ( x 4 + y 4 ) + 96 x 4 y 4 = ( x 4 + y 4 ) +
2 2

+16 x 2 y 2 ( x 4 + y 4 ) + 64 x 4 y 4 − 16 x 2 y 2 ( x 4 + y 4 ) + 32 x 4 y 4 =

(x + y 4 + 8 x 2 y 2 ) − 16 x 2 y 2 ( x 4 − 2 x 2 y 2 + y 2 ) =
2
= 4

= (x ) ( )
2 2
+ y 4 + 8x 2 y 2 − 4 x 3 y − 4 xy 3 =
4

= (x4
− 4 x 3 y + 8x 2 y 2 + 4 xy 3 + y 4 ⋅ x 4 + 4 x 3 y + )(
+ 8x 2 y 2 − 4 xy 3 + y 4 . )
256. Məlumdur ki, 332 − 232 = (316 − 216 ) ⋅ (316 + 216 ).
Bu qayda ilə )( )(( )
332 − 232 = 316 + 216 38 + 28 38 − 28 =
= (3 + 2 )(3
+ 2 )(3 + 2 )(3 − 2 )=
16 16 8 8 4 4 4 4

= (3 + 2 )(3 + 2 )(3 + 2 )(3 + 2 )(3 − 2 )=


16 16 8 8 4 4 2 2 2 2

= (3 + 2 )(3 + 2 )(3 + 2 )(3 + 2 )(3 + 2 ) ⋅ (3 − 2 ).


16 16 8 8 4 4 2 2

Qeyd edək ki, 3+2=5, 32+22=13, 34+24=97 ədədləri sadə


ədədlərdir. Digər tərəfdən 38 + 28 = 6817 = 401⋅17 olduğundan
bir sadə vuruq da 17 ədədidir.

257. ƏBOB(5n+6, 8n+7) = ƏBOB(5n+6, 3n+1)=


ƏBOB(3n+1, 2n+5)= ƏBOB(2n+5, n−4) = ƏBOB(n−4, n+9)
= ƏBOB(n−4, 13).

13 sadə ədəd olduğundan ƏBOB (n−4; 13) ya 13, ya da 1-dir.


Bu halların hər ikisi mümkündür: n = 1 ⇒ ƏBOB (11;15) = 1,
n = 4 ⇒ ƏBOB (26;39) = 13.

165
m n
258. Viyet teoreminə görə x1 + x2 = − ; x1 ⋅ x2 = 1 − .
2 2
Buradan m = − 2( x1 + x2 ), n = 2 ⋅ (1 − x1 x2 ). Onda
m2 + n2
= ( x1 + x2 ) − (1 − x1 x2 ) =
2 2

4
( )( )
= x12 + x22 + 1 + x12 ⋅ x22 = x12 + 1 x22 + 1 .
x1 ≠ 0 və x2 ≠ 0 olduğundan, bu axırıncı hasil mürəkkəb
m2 + n2
ədəddir. Deməli, ədədi mürəkkəb ədəddir.
4
259 sin x + cos 2 x = A və sin 2 x + cos x = B olsun. Onda
verilmiş cəmləri bir-birinə vursaq:
1 1
A⋅ B = sin 2 x + sin 4 x + sin x ⋅ sin 2 x + cos x ⋅ cos 2 x =
2 2 alarıq.
⋅ 2 sin 3x ⋅ cos x + cos x = (sin 3x + 1) cos x∈Q
1
=
2
Digər tərəfdən
( ) (
A2 + B 2 = sin 2 x + cos 2 x + sin 2 2 x + cos 2 2 x + )
+ 2(sin x ⋅ cos 2 x + cos x ⋅ sin 2 x ) = 2(1 + sin 3 x )∈Q
olur. Onda surət və məxrəc rasional olduğundan
A⋅ B
=
(sin 3x + 1) ⋅ cos x = 1 cos x rasionaldır.
A +B
2 2
2(1 + sin 3 x ) 2
( ) (
Deməli, A = sin x + 2 cos 2 x − 1 ⇒ sin x = A + 1 + 2 cos 2 x ∈Q . )
260. Hər dəfə qalıqda 1 almaqla ədədin 2, 3, 4, 5, 6
ədədlərinə bölünməsi üçün 2, 3, 4, 5, 6 ədədləri üçün ƏKOB
tapıb, üstünə 1 gəlmək lazımdır. Belə ədəd 60+1 = 61

166
olacaqdır. Lakin şərtə görə həmin ədəd 7-yə qalıqsız
bölünməlidir, 61 isə bu şərti ödəmir. Bu şərti ödəyən ədədi 0
tapmaq üçün belə mühakimə aparaq:
60 = 56+4 şəklində yazsaq, 56 ədədi 7-yə qalıqsız bölünür.
İndi 4-ü elə ədədə vurmaq lazımdır ki, hasilin üstünə 1
gəldikdə 7-yə bölünən ən kiçik ədəd alınsın. Bu şərti 5 ədədi
ödəyir. Çünki 4⋅5+1=21. Deməli, (56+4) cəmini 5-ə vurub
üstünə 1 gəlmək lazımdır. Onda (56⋅5)+(4⋅5+1) alarıq.
Toplananların hər biri 7-yə bölünür. Deməli, 280+21 = 301
cəmi də 7-yə qalıqsız bölünür.

261. Şərtə görə ab = a+b. Hər tərəfi kuba yüksəltsək:


a 3 b 3 = ( a + b) 3 . Onda
a 3b3 − a 3b3 = a 3 + b3 − (a + b ) =
3

= a 3 + b3 − a 3 − b3 − 3ab(a + b ) = − 3ab(a + b ) = − 3a 2b 2 .
Bunu nəzərə alsaq:
(a )3
+ b 3 − a 3 b 3 + 27a 6 b 6 = − 3a 2 b 2
3
( )
3
+ 27a 6 b 6 =
= −27 a 6 b 6 + 27a 6 b 6 = 0
olur.
262. Verilən həndəsi silsilədə b = aq, c = bq, d = cq olsun.
(
Onda b 2 + c 2 + d 2 = a 2 q 2 + b 2 q 2 + c 2 q 2 = a 2 + b 2 + c 2 ⋅ q 2 . )
Bərabərliyin hər tərəfini a 2 + b 2 + c 2 ifadəsinə vursaq:
(b 2
)( ) (
+ с2 + d 2 a 2 + b2 + c2 = a 2 + b2 + c2 q 2. )2

b = aq, c = bq, d = cq bərabərliklərini ardıcıl olaraq,


a, b və c -yə vurub, alınan ifadələri tərəf-tərəfə toplasaq:
(
ab + bc + cd = a 2 + b 2 + c 2 ⋅ q. Buradan )
167
(ab + bc + cd )2 = (a 2 + b 2 + c 2 )2 ⋅ q 2 , bunu yuxarıda nəzərə
alsaq: (a 2 + b 2 + c 2 )⋅ (b 2 + c 2 + d 2 )= (ab + bc + cd ) alarıq.
2

263. x − y = a, y − z = b qəbul etsək; z − x = − (a + b ) olar.


Verilmiş ifadəni aşağıdakı kimi çevirsək:
(x − y )7 + ( y − z )7 + (z − x )7 = a 7 + b 7 − (a + b ) =
7

= a 7 + b 7 − a 7 − 7a 6b −  − 7ab 6 − b 7 =
(
= − 7ab a 5 + 3a 4b +  + 3ab 4 + b5 = )
= − 7ab((a 5
) (
+ b + 3ab a + b + 5a 2b 2 (a + b ) =
5 3 3
) )
(
= −7ab(a + b ) a + 2a b + 3a b + 2ab + b
4 3 2 2 3 4
)
alarıq. Axırıncı ifadənin a, b və (a+b)-yə, yəni (x−y), (y−z)
və (z−x)-ə bölünməsi aydın görünür.
Deməli, verilmiş ifadə 7(x − y)(y − z)( z − x ) ifadəsinə
bölünür.

x− y x y
264. Verilmiş ifadəni = −
x2 + y 2 x2 + y 2 x2 + y 2
x y
şəklində yazıb, = cos α , = sin α qəbul
x +y2 2
x + y2
2

edək. Aydındır ki, sin 2 α + cos 2 α = 1 eyniliyi ödənir.


 π π
Onda cos α − sin α = 2 cosα +  ≤ 2 ; α = − olduqda
 4 4
ən böyük qiymət 2 olur.

265. Sistemin II tənliyindən x = y+z−3 alarıq. Bunu


sistemin I tənliyində nəzərə alsaq:

168
y 2 + z 2 + 9 + 2 yz − 6 y − 6 z − y 2 − z 2 = 1,
2 yz − 6 y − 6 z = − 8,
y (z − 3) = 3z − 4,
3z − 4 3z − 9 + 5 5
y= = = 3+ .
z −3 z −3 z −3
5-in tam bölənləri: ±1 və ±5 olduğundan z − 1 = ±1 və
z − 1 = ±5 olar. Onda z1 = 4, z2 = 2, z3 = 8, z4 = −2;
x və y -in uyğun qiymətlərini tapa bilərik.
Cavab: {(9;8;4), (− 3;−2;2), (9;4;8), (− 3; 2; − 2)}.

xxyy =100 x + 100 x + 10 y + y =1100 x + 1 y =11(100 x + y ) =


266.
=11 ⋅ x0 y
olduğundan xxyy ədədi yalnız o halda natural ədədin
kvadratı olar ki, xoy ədədi 11-ə bölünsün və qismət də
natural ədədin kvadratı olsun.
Onda x0 y =100 x + y = (x + y ) + 99 x olduğundan x+y cəmi
11-ə bölünməlidir. Yəni, x + y = 11 olmalıdır. Seçmə yolu ilə
x = 7 və y = 4 alarıq.

267. Verilmiş funksiyanı


x5 x3 6
f (x ) =
1 1 1 1 1 1
+ + = x4 + x2 + + + + + +
x x x x x x x x x
şəklində yazıb, Koşi bərabərsizliyini tətbiq etsək:
1 1 1 1 1 1
x4 + x2 + + + + + + ≥
x x x x x x

169
1 1 1 1 1 1
≥ 8 x4 ⋅ x2 ⋅ ⋅ ⋅ ⋅ ⋅ ⋅ =8
x x x x x x
alarıq. Deməli, verilən funksiyanın ən kiçik qiyməti 8-dir.
1
Bərabərlik halı x 4 = x 2 = , buradan x = 1 olduqda mümkündür.
x

268. x 2 + 3 x − 4 = a və 2 x 2 − 5 x + 3 = b olsun.
Onda 3 x 2 − 2 x − 1= a + b olar. Tənlik aşağıdakı şəkilə düşər:
a 3 + b3 = (a + b ) ⇔ a 3 + b3 = a 3 + 3ab(a + b ) + b3 ⇔
3

⇔ 3ab(a + b ) = 0 .
I hal: a=0 olduqda, x 2 + 3 x − 4 = 0, x1 = −4 və x2 = 1 olur;
3
II hal: b=0 olduqda, 2 x 2 − 5 x + 3 = 0, x3 = 1 və x4 = olur?
2
1
III hal: a+b=0 olduqda 3 x 2 − 2 x − 1 = 0, x5 = 1 və x6 = − olur.
3
 1 3
Cavab: − 4; − ; 1;  .
 3 2

269. Şərtə görə:


(a + b )2 − 2ab = (c + d )2 − 2cd ,
(a + b )2 − (c + d )2 = 2(ab − cd ),
(a + b + c + d )(a + b − c − d ) = 2(ab − cd )  2 .
a + b = x və c + d = y qəbul etsək: ( x + y )( x − y )  2 .
Əgər x− y cütdürsə, x + y cəmi 2-dən böyük cüt ədəddir.
Deməli, x + y = a + b + c + d ədədi mürəkkəbdir.

170
270. Axtarılan ədəd xy olsun. Onda şərtə görə
xy = x 2 + y 3 ,
10 x + y = x 2 + y 3 ,
10 x − x 2 = y 3 − y,
x(10 − x ) = ( y − 1) y ( y + 1)
alarıq. Sağ tərəf 3 ardıcıl ədədin hasili olduğundan, 6-ya
bölünür. Onda x(10 − x ) hasilinin 6 -ya bölünməsi üçün ya
x 6, ya da (10 − x ) 6 olmalıdır. 0< x ≤ 9 şərtini nəzərə
alsaq: x = 6 və ya x = 4 olar. Bunları nəzərə alsaq:
( y − 1) ⋅ y( y + 1) = 4 ⋅ 6,
( y − 1) ⋅ y( y + 1) = 2 ⋅ 3 ⋅ 4.
Buradan y = 3 alarıq. Deməli, xy = 43 və ya xy = 63 olur.

271. Həmin ədəd 7 xy olsun. Onda sonra alınmış ədəd xy 7


olar. Şərtə görə 7 xy − xy 7 =117,
(
700 + xy − 10 ⋅ xy + 7 =117,)
9 ⋅ xy = 576,
xy = 64 alarıq.
Deməli, əvvəlki ədəd 7 xy = 764 olur.

272. Tutaq ki, elə x∈Q var ki, α β = x -dir.


β
Onda logα α = logα x, β = logα x şəklində ola bilər.
α = 2 və x = 3 götürsək: β = log 2 3 olar. β = log 2 3
ədədinin irrasional olub - olmamasını göstərək.

171
p
Tutaq ki, log 2 3 = ∈Q , burada p, q∈N. Onda
q

( 2) ( 2) =3 , 2
p
p
= 3, = 9q .
q q p

Burada sol tərəf cüt, sağ tərəf isə təkdir. Ziddiyət aldıq.
Deməli, β = log 2 3 irrasionaldır. Onda
log 3
αβ = 2 2
= 3∈Q .

273. Aydındır ki, x ≠ 49, x ≠ 50. x − 49 = t əvəz etsək:


t t −1 49 50 49t + 50(t − 1) 49t + 50(t − 1)
+ = + , = ,
50 49 t −1 t 50 ⋅ 49 t (t − 1)
99t − 50 99t − 50
= 2 ,
50 ⋅ 49 t −t
(99t − 50)(t 2 − t ) − (99t − 50) ⋅ 50 ⋅ 49 = 0,
(99t − 50)(t 2 − t − 50 ⋅ 49)= 0.
Buradan 99 t − 50 = 0 və ya t 2 − t − 50 ⋅ 49 = 0 ;
50
t1 = , t 2 = 50, t 3 = − 49 .
99
50 50
Onda əvəzləmədən x = 49 + t və x1 = 49 + = 49 ;
99 99
x2 = 49 + 50 = 99, x3 = 49 + (− 49 ) = 0 alarıq.

274. Axtarılan ədəd xy olsun. Onda şərtə görə


xy = x + y 2 alarıq. Buradan 10 x + y = x + y 2 ,
9 x = y 2 − y,
y ( y − 1)
x= .
9

172
Şərtə görə x, y∈N; 0 < x ≤9 və 0< y ≤ 9 olduğunu nəzərə
y ( y − 1)
alsaq: x = ∈ N olması üçün yalnız y = 9 -dur.
9
Onda x = 8 olur. Deməli, xy = 89.

275. 50-yə bölünməyən ədədləri 50-yə böldükdə qalıqda 1-


dən 49-a qədər (49 da daxil olmaqla) olan ədədlərdən biri
alınar. Yəni, 49 müxtəlif ədəd ola bilər. Lakin verilən
ədədlərin sayı isə 50 olduğundan həmin qalıqlardan biri
təkrarlanacaqdır.
276. Məsələnin şərtindən görünür ki, kiçik və böyük ədəd
cüt ədəddir. Ona görə də bu ədədlərdən biri 4-ə bölünür.
Onda həmin ədədlərin hasili də 8-ə bölünməlidir.
Digər tərəfdən üç ardıcıl ədəddən biri həmişə 3-ə bölünür.
Deməli, həmin ədədlərin hasili 8⋅3 = 24-ə bölünəcəkdir.
277. Ədədin 2-yə, 4-ə, 5-ə, 7-yə və 8-ə bölünüb, qalıqda 1
olması üçün həmin ədəd 5⋅7⋅8n+1 və ya 280n+1 şəklində
olmalıdır. Burada n-i elə seçmək lazımdır ki, 280n+1 ifadəsi
9-a bölünən ən kiçik ədəd olsun. Bu qayda ilə n=8 alarıq.
Beləliklə, axtarılan ədəd 2241 olur.

278. Tənliyin kökləri x1 , x2 , x3 , x4 , x5 , x6 olsun. Onda


 x1 + x2 +  + x6 = 6,
Viyet teoreminə görə 
 x1 x2 x3  x6 =1
Ədədi orta və həndəsi orta haqqında bərabərsizliyə görə
x1 + x2 +  + x6 6
≥ x1 ⋅ x2  x6 ,
6

173
6 6
buradan ≥ 1 ⇒ 1 ≥ 1 olduğundan x1 = x2 = x3 = = x6 =1 olur.
6
Onda
(x − 1) = 0 , (x − 1)2 = 0,, (x − 1)6 = 0;
(x − 1)6 = x 6 − 6 x5 + 15 x 4 − 20 x3 + 15 x 2 − 6 x + 1= 0.
↓ ↓ ↓ ↓
a b c d
Deməli, a =15, b = −20, c =15, d = −6 -dır.

279. a + b = 2 − c və a 2 + b 2 = 2 − c 2 . Koşi-Bunyakovski
bərabərsizliyinə görə
(a + b )2 = (a ⋅ 1 + b ⋅ 1)2 ≤ (12 + 12 )⋅ (a 2 + b 2 )= 2(a 2 + b 2 ).
Onda (2 − с )2 ≤ 2(2 − с 2 ), 4 + с 2 − 4с ≤ 4 − 2с 2 ,
3с 2 − 4с ≤ 0 ,
4 4
buradan isə 0 ≤ с ≤ ⇒ сmax = alarıq.
3 3
280. Qeyd edək ki,
1 − a 2
1 + a = − 1 + 1 + a ,

1 − b 2 1− a 1− b 1− c
 = − 1+ ⇒ + + =
1 + b 1+ b 1+ a 1+ b 1+ c
1 − c 2
 = −1+ =
1 + c 1+ c
 1 1 1 
= − 3 + 2 + + .
1+ a 1+ b 1+ c 

174
Digər tərəfdən 1+a, 1+b, 1+c üçün
1+ a +1+ b +1+ c 3
≥ , buradan
3 1 1 1
+ +
1+ a 1+ b 1+ c
1 1 1 9
+ + ≥ .
1+ a 1+ b 1+ c 4
1− a 1− b 1− c 9 3
Onda + + ≥ − 3 + 2 ⋅ = olur.
1+ a 1+ b 1+ c 4 2

281. Tənliyin sol tərəfi müsbətdir. Onda x − 4 > 0 ⇔ x > 4 olur.


(4;+∞) aralığında f ( x ) = 2 x 2 − 16 + x 2 − 9 funksiyası
artır, g ( x ) = isə azalır. Onda f ( x ) = g ( x ) tənliyinin
10
x−4
kökü varsa, həmin kök yeganədir. Deməli, x = 5 tənliyin
yeganə köküdür.
282. Məlumdur ki, ( a − 1) ≥ 0,
2
( b − 1) ≥ 0,
2

( c − 1) ≥ 0,
2
( d − 1) ≥ 0.
2

Buradan
a + 1 ≥ 2 a , b + 1 ≥ 2 b , c + 1≥ 2 c , d + 1 ≥ 2 d .
Bunları tərəf-tərəfə toplasaq:
a+
b + c
+ d + 4≥ 2 a + b + c + d ;
8

(
2 a + b + c + d ≤12; )
a + b + c + d ≤6

283. Məlumdur ki, a 2 + c 2 ≥ 2ac, b 2 + d 2 ≥ 2bd .


Tərəf-tərəfə toplasaq:
175
+
a2b 2 + c
2
+ d2 ≥ 2ac + 2bd ,

1 1

2 ≥ 2(ac + bd ) ⇔ ac + bd ≤1.

284. a + b + c + d = 8 − e və a 2 + b 2 + c 2 + d 2 = 16 − e 2
a+b+c+d = 1⋅a+1⋅b+1⋅c+1⋅d nəzərə alsaq,
Koşi-Bunyakovski bərabərsizliyinə görə
(a + b + c + d )2 ≤ (12 + 12 + 12 + 12 )⋅ (a 2 + b 2 + c 2 + d 2 ),
(8 − e)2 ≤ 4 ⋅ (16 − e 2 ), 64e 2 − 16e ≤ 64 − 4e 2 ,
5e 2 − 16e ≤ 0,
16 16
buradan 0 ≤ e ≤ və emax = alarıq.
5 5

285. Koşi-Bunyakovski bərabərsizliyinə görə


(x1 + x2 +  + xn ) = (1⋅ x1 + 1⋅ x2 +  + 1⋅ xn )2 ≤
( )( ) (
≤ 12 + 12 + 12 ⋅ x12 + x22 +  + xn2 = n ⋅ x12 + x22 +  + xn2 . )
Buradan ( x1 + x2 +  + xn ) ≤ n(x12 + x22 +  + xn2 ).
2

286. Ədədi orta və həndəsi orta haqqında bərabərsizliyə görə


1+ 2 + 3 ++ n
n
1⋅ 2 ⋅ 3 n ≤ ;
n
n(n + 1)  n +1
n
n n! ≤ ⇔ n !≤   .
2n  2 
287. Ədədi orta və harmonik orta arasındakı bərabərsizliyə
görə

176
a1 + a2 +  + an n
≥ , buradan
n 1 1 1
+ ++
a1 a2 an
 1
(a1 + a2 +  + an ) ⋅  1 +
1
+  +  ≥ n 2 alarıq.
 a1 a2 an 

288. Kökaltı ifadəni çevirək:


2

( )  2011 
2011 + 2 ⋅ 2011 + 1 − 2 ⋅ 2011 + 
2
 =
 2012 
2 2
2011  2011   2011 
= 2012 2 − 2 ⋅ 2012 ⋅ +  =  2012 −  .
2012  2012   2012 
Onda verilən ifadənin qiyməti
 2011  2011
 2012 − + = 2012 olar.
 2012  2012
289. Çevirmə aparsaq, sol tərəf
( ) (
x 4 + 2 x 3 + 2 x 2 + 2 x + 1= x 4 + 2 x 3 + 2 x 2 + x 2 + 2 x + 1 = )
= x (x + 2 x + 1) + (x + 2 x + 1) = (x + 1)(x + 1)
2 2 2 2 2

şəklinə düşər. Onda (x + 1) ⋅ ( x + 1) = y alarıq.


2 2 2

Tənliyin sağ tərəfi tam kvadrat olduğundan sol tərəfdə tam


kvadratdır. Bu iki halda mümkündür:
1) х 2 + 1 tam kvadratdır, onda x = 0 və y2 = 1;
2) х + 1 = 0, onda x = −1, y = 0.
Deməli, x = 0, y = ±1 və ya x = −1, y = 0 olur.

290. 100017 ≡ −117 ( mod 1001) ,

177
99917 ≡ −217 ( mod 1001) ,
99817 ≡ −317 ( mod 1001) ,

1 ilə −1 , 2 ilə −217 bir-birini islah edir və i.a.
17 17 17

Deməli, verilən cəm 1001 moduluna görə sıfıra bərabərdir:


117 + 217 + 317 +  + 100017 ≡ 0 (mod 1001).
Bununla da isbat tamam oldu.

291. [x ]= a, {x}= b qəbul edək. Onda x = a+b olur.


Bunları nəzərə alsaq, verilmiş bərabərsizlik aşağıdakı şəkilə
düşər: ab < a + b − 1 ⇔ (a − 1)(b − 1)< 0.
0 ≤ {x}<1 olduğundan b < 1 ⇔ b − 1 < 0 .
Onda a − 1 > 0 ⇔ a > 1, yəni [x ]>1. Aydındır ki, x ≥ 2 olur.

292. 2011 ⋅ 211 + 2500 = 2011 ⋅ ( 2011 + 100 ) + 2500 =


= 20112 + 2 ⋅ 2011 ⋅ 50 + 50= ( 2011 + 50=
)
2 2
20612.
Deməli, verilmiş ədəd sadə deyil.

293. Aydındır ki, x ≠ 3. Hər tərəfi (x−3)2 -na bölsək:


2
 3x 
x +  = 16 alarıq.
2

 x −3
a 2 + b 2 = (a + b ) − 2ab eyniliyini tətbiq etsək:
2

2
 3x  3x
x+  − 2x ⋅ =16,
 x −3 x −3

178
2
 x2  x2
  − 6 ⋅ =16.
 x −3 x −3
x2 t 2 − 6t − 16 = 0
= t qəbul edək. Onda:
x−3 t1 = −2; t2 = 8.
x2
1) = − 2 ⇒ x 2 + x − 6 = 0 buradan x1, 2 = −1 + 7 .
x−3
x2
2) = 8 ⇒ x 2 − 8 x + 24 = 0 bu halda isə həll yoxdur.
x −3

294. Verilən nisbətlərə əsasən: y = 2x, z = 3x, t = 4x olar.


Bunları II tənlikdə nəzərə alsaq:
9 x + 14 x + 9 x + 8 x = 200,
40 x = 200, x = 5 alınır.
Onda y = 2 ⋅ 5 = 10, z = 3 ⋅ 5 =15, t = 4 ⋅ 5 = 20 olur.

295. Ədədi orta və həndəsi orta arasındakı bərabərsizliyə


görə x + y + z ≥ 33 xyz .
Digər tərəfdən 3 = log3 x + log3 y + log3 z = log3 ( xyz )
olduğundan x y z = 27. Onda x + y + z ≥ 3 3 27 = 9.
Deməli, x + y + z ≥ 9 olacaqdır.

296. Tutaq ki, şərti ödəyən ədəd var. Onda onun rəqəmləri
cəmi: 1 ⋅ 1 + 2 ⋅ 2 + 3 ⋅ 3 +  + 9 ⋅ 9 = 285 olur.
Bu isə 3-ə bölünür, lakin 9-a bölünmür. Deməli, şərti ödəyən
ədəd yoxdur.

179
297. Aydındır ki, iki qonşu natural ədədin cəmi tək ədəddir.
Onda 2002 ardıcıl natural ədədin cəmi, 1001 tək ədədin cəmi
olur ki, bu da həmin cəmin tək olması deməkdir. Deməli,
həmin cəm 2002-yə bölünə bilməz.

298. 32 x − y + 6 ≥1 və 5 x + y − 3 ≥1 olduğundan verilmiş bərabərlik


32 x − y+6 = 1,
yalnız  x + y−3 olduqda mümkündür. Onda
5 =1
2 x − y + 6 = 0  x = −1,
 ⇒ olur.
x + y − 3 y = 4

299. Məlum a 2 + b 2 ≥ 2ab, b 2 + c 2 ≥ 2 bc, c 2 + a 2 ≥ 2 ca


bərabərsizliklərini nəzərə alsaq:
1 1 1 1 1 1
+ + ≥ + + .
1 + 2ab 1 + 2bc 1 + 2ca 1 + a + b 1 + b + c 1 + c + a 2
2 2 2 2 2

Ədədi-harmonik orta bərabərsizliyini tətbiq etsək:


1 1 1
+ + ≥
1+ a + b 1+ b + c 1+ c + a2
2 2 2 2 2

3
≥ 3⋅
(1 + a + b ) + (1 + b 2 + c 2 ) + (1 + c 2 + a 2 ) =
2 2

9 9
= = =1 alarıq.
(
3+ 2 a +b + c
22 2
)
3 + 2⋅3
Əvvəl və axırı birləşdirsək, verilən bərabərsizliyi isbat etmiş
oluruq.
300. Üçbucağın katetləri a, b, hipotenuzu c olsun.
Onda şərtə görə b = a+d, c = a+2d (d >0 silsilə fərqidir).
Pifaqor teoreminə görə

180
( a + 2d ) 2 = a 2 + ( a + d ) 2 ,
a 2 + 4ad + 4d 2 = 2a 2 + 2ad + d 2 ,
a 2 − 2ad + 3d 2 = 0.
Buradan a = 3d, onda b = 4d, c = 5d olur.
a+b+c
S = ab və S = ⋅ r düsturlarından istifadə etsək:
2
2S ab 3d·4d 12d 2
r= = = = =d.
a + b + c a + b + c 3d + 4d + 5d 12d
Beləliklə, r = d aldıq.
301. 11
1 = n işarə edək. Onda verilmiş ədədi
10

( )
n ⋅ 10 + 2n = n 1010 − 1 + 3n = n ⋅ 9n + 3n = (3n ) ⋅ (3n + 1)
10

yazmaq olar. Bununla da isbat tamam oldu.

302. a(b + c ) + b(c + a ) + c(a + b ) − 4abc =


2 2 2

= ab 2 + ac 2 + bc 2 + ba 2 + c(a + b ) =
2

( ) ( )
= ab 2 − ba 2 + ac 2 + bc 2 + c(a + b ) =
2

( )
= (a + b ) ab + c 2 + ac + bc = (a + b )(b + c )(c + a ).

303. Verilən bərabərliyin sol tərəfi 9 -a bölündüyündən sağ


tərəfin də, yəni 194 y -in 9 -a bölünməsi üçün yalnız y = 4
olur. Bunu verilən bərabərlikdə yerinə yazsaq:
9( x − 2) = 1944, buradan (x−2)3 = 216,
3

x − 2 = 6 və x = 8 alarıq.
304. Diqqət yetirsək:

181
H1 =1 =13 − 03 ; H 2 = 7 = 23 − 13 ;
a)
H 3 = 19 = 33 − 23 ; H 4 = 37 = 43 − 33 ;
və sairə H n = n3 − (n − 1) alarıq.
3

H1 + H 2 + H 3 +  + H n =
b)
= 13 + 23 − 13 + 33 − 23 + 43 − 33 +  + n3 − (n − 1) = n 3
3

305. Əvvəlcə 5 müxtəlif natural ədəddən ibarət istənilən


ədədi silsilə götürək. Məsələn: 1, 2, 3, 4, 5. Bu ədədlərin
hasili 120-yə bərabərdir və natural ədədin 2008-ci qüvvəti
deyil. Hər bir həddi 120n-ə vursaq:
120n , 2 ⋅ 120n , 3 ⋅ 120n , 4 ⋅ 120n , 5 ⋅ 120n alarıq.
Bu ədədlər də əvvəlki kimi ədədi silsilə əmələ gətirir və
onların hasili 1205n+1-ə bərabərdir. İndi n-i elə seçək ki, 5n+1
ədədi 2008 -ə bölünsün. Yəni, 5n + 1 = 2008 m, buradan,
məsələn, m = 2 və n = 803 alarıq.
Onda həmin hasil: 1205 n+1 =120 2008⋅2 = (120 2 )
2008
olacaqdır.

306. x, y, z ∈ [1; 2] olmaqla, aşağıdakı bərabərsizlikləri


düzəldək:
(x − 1)( y − 2) + ( y − 1)(x − 2) ≤ 0,
( y − 1)(z − 2) + (z − 1)( y − 2) ≤ 0,
(z − 1)(x − 2) + (x − 1)(z − 2) ≤ 0 .
x2 2 x2 y2 2 y2 z2 2 z2
Buradan ≥ ⋅ ; ≥ ⋅ ; ≥ ⋅ ;
yz + 2 3 y + z zx + 2 3 z + x yx + 2 3 x + y
x2 y2 z2
+ + ≥ x + y + z olduğunu isbat etməliyik.
y+z x+z y+z

182
x2 y2 z2 x y z x+ y+z
+ + = + + ≥ .
y+z x+z y+z y z z x x y
+ + +
2
x x y y z z
Deməli, isbat tamam oldu.

307. Koşi-Bunyakovski bərabərsizliyinə görə


c 2 + ab ≤ c 2 + a 2 ⋅ c 2 + b 2 ,
a 2 + bc ≤ a 2 + b 2 ⋅ a 2 + c 2 ,
b 2 + ca ≤ b 2 + c 2 ⋅ b 2 + a 2
olduğundan ədədi və həndəsi orta haqqında teoremə görə
a 2 + b2 b2 + c2 c2 + a 2
+ + ≥
c 2 + ab a 2 + bc b 2 + ca
a 2 + b2 b2 + c2 c2 + a2
≥ + + ≥
c2 + a 2 ⋅ c2 + b2 a 2 + b2 ⋅ a 2 + c2 b2 + c2 ⋅ b2 + a 2

a2 + b2 b2 + c2 c2 + a2
≥ 33 ⋅ ⋅ = 3.
c2 + a2 ⋅ c2 + b2 a2 + b2 ⋅ a2 + c2 b2 + c2 ⋅ b2 + a2
Beləliklə, isbat tamam oldu.
f (1)
308. y = x olsun, onda x 2 f (x ) = f (1) ⇒ f (x ) = olar.
x2
F(1) istənilən müsbət həqiqi ədəddir.

Deməli, f ( x ) = 2 , burada k ∈R + .
k
x
1 + 2 + 3 +  + 2011
309. 2011! = 2011 1 ⋅ 2 ⋅ 3 ⋅⋅ 2011 < = 1006 .
2011
Buradan, 2011! <10062011 alarıq.

183
310. D(arccos x ) = [− 1;1] və D(lg x ) = (0;+∞ ) olduğundan
tənliyin sol tərəfinin təyin oblastı (0;1] olur. Sağ tərəfin təyin
oblastı isə x − 2≥ 0 və ya x ≥ 2 -dir. Onda x ≤ 1 və x ≤ 2
bir-birinə ziddir. Deməli, verilmiş tənliyin həlli yoxdur.

311. Həmin ədəd xy olsun. Onda şərtə görə


xy + yx = n 2 , n∈ N .
11( x + y ) = n 2 ,
n2
x+ y=
∈N .
11
Buradan, 1≤ x ≤ 9 və 1≤ y ≤ 9 onda, n =11 və x + y =11
alarıq. Onda xy∈{29, 38, 47, 56, 65, 74, 83, 92} olur.

6 8
312. Verilən tənliyi x 2 + y 2 + 1= x + y şəklində yazaq.
5 5
Buradan
 2 3 3 3   2
2 2
4
2
4 4
2

 x − 2 ⋅ x +   −    +  y − 2 ⋅ y +   −   + 1 =0,
 5 5 5   5 5 5 
2 2
 3 9  4  16
 x −  − +  y −  − +1 = 0,
 5  25  5  25
2 2
 3  4
x−  + y−  =
0,
 5  5
3 4
Buradan x = və y = aldıq.
5 5

184
Deməli, tənliyin yalnız  3 , 4  həlli var.

5 5 
313. Viyet teoreminə görə
a + b + c = −3, ab + bc + ca = −7, abc = −1.
(a + b + c )2 = a 2 + b 2 + c 2 + 2(ab + bc + ca ) eyniliyinin
nəzərə alsaq: a 2 + b 2 + c 2 = (a + b + c ) − 2(ab + bc + ca ) =
2

= (− 3) − 2 ⋅ (− 7 ) = 9 + 14 = 23 taparıq.
2

314. Məlum bərabərsizliklərə görə


1 + a1 ≥ 2 a1 ,
2 + a2 ≥ 2 2a2 ,
3 + a3 ≥ 2 3a3 ,

n + an ≥ 2 nan .

(1 + a1 )(2 + a2 )(3 + a3 )(n + an )≥


.
≥ 2n (1 ⋅ 2 ⋅ 3 n ) ⋅ (a1 ⋅ a2 ⋅ a3  an )

(1 + a1 )(2 + a2 )(3 + a3 )(n + an )≥ 2n n! .

315. Verilmiş ifadəni aşağıdakı kimi çevirsək:


( ) (
29 + 299 = 29 1 + 290 = 29 1 + 210( ) )=
9

( )
= 29 110 + 102410 = 29 ⋅ (1 + 1024 ) ⋅ A =
= 2 ⋅ 1025 ⋅ A
9

hasilini alarıq. Sonuncu hasil həm 4-ə, həm də 25-ə bölünür.


Deməli, 29+299 cəmi 4⋅25 = 100 -ə bölünür.
185
316. Sistemi aşağıdakı şəkildə yazaq:
 х + у = 2,

 хy = 1 + z .
2

Viyet teoreminin tərsinə görə x və y ədədləri


t 2 − 2t + 1 + z 2 = 0
tənliyinin kökləridir. Onda (t − 1) + z 2 = 0 , buradan
2

t − 1= 0, t = 1,
 ⇒  və t1 = x = 1, t2 = y = 1 alarıq.
z = 0 z = 0
Onda verilmiş sistemin tam həlli: (1; 1; 0) üçlüyü olur.

317. Qeyd edək ki,


111
 11 = 111

  11⋅ 1 00

 
001 00
 
001.

27 9 8 8

Birinci vuruq 9-a, ikinci vuruq isə 3-ə bölünür. Buna görə də
bu hasil 27-yə bölünür.

318. Qeyd edək ki,


ab ⋅ cd ≤ ab ⋅ 99 < ab ⋅100 < ab00 + cd = abcd .
Deməli, şərti ödəyən ikirəqəmli ədədlər yoxdur.

319. Verilmiş ifadəni vuruqlara ayıraq:


( ) (
n3 + 3n 2 + 6n + 8 = n3 + 8 + 3n 2 + 6n = )
= (n + 2 )(n − 2n + 4 ) + 3n(n + 2 ) = (n + 2 )(n
2 2
)
+n+4 .
Aldığımız hasildəki vuruqların hər biri n-in istənilən natural
qiymətində 1-dən böyükdür. Deməli, verilmiş ədəd mürəkkəb
ədəddir.

186
ab + ac + bc = 3 ⇔ (ab + bc + ca ) = 9,
2
320.
(
a 2b 2 + b 2c 2 + c 2 a 2 + 2 a 2bc + ab 2c + abc 2 = 9 , )
a 2b 2 + a 2c 2 + b 2c 2 + 2 ⋅ abc(a + b + c ) = 9 ,
a 2b 2 + a 2c 2 + b 2c 2 + 2 ⋅ 9 ⋅ (− 5) = 9 ,
a 2b 2 + a 2c 2 + b 2c 2 = 99 .
Axırıncı bərabərliyin hər tərəfini abc = 9 ilə tərəf-tərəfə
ab ac bc
bölsək: + + = 11 alarıq.
c b a

321. 20000 = 2 ⋅ 10 4 = 2 ⋅ 54 ⋅ 2 4 olduğundan


58 + 28 + 20000 = 58 + 2 ⋅ 54 ⋅ 2 4 + 28 =

= (5 4
+ 24 )
2
= 54 + 2 4 = 625 + 16 = 641.
322. Qeyd edək ki,
x4 + x2 + 1 = x4 + 2x2 + 1 − x2 = x2 + 1 − x2 = ( ) 2

( )(
= x2 − x + 1 x2 + x + 1 . )
Onda

4= =
( =
)(
x4 + x2 + 1 x2 − x + 1 x2 + x + 1 x2 + x + 1 x2
= +1
)
x3 + 1 (x + 1) x 2 − x + 1 (x +1 x +1 )
x2 x2 x +1 1
olar. Buradan + 1= 4 ⇒ =3 ⇒ 2 = ⇒
x +1 x +1 x 3
1 1 1
⇒ + 2 = alarıq.
x x 3

323. Aşağıdakı kimi mühakimə aparaq:

187
7 !>1000 ⇒ abc < 700;
6 != 720 > 700 > abc ⇒ a < 6, b < 6, c < 6;
abc ≤ 555 olarsa, 5!+5!+5! = 360 ≠ 555;
abc ≤ 355 olarsa, 3!+5!+5! = 246 ≠ 355;
abc ≤ 245 olarsa, 2!+4!+5! = 146 ≠ 245;
abc ≤145 olarsa, 1!+4!+5! = 145, bu şərti ödəyir.
Deməli, abc = 145, buradan a=1, b=4, c=5 olur.

324. Verilmiş tənliyi aşağıdakı kimi çevirək:


x 5 − 2 x 2 = − 3,
( )
x 2 x 3 − 2 = − 3.
Qeyd edək ki, − 3 = 1 ⋅ (− 3) və ya − 3 = 3 ⋅ (− 1).
 x 2 = 1,  x = ±1,
Onda 1)  3 ⇒ 3 ⇒ x = −1;
 x − 2 = −3  x = −1
 x 2 = 3, x = ± 3
2)  3 ⇒ ⇒ x∈∅.
 x − 2 = −1  x = 1
Deməli, tənliyin tam həlli x =1-dir.

325. − 1 ≤ sin x ≤ 1 olduğundan − 1 ≤ [x ] ≤ 1 olur.


Buradan [x ]= − 1; [x ] = 0 və [x ]=1 .
π
1) [x ]= − 1 ⇒ − 1≤ x < 0, onda sin x = − 1⇒ x = − ∉[− 1, 0),
2
bu halda həll yoxdur;
2) [x]= 0 ⇒ 0 ≤ x < 1, onda sin x = 0 ⇒ x = 0 ∈[0;1);

188
π
3) [x]=1 ⇒ 1 ≤ x < 2, onda sin x =1 ⇒ x = ∈[1;2 ).
2
 π
Deməli, x∈0;  .
 2
326.. Hər bir toplananı aşağıdakı kimi qiymətləndirək:
a a a + (2a + b ) 3a + b
≤ ≤ =
1 + b + a 2c + b 2c + b + (2a + b ) 2(a + b + c )
2

burada 1 + c 2 ≥ 2c nəzərə alınmışdır.


b 3b + c
Analoji olaraq: ≤
1+ c + a 2
2(a + b + c )
с 3c + a
≤ .
1+ a + b 2
2(a + b + c )
Aldığımız 3 bərabərsizliyi tərəf-tərəfə toplasaq:
a b c
+ + ≤
1 + b + c 1 + c + a 1 + a + b2
2 2

3a + b + 3b + c + 3c + a 4a + 4b + 4c
≤ = = 2.
2(a + b + c ) 2(a + b + c )
Beləliklə, isbat tamam oldu.

1
327. Asanlıqla yoxlamaq olar ki, və 1 cx 2 + bx + a = 0
p1 p2
tənliyinin kökləridir. Şərtdəki silsilə ardıcıllığını nəzərə alaq.
1 1 1
Əgər q1 = olarsa, onda p1 + = p2 + və buradan
p2 p1 p2
p1 = p2 , ya da p1 = q1; hər iki hal köklərin fərqli olmasına

189
1 1 1
ziddir. Əgər q2 = , onda p1 − = p2 − . Buradan
p2 p1 p2
p1 = p2 (yenə ziddiyyətdir), ya da
1 c
p1 = − ⇒ p1 ⋅ p2 = − 1 = ⇒ a + c = 0 .
p2 a
tələb olunan isbatı aldıq.

328. Koşi bərabərsizliyinə görə

x2 + x − 1 ≤
(x 2
)
+ x − 1 + 1 x2 + x
= ;
2 2

1 + x − x2 ≤
(1 + x − x ) + 1 = 2 + x − x
2 2
.
2 2
Onda x 2 + x − 1 + 1 + x − x 2 ≤ x + 1 olur.
Verilmiş tənlikdə bunları nəzərə alsaq:
x 2 − x + 2 ≤ x + 1,
x 2 − 2 x + 1≤ 0,
(x − 1)2 ≤ 0 ⇒ x = 1.
Deməli, x =1 verilmiş tənliyin yeganə həllidir.

329. Üçbucağın sahə düsturlarını:


abc a+b+c
S∆ = , S ∆ = pr və p =
4R 2

nəzərə alsaq: abc = a + b + c ⋅ r. Buradan


abc
= 2Rr ,
4R 2 a+b+c
abc
R ≥ 2 r olduğundan ≥ 4r 2 alarıq.
a+b+c
190
330. Sinuslar teoreminə görə
a = 2 R ⋅ sin α , b = 2 R ⋅ sin β , c = 2 R ⋅ sin γ .
Hər bir bərabərliyi kvadrata yüksəldib, tərəf-tərəfə toplasaq:
( )
a 2 + b 2 + c 2 = 4 R 2 sin 2 α + sin 2 β + sin 2 γ =
= 8R (1 + cos α ⋅ cos β ⋅ cos γ );
2

olduğundan a 2 + b 2 + c 2 ≥ 9 R 2 , (R ≥ 2r ).
1
cos α ⋅ cos β ⋅ cos γ ≤
8
Onda a 2 + b 2 + c 2 ≥18 Rr alarıq.

331. a(b + c )2 = a (b 2 + 2bc + c 2 ) = ab 2 + 2abc + ac 2 ,


( )
b(a + c ) = b a 2 + 2ac + c 2 = a 2b + 2abc + bc 2 ,
2

( )
c(a + b ) = a 2 + 2ab + b 2 = a 2 c + 2abc + b 2 c.
2

Bunları nəzərə alsaq:


a(b + c ) + b(a + c ) + c(a + b ) − 3abc =
2 2 2

= ab 2 + ba 2 + ac 2 + ca 2 + bc 2 + cb 2 − 3abc =
= (ab(a + b ) + abc ) + (ac(a + c ) + abc ) + (bc(b + c ) + abc ) =
= ab(a + b + c ) + ac(a + b + c ) + bc(b + c + a ) =
= (a + b + c )(ab + ac + bc )
alarıq.

332. P(x) çoxhədlisinin kökləri x1, x2, x3 olsun. Onda Viyet


teoreminə görə x1 + x2 + x3 = − 6 və şərtə görə x1 + x3 = 2x2
alarıq. Buradan 3x2 = −6 ⇔ x2 = − 2 .
Deməli, P(x) çoxhədlisinin bir kökü (−2) -dir. Onda
P(− 2) = 0 ⇔ − 8 + 24 − 10 + a = 0 ⇔ a = −6 alarıq.

191
Beləliklə, P( x ) = x 3 + 6 x 2 + 5 x − 6 olur.
( )
P( x ) = ( x + 2 ) x 2 + 4 x − 3 olduğundan köklər
x1 = −2, x2 = − 2 + 7 , x3 = − 2 − 7 olur.

1 2 2010
333. + ++ =
2! 3! 2012
2 1 3 1  2012 1 
=  −  +  −  +  +  −  =
 2! 2!   3! 3!   2012 ! 2012 ! 
 1 1 1  1 1 
= 1 −  +  −  +  +  −  =
 2!   2! 3!   2011! 2012 ! 
1
= 1− < 1.
2012 !

334. Qeyd edək ki, 10m −1 < 21000 < 10m və 10n −1 < 51000 < 10n.
Bu bərabərsizlikləri hədbəhəd vursaq:
10m + n − 2 <101000 <10m + n alarıq.
Buradan m + n − 1<1001< m + n + 1 və m + n = 1001 alınır.
2

335.  x +  = x 2 + 2 + 2 eyniliyindən x + 1 = 3 alarıq.


1 1
 x x x
Hər tərəfi kuba yüksəltsək:
3
 1 1 1 1 1
27 =  x +  = x 3 + 3x ⋅  x +  + 3 = x 3 + 3 + 3 ⋅ 3.
 x x x x x
1 1 1
Buradan x 3 + = 27 − 9 =18 . x2 + 2
= 7 və x 3 + 3 = 18
x3 x x
bərabərliklərini tərəf-tərəfə vuraq:

192
 1  1 1 1 1
7 ⋅18 =  x 2 + 2  x 3 + 3  = x 5 + x 2 ⋅ 3 + 2 ⋅ x 3 + 5 =
 x  x  x x x
1 1 1
= x5 + 5 + x + = x5 + 5 + 3 ,
x x x
1
buradan x 5 + =126 − 3 =123 olur.
x5
336. Verilmiş ifadəni, n!(n + 2 ) = n!(n + 1 + 1) = (n + 1)!+ n!
eyniliyinə əsasən aşağıdakı kimi yazsaq:
S = 2!+ 1!−3!−2!+4!+3!−5!−4!+  + 2000!+
+ 1999!−2000!− 2000!+ 2001!=1
alarıq.

337. Tutaq ki, α iti bucaq və tgα = 0,2 -dir.


2tgα 2 ⋅ 0,2 5
Onda tg 2α = = = və
1 − tg α 1 − 0,2 2 12
2

2tg 2α 120
tg 4α = = >1 olur.
1 − tg 2α 119
2

Buradan tg 4α >1 ⇒ tg 4α > tg 45o ⇒ 4α > 45o ⇒ α > 11o.


Onda tg11o < tgα = 0,2 və tg11o < 0,2 alarıq.
1
338. Tutaq ki, α − iti bucaq və sin α = -dir. Onda
6
3
1  1  1 1 13 13 1
sin α = 3 sin α − 4 sin α = 3 ⋅ − 4 ⋅   = − = < =
3

6  6  2 54 27 26 2
1 1
və sin 3α < olur. sin 3α < = sin 30 o ⇒ 3α < 30 o ⇒ α < 10 o.
2 2

193
1
Beləliklə, sin α < sin 10 o və buradan sin 10 o > alırıq.
6
339. Bərabərsizliyin sol tərəfini aşağıdakı kimi çevirək:
x6 + x 4 + x 2 + 2 x3 + 2 x + 2 x5 + x 4 + x 2 + 1 =
( ) ( ) ( )
= x6 + 2 x5 + x 4 + x 4 + 2 x3 + x 2 + x 2 + 2 x + 1 =
= x (x + 2 x + 1) + x (x + 2 x + 1) + (x + 2 x + 1) =
4 2 2 2 2

= (x + 2 x + 1)(x + x + 1) = ( x + 1) ⋅ ((x + 1) − x )=
2 4 2 2 2 2 2

= ( x + 1) ⋅ (x + x + 1)(x − x + 1)≥ 0.
2 2 2

x 2 + x + 1 və x 2 − x + 1 kvadrat üç hədlilərinin hər birinin


diskriminantı mənfidir. Onda bu vuruqların hər biri müsbət
olur. Deməli, x -in istənilən qiymətində verilmiş bərabərsizlik
doğrudur.

340. Verilmiş ifadələri çevirək:


(
n 5 + n + 1 = n 5 − n 2 + n 2 + n + 1 = n 2 (n − 1) n 2 + n + 1 + + n 2 + n + 1 = ) ( )
( )(
= n + n + 1 n (n − 1) + 1 ,
2 2
)
n11 + n + 1 = n11 − n 5 + n 5 + n + 1 =
( )( ) (
= n5 n3 − 1 n3 + 1 + n 2 + n + 1 n3 − n 2 + 1 = )( )
= (n + n + 1)(n (n − 1)(n
2 5 3
)
+1 + n − n 3 2
+ 1).
n >1 olduqda hər iki halda vuruqlar vahiddən böyük olur.
Deməli, n >1 olduqda verilmiş ədədlər mürəkkəb ədədlərdir.
n =1 olduqda hər iki ədəd 3-ə bərabərdir. Deməli, yalnız n =1
olduqda hər iki ədəd sadədir.
341. x + a x + b x − 8 x + 4 = x + m x + n
4 3 2 2
( )
2
olsun. Onda
(
x 4 + a x3 + b x 2 − 8 x + 4 = x 4 + 2m x3 + m2 + 2n x 2 + 2mn x + n 2 . )
194
Buradan, qeyri-müəyyən əmsallar üsuluna görə
n 2 = 4,

mn = −4,

b = m + 2n,
2

a = 2 m .

Bu sistemi həll etsək: n = ± 2, m =  2, b = 4 ± 4, a =  4 alarıq.
Onda 2 hal alınır : x 4 − 4 x 3 + 8 x 2 − 8 x + 4 = (x 2 − 2 x + 2)
2

( )
2
x 4 + 4 x3 − 8x + 4 = x 2 + 2 x − 2 .
Deməli, a = − 4, b = 8 və a = 4, b = 0 .

342. Verilmiş funksiyanın böhran nöqtələrini tapaq:



(
y′ = 0 ⇔ sin100 x + cos100 x = 0, )
100 sin x ⋅ cos x − 100 ⋅ cos x ⋅ sin x = 0,
99 99

( )
sin 2 x sin 98 x − cos 98 x = 0,
πk π
sin 2 x = 0 ⇔ x = və ya tg 98 x =1 ⇔ x = ± + πn, n∈ Z .
2 4
(− π; π) aralığını nəzərə alsaq:
π π π π
x1 = 0, x 2 = , x 3 = − , x 4 = − , x 5 = ;
2 2 4 4
π  π
f (0) = f   = f  −  = 1;
2  2
100
π  π  1  1
f =f−  = 2⋅  = 49 .
4  4  2 2
1
Deməli, ymin = -dur.
249

195
343. [x ]⋅ {x}= f ( x ) işarə edək. Onda f ( x ) = 2005 x tənliyini
həll etməli oluruq. Aşkardır ki, x = 0 bu tənliyi ödəyir.
Əgər x>0 olarsa, onda 0 < [x ] ≤ x və 0 ≤ {x} < 1 olar, yəni
f ( x ) < x < 2005x.
Əgər x ≤ −1 olarsa, onda x − 1 < [x ] < 0 və 0 ≤ {x} < 1 olar,
yəni f ( x ) > x − 1 > 2005 x. Əgər − 1 < x < 0 olarsa, onda
[x] = −1 və {x} = x − [x]= x + 1 olar. Buradan f (x ) = − x − 1.
Digər tərəfdən f ( x ) = 2005 x olduğundan
1
2005 x = − x − 1 ⇔ x = − .
2006
1
Beləliklə, verilmiş tənliyin həlləri: x = 0 və x = − olur.
2006
344. n8 + n + 1 = n8 − n 2 + n 2 + n + 1 =
( ) ( )( )
= n 2 n 6 − 1 + n 2 + n + 1= n 2 n3 − 1 n3 + 1 +
+ n + n + 1 = (n − 1)(n + n + 1)(n + n )+
2 2 5 2

+ (n + n + 1) = (n + n + 1)((n − 1)(n + n ) + 1);


2 2 5 2

buradan aydındır ki, bütün natural n >1 ədədləri üçün


n8 + n + 1 ədədi mürəkkəb ədəddir.
Deməli, n8 + n + 1 ifadəsi yalnız n =1 olduqda sadədir.

(
345. 32 n+1 − 2 2 n+1 − 3n ⋅ 2 n = 32 n+1 − 3 ⋅ 3n ⋅ 2 n + )
( ) ( )
+ 2 ⋅ 3n ⋅ 2n − 22 n +1 = 3n +1 ⋅ 3n − 2n + 2n +1 (3 n
)
− 2n =
( )( )
= 3n − 2 n ⋅ 3n+1 + 2 n+1 şəklində yazsaq, n >1 olduqda,
3 − 2 >1 olduğundan verilmiş ədəd mürəkkəb ədəddir.
n n

196
346. Həmin ədədlərin sayı 9!-dır. Hər bir mərtəbədə hər bir
rəqəm 8! dəfə iştirak edir. Ona görə də istənilən mərtəbədəki
rəqəmlərin cəmi:
1 ⋅ 8!+2 ⋅ 8!+3 ⋅ 8!+  + 9 ⋅ 8! = (1 + 2 + 3 +  + 9) ⋅ 8! = 45 ⋅ 8! = 5 ⋅ 9 !
olar. Onda axtarılan bütün bu ədədlərin cəmi:
109 − 1
(
5 ⋅ 9!⋅ 1 + 10 + 10 2 +  + 108 = 5 ⋅ 9!⋅ ) 10 − 1
= 5 ⋅ 8!⋅ 999999999

hasilinə bərabər olar ki, bu da 999999999 ədədinə bölünür.

347. Çevirmə aparsaq:


x 3 y − x 3 z + y 3 z − xy 3 + z 3 ( x − y ) =
( ) (
= xy x 2 − y 2 − z ( x − y ) x 2 + xy + y 2 + )
3
(
+ z ( x − y ) = ( x − y ) x y + xy − x z − xyz − y 2 z + z 3 =
2 2 2
)
( (
= (x − y ) x 2 ( y − z ) + xy ( y − z ) − z y 2 − z 2 = ))
= (x − y )( y − z ) (x + xy − yz − z ) =
2 2

= (x − y )( y − z ) ( y (x − z ) + (x − z )) = 2 2

= (x − y )( y − z )(x − z )(x + y + z )

348. Ədədi silsilənin xassəsinə görə a + c = 2b.


Onda a 2 (b + c ) + b 2 (a + c ) + c 2 (a + b ) =
(
= a 2b + a 2 c + 2b 3 + ac 2 + bc 2 = b a 2 + c 2 + ac(a + c ) + 2b 3 = )
( 2
)
= b 4b − 2ac + 2abc + 2b = 4b − 2abc + 2abc + 2b 3 =
3 3

= 6b 3 = ⋅ (3b ) = (2b + b ) = (a + b + c ) .
2 3 2 3 2 3

9 9 9
α +β α −β
349. sin α + sin β = 2 sin ⋅ cos ,
2 2

197
α +β α − β nəzərə alıb, tərəf-tərəfə
cos α + cos β = 2 cos ⋅ cos
2 2
ϕ
α +β 2 6 1 2 tg
bölsək: tg = : = . sin ϕ = 2 düsturunu
2 2 2 3 2 ϕ
1 + tg
2
2 2 3
tətbiq etsək: sin (α + β) = 3 2 3 3
2
= 3 = = alarıq.
 1  1+ 1 4 2
1+   3
 3

x
350. I üsul: y = götürək. Onda
2
 x  x  x  x
f  x −  = f (x ) ⋅ f  , f   = f ( x )⋅ f   .
 2 2 2 2
 x
f   ≠ 0 olduğundan f (x) = 1. Buradan f(3) = 1 alarıq.
2

II üsul: y = 0 olsun, onda


f ( x − 0 ) = f ( x ) ⋅ f (0 )⇒ f ( x ) = f ( x ) ⋅ f (0 ), buradan
f ( x ) ≠ 0 olduğundan f (0) = 1 alarıq.
1 = f (0) = f (3 − 3) = f (3) ⋅ f (3) ⇒ ( f (3)) = 1⇒ f (3) = 1
2

olur.

f (a ) a 2
351. b 2 ⋅ f (a ) = a 2 ⋅ f ( x ) ⇒ = . Onda
f (b ) b 2

198
f (5) − f (1) f (5) f (1)  5   1 
2 2
25 1 24
= − =  −   = − = =6
f (2 ) f (2 ) f (2 )  2   2  4 4 4
olur.

352. Tənliyi cos10 x = 1 + sin10 x şəklində yazaq.


0 ≤ cos10 x ≤ 1 və 1 + sin10 x ≥ 1 olduğundan
cos10 x = 1 cos x = ±1
 ⇒  ⇒ sin x = 0
1 + sin x = 1 sin x = 0
10

Buradan x = πk , k ∈ Z . Onda verilmiş tənliyin [0; π ]


parçasındakı həlləri x1 = 0, x2 = π olar.

353. Kosinuslar teoreminə görə


b2 + c2 − a 2 c2 + a2 − b2 a2 + b2 − c2
cos A = , cos B = , cos C = .
2bc 2ca 2ab
Onda
( ) ( ) (
ab 2 b 2 + c 2 − a 2 bc 2 c 2 + a 2 − b 2 ca 2 ⋅ a 2 + b 2 − c 2
= =
) alarıq.
2bc 2ca 2ab
Buradan
( ) ( ) ( )
a 2b 2 b 2 + c 2 − a 2 = b 2 c 2 c 2 + a 2 − b 2 = c 2 a 2 a 2 + b 2 − c 2 .
2 2 2
Hər tərəfi a b c ifadəsinə bölsək:
b2 + c2 − a 2 c2 + a 2 − b2 a 2 + b2 − c2
= = (∗)
c2 a2 b2
a c a+c
= = nəzərə alsaq, (∗) ifadəsindən
b d b+d

199
(b 2
) ( ) ( = 2 2 2 =1
)
+ c2 − a2 + c2 + a2 − b2 + a 2 + b2 − c2 c2 + a2 + b2
c2 + a2 + b2 c +a +b
alarıq. Bərabər nisbətlərin xassəsinə görə (∗) -dakı kəsrlərin
hər biri 1-ə bərabərdir.
Onda b 2 + c 2 − a 2 = c 2 və b 2 = a 2 və ya b = a olur.
Eyni şəkildə a = c və a = b = c alarıq ki, bu da verilmiş
üçbucağın bərabəryanlı olması deməkdir.

354. Hər tərəfi sin x ⋅ cos 2 x hasilinə vuraq:


8 ⋅ sin x ⋅ cos 2 x ⋅ cos x ⋅ cos 4 x ⋅ cos 5 x = sin x ⋅ cos 2
sin 8 x ⋅ cos 5 x = sin 3x − sin x,
sin 7 x ⋅ cos 6 x = 0.
πk π πn
Buradan xk = , xn = + , (k , n∈ Z ).
7 12 6
355. Tənliyin hər tərəfini 2-yə vurub, çevirmə aparsaq:
2 cos 2 x + 2 cos 2 y − 2 cos x ⋅ cos y − 2 cos x − 2 cos y + 2 = 0.
Qruplaşdırma aparsaq:
(cos x − cos y )2 + (cos x − 1)2 + (cos y − 1)2 = 0.
Buradan cos x = cos y = 1 və
x = 2πk , k ∈ Z , y = 2πn, n∈ Z alarıq.

356. tgA − tgB = tgA ⋅ tgB ⋅ tgC + tgC ,


tgA − tgB = (tgA ⋅ tgB + 1) ⋅ tgC ,
tgA − tgB
= tgC ⇔ tg ( A − B ) = tgC.
1 + tgA ⋅ tgB
Buradan A − B = C ⇔ A = B + C ⇒ A = 90o.
Deməli, həmin üçbucaq düzbucaqlı üçbucaqdır.
200
357. a + b + c = 0, olduğundan (a + b + c )(aα + bβ + cγ ) = 0,
a 2α + b 2 β + c 2γ + ab(α + β ) + ac(α + γ ) + bc(β + γ ) = 0.
Şərti nəzərə alsaq: α + β = −γ , α + γ = − β , β + γ = −α .
Onda a 2α + b 2 β + c 2γ − abγ − acβ − bcα = 0,
α β γ 
a 2α + b 2 β + c 2γ − abc + +  = 0.
a b c
α β γ
Digər tərəfdən + + = 0 olduğundan
a b c
α ⋅ a 2 + β ⋅ b 2 + γ ⋅ c 2 = 0 alarıq.

358. f ( x − y ) = f ( x ) ⋅ f ( y )⇒ f ( x ) = f (x + y ) ⋅ f ( y ),
f (x )
f (x + y ) = .
f (y)
(a + b + c )(aα + bβ + cγ ) = 0,
Burada, şərtə görə y -in, (−∞; +∞) alalığından götürülmüş
hər hansı qiyməti üçün f (y) ≠ 0. Əks halda f (x)=0 olardı ki,
bu isə ola bilməz. Onda
 x
f 
f (x )  x x
f (x + y ) = ⇒ f  +  =   = 1⇒ f (x ) = 1
2
f (y) 2 2 f  x
 
2
alarıq.

359. x = y qəbul etsək:


f ( x ) + f ( x ) = f (0 ) + 2 x 2 ⇒ 2 f ( x ) = 2 x 2 + f (0 ),

f (x ) = x 2 + f (0 ) = x 2 + a, (burada, a = f (0)).
1 1
2 2

201
x = y = 0 olduqda f (0 ) = 0 + a ⇒ a = ⇒ a = 0 alarıq.
1 a
2 2
Onda f ( x ) = x olur.
2

360. Verilmiş tənliyin kökləri x1, x2 və x3 olsun. Onda Viyet


teoreminə görə
 x1 + x2 + x3 = 3,

 x1 x2 + x2 x3 + x3 x1 = − 1,
x ⋅ x ⋅ x = − a
 1 2 3
Digər tərəfdən x1, x2, x3 ədədləri ədədi silsilə əmələ
gətirdiyindən, x1 + x3 = 2 x2 . Onda
(x1 + x3 ) + x2 = 3 ⇔ 3x2 = 3 ⇒ x2 = 1 və x1 + x3 = 2 olar.
x2 = 1 qiymətini yuxarıdakı sistemdə nəzərə alsaq:
 x1 + x3 + x1 x3 = −1 2 + x1 x3 = −1  x x = −3,
 ⇒  ⇒ 1 3 olar.
 x1 ⋅ x3 = −a a = − x1 ⋅ x3 a = 3
Digər tərəfdən x1 + x3 = 2 və x1 ⋅ x3 = −3 tənliklərini birlikdə
həll etsək: x1 = −1, x3 = 3 alarıq.
Beləliklə, x1 = −1, x2 = 1, x3 = 3 və a = 3 olur.

1 + f (x + a )
361. f (x + 2a ) = f ((x + a ) + a ) = =
1 − f (x + a )
1+ f (x )
1+
1− f (x ) 1 − f (x ) + 1 + f (x ) 2 1
= = =− =−
f (x ) 1 − f (x ) − 1 − f (x )
.
1−
1+ 2 f (x ) f (x )
1− f (x )

202
Bu qayda ilə f (x + 4a ) = f ((x + 2a ) + 2a ) = − = f (x )
1
f ( x + 2a )
alarıq. Deməli, f (x + 4a ) = f (x ) olduğundan f (x) funksiyası
dövridir və onun dövrü T = 4a -dır.
362. Dərəcəni azaltma düsturunu tətbiq edək:
1 − cos 20o 1 − cos100o 1 − cos140o
tg 210o + tg 2 50o + tg 2 70=
o
+ + =
1 + cos 20o 1 + cos100o 1 + cos140o
3−
1
2
( cos80o + 2 cos120o + cos160o + cos 40o + cos 240o )
=
8cos 2 10o cos 2 50o cos 2 70o
3 3
3+ −
−3cos 20 ⋅ cos 40 ⋅ cos80
o o o
4 8
= = 9
→ 3
8
olar.

363. a 2 + b 2 = 2 + 2(cos α ⋅ cos β + sin α ⋅ sin β ) =


α −β
= 2 + 2 cos(α − β ) = 4 cos 2 .
2
4a 4(sin α + sin β )
= =
a + b + 2b 4 cos 2 − β + 2(cos α + cos β )
22
α
2
α +β α −β α +β
2 sin ⋅ cos sin
= 2 2 = 2 =
α −β  α −β α +β α β
cos  cos + cos  cos ⋅ cos
2  2 2  2 2
α β β α
sin ⋅ cos + sin ⋅ cos
= 2 2 2 2 = tg α + tg β .
α β 2 2
cos ⋅ cos
2 2

203
İsbat tamam oldu.

364. Çevirmə aparsaq:


α β α β γ
tg ⋅ tg + tg
 tg + tg  =
2 2 2 2 2
α β α β   α β
= tg ⋅ tg + ctg  +  ⋅  tg + tg  =
2 2 2 2  2 2
α β
1 − tg ⋅ tg
α β 22 ⋅  tg α + tg β  =
= tg ⋅ tg +
2 2 α β  2 2
tg + tg
2 2
α β α β
= tg ⋅ tg + 1 − tg ⋅ tg =1
2 2 2 2
alarıq.

365. y = 0 olduqda f (0 ) = f ( x ) ⋅ cos 0, buradan f ( x ) = f (0 ).


f (0) = c (c − const ) ilə işarə etsək: f ( x ) = c olur.
Aydındır ki, əgər varsa, f funksiyası sabitdir. Onda verilmiş
tənlikdə yoxlama aparsaq: c = c ⋅ cos y, bu axırıncı tənlik isə
yalnız c = 0 olduqda eynilikdir. Deməli, f (x ) = 0 olur.

t −1
366. 2 x +1= t qəbul edək. Onda x = . Yeni dəyişəni
2
verilmiş tənlikdə nəzərə alsaq:
 t −1 t −1
2

f (t ) = 4 ⋅   + 14 ⋅ + 7;
 2  2
f (t ) = t 2 − 2t + 1 + 7t − 7 + 7 ;
f (t ) = t 2 + 5t + 1
alarıq. Onda t hərfini x hərfi ilə əvəz etsək:
204
f ( x ) = x 2 + 5 x + 1 olur.

367. ( x + y + z ) = x 2 + y 2 + z 2 + 2( xy + yz + zx ) eyniliyini
2

nəzərə alsaq:
x + y + z = 6
x 2 + y 2 + z 2 = 62 − 2 ⋅ 11 = 14 və  2 alarıq.
 x + y + z = 14
2 2

x 2 + y 2 + z 2 =14 sferası ilə x + y + z = 6 müstəvisinin


kəsişməsi ya çevrə, ya nöqtə, ya da boş çoxluqlar. Sferanın
mərkəzindən müstəviyə qədər məsafə:
2
 6 
d =  =12, sferanın radiusundan R = 14 kiçik
2 2

 3
olduğundan axtarılan nöqtələrin həndəsi yeri çevrədir.
Pifaqor teoreminə görə bu çevrənin radiusu:
r = R 2 − d 2 = 14 − 12 = 2 olur.

1 1 1
368. 1 + = t əvəz edək. Onda = t − 1⇒ x = olar.
x x t −1

Buradan f (t ) =  1  − 1 = 1 − (t − 12) = 1 − t + 22t − 1 = 2t − t 2 .


2 2 2 2

 t −1 (t − 1) (t − 1) (t − 1)
2x − x2
Deməli, f ( x ) = olar.
(x − 1)2
369. Üçbucaq bərabərsizliyinə görə
b − c < a < b + c.
Çevirmə aparıb, gücləndirmə tətbiq etsək:
( b− c ⋅ )( b+ c < ) ( a ) < ( b) + ( c) ,
2 2 2

( b− c) < ( a ) < ( b + c) ,
2 2 2

205
Buradan b − c < a < b + c alarıq. Üçbucaq
bərabərsizliyi ödəndiyinə görə isbat tamam olur.

370. Ədədi və həndəsi orta haqqında bərabərsizliyi tətbiq


etməklə, aşağıdakı çevirməni aparsaq:
n 2 1 + 3 + 5 + 7 +  + (2n − 1) n
n= = > 1 ⋅ 3 ⋅ 5 ⋅ 7  (2n − 1),
n n
buradan n n > 1 ⋅ 3 ⋅ 5 ⋅ 7(2n − 1) alarıq.

371. Sol tərəfi aşağıdakı kimi çevirək:


( )
sin 2 n x + 2n − 2 sin n x ⋅ cos n x + cos 2 n x.
Digər tərəfdən 1 = (sin x + cos x ) =
2 2 n

= sin x + cos x + n(sin x ⋅ cos


2n 2n
x + cos x ⋅ sin
2 2n − 2 2 2n − 2
)
x+
+ c (sin x ⋅ cos
2
n
4 2n − 4
x + cos x ⋅ sin x )+ ≥
4 2n − 4

≥ sin x + cos x + (2 − 2 ) sin x ⋅ cos x .


2n 2n n n n

Burada hər bir mötərizə 2 sin n x ⋅ cos n x -dən kiçik deyil,


1
(
əmsallar cəmi isə 2n − 2 -yə bərabərdir.
2
)
372. Tənliyi aşağıdakı kimi çevirək:
= − ( y − 3) + 2 , (tgx > 0).
1
tgx +
2

tgx
1
Bu tənlikdə sol tərəf: tgx + ≥ 2, sağ tərəf isə
tgx
2 − ( y − 3) ≤ 2 olduğundan mümkün hal
2

206
 1
tgx + tgx = 2, olduqda olur.

2 − ( y − 3)2 = 2

π
Onda tgx = 1 ⇒ x = + π k , k ∈ Z və y − 3 = 0 ⇒ y = 3 olur.
4
1
373. I üsul: sin α ⋅ sin β = (cos(α − β ) − cos(α + β )) və
2
α −β α +β
cos α − cos β = −2 sin ⋅ sin düsturlarını tətbiq
2 2
etməklə fərqi qiymətləndirək:
sin 1o sin 3o sin 1o ⋅ sin 4o − sin 2o ⋅ sin 3o
− = =
sin 2o sin 4o sin 2o ⋅ sin 4o

=
(cos 3 o
) ( =
)
− cos 5o − cos1o − cos 5o cos 3o − cos1o
=
2 sin 2o ⋅ sin 4o 2 sin 2o ⋅ sin 4o
− 2 sin 1o ⋅ sin 2o sin 1o
= = − < 0.
2 ⋅ sin 2o ⋅ sin 4o sin 4o
sin 1o sin 3o
Beləliklə, < alarıq.
sin 2o sin 4o

II üsul: Hasili cəmə çevirməklə, aşağıdakı nisbəti


qiymətləndirək:
sin 1o sin 3o sin 1o ⋅ sin 4o cos 3o − cos 5o
: = =
sin 2o sin 4o sin 2o ⋅ sin 3o cos1o − cos 5o
y=cos x funksiyası (0°; 90°) aralığında azalan olduğundan
cos1o > cos 3o ⇒ cos1o − cos 5o > cos 3o − cos 5o > 0 olur.
cos 3o − cos 5o sin 1o sin 3o
Onda <1 və buradan < alarıq.
cos1o − cos 5o sin 2o sin 4o
207
374. P( x ) = ax 4 + bx 3 + cx 2 + dx + c olsun. Onda
P(1) = P(− 1) ⇒ a + b + c + d + e = a − b + c − d + e ⇒ b + d = 0 ;
P(2) = P(− 2) ⇒16a + 8b + 4c + 2d + e = 16a − 8b + 4c − 2d + e,
buradan 16b + 4d = 0 ⇒ 4b + d = 0 alınır. Onda
b + d = 0
 ⇒ b = d = 0 və P( x ) = ax 4 + cx 2 + e olur.
 4b + d = 0

375. Üç parçanın uzunluqları həndəsi silsilə əmələ gətirirsə,


onda, onlar a, a q, a q2 (a > 0) olar. Buradan
5 +1
a) q ≥ 1 olduqda aq 2 < a + aq ⇔ q 2 − q − 1 < 0 ⇔ 1 < q < ;
2
b) 0 < q < 1 olduqda isə
5 −1
a < aq + aq 2 ⇔ q 2 + q − 1 > 0 ⇔ < q < 1.
2
 5 −1   5 + 1
Deməli, q∈ ;1 ∪ 1; .
2 2 
   
376. Verilmiş bərabərliyi aşağıdakı şəkildə yazaq:
1 + cos 2α 1 + cos 2β
+ + cos 2 γ = 1,
2 2
cos 2α + cos 2β + cos 2 (α + β ) = 0,
1 1
2 2
cos(α + β ) ⋅ cos(α − β ) + cos 2 (α + β ) = 0,
cos(α + β )(cos(α − β ) + cos(α + β )) = 0,
2 cos(α + β ) ⋅ cos α ⋅ cos β = 0.

208
Buradan cos α = 0 olarsa, α = 90o , cos β = 0 olarsa,
β = 90o , cos(α + β ) = cos γ = 0 olarsa, γ = 90o olur.
Bu halların hər birində üçbucaq düzbucaqlı olur.

377. sin 7 x + cos 7 x = (sin x + cos x )(sin 6 x − sin 5 x ⋅ cos x +


+ sin 4 x ⋅ cos 2 x − sin 3 x ⋅ cos3 x + sin 2 x ⋅ cos 4 x −
− sin x ⋅ cos5 x + cos 6 x .)
Digər tərəfdən
3
sin 6 x + cos 6 x =1 − 3 sin 2 x ⋅ cos 2 x = 1 − sin 2 2 x.
4
Şərtə əsasən sin 2 x = 1 olduğundan
3 1
sin 6 x + cos 6 x = 1 − ⋅ 1 = ,
4 4
( )
sin 4 x ⋅ cos 2 x + sin 2 x ⋅ cos 4 x = sin 2 x ⋅ cos 2 x sin 2 x + cos 2 x =
1 1 1
= sin 2 2 x = ⋅ 1 = ,
4 4 4
1 1
sin 3 x ⋅ cos3 x = sin 3 2 x = ,
8 8
(
sin 5 x ⋅ cos x + sin x ⋅ cos 5 x = sin x cos x sin 4 x + cos 4 x = )
 1  1  1  1 1 1
= sin x ⋅ cos x1 − sin 2 2 x  = sin 2 x1 − sin 2 2 x  = 1 −  = .
 2  2  2  2 2 4

1 1 1 1 1 2
Beləliklə, sin 7 x + cos 7 x = 2  + − −  = 2 ⋅ = olur.
4 4 8 4 8 8

1 1
378. = t qəbul edək. Onda x = olar. Yeni dəyişəni
x t
verilmiş tənlikdə nəzərə alsaq:
209
1  1 1  1
 − 1 ⋅ f   + f (t ) =  − 1 f   + f (x ) =
1 yaxud 1
, .
t  t  1
−1 x  x 1
−1
t x
 1 1
( x − 1) f ( x ) + f  x  = x − 1 ,
Onda    tənliklər sistemini alarıq.

1 − x ⋅ f  1  + f ( x ) = x ,
 x  x 1− x

Buradan
(1 − x )2 ⋅ f (x ) + f (x ) = x 1
+ ,
x 1− x x
1 − 2x + x2 + x x2 − x + 1
⋅ f (x ) = ,
x x(1 − x )
x2 − x + 1 x2 − x + 1
⋅ f (x ) = f (x ) =
1
, alarıq.
x x(1 − x ) 1− x

379. Tənliyin kökləri a, b, c olsun.


Onda a 3 − a + 1 = 0, b3 − b + 1= 0, c 3 − c + 1 = 0 .
a + b + c = 0,
Viyet teoreminə görə ab + bc + ca = −1,
abc = −1.

Onda a 3 = a − 1, a 4 = a 2 − a,
( )
a 8 = a 4 − 2a 3 + a 2 = a 2 − a − 2(a − 1) + a 2 = 2a 2 − 3a + 2.
a 16 = 4a 4 + 9a 2 + 4 − 12a 3 + 8a 2 − 12a =
( )
= 4 a 2 − a − 12(a − 1) + 17 a 2 − 12a + 4 =
= 4a 2 − 4a − 12a + 12 + 17 a 2 − 12a + 4 =
= 21a 2 − 28a + 16.

210
Analoji qayda ilə b16 = 21b 2 − 28b + 16;
c16 = 21c 2 − 28c + 16 alarıq.
Aldığımız bərabərlikləri tərəf-tərəfə toplasaq:
( )
a16 + b16 + c16 = 21 a 2 + b 2 + c 2 − 28(a + b + c ) + 48 =
(
= 21 (a + b + c ) − 2(ab + bc + ca ) − 28(a + b + c ) + 48 =
2

( ))
= 21 0 2 − 2 ⋅ (− 1) − 28 ⋅ 0 + 48 = 42 + 48 = 90
olur. Deməli, a16 + b16 + c16 = 90 -dır.

380. Ardıcıl olaraq, verilmiş tənlikdə


π π π π
x = 0, y = t , x = , y = + t , x = + t , y = yazsaq
2 2 2 2
(t − yeni dəyişəndir):
 f (t ) − f (− t ) = 2 f (t ),
 sistemini alarıq.
 f (t + π ) − f (− t ) = 0,
 f (t + π ) − f (t ) = −2a ⋅ sin t

π 
Burada f   = a qəbul edək. Bu sistemin I tənliyindən
2
f (− t ) = − f (t ) alınır. II və III tənlikdən
f (t ) − f (− t ) = 2a ⋅ sin t , f (t ) = a ⋅ sin t.
Yoxlama aparsaq: a sin ( x + y ) − a sin ( x − y ) = 2a sin y ⋅ cos x
alarıq ki, a -nın istənilən qiymətində bu bərabərlik eynilikdir.
π 
Bundan başqa, f ( x ) = a sin x funksiyası üçün f   = a
2
ödənir. Deməli, f ( x ) = a ⋅ sin x, burada a istənilən sabitdir.

211
(
381. f (sin x ) + f (cos x ) = 3 = 3 sin 2 x + cos 2 x = )
3 sin 2 x + 3 cos 2 x olduğundan f ( x ) = 3x 2 şəklində ola bilər.

382. Aşağıdakı çevirmələri aparaq:


cos( x + y ) + cos( y + z ) + cos( z + x ) =
= cos(( x + y + z ) − z ) + cos(( x + y + z ) − x ) +
+ cos(( x + y + z ) − y ) = cos( x + y + z )(cos x + cos y + cos z ) +
+ sin ( x + y + z )(sin x + sin y + sin z ) = a cos 2 ( x + y + z ) +
+ a sin 2 ( x + y + z ) = a.
Beləliklə, isbat tamam oldu.

383. Viyet teoreminə görə


x1 + x2 = 3 və x1 ⋅ x2 = 1.
Onda
x1 + x2 = ( x1 + x2 )= 2
x1 + x2 + 2 x1 x2 =
= 3 + 2 ⋅1 = 5 .
Bu qayda ilə
4 x1 + 4 x2 = x1 + x2 + 24 x1 x2 =

= 5 + 2 ⋅1 = 2 + 5 ,
8 x1 + 8 x2 = 2 + 2 + 5 ,
prosesi davam etdirsək:
2n x1 + 2 n x2 = 2 + 2 +  + 2 + 5 alarıq.
Digər tərəfdən

212
2 + 2 + + 2 + 5 > 2 + 2 + + 2 + 4 = 2

olduğundan 2n x1 + 2 n x2 > 2 olur.

384. x = y = z = 0 olduqda
sin x ⋅ sin y ⋅ sin z ⋅ + cos x ⋅ cos y ⋅ cos z = 1 olur.
Göstərək ki, verilmiş ifadənin qiyməti 1-dən böyük ola
bilməz. Aydındır ki,
sin x ⋅ sin y ⋅ sin z + cos x ⋅ cos y ⋅ cos z ≤
≤ sin x ⋅ sin y + cos x ⋅ cos y .
 π
0; 2  parçasında elə x ′ və y′ bucaqları tapmaq olar ki,

sin x′ = sin x və sin y′ = sin y .


Onda sin x ⋅ sin y + cos x ⋅ cos y =
= sin x ′ ⋅ sin y′ + cos x ′ ⋅ cos y′ = cos(x ′ − y′) ≤1.
Deməli, verilmiş ifadənin ən böyük qiyməti 1-dir.

x2 + x
385. Tənliyin sağ tərəfi 0 ≤ 2 cos 2 ≤ 2, sol tərəfi isə
6
iki müsbət qarşılıqlı tərs ədədin cəmi olduğundan
3x + 3− x ≥ 2 , onda
 2 x + x
2
 2 x + x
2

 2 cos ≤ 2,  2 cos = 2,
 6 ⇒ 6
3x + 3− x ≥ 2 3x + 3− x = 2.
 
3x + 3− x = 2 tənliyinin yeganə x = 0 kökü var. x = 0 həm
213
də I tənliyi ödədiyindən, yəni, 2 cos 2 0 = 2 ⇒ cos 0 = 1
olduğundan verilmiş tənliyin kökü x = 0 olur.

386. 2010 = n qəbul edək. Onda


( )(
n 2 + (n − 2)(n − 1)(n + 1)(n + 2) = n 2 + n 2 − 4 n 2 − 1 = )
= n + n − n − 4n + 4 = n − 4n + 4 = n − 2 .
2 4 2 2 4 2
( 2
)
2

Buradan alınır ki, verilmiş ədəd (20102−2)2 ədədinə


bərabərdir.

387. Çevirmə aparsaq:


( )
log5 1 + x > log 4 x , ( x > 0).
log 4 x = t qəbul edək, onda x = 4t və
( )
log5 1 + 4t > t ⇔ 1 + 4t > 5t alarıq.
t t
1 4
f (t ) =   +   >1 funksiyasına baxaq.
5  5
t = 1 olduqda f (t) = 1,
t < 1 olduqda f (t) > 1,
t > 1 olduqda f (t) < 1.
Buradan t < 1 olur. Onda x < 41 ⇒ 0 < x < 16 alarıq.

388. Tənliyin sol tərəfi cüt funksiyadır. Onda x=xo kökdürsə,


x = −xo da kök olacaq. Əksi özünə bərabər olan yeganə ədəd
x = 0 olduğundan
−a ⋅ tg ( cos 0 ) + 1 =0,
a ⋅ tg 1 = 1,
a = ctg 1 .
214
Yoxlasaq, x = 0 tənliyin yeganə kökü olduğunu görərik.

389. f (999) = f ( f (1000)) =


1 1
= .
f (1000) 999
1
Aydındır ki, funksiya 999 və qiymətlərini alır.
999
Deməli, elə a ədədi tapılar ki, f (a ) = 500 olar. Lakin

f (500 ) = f ( f (a )) =
1 1
= .
f (a ) 500

Beləliklə, f (500) = 1 olur.


500

390. Tutaq ki, x = y = 1 , onda 2 f (1) = 2 f 2 (1) alarıq.


Buradan f (1) = 0 və ya f (1) = 1.
a) f (1) = 0 olduqda y =1 üçün f (x)=0 (∀x ∈ R);
b) f (1) = 1 olduqda y = 1 üçün
f (x ) + x = (x + 1) ⋅ f (x ) ,
x ( f (x ) − 1) = 0.
Əgər x ≠ 0 olarsa, f (x) = 1; əgər x = 0 olarsa, f (x) = a olur.
1, əgər x ≠ 0;
Onda f (x ) =  a∈R.
a , əgər x = 0,
391. y = 0 olduqda f ( x ) + f ( x ) − f ( x ) − x 3 + 0 = 0,
buradan f ( x ) = x 3 alarıq.
Yoxlama aparsaq: ( x + y ) + ( x − y ) − x 3 − x 3 − 6 xy 2 = 0
3 3

olduğu aşkar görünür.

215
392. y = 0 olduqda f ( x + 0 ) = ( f (x )) 0 = 1.
Deməli, f (x) = 1.

393. x = 1 və f (1) = a olduqda,


f ( y ) = f (1 ⋅ y ) = ( f (1)) = a y .
y

Deməli, f ( x ) = a x olur.

394. x = 0 və f (0) = a olduqda


f ( y ) = f (0 + y ) = f (0 ) + y = y + a.
Beləliklə, f (x) = x + a olur.
Yoxlama aparsaq: f (x + y ) = x + y + a = (x + a ) + y = f (x ) + y
alarıq.

1+ x
395. Qeyd edək ki, − 1 < x < 1 ⇔ 0 < < 1,
2
1− x
bu qayda ilə 0 < < 1. Onda
2
1+ x  1+ x 1− x  1− x
n n

  ≤ ;   ≤ olar.
 2  2  2  2

Bunları tərəf-tərəfə toplasaq:

1+ x  1− x 
n n

  +  ≤ 1, buradan
 2   2 
(1 + x )n + (1 − x )n ≤ 2n alarıq.

216
396. Aşkardır ki,
1 1 1 1 1 1 1 1 1
+ + + ++ ≤ 1+ + + ++ =
1! 2 ! 3! 4 ! n! 1⋅ 2 2 ⋅ 3 3 ⋅ 4 (n − 1)n
 1 1 1 1 1  1 1
= 1 + 1 −  +  −  +  −  +  +  − =
 2   2 3   3 4   n − 1 n
1 2n − 1
= 2− = .
n n
Əvvəl ilə axırı birləşdirsək:
1 1 1 1 2n − 1
+ + ++ ≤ alarıq.
1! 2! 3! n! n

397. Verilən bərabərsizliyi aşağıdakı kimi yazaq:


n! =1 ⋅ 2
⋅
3
n ≥ 2⋅ 
2 ⋅⋅
2⋅ ⋅ 2 = 2n −1.
2
n −1 n −1

Buradan n! ≥ 2n −1 alarıq.

398. 3
3 + 3 3 = a və 3
3 − 3 3 = b olsun.
Onda a 3 + b3 = 3 + 3 3 + 3 − 3 3 = 6.
Aydındır ki, ab 2 + a 2b < a 3 + b3 (burada a ≠ b olduğundan
bərabərsizlik ciddidir), buradan
ab(a + b ) < a 3 + b3 = 6,
3ab(a + b ) < 18,
a 3 + b3 = 6,
⇒ (a + b ) < 24 ⇒ a + b < 23 3 .
3

3ab(a + b ) < 18
Deməli, 3
3 + 3 3 + 3 3 − 3 3 < 23 3 doğrudur.

217
399. x y =1 münasibətindən x və y eyniişarəli olduğu alınır.
Digər tərəfdən x + y = 2 − cos 2 z ⇒ 0 < x + y ≤ 2.
Deməli, x və y müsbətdir. Ədədi və həndəsi orta haqqında
bərabərsizliyə görə x + y ≥ 2 xy = 2 ⋅ 1 = 2 olar.
 x + y ≤ 2,
Onda  ⇒ x + y = 2 alarıq.
x + y ≥ 2
Buradan və x y = 1 şərtindən x = y = 1 olur.
π
Onda cos 2 z = 0 ⇔ cos z = 0 ⇒ z = + πn, n ∈ Z .
2
π
Deməli, x=1, y =1, z = + πn, n∈ Z .
2

400. Qeyd edək ki, x = [x ] + {x} və 0 ≤ {x}<1.


{x} = α olsun, onda x = [x ] + α ⇔ n x = n ⋅ [x ] + n ⋅ α ,
0 ≤ n ⋅ α < n. Buradan n ⋅ [x ] ≤ [nx ] ≤ n ⋅ [x ] + n − 1 yaza
bilərik. Burada n = 1, 2, 4, 8, 16 və 32 qiymətlərini növbə
ilə yazsaq:
63 ⋅ [x ]≤ [x ] + [2 x ] + [4 x ] + [8 x ] + [16 x ] + [32 x ]≤ 63 ⋅ [x ] + 57,
63 ⋅ [x ]≤12345 ≤ 63 ⋅ [x ] + 57
olduğundan 12288 ≤ 63 ⋅ [х ] ≤ 12345 və buradan

≤ [х ] ≤ 195
1 20
195 ⇒ х ∈∅,
21 21
çünki, baxılan aralıqda heç bir tam ədəd yoxdur.

401. Qeyd edək ki, x < 0 ⇒ 2 x < 0, 3 x < 0, , n x < 0 .


Mənfi ədədlərin cəmi heç vaxt 1 ola bilməz.

218
x > 0 ⇒ 0 ≤ [x ] ≤ [2 x ] ≤  ≤ [n x ];
n dənə tam ədədin cəmi 1 olduğuna görə,
[x] = [2 x] =  = [(n − 1) x] = 0 və [n x] = 1 olmalıdır.
1 
[(n − 1) x] = 0 ⇒ 0 ≤ (n − 1) x < 1 ⇒ 0 ≤ x <
n − 1
⇒
[n x] = 1 ⇒ 1 ≤ n x < 2 ⇒ 1 ≤ x < 2 

n n
1 1 
⇒ x∈ ; , (n ≥ 2, n ∈ N ).
 n n − 1

402. Tutaq ki, x verilən tənliyin həllidir və [x ] = n , (n ∈ Z).


Onda tənlikdən x 2 + 7 = 8n alarıq ki, buradan n ≥ 0.
Bu halda n ≤ x < n + 1 bərabərsizliyini tətbiq etsək:
n 2 + 7 ≤ x 2 + 7 < (n + 1) + 7 = n 2 + 2n + 8.
2

n 2 + 7 = 8n nəzərə alsaq:
n 2 + 7 ≤ 8n < n 2 + 2n + 8, buradan
n 2 + 7 ≤ 8n 1 ≤ n ≤ 7,
 ⇒
8n < n + 2n + 8 n < 2
2

Buradan 1 ≤ n < 2 və ya 4 < n ≤ 7, n tam ədəd olduğundan


n=1, 5, 6, 7 alarıq. Onda x 2 = 1, 33, 41, 49 olar.
Yoxlama göstərir ki, 1, 33 , 41 və 7 verilmiş tənliyin
həllidir. Lakin (− 1), (− 33 ), (− 41) və (−7) kənar
köklərdir.

403. Kəsr hissənin xassəsinə görə 0 ≤ x 3 < 1⇔ 0 ≤ x < 1.

219
Digər tərəfdən {a + b}= a yalnız o vaxt ödənir ki, b∈ Z
{ }
olsun. Buna görə x 3 + 3 x 2 + 3 x + 1 = x 3 və buradan
3x 2 + 3x = n ∈ Z .
f ( x ) = 3 x 3 + 3 x funksiyası [0;1) -də artır.
Onda f (0) = 0, f (1) = 6 və 0 ≤ f ( x ) < 6, yəni, 0 ≤ n < 6 .
− 3 ± 9 + 12n
3 x 2 + 3 x + n = 0 tənliyinin kökləri x = .
6
Buradan n = 0; 1; 2; 3; 4; 5 olmaqla, x -in qiymətləri tapılır.

x
404. Məlum, 1 − cos x = 2sin 2 ,
2
x x x
1 + cos x = 2 cos 2 , sin x = 2sin ⋅ cos eyniliklərini
2 2 2
nəzərə alsaq:
x x x
1 + sin x −
= cos x 2sin  sin + cos  ,
2 2 2
x x x
1 + sin x =
+ cos x 2 cos  sin + cos  alarıq.
2 2 2
x π
Digər tərəfdən sin + cos =2 ⋅ sin  +  ifadəsi
x x
2 2 2 4
 π π
 − ,  aralığında müsbətdir. Beləliklə, hər bir
 2 2
x x x
2sin  sin + cos 
 π π  üçün 2 2 2 x
x ∈ =
− ,  f ( x) = tg olur.
 2 2 x x x 2
2 cos  sin + cos 
2 2 2
220
x  π π
f ( x ) = tgfunksiyası isə  − ;  aralığında tək funksiya
2  2 2
olduğu aşkardır.

405. [3 m ] = k ; m, k ∈ N olduqda, k 3 ≤ m < (k + 1) . Buradan


3

k 3 ≤ m ≤ (k + 1) − 1 alarıq.
3

m ∈ N üçün (k + 1) − k 3 = 3k 2 + 3k + 1.
3

x −1
Verilən bərabərliyin sol tərəfi ∑S
k =1
k olsun.

( )
Burada S k = k 3k 2 + 3k + 3 olar. S k > 0, k ∈ N üçün
S1 =1 ⋅ 7 = 7, S 2 = 2 ⋅ 19 = 38, S3 = 3 ⋅ 37 = 111 və
S4 = 4 ⋅ 61 = 244 olduğundan S1 + S2 + S3 + S4 = 400 olur.
Onda verilən tənliyin natural ədədlər çoxluğunda yalnız bir
həlli vardır: x − 1 = 4 ⇒ x = 5.

406. Sistemin I tənliyini y-ə, II tənliyini isə x -ə vurub,


tərəf-tərəfə toplasaq:
 x + 2y   2x − y 
y  x + 2  + x  y + 2
2 
 = 2 y,
 x +y   x + y 2 

buradan xy + 1 = y ⇒ x = 1 −
1
( y ≠ 0).
y
İndi sistemin I tənliyini x -ə, II tənliyini isə y -ə vurub, tərəf-
tərəfə çıxsaq:
 x + 2y   2x − y 
x  x + 2  − y  y + 2
2 
 = 2 x,
 x +y   x + y 2 

221
buradan x 2 − y 2 + 1 = 2 x ⇔ ( x − 1) = y 2 , burada
2

1 1
x =1− nəzərə alsaq: 2 = y 2 ⇔ y 4 = 1 ⇔ y = ±1.
y y
1
Beləliklə, y1 = 1 olduqda x1 = 1 − = 0;
1
1
y2 = −1 olduqda x2 = 1 − = 2 olur.
−1
Asanlıqla yoxlamaq olar ki, (0; 1) və (−1; 2) cütü verilən
sistemin həllidir.

407. Sistemin hər bir tənliyini kvadrata yüksəldib, toplasaq:


sin 2 x + sin 2 y + sin 2 z + 2 sin x ⋅ sin y + 2 sin y ⋅ sin z +
+ 2 sin z ⋅ sin x + cos 2 x + cos 2 y + cos 2 z + 2 cos x ⋅ cos y +
+ 2 cos y ⋅ cos z + 2 cos z ⋅ cos x = 9 .
Buradan cos( x − y ) + cos( y − z ) + cos( x − z ) = 3 bərabərlik
halı yalnız cos( x − y ) = 1, cos( y − z ) =1 və cos( x − z ) = 1
olduqda mümkündür. Onda y = x + 2πm, z = x + 2πn,
3 1 π
buradan cos x = , sin x = və x = + 2πk , k ∈ Z .
2 2 6
π π
Bunları nəzərə alsaq: y = + 2π l , z = + 2π q alarıq.
6 6

408. Hər tərəfi loqarifmləsək:


ln e x ≥ ln x e ⇔ x ≥ e ln x.
İndi x ≥ e ⋅ ln x bərabərsizliyini isbat edək.
f (x ) = x − e ln x funksiyasına baxaq.

222
x > 0 olduqda f ′( x ) =1 − ⇒ f ′( x ) > 0, ( x > e ) ,
e
x
yəni , (e, + ∞ ) aralığında f (x) monoton artır;
x < e olduqda (0; e) aralığında f (x) monoton azalır;
xo = e olduqda f ′(e ) = 0 olur.
Aydındır ki, xo = e f ( x ) -in minimum nöqtəsidir.
Buna görə istənilən x ∈ (0; + ∞ ) üçün f ( x ) ≥ f (e ) = 0 ,
yəni, x − elnx ≥ 0 ⇔ x ≥ e ⋅ lnx . Onda
e x ≥ eelnx ⇔ e x ≥ x e , ( x > 0 ) alarıq.
Tələb olunan bərabərsizliyi aldıq.

409. α 3 = (
3
2 +3 3 )
3
= 5 + 33 2 ⋅ 3 3 ⋅ (
3
)
2 +3 3 =
)(
= 5 + 33 6 α , onda α 3 − 5 = 33 6 α və buradan
(α − 5) = (3 6 α ) , (α
3 3 3
3
− 5 = 162 α 3 ,
3
)
3

α 9 − 15α 6 − 87α 3 − 125 = 0 alarıq.


Deməli, axtarılan çoxhədli
P( x ) = x 9 − 15 x 6 − 87 x 3 − 125 oldu.
Əgər bu çoxhədlini istənilən tam əmsallı çoxhədliyə
vursaq, yenidən kökü α = 3 2 + 3 3 olan çoxhədli alarıq.

410. Belə ki, istənilən x ∈ R üçün


1 + x 2 + x + 1 ≥ x + x + 1≥1.
Buradan aydın olur ki, verilən funksiyanın təyin oblastı
R -dir. Qeyd edək ki, istənilən x ∈ R üçün

223
1 + x 2 − x + 1 = x − x + 1 ≥ 1,
onda verilən funksiyanın surət və məxrəcini
1 + x 2 − x + 1 ifadəsinə vursaq:

f (x ) =
( 1+ x 2
)(
+ x −1 1 + x2 − x +1 )=
( 1+ x 2
+ x + 1)( 1 + x 2
− x + 1)
1 + x 2 − ( x − 1)
2
2x x
= = =
( 1 + x + 1) − x
2
.
2 2 2 + 2 1+ x 2
1 + 1 + x2
Arqumentin işarəsini dəyişsək:
−x
f (− x ) =
x
=− = − f ( x) alarıq.
1 + 1 + x2 1 + 1 + x2
Deməli, verilən funksiya tək funksiyadır.

411. Tənliyi aşağıdakı kimi çevirək:


3 x 3 − 3 x 2 − 3 x =1,
4 x 3 = x 3 + 3 x 2 + 3 x + 1,
4 x 3 = ( x + 1) ,
3

1
buradan 3
4x = x + 1 ⇔ x = 3 alarıq.
4 −1

412. Aydındır ki, x = 0 ax 5 + bx 4 + c = 0 tənliyinin kökü


ola bilməz, əks halda c = 0 olardı və tənliyin 2 -dən artıq
kökü olmazdı.
Bu tənliyin hər tərəfini x 5 ≠ 0 -ə bölsək:

224
b c
a+ + = 0, buradan aydın olur ki, x1, x2 , x3 ədədləri
x x5
1 1 1
ax 5 + bx 4 + c = 0 tənliyinin köküdürsə, , , ədədləri
x1 x2 x3
cx5 + bx + a = 0 tənliyinin köküdür.

413. a + b = x, b + c = y, c + a = z işarə etsək;


(a − b )(b − c )(c − a ) = (z − y )(x − z )( y − x ) olar.
(a + b )(b + c )(c + a ) xyz
Digər tərəfdən

a = (x − y + z ), b = ( y − z + x ), c = (z − x + y ).
1 1 1
2 2 2
Onda
a b c x− y+z y−z+x z−x+ y
+ + = + + =
a+b b+c c+a 2x 2y 2z
1 y−z 1 z−x 1 x− y
= − + − + − =
2 2x 2 2y 2 2z
3 1 y−z z−x x− y
= −  + + =
2 2 x y z 
3 1 ( y − z ) yz + ( z − x )zx + ( x − y )xy
= − ⋅ =
2 2 xyz
3 1 ( z − y )( x − z )( y − x ) 3 1 12
= − ⋅ = − ⋅ =
2 2 xyz 2 2 2012
3 3 1506 753 250
= − = = =1 .
2 1006 1006 503 503

414. (cos x + i sin x)7 ifadəsinə baxaq. Muavr düsturuna görə

225
(cos x + i sin x )7 = cos 7 x + i sin 7 x .
Digər tərəfdən
(cos x + i sin x )7 = cos7 x + 7i cos6 x ⋅ sin x − 21cos5 x ⋅ sin 5 x −
− 7 cos x ⋅ sin 6 x − i sin 7 x =
(
= cos 7 x − 21cos5 x ⋅ sin 2 x + 35 cos3 x sin 4 x − 7 cos x sin 6 x + )
+ i (7 cos 6
x sin x − 35 cos x sin x + 21cos x sin x − sin x .
4 3 2 5 7
)
(
Buradan cos 7 x + i sin 7 x = cos7 x − 21cos5 x ⋅ sin 2 x +
+ 35 cos3 x sin 4 x − 7 cos x sin 6 x + )
(
+ i 7 ⋅ cos x sin x − 35 cos x ⋅ sin x + 21cos x sin x −sin 7 x ).
6 4 3 2 5

İki kompleks ədədin bərabərliyinə görə


sin 7 x = 7 cos6 x ⋅ sin x − 35 cos 4 x ⋅ sin 3 x +
+ 21cos 2 x ⋅ sin 5 x − sin 7 x .

415. Dəyişənin mümkün qiymətləri çoxluğu


sin x > 0, cos x > 0.
1
Tənliyi çevirsək: og sin x cos x + =2
og sin x cos x
(logsin x cos x − 1)2 = 0 ⇔ logsin x cos x = 1 ⇔ cos x = sin x
π
buradan da x = + πk , k ∈ Z .
4
Dəyişənin mümkün qiymətləri çoxluğunu nəzərə alsaq:
π
x= + 2π k olur.
4

226
416. Qeyd edək ki, 1 + tgα = tg 45o + tgα =
(
sin 45o + α
.
)
cos α ⋅ cos 45o
Onda
sin 46o
1 + tg1o = ,
cos1o ⋅ cos 45o
sin 47 o
1 + tg 2o = ,
cos 2o ⋅ cos 45o

sin 88o
1 + tg 43o = ,
cos 43o ⋅ cos 45o
sin 89o
1 + tg 44o = ,
cos 44o ⋅ cos 45o
1 + tg 45o = 2.
Bu bərabərlikləri tərəf-tərəfə vurub,
sin 89o = cos1o , sin 88o = cos 2o ,, sin 46o = cos 44o
nəzərə alsaq: (1 + tg1o )⋅ (1 + tg 2o )⋅  ⋅ (1 + tg 44o )⋅ (1 + tg 45o ) =
sin 46o ⋅ sin 47 o ⋅  ⋅ sin 89o 2
= ⋅2= = 2 23 .
cos1 ⋅ cos 2 ⋅  ⋅ cos 44 cos 45
o o o
( )
o 44
 1 
 
44

 2
Onda 2 x = 223 ⇒ x = 23 alarıq.

( )
417. a (sin x; cos x ) və b sin x ; cos x vektorlarını qəbul
edək. Onda а ⋅ b ≤ a ⋅ b bərabərsizliyini tətbiq etsək:

sin x sin x + cos x cos x ≤ sin 2 x + cos 2 x ⋅ sin x + cos x .


Burada bərabərlik halı, vektorlar yalnız kollinear olduqda

227
mümkün olduğundan
sin x cos x π
= ⇔ sin x = cos x ⇔ x = + 2πn, n ∈ Z .
sin x cos x 4

418. Qeyd edək ki, − 2 ≤ sin x + cos x ≤ 2 .


1
Digər tərəfdən tg x + ctg x = tg x + olduğundan
tg x
tg x + ctg x ≥ 2 və ya tg x + ctg x ≤ − 2.
Verilmiş tənliyin sol tərəfi [− 2 ; 2 ] , sağ tərəfi isə
(− ∞;−2]∪ [2;+∞ ) aralığında qiymətlər aldığına görə,
həlli yoxdur.

419. Tənliyin hər tərəfini xyz ifadəsinə bölüb, bütün


toplananları sol tərəfə keçirsək:
1 1 1 1 1 1
2
+ 2+ 2− − − = 0 alarıq.
z x y yz xz xy
Hər tərəfi 2-yə vurub, qruplaşdırsaq:
2 2 2 2 2 2
2
+ 2+ 2− − − = 0,
x y z xy xz zx
2 2
1 1  1 1 1 1
2

 −  +  −  +  −  = 0,
 x y  y z  z x
buradan da x = y = z taparıq (x ≠ 0, y ≠ 0, z ≠ 0) .
Deməli, tənliyin sonsuz sayda həlli var.

420. x 4 = t işarə edək. Onda x = 4 t olur.

228
Yeni dəyişənə keçsək: ( t ) = 4 ⇔ t = 4 , buradan
4
t t 4

t = 4 və x 4 = 4 ⇔ x = 2 alarıq.

421. Tənliyin hər tərəfini (1 − x ) ifadəsinə vurub, çevirmə


2

aparsaq: (1 − x )(1 − x )= (1 − x ) ,
3 11 7 2

− x 3 − x11 = −2 x 7 ,
(
x3 1 − x 4 = 0 , )2

x1 = 0, x2 = −1, x3 = 1.
x3 = 1 kökü, verilmiş tənliyi (1− x) -ə vurmaqla alındığına
görə kənar kökdür. Onda x1 = 0, x2 = −1 olur.

(
422. Verilən bərabərliyin hər tərəfini x − x 2 + 1 ifadəsinə )
vursaq: − y − y 2 + 1 = x − x2 + 1. (1)

( )
Analoji olaraq, hər tərəfi y − y 2 + 1 ifadəsinə vursaq:

− x − x2 + 1 = y − y2 + 1 (2)
alarıq. (1) və (2) -ni tərəf - tərəfə toplasaq:
− ( x + y ) = x + y, buradan x + y = 0 alarıq.

423. Qeyd edək ki, 1 − x 2 ⇒ x ≤1 və 1 − y 2 ⇒ y ≤1.

Onda elə α bucağı var ki, x = cos α və 1 − x 2 = sin α ;

y = cos β və 1 − y 2 = sin β olar. Bunları verilmiş


ifadədə nəzərə alsaq:

229
x 1 − y 2 + y 1 − x 2 = cos α ⋅ sin β + sin α ⋅ cos β =
= sin (α + β )≤1.
Bərabərlik halı x =1 və y = 0 olduqda mümkündür.
Deməli, verilən ifadənin ən böyük qiyməti 1-dir.

424. Verilmiş ədədi aşağıdakı kimi çevirək:


3
 
 = 100 00 − 1 = 1000 00 − 3 00 00 +
3
999
  99 
 
    
  
2002  2002  6006 4004

+ 3 00

 00 − 1 = 999

  
99 7 00
 
 
00 2 99
  
99 .
2002 2001 2001 2002

Onda verilən ədədin rəqəmləri cəmi:


9 ⋅ 4003 + 7 + 2 = 9 ⋅ 4004 = 360036 olur.

425. Ədədi silsilənin birinci həddini a, fərqini d ilə işarə


edək. Onda məsələnin şərtinə görə
a + d ( p − 1), a + d (q − 1), a + d (r − 1), a + d ( s − 1)
ədədləri həndəsi silsilə təşkil edər. Beləliklə,
a + d (q − 1) = (a + d ( p − 1))t (1)
a + d (r − 1) = (a + d (q − 1)t (2)
a + d ( s − 1) = (a + d (r − 1))t (3)
burada t həndəsi silsilənin vuruğudur.
Əgər (1) bərabərliyindən (2) -ni çıxsaq,
d (q − r )= d ( p − q )t (4)
(2) bərabərliyindən (3) -ü çıxsaq,
d (r − s ) = d (q − r )t . (5)
Nəhayət, (4) və (5) bərabərliklərini tərəf-tərəfə bölsək:
230
p−q q−r
=
q−r r −s
Deməli, p − q, q − r , r − s ədədləri həndəsi silsilə əmələ
gətirir.

426. Bərabərliyin sol tərəfini aşağıdakı şəkildə yazaq:


2 3 4  1 2 3 4  1 2 3 4 
 + 2 + 3  + 3  + 2 + 3  + 6  + 2 + 3  + (1)
7 7 7  7 7 7 7  7 7 7 7 
2 3 4 123 123
Lakin + + = = olduğundan, (1) ifadəsi
7 7 2 73 73 73
123 1 123 1 123
aşağıdakı şəklə düşər: 3 + 3 ⋅ 3 + 6 ⋅ 3 +
7 7 7 7 7
123 1
Sonuncu cəm, birinci həddi 3 , silsilə vuruğu 3 olan
7 7
sonsuz azalan həndəsi silsilədir. Onda axtarılan cəm
123  1  41
: 1 − 3  = olar.
7  7  114
3

427. Göstərək ki, a 2 + b 2 + c 2 ≤ (| a | + | b | + | c |) 2


Həqiqətən də
a 2 + b 2 + c 2 ≤| a |2 + | b |2 + | c |2 +2 | a | ⋅ | b | +2 | b | ⋅ | c | +
+2 | a | ⋅ | c |
və ya
a + b + c ≤ a + b + c + 2 | a | ⋅ | b | +2 | b | ⋅ | c | +2 | a | ⋅ | c |,
2 2 2 2 2 2

2 (| a | ⋅ | b | + | b | ⋅ | c | + | a | ⋅ | c |) ≥ 0 .
Sonuncu bərabərsizlik doğrudur. Bərabərlik halı a, b, c

231
ədədlərindən ən azı hər hansı ikisi sıfır olduqda mümkündür.

428. x > ab və a > b > 0 olduğundan, x > 0 və


x+a x+b
x + a > x + b > 0. > bərabərsizliyinin
x +a
2 2
x2 + b2
doğru olması üçün aşağıdakılar doğru olmalıdır:
( x + a ) 2 ( x + b) 2 ( x 2 + a 2 ) + 2ax ( x 2 + b 2 ) + 2bx
> ⇒ > ⇒ 1+
x2 + a2 x2 + b2 x2 + a2 x2 + b2
2ax 2bx
+ 2 2 > 1 + 12 2 ⇒
x +a x +b
a b
⇒ 2 2> 2 2⇒
x +a x +b
⇒ a( x + b ) > b( x 2 + a 2 ) ⇒ x 2 (a − b) > ab(a − b)
2 2

Lakin a > b olduğundan, x 2 > ab və ya x > ab .

429. Aşağıdakı şəkildə çevirmələr edək:


S=
−(a22 − a12 ) − (a42 − a32 ) −  − (a22n − a22n −1 ) =
=
−((a2 − a1 )(a2 + a1 ) + (a4 − a3 )(a4 + a3 ) +
+  + (a2 n − a2 n −1 )(a2 n + a2 n −1 )) =
− − d (a1 + a2 + a3 + a4 +  + a2 n −1 + a2 n ),
burada d = a2 − a1 = a3 − a2 = a4 − a3 =  = a2 n − a2 n −1 .
2a1 + d (2n − 1)
Beləliklə, S =−d ⋅ ⋅ 2n =dn(d (1 − 2n) − 2a1 )
2

430. 4 ≥ 2( x 2 + y 2 ) = ( x + y ) 2 + ( x − y ) 2 ≥ ( x + y ) 2
olduğundan, | x + y |≤ 2

232
431. (n + 1) sayda doğru bərabərliyi aşağıdakı şəkildə yazaq:
1 1 2
− =
a −1 a +1 a −1
2

2 2 22
− =
a2 −1 − a2 + 1 a4 −1

22 22 23
− =
a 4 − 1 a 4 + 1 a8 − 1
..................

2n 2n +1
2n
− = n+1
a2 −1 a2 + 1 a2 −1
n n

Tənlikləri tərəf-tərəfə toplasaq, alarıq:


1 2 4 2n 1 2n +1
+ + + ... + 2n = − n+1
a + 1 a2 + 1 a4 + 1 a + 1 a −1 a2 −1

432. Verilmiş çoxhədlini x n − 1 − ax( x n − 2 − 1) şəklində


x k +1 − 1
yazaq və onu = x k + x k −1 +  + x + 1
x −1
düsturundan istifadə edərək, ( x − 1) ikihədlisinə bölək:

x n − 1 − ax( x n − 2 − 1)
= x n −1 + x n − 2 +  + x + 1 −
x −1
−ax( x + x n − 4 +  + x + 1)
n −3

Bərabərliyin sağ tərəfinin ( x − 1) -ə bölünməsi üçün Bezu


teoreminə görə n − (n − 2)a = 0 bərabərliyi ödənilməlidir.

233
n
Deməli, verilmiş çoxhədli ixtiyari n ∈ N , n > 2, a =
n−2
üçün ( x − 1) ifadəsinə bölünür.
2

433. Verilmiş bərabərsizliyin doğru olması üçün aşağıdakı


bərabərsizliklər ödənilməlidir:
( y 2 + 1) 2 x 2 − 4 y ( y 2 − 1) x + 4 y 2 ≥ 0 ya
x 2 ( y 2 − 1) 2 − 4 xy ( y 2 − 1) + 4 y 2 + 4 x 2 y 2 ≥ 0
və ya (( y 2 − 1) x − 2 y ) 2 + (2 xy ) 2 ≥ 0 .
Bərabərlik halı yalnız x= y= 0 olduqda mümkündür.

434. Aşağıdakı doğru bərabərsizliklərdən istifadə edək:


1 1 1
< < (1)
(n + k + 1)(n + k ) (n + k ) 2
(n + k − 1)(n + k )
1 1 1
Burada − − və
(n + k + 1)(n + k ) n + k n + k + 1
1 1 1
= −
(n + k − 1)(n + k ) n + k − 1 n + k

olduğunu nəzərə alsaq, (1) bərabərsizliyini aşağıdakı şəkildə


yazmaq olar:
1 1 1 1 1
− < < −
n + k n + k + 1 (n + k ) 2
n + k −1 n + k
k - ya 1, 2, 3, ..., p qiymətlərini versək,

234
1 1 1 1 1 
− < < − 
n + 1 n + 2 (n + 1) 2
n n +1

1 1 1 1 1 
− < < − 
n + 2 n + 3 (n + 2) 2
n +1 n + 2  (2)
.......................................................... 

1 1 1 1 1 
− < < −
n + p n + p + 1 (n + p ) 2 n + p − 1 n + p 
(2) bərabərsizliklərini tərəf-tərəfə toplasaq:
1 1 1 1 1
− < + + + <
n + 1 n + p − 1 (n + 1) (n + 2)
2 2
(n + p) 2
1 1
< −
n n+ p
axtarılan bərabərsizliyi almış olarıq.

435. Burada (n − 1) cüt ədəd olduğundan, aşağıdakı şəkildə


çevirmələr edək:
1k + 2k + 3k +  + (n − 1) k = (1k + (n − 1) k ) +
  n − 1 k  n + 1 k 
+(2 + (n − 2) ) + (3 + (n − 3) ) +  +   +
  2   2  
k k k k

 
 n −1   n +1 
k k

Burada   və   cəmini orta hədləridir.


 2   2 
Mötərizə içərisində olan bütün cəmlər k tək ədəd olduğu
üçün Bezu teoreminə görə
n −1 n +1
1 + (n − 1) =2 + (n − 2) =3 + (n − 3) = = + =n
2 2
ədədinə bölünür.

235
436. Aşağıdakı doğru bərabərsizlikləri yazsaq:
1 1 1 1 1 1
< , < , <
a+b+c a+b a+b+c a+c a+b+c b+c
Bu bərabərsizlikləri tərəf-tərəfə toplasaq, axtarılan
bərabərsizliyi alarıq.

437. Birinci 600 natural ədəd arasında 7-yə bölünən


 600  = 85 sayda ədəd vardır, deməli, 600! ədədi 785 -ə
 7 
bölünür. Uyğun qayda ilə bu ədədlər arasında 72 -na bölünən
 600  =12, 73 -na bölünən  600  = 1 ədəd vardır.
 7 2   73 
Deməli, 600! ədədində 7-nin qüvvəti 85 + 12 + 1 = 98 -dir.
Beləliklə, 600! ədədi 799 -a bölünmür.

438. Tutaq ki, a = −3 -dir. Onda adi bölmə əməlini yerinə


yetirməklə
x3 + y 3 + z 3 − 3 xyz
= x3 + y 3 + z 3 − xy − yz − xz almaq olar.
x+ y+z
İsbat edək ki, a ≠ −3 olduqda x3 + y 3 + z 3 + axyz çoxhədlisi
x + y + z ifadəsinə bölünmür. Əksini fərz edək.
Onda x3 + y 3 + z 3 + axyz = ( x + y + z ) A yazmaq olar, burada
A qismətdir. Bu halda x = −( y + z ) olduqda çoxhədli sıfra
bərabər olmalıdır, yəni, −( y + z )3 + y 3 + z 3 − ayz ⋅ ( y + z ) =
0.
Sadələşdirmə aparsaq: (a + 3)( y + z ) yz = 0 alarıq.
Sonuncu bərabərlik eynilik olduğundan y və z -in istənilən
qiymətində ödənilməlidir. Onda a + 3 =0, a =−3 olur.

236
Beləliklə, ancaq a = −3 olduqda verilmiş çoxhədli x + y + z
cəminə bölünür.

439. Verilmiş bərabərsizliyin doğru olması üçün aşağıdakılar


ödənilməlidir.
2 2+a+b 2 (1 + a) + (1 + b)
≤ , ≤ ,
1 + a + b 1 + a + b + ab 1 + a + b (1 + a)(1 + b)
2 1 1 1 1 1 1
≤ + , + ≤ +
1+ a + b 1+ b 1+ a 1+ a + b 1+ a + b 1+ b 1+ a
Sonuncu bərabərsizliyin doğruluğu isə aşkardır. Bərabərlik
halı a= b= 0 olduqda mümkündür.

440. Şərtə görə a > b olduğundan, a − b > 0 .


Tutaq ki, a − b = c > 0, onda a= b + c .
Beləliklə bərabərsizlik aşağıdakı şəklə düşər:
n
b+c < n b + n c (1)
(1) bərabərsizliyinin hər iki tərəfi müsbət olduğu üçün,

( ) <( )
n n
n
b+c n
b+nc və ya b + c < b + c + M .
M > 0 olduğu üçün, sonuncu bərabərsizlik doğrudur.
Beləliklə, (1) bərabərsizliyi və deməli verilmiş bərabərsizlik
doğrudur.

441. n = 0 qiymətində verilmiş kəsr ixtisar olunmayandır və


3
-ə bərabərdir. n ≠ 0 qiymətlərini nəzərdən keçirək. n − in
7
2n + 3
digər bütün qiymətlərində ixtisar olunmayandır.
5n + 7

237
2n + 3
Həqiqətən də əgər ixtisar olunandırsa, onda
5n + 7
5n + 7 n +1
= 2+ kəsri
2n + 3 2n + 3
n +1
də ixtisar olunandır. Lakin ixtisar olunandırsa,
2n + 1
2n + 3 1
= 2+ kəsri də ixtisar olunan olmalıdır.
n +1 n +1
1 2n + 3
Lakin ixtisar olunmayan kəsr olduğundan,
n +1 5n + 7
kəsri də ixtisar olunmayandır.

442.
a 2b 2 + b 2 c 2 + c 2 a 2 =

=
1
2
( ( )
a 2b 2 + b 2 c 2 ) + ( a 2b 2 + a 2 c 2 ) + ( b 2 c 2 + a 2 c 2 ) ≥

1
≥ (2ab 2 c + 2a 2bc + 2abc= 2
) abc(a + b + c)
2
Bərabərlik halı a, b, c ədədlərindən ən azı ikisi sıfra bərabər
olduqda mümkündür.

443. Tutaq ki, f ( x, y=


) ( x − y )Q( x, y ) (1)
burada Q( x, y ) hər hansı çoxhədlidir. (1) bərabərliyi eynilik
olduğundan, x və y -in istənilən qiymətində doğrudur.
f ( x, y ) -in x və y dəyişmələrinə nəzərən simmetrik
olduğunu nəzərə alsaq, (1) bərabərliyini
f ( x, y=
) ( y − x)Q( y, x) (2)

238
şəklində yazmaq olar. Buradan
( x − y )Q( x, y ) =
− ( y − x)Q( y, x) və ya Q( x, y ) = −Q( y, x)
x = y olduqda Q( y, y ) = −Q( y, y ) və ya Q( y, y ) = 0
Bu onu göstərir ki, Q( x, y ) çoxhədlisində x -i y -lə əvəz
etdikdə, o sıfıra çevrilir, yəni Q( x, y ), ( x, y ) ikihədlisinə
bölünür, başqa sözlə Q( x, y= ) ( x − y )Q1 ( x, y ) (3)
(2) və (3) bərabərliyini istifadə etsək,
) ( x − y ) 2 Q1 ( x, y ) alınar.
(1) f ( x, y=
Bunu da isbat etmək tələb edilirdi.

444. Tənliyi aşağıdakı şəkildə yazaq: x 2 − 17 = 3 y 2 . İki halı


nəzərdən keçirək: x = 3k , = x 3k ± 1 . Birinci halda sol tərəf
9k − 17 şəklinə düşür və bu ifadə 3-ə bölünmür. İkinci
2

halda, yəni =x 3k ± 1 olduqda bərabərliyin sol tərəfi


9k ± 6k − 16 şəklinə düşür ki, bu da 3-ə bölünmür.
2

Beləliklə, tənliyin sağ tərəfi həmişə 3-ə bölündüyü üçün


verilmiş tənliyin tam həlli yoxdur.

445. a 3 − a ifadəsini xətti vuruqlarına ayıraq:


a 3 − a = a (a 2 − 1) = (a − 1)a (a + 1) .
Üç ardıcıl a − 1, a və a + 1 ədədlərdən biri və yalnız
biri 3-ə bölündüyündən, bütün hasil də 3-ə bölünəcək.

446. a 3 − =a a (a 2 − 1) ifadəsinin 3-ə bölünməsi faktından


istifadə edək. Burada 3 –sadə ədəd olduğundan ya a , ya da
a 2 − 1 ifadəsi 3-ə bölünməlidir. Əgər a ədədi 3-ə bölünərsə,

239
onda a 2 ədədi də 3-ə bölünər və a 2 -nın 3-ə bölünməsindən
alınan qalıq 0 olar. Əgər a 2 − 1 ədədi 3-ə bölünərsə, onda
a 2 − 1= 3k (k ∈ Zz ), a 2 = 3k + 1 olar,
yəni, a 2 -nin 3 -ə bölünməsindən alınan qalıq 1 olar.

447. a 3 + 2a = (a 3 − a ) + 3a olduğundan və hər toplanan


3-ə bölündüyündən a 3 + 2a cəmi də 3-ə bölünür.

448. a 3 + b3 + c3 = (a 3 − a ) + (b3 − b) + (c3 − c) + (a + b + c)


Sağ tərəfdəki toplananlardan hər biri 3-ə bölündüyü qçqn,
bütün cəm də 3-ə bölünür.

449. Həlli: Tam ədədin kvadratının 3-ə bölünməsindən


alınan qalıq 0 və ya 1-dir. Şərtə görə a ədədi 3-ə bölünmədiyi
üçün a 2 =3k + 1, (k − mənfi olmayan tam ədəddir) şəklində
göstərmək olar. Onda 5a 2 + 1= 5(3k + 1) + 1= 15k + 6 .
Sonunda cəm 3-ə bölünür.

450. Fərz edək ki a və b ədədləri 3-ə bölünmür.


Onda a 2 =+
3k 1, b 2 =+ 3b 1 şəklində yazmaq olar.
Burada k və l tam ədədlərdir. Bu bərabərlikləri tərəf-tərəfə
toplasaq: a 2 + b 2 = 3(k + l ) + 2 alarıq.
Göründüyü kimi bu halda a 2 + b 2 cəmi 3-ə bölünmür.
Aydındır ki, əgər bu ədədlərdən biri 3-ə bölünüb, digəri
bölünməzsə, a 2 + b 2 cəmi 3-ə bölünməz. Həqiqətən də,
məsələn, a=2
3k , b=2
3n + 1 olarsa, a 2 + b 2 = 3(k + n) + 1.
Deməli, a 2 + b 2 cəminin 3-ə bölünməsi üçün a və b
ədədlərindən hər biri 3-ə bölünməlidir.
240
451. a 5 − =a a(a 4 − 1)= a(a 2 − 1)(a 2 + 1)= (a − 1)a(a + 1)(a 2 + 1) .
Burada a 2 + 1 vuruğunu a 2 + 1= (a 2 − 4) + 5 şəklində göstərək.
Onda
a 5 − a = (a − 1)a (a + 1)((a 2 − 4) + 5) = (a − 1)a (a + 1)(a 2 − 4) +
+5(a − 1)a (a + 1) = (a − 2)(a − 1)a(a + 1)(a + 2) +
+5(a − 1)a (a + 1)
Sonuncu cəmdə birinci toplanan 5 ardıcıl tam ədədin hasili
olduğundan, ikinci toplananda isə 4 vuruğu olduğu üçün
bütün cəm, yəni, a 5 − a ifadəsi 5-ə bölünür.

452. Həqiqətən də a 5 − =
a a (a 4 − 1) ifadəsi 5-ə
bölündüyündən ya a , ya da a 4 − 1 ədədi 5-ə bölünməlidir.
a ədədi 5-ə bölünərsə a 4 ədədinin 5-ə bölünməsindən alınan
qalıq 0, a 4 − 1 ədədi 5-ə bölünərsə,
a 4 − 1= 3k (k ∈ z ), a 4 = 3k + 1 yazmaq olar.
Yəni, a 4 -ün 5-ə bölünməsindən alınan qalıq bu halda 1
olacaq.
453. İsbat etmək lazımdır ki, bu cəm həm 3-ə, həm də
5-ə bölünür.
1) 9a 5 ədədi 3-ə bölündüyündən, 5a 3 + 4a cəminin 3-ə
bölündüyünü göstərmək kifayətdir:
5a 3 + 4a= (5a 3 − 5a ) + (5a + 4a )= 5(a 3 − a ) + 9a
Toplananlardan hər biri 3-ə bölünür, onda cəm də 3-ə bölünür.
2) Verilmiş cəmin 5-ə bölündüyünü göstərmək üçün
9a 5 − 4a ifadəsinin 5-ə bölündüyünü göstərmək kifayətdir:
9a 5 − 4a = 9a 5 − 9a + 5a = 9(a 5 − a ) + 5a
Göründüyü kimi həqiqətən də bu cəm 5-ə bölünür.

241
454. Bu ifadələrdən ən azı birinin 5-ə bölünməsi ilə
eynigüclüdür. Aşağıdakı şəkildə çevirmələr edək:
( x 2 − 2 xy + 2 y 2 )( x 2 + 2 xy + 2 y 2 ) =
=(( x 2 + 2 y 2 ) − 2 xy )( x 2 + 2 y 2 ) + 2 xy ) =( x 2 + 2 y 2 ) 2 − 4 x 2 y 2 =A
=x 4 + 4 x 2 y 2 + 4 y 4 − 4 x 2 y 2 =x 4 + 4 y 4
lınan cəmin 5-ə bölünməsi üçün ya x və y ədədlərinin
hər biri 5-ə bölünməlidir, ya da hər biri 5-ə bölünməməlidir.
1) Əgər x və y 5-ə bölünərsə, onda hər şey aydındır.
2) Əgər x və y 5-ə bölünürsə, onda x= 4
5k + 1, y =
4
5l + 1
şəklində yazmaq mümkün olduğundan,
x 4 + 4 y 4 = (5k + 1) + 4(5l + 1) = 5k + 20l + 5 olar.
Sonuncu cəm 5-ə bölündüyündən bu hal da mümkündür.
Deməli, verilmiş ifadələrin ən azı birinin 5-ə bölünməsi
üçün ya hər ikisi 5-ə bölünən, ya da hər ikisi 5-ə
bölünməyən istənilən x və y ədədləri götürülə bilər.

455. =
a n 2 + 1 olsun. Onda
(n 2 + 1)(n + 1) − (n 2 + n + 1) =n3 alarıq.

456. Şərtə görə aa + 2bc = d (2d ) . Buradan göründüyü kimi


a cüt ədəddir. Əgər a ≥ 4 olarsa, d ≥ 6 və 2d ikirəqəmli
ədəd olar. Ona görə də a = 2 -dir. Bu halda d ≥ 3 olmalıdır.
1
Əgər d = 3 olarsa, əkizlərin yaşı (36 − 22) = 7 olar. Bu isə
2
olimpiadada iştirak etmək üçün çox azdır. Əgər d = 4 olarsa,
1
əkizlərin yaşı (48 − 22) = 13 olar. d = 5 olduqda 2d rəqəm
2
olmadığı üçün, bu hal mümkün deyil. Beləliklə, qardaşların

242
yaşı 13, 13 və 22 -dir.
457. Bütün toplananları sol tərəfə keçirərək, alınmış ifadəni
sadələşdirək:
a 2b + b 2 c + c 2 a − b 2 a − a 2 c − c 2b = ab(a − b) − c(a 2 − b 2 ) −
+
−c(a 2 − b 2 ) + c 2 (a − b) = (a − b)(ab − c(a + b) + c 2 ) =
= (a − b)(ab − ac − bc + c 2 ) =
= (a − b)(a (b − c) − c(b − c)) = (a − b)(b − c)(a − c)
Son nəticədə (a − b)(b − c)(a − c) > 0 bərabərsizliyi alınır.
Burada a > b > c olduğundan alınmış bərabərsizlik doğrudur.

458. Verilmiş üçhədlinin x0 = 2 nöqtəsində qiymətini tapaq.


 q
Onda y0 = 4 + 2 p + q = 4 + 2  p +  = 4006
 2
Göründüyü kimi bu şəkildə olan bütün üçhədlilərin qrafikləri
(2; 4006) nöqtəsindən keçir.

459. Aşağıdakı eynilikdən istifadə edək:


n !(n + 2)
= n !(n + 1 + 1)
= n !(n + 1) + n=! (n + 1)!+ n !
Onda verilmiş ifadə aşağıdakı şəklə düşər:
2!+ 1!− 3!− 2!+ 4!+ 3!− 5!− 4!+  + 2000!+ 1999!− 2001!−
−2000!+ 2001! = 1
460. Əksini fərz edək. Tutaq ki, elə x və y natural ədədləri
var ki, x + y =201 , və xy hasili 201-ə bölünür.
201 = 3 ⋅ 67 olduğu üçün bu ədədlərdən biri, məsələn, x
ədədi 3-ə bölünür. Onda= y 201 − x olan digər ədəd də 3-ə
bölünməlidir. Analoji olaraq, həm x, həm də y ədədləri 67-
yə bölünməlidir. Bu halda x və y ədədlərindən hər biri
243
3 ⋅ 67 = 201 ədədindən kiçik deyil. Bu isə məsələnin şərtinə
ziddir.

cos 2 x + 3 2 cos 2 x − 1 + 3 1
461. = = 2 cos x + ≥=
2⋅2 4
cos x cos x cos x

462. Bu ədədin sonuncu iki rəqəmi sıfır olduğu üçün 4-ə


bölünür. Verilmiş ədədin 9-a bölünüb bölünmədiyini
araşdıraq. Bu ədədi aşağıdakı şəkildə yazaq:
1 ⋅10k + 2 ⋅10k −1 + 3 ⋅10k − 2 +  + 98 ⋅104 + 99 ⋅102 +
+100 = 1 + 99
 9 + 2 + 2 ⋅ 99 9 + 3 + 3 ⋅ 99
 9 +
k k −1 k −2

+  + 98 + 98 ⋅ 999 9+99+99⋅99+100=(1+2+3+…100)+
+ 99 + 99 ⋅ 99 + 100 = (1 + 2 + 3 + 100) +
 
+  99
 9 + 2 ⋅ 99
 9 + 3 ⋅ 99
 9 +  + 98 ⋅ 9999 + 99 ⋅ 99 
 k k −1 k −2 
Burada k ədədi verilmiş ədədin rəqəmləri sayıdır. İkinci
mötərizə 9-a bölünür. Birinci mötərizə isə
100(1 + 100)
1 + 2 + 3 +  + 100 = olduğundan, 9-a bölünmür.
2
Deməli, verilmiş ədəd 9 -a və deməli 36 -ya bölünmür.

463. Tənliklər sisteminin aşkar olan sıfır həllərini tapaq:


x=
1 y=
1 z=
1 o0
Sistemin birinci tənliyindən görünür ki, əgər x = 0 olarsa,
onda ya z = 0 və ya y = 0 -dır. Əgər x= y= 0 -dırsa, onda
sistemin üçüncü tənliyi z 2 = 3 z şəklində olar ki, buradan
da z = 3 olduğu tapılır. Beləliklə, x=
2 y=
2 0 , z2 = 3 .
Analoji qayda ilə x=
3 z=
3 0, y=
3 2 və y=
4 z=
4 0 , x4 = 1

244
həlləri tapılır. Daha sonra sistemin birinci tənliyini ikinciyə
və yenə də birincini üçüncü tənliyə böldükdən sonra
aşağıdakı tənliklər sistemini alarıq:
x+z x  z 2z
 y + z = 2y y − x = 1
 
x+ y x  3z 2 z
 =  + = 1
 z + y 3z x y
( x + y )( x + z ) =
x ( x + y )( x + z ) =
x
 
 
 xy + 2 yz − xz = 0

Sonuncu sistemdən də 2 xz + 3 yz − xy = 0 alarıq.
( x + y )( x + z ) =
 x
35 7 5
Buradan isə x5 = , y5 =
− , z5 =
− olur.
24 24 24

464. Tənlikdən göründüyü kimi x ≥ 0 və


c ≥=0 . c 0,= b 0 olduqda verilmiş tənliyin yeganə x = 0
həlli vardır. c = 0 və b ≠ 0 olduqda isə tənliyin həlli yoxdur.
Asanlıqla göstərmək olar ki, tənliyin həlli ancaq
x + a x + b = ( xc − x ) 2 olduqda olar.
x + a x + b = c 2 c− 22c − 2c x + x bərabərliyinin a və b -nın
hansı qiymətlərində doğru olduğunu müəyyən edək. Bu
b = c 2
ifadələrin eyni ilə bərabər olması üçün  (1)
 a = −2 c
olmalıdır. Bunları nəzərə alsaq, tənlik
(c − x ) 2 + x + c və ya | c − x | + x =
c (2)

245
şəklində olar. Sonuncu tənlik aşağıdakı iki qarışıq sistemə
ayrılır:
c − x ≥ 0 c − x < 0
1)  2) 
c − x + x =
c  x − c + x =c
c > 0 olduğu üçün birinci sistemin 0 ≤ x ≤ c 2 (3) şəklində
sonsuz sayda həlli vardır.
İkinci sistemin həlli yoxdur, lakin (2) tənliyi və ona görə də
verilən tənliyin (1) bərabərlikləri ilə seçilmiş a və b üçün
(3) şərtini ödəyən sonsuz sayda həlli vardır.

465. Məchulun mümkün qiymətləri çoxluğu aşağıdakı


bərabərsizliklər sistemindən tapılır:
 x =+ 1 ≥ 0,

 x −1 Buradan x ≥ 1 və ya −1 ≤ x < 0 alınır.
 x ≥ 0.

−1 ≤ x < 0 olduqda x + 1 − 1 < 0 olur. Buna görə də [ −1;0]


aralığında bərabərsizliyin sol tərəfi mənfi, sağ tərəfi isə
müsbət olduğundan, həlli yoxdur.
x ≥ 1 olduqda bərabərsizliyin hər iki tərəfi müsbətdir.
Bərabərsizliyin hər iki tərəfini kvadrata yüksəldək:
1
x +1− 2 x +1 > 1−
x
x2 + x − 2x x + 1 + 1 > 0
( x − x + 1) 2 > 0
Sonuncu bərabərsizlik x -in, x − x + 1 ifadəsini sıfıra
çevirən qiymətlərdən başqa bütün mümkün qiymətlərində

246
1+ 3
doğrudur. x − x + 1 =0 tənliyini həll etsək, x =
2
alınar. Məchulun mümkün qiymətləri çoxluğunu nəzərə
alsaq, verilmiş bərabərsizliyin həlli x -in aşağıdakı şərtləri
1+ 5 1+ 5
ödəyən qiymətləri olar: 1 ≤ x < və x > .
2 2
466. Şəkildən göründüyü kimi böyük düzbucaqlının sahəsi
1 1 1
⋅ (52 − 51) = ⋅1= ,
51 51 51
1 1 1
kiçik düzbucaqlının sahəsi (52 − 51)
= , =y ,
51 52 x
= =
x 51, x 52 xətləri ilə hüdudlanmış fiqurun (əyrixətli
trapesiyanın) sahəsi
52
dx 52
∫ =nx ==
|52
51 n52 − n51 = n olduğundan,
51 x 51
1 52 1
< n < .
52 51 51

467. Şəkildən göründüyü kimi əyrixətli trapesiyanın sahəsi


202
dx 202 101
S=∫ = 200 =
nx |202 n 202 − n 200 = n = n olur.
200 x 200 100
1 2
Digər tərəfdən S ABCD = ⋅ (202 − 200) = = S AB1C1D < S
201 201
2 101
olduğundan, < n .
201 100

468. Burada arcsin-ların fərqi müəyyən inteqraldır.

247
0,8
dx
Həqiqətən də arcsin 0,8 − arcsin 0, 6 = 0,6 =
arcsin x |0,8 ∫ .
0,6
1 − x2
funksiyasının qrafiki ilə hüdudlanmış ABCD əyrixətli
0,8
dx
trapesiyasının sahəsinə bərabərdir: S =
∫ =
S ABCD .
0,6
1 − x2
Nəzərə alsaq ki, S ABC1D < S AB1CD və
1 5 1 1
S ABC1D = f (0, 6) ⋅ (0,8 − 0, 6) = ⋅ 0, 2 = ⋅ = ,
1 − 0, 62 4 5 2
1 5 1 1 1 1
S AB1CD = ⋅ 0, 2 = ⋅ = , onda < S < və ya
1 − 0,82 3 5 3 4 3
1 1
< arcsin 0,8 − arcsin 0, 6 < alarıq.
4 3
π
π 9
469. a) Burada sin 200 =
sin =∫ cos xdx =
S.
9 0

A B
1
C
0 D
π π x
9 2
Şəkildən göründüyü kimi S = S0 ACD .
π π π
Digər tərəfdən S1 = S0 ABD = ⋅1 = olduğundan, sin 200 < .
9 9 9

248
π 7
Burada < doğru bərabərsizliyini nəzərə alsaq,
9 20
7
sin 200 < alarıq.
20
b) Tutaq ki, düzbucaqlı OACD trapesiyasının sahəsi S 2 − dir.
π
1 + cos
π 9 ⋅π .
=
OA 1,=
CD cos olduğundan,
= S2
9 2 9
π
1 + cos
Burada S 2 < S , yəni, 9 ⋅ π < sin π .
2 9 9
π
sin= a (a > 0) qəbul edərək, alınmış bərabərsizliyi a -ya
9
π 1 + 1 − a2 π
görə həll etsək: cos = 1 − a 2 , ⋅ < a və ya
9 2 9
36π
a> 2 alarıq. Beləliklə,
π + 182
π 36π 36 ⋅ 3,1 111, 6 1
sin > 2 > = > 0,3338 > .
9 18 + π 324 ⋅ 3, 2 334, 24
2 3
1
Deməli, sin 200 > .
3

470. x = 1 olduqda e x ≥ ex bərabərsizliyi doğrudur. Tutaq ki,


x > 1 . Onda
x x
e x ≥ ex ⇔ e x − e ≥ ex − e ⇔ ∫ et dt ≥ ∫ edt. Burada
1 1

249
x x
S1 =
∫ e dt =
S ABED , S 2 =
t
∫ edt =
S ABCD
1 1

olduğundan, S1 > S 2 ⇔ e ≥ ex bərabərsizliyi alınır.


x

1 e
471. f ( x) = 2
və g ( x) = funksiyalarının qrafikini quraq
x x
və [ e; π ] parçasında bu funksiyalara uyğun əyrixətli
trapesiyaların sahəsini tapaq:
π
dx 1 1 eπ
π
S1 =∫ 2 =− , S2 =
∫ dx =
en .
e x e π e x e
y
4
3
e
g ( x) =
2 x

1
1
f ( x) =
x2
0 2 3 x
1

Şəkildən göründüyü kimi S1 < S 2

π
e
e
1 1 e 1 1
1 1 − en −
Onda − < en ⇔ e π e > e π ⇔ e π e >  
e π e π 

472. Tutaq ki, a1 və an silsilənin kənar hədləridir.


ak ⋅ an − k +1 ≥ a1 ⋅ an bərabərsizliyinin isbat edilməsi tələb edilir.

250
Məlum düsturlara görə:
ak =a1 + d (k − 1), an − k +1 =a1 + d (n − k ), an =a1 + d (n − 1) .
Onda ak ⋅ an − k +1 = a12 + a1d (n − 1) + d 2 (k − 1)(n − k ) və
a1 ⋅ an = a12 + a1d (n − 1) olduğundan,
ak ⋅ an − k +1 − a1an = d 2 (k − 1)(n − k ) alarıq.
d 2 (k − 1)(n − k ) ≥ 0 bərabərsizliyinin d və k -nın hansı
qiymətlərindən ödənə bilməsini araşdıraq. d = 0 olduqda və
istənilən d üçün k = 1 olduqda bərabərlik halı alınır.
1 < k < n və d ≠ 0 olduqda isə d 2 (k − 1)(n − k ) > 0 olur.
k = n olduqda da ixtiyari d üçün bərabərlik halı alınır.
Deməli, ak ⋅ an − k +1 ≥ a1 ⋅ an bərabərsizliyi doğrudur.

473. Məsələnin şərtinə görə


x3 + ( x + d )3 + ( x + 2d )3 = ( x + 3d )3 , d > 0 . Buradan
x3 + x3 + 3 x 2 d + 3 xd 2 + d 3 + x3 + 6 x 2 d + 12 xd 2 + 8d 3 =
x3 + 9 x3 d + 27 xd 2 + 27 d 3 ;
2 x3 − 12 xd 2 − 18d 3 =
0;
x3 − 6 xd 2 − 9d 3 =0
Aşkardır ki, əgər x və d ədədlərinin ortaq böləni olarsa,
onda bu bölən axtarılan dörd ədədin də ortaq böləni olar
Sonuncu bərabərlikdən görünür ki, d ≠ 1 olduqda x 3 ədədi
d − yə bölünür. Lakin bu mümkün deyil, çünki şərtə görə
x və d qarşılıqlı sadə ədədlərdir. Deməli d = 1 olmalıdır.
Onda alınan bərabərlik x3 − 6 x − 9 = 0 şəklinə düşür.
Bu tənliyi həll edək:
251
x3 − 3x 2 + 3x 2 − 9 x + 3x − 9 = 0
x ( x − 3) + 3 x( x − 3) + 3( x − 3) =
2
0
( x − 3)( x 2 + 3 x + 3) =
0
Burada x3 + 3 x + 3 = 0 tənliyinin həqiqi kökü yoxdur.
Deməli, x = 3 tənliyin yeganə həllidir.
Beləliklə, axtarılan ədədlər 3, 4, 5 və 6 -dır.

474. Məsələni həll etmək üçün


(−∞; −3],[−3; −2],[−2; 2],[2;3] və [3; +∞) aralıqları daxilində
funksiyanın ən kiçik qiymətini seçmək lazımdır.
1) −∞ < x ≤ − 3x olduqda f ( x) = 2 − x + 3 − x + x 2 − 4 − x 2 + 9
və ya f ( x) =−2 x + 10 alarıq. Funksiya azalan olduğu üçün
onun həmin aralıq daxilindəki ən kiçik qiyməti x = −3
olduqda alınır, yəni f (−3) = 16 olar.
2) −3 ≤ x ≤ −2 olduqda,
f ( x) = 2 − x + 3 − x − x + x 2 − 4 − 9 + x 2 = 2 x 2 − 2 x − 8
−2 1
Bu parabolanın təpə nöqtəsi x = − =olduğundan və bu
4 2
nöqtə isə baxılan aralığın sağ tərəfində yerləşdiyi üçün bu
aralıqda funksiya azalır. Onda x = −2 nöqtəsində funksiyanın
f (−2) =4 olan ən kiçik qiyməti vardır.
3) −2 ≤ x ≤ 2 olduqda
f ( x) =2 − x + 3 − x + 4 − x 2 + x 2 − 9 =−2 x funksiyası
azalandır və baxılan aralığın x = 2 nöqtəsində f (2) = −4
olan ən kiçik qiymətini alır.
4) 2 ≤ x ≤ 3 olduqda
252
f ( x) = x − 2 + 3 − x + x 2 − 4 + x 2 − 9 = 2 x 2 − 12 .
Bu parabolanın təpə nöqtəsi koordinat oxu üzərindədir. Eyni
zamanda x 2 -ın əmsalı müsbət olduğu üçün [ 2;3] aralığında
funksiya artan olar və ən kiçik qiymətini isə aralığın sol
kənarında olan x = 2 nöqtəsində alır: f (2) = −4 olur.
5) x ≥ 3 olduqda
f ( x) = x − 2 + x − 3 + x 2 − 4 − x 2 + 9 = 2 x alırıq.
Funksiya artan olduğu üçün, o ən kiçik qiymətini baxılan
aralığın x = 3 nöqtəsində alır, yəni, f (3) = 6 olur.
Beləliklə, verilmiş funksiya ən kiçik qiymətini x = 2
nöqtəsində alır və f (2) =−4 − dür.

475. Verilən tənliklərdən çevirmə ilə alınan


sin x + 2 cos a cos x =
2 (1)

Tənliyinin tərəflərini 1 + 4 cos 2 a ≠ 0 ifadəsinə bölək. Onda


1 2 cos a 2
⋅ sin x + cos x = (2)
1 + 4 cos 2 a 1 + 4 cos 2 a 1 + 4 cos 2 a
2 2
 1   2 cos a 
alınır. Burada   +  = 1 oldu-
 1 + 4 cos a   1 + 4 cos a 
2 2

1 2 cos a
ğundan, = sin ϕ və = cos ϕ qəbul
1 + 4 cos a
2
1 + 4 cos 2 a
etmək olar. Bunları nəzərə alsaq, (2) tənliyi
2
sin x sin ϕ + cos ϕ cos x = və ya
1 + 4 cos 2 a

253
2
cos( x − ϕ ) = (3) şəklinə düşər.
1 + 4 cos 2 a
2
≤ 1 olarsa, (3) tənliyinin və ona görə də
1 + 4 cos 2 a
verilən tənliyin həlli var, əks halda tənliyin həlli yoxdur.
Sonuncu bərabərsizliyi həll etsək:
4 ≤ 1 + cos 2 a ⇒ 3 ≤ 2(1 + cos 2a ) ⇒ 3 ≤ 2 + 2 cos 2a
1
və ya cos 2α ≥ alarıq. Buradan da
2
π π π π
− + 2π k ≤ 2a ≤ + 2π k və ya − +πk ≤ a ≤
+πk
3 3 6 6
alınar. Deməli, a -nın tapılan və şərti ödəyən qiymətlərində
verilmiş tənliyin həlli vardır və
2
x= ± arccos + 2π m + ϕ .
1 + 4 cos 2 a
2 cos a
Burada ϕ = arccos götürmək olar,
1 + 4 cos 2 a
çünki, sin ϕ > 0, 0 < ϕ < π -dir.

476. Tutaq ki, f ( x) dövrü T olan funksiyadır. Onda


istənilən x üçün f ( x + T ) = f ( x) və ya
sin a ( x + T ) + cos( x + T=
) sin ax + cos x (1) bərabərliyi
doğru olacaqdır. x -ə 0 və −2T qiymətləri verək. Onda
sin aT + cos T = 1,

− sin aT + cos T = 1

254
Buradan da cos T = 1 və ya T = 2π n, n ∈ Zz olduğu tapılır.
T -nin bu qiymətini (1) tənlikdə yerinə yazsaq,
sin 2π na = 0 və ya 2=π na π m, m ∈ Zz alınır.
π
Buradan da a = alırıq, yəni a rasional ədəddir.
2n

477. Bərabərsizliyi aşağıdakı şəkildə çevirək:


( x + y ) 2 + 2( x + y ) sin x + 2(1 − cos 2 x) ≥ 0
( x + y + sin x) 2 + sin 2 x ≥ 0
Sonuncu bərabərsizliyin doğruluğu aşkardır. Burada
 x + y + sin x =0
 şərtləri ödənildikdə də bərabərsizlik
sin x = 0
doğrudur. Buradan da x = π k, y =−π n, k , n ∈ Zz alınır.

478. Tənliyi aşağıdakı şəkildə çevirək:


4 3
2sin 2 x 2 + cos 2 x + sin 2 x − 1 =0
5 5
4  π 3
cos ϕ = qəbul edək  0 < ϕ <  , onda sin ϕ = olar. Bu
5  2 5
ifadələri tənlikdə nəzərə alsaq,
2sin 2 x 2 + cos(2 x − ϕ ) − 1 =0
cos(2 x − ϕ ) =
cos 2 x 2
Buradan −(2 x − ϕ ) + 2π k , k ∈ zZ
2x2 =
2 x 2 = 2 x − ϕ + 2π m, k∈Z
z
ϕ
x2 + x − =πk (1)
2

255
ϕ
x2 − x + +πm =0 (2)
2
ϕ 
−1 ± 1 + 4  + π k 
alınar. (1)=
bərabərsizliyindən x  2 
, k ≥ 0,
2
ϕ 
1± 1− 4 + π m 
2 
=
(2) bərabərsizliyindən isə x2 ,m<0
2
qiymətləri tapılır.

479. Məlumdur ki, vahiddən kiçik ədədin kvadrat kökü


özündən böyük, lakin vahiddən kiçikdir. Deməli,
0,99
 9 < A < 1 olmalıdır. Buradan da A = 0,99
 9 alınar.

100 100

Bununla da, vergüldən sonra ilk 100 rəqəm tapılır. 101-ci


rəqəmi tapaq. Tutaq ki, bu rəqəm a -dır. Onda A = 0,99 9a
kimi yazmaq olar.
 10 − a  2(10 − a) 2 (10 − a) 2
2

A =
(0,99 9a) =
2
 1 − 2
101 
=
1 − +
 10  10101 10202
A2 -nın kökaıltı ədəddən kiçik olması üçün 10 − a > 5 olmağı
kifayətdir. Buradan da a < 5 olduğu tapılır. Beləliklə də
0,99
9 4 < A < 0,99
 09 5 olduğundan 101-ci rəqəm 4
100 100

olmalıdır.
480. Məchulun mümkün qiymətləri çoxluğu x ≥ 2 -dir.
Verilmiş tənliklər sistemi aşağıdakı iki sistem ilə eyni
güclüdür.

256
 π   π 
sin  2 x  = 1 sin  2 x  = −1
   və   

 (
cos π x − 2 =
) 1 (
cos π x − 2 =
 ) −1
Birinci sistemdən alırıq:
π π
= x (4m + 1)  =
x 4m + 1
2 2 və ya 
π x − 2 = 2π n  x − 2 =2n

Burada m və n mənfi olmayan tam ədədlərdir.
= x (4m + 1) 2

=  x 4n + 2
2

Sonuncu sistemin birinci tənliyindən görünür ki, x tək


ədəddir, ikinci tənlikdən x -ın cüt ədəd olduğu görünür ki, bu
da mümkün deyildir. İkinci sistemi həll edək:
π π
= x ( 4k − 1)  =x 4k − 1
2 2 və ya 
π x −=
2 π (2 p + 1)  x − 2 = 2 p + 1

Burada k natural ədəd, p -isə mənfi olmayan tam ədəddir.
Deməli
=xk (4k − 1)
2


 x = (4 p + 1) + 2
2

Buradan da (4k − 1) 2 = (2 p + 1) 2 + 2
(4k − 1) 2 − (2k + 1) 2 =
2
(4k − 2 p − 2)(4k + 2 p ) =2
2(2k − p − 12)(2k + p ) =1

257
Bu isə mümkün deyildir. Beləliklə, verilmiş tənliyin həlli
yoxdur.
481. 17 – 13 = 14 fərqi 2-yə bölünür, lakin 4-ə bölünmür.
Onda Bezu teoreminə görə 1715 - 315 fərqi 2-yə bölünür.
Lakin bu o demək deyildir ki, verilmiş fərq 4-ə bölünmür. Bu
fərqi aşağıdakı şəkildə vuruqlara ayıraq:
1715 − 315= 14(1714 + 1714 ⋅ 3 + 1712 ⋅ 32 +  + 314 ) . Mötərizə
içərisində yalnız tək toplananlar vardır. Bu toplananların sayı
nə qədərdir? Məlumdur ki, a n − b n fərqini vuruqlara
ayırdıqda ikinci vuruqda n = 15 olduğundan, toplananların
sayı da 15-dir. Deməli, mötərizə içərisindəki toplananların
cəmi tək ədəddir və 1715 − 1315 fərqi 4-ə bölünmür.

482. Verilmiş ifadəni aşağıdakı şəkildə çevirək:


2 ⋅ 7 n + 1 =2 ⋅ 7 n + 3 − 2 =2(7 n − 1) + 3 .
Burada 7 n − 1 fərqi 7 − 1 =6 -ya bölündüyündən bütün cəm
də 3-ə bölünür.

483. Veilmiş cəmi aşağıdakı şəkilə çevirək:


2n + 2 ⋅ 3n + 5n − 4 =4 ⋅ 2n ⋅ 3n − 4 + 5n =4(6n − 1) + 5n =
= 5(4 ⋅ (6n −1 + 6n − 2 +  + 6 + 1) + n)=
= 5(4(6n −1 − 1) + 4(6n − 2 − 1) +  + 4(6 − 1) + 5n)
Sonuncu cəmdə hər bir toplanan 5-ə bölündüyündən bütün
hasil 25-ə bölünür. Qeyd edək ki, sonuncu çevirmədə
n= −4n + 5n şəklində göstərildikdən sonra −4n hər biri −4
-ə bərabər olan n sayda toplananın cəmi şəklində

258
göstərilmiş və 4 ⋅ 6n−1 − 4, 4 ⋅ 6n− 2 − 4, , 4 ⋅ 6 − 4 fərqləri
yaradılmışdır.

484. Verilmiş cəmi aşağıdakı şəkildə çevirək:


11n + 2 + 122 n +1 = 112 ⋅11n + 12 ⋅122 n = 121 ⋅11n + 12 ⋅144n =
=12 ⋅144n − 12 ⋅11n + (12 ⋅11n + 121 ⋅11n ) =
= 12(144n − 11n ) + 133 ⋅11n
Burada toplananların hər biri 133-ə bölündüyü üçün, verilmiş
cəm də 133-ə bölünür.

485. Verilmiş üçrəqəmli ədədləri a və b ilə işarə edək. On-


da şərtə görə a + b cəmi 37-yə bölünür. Altı rəqəmli ədədi
1000a + b şəklində yazmaq olar. Aşağıdakı şəkildə çevirmə
edək: 1000a +=b 999a + (a + b) . Alınmış cəmdə birinci top-
lanan 999a =111 ⋅ 9a =3 ⋅ 37 ⋅ 9a =27 a ⋅ 37 olduğundan, 37-
yə bölünür. Məsələnin şərtinə görə ikinci toplanan da 37-yə
bölündüyündən, bütün cəm də 37-yə bölünür.
486. Bütün belə üçrəqəmli ədədlərin sayı 999-110=889-dur.
Verilmiş ədədi aşağıdakı şəkildə yazaq:
S = a1 + a2 ⋅103 + a3 ⋅106 +  + a888 ⋅102661 + a889 ⋅102664 ,
burada a1 , a2 , , a889 . Məsələnin şərtində verilmiş üçrəqəmli
ədədlərdir. Bu cəmi aşağıdakı şəkildə çevirək:
S = (a1 + a2 +  + a889 ) + a2 (103 − 1) + a3 (106 − 1) +  +
+ a889 (102664 − 1)
Sonuncu cəmdə 2-ci toplanandan başlayaraq bütün cəmlər
103 − 1 = 999 = 27 ⋅ 37 olduğu üçün 37–yə bölünür. Birinci
toplanan isə a1 + a2 +  + a889 = 111 + 112 +  + 999

259
olduğundan kənarlardan bərabər məsafədə yerləşən
toplananların 111 + 999 ≠= 112 + 992
998 =113 + 997
999 = =1110
cəmi 37 yə bölünür. Burada ortada 445 ədədi tək qalsa da,
o da 37 -yə bölündüyü üçün, a1 + a2 +  + a 889 cəmi və
deməli S cəmi də 37 -yə qalıqsız bölünür.
487. a + 7b cəmini 6-ya vuraraq 6a + 11b cəmindən çıxaq.
Onda 6a + 11b − 6(a + 7b) =+
6a 11b − 6a − 42b =−31b.
Alınmış fərq və 6a + 11b cəmi 31-ə bölündüyündən
6a + 42b və deməli, a + 7b cəmi 31-ə bölünür.
488. Axtarılan rəqəmləri x, y və z ilə işarə edək. Onda
altırəqəmli ədəd 456xyz şəklinə düşər. Burada
=
456 xyz 456000 + xyz və 456000 = 504 ⋅ 904 +384 olduğu
üçün, 456 xyz = 504 ⋅ 904 + 384 + xyz olar. Verilmiş ədədin
504-ə bölünməsi üçün, 384 + xyz cəmi 504-ə bölünməlidir.
Bu iki halda ola bilər: 384 + xyz =
504 və 384 + xyz =⋅
2 504 .
Birinci halda xyz = 504 − 384 = 120, ikinci halda
xyz = 1008 − 384 = 624 alarıq.

489. = a 2k + 1 , k ∈Zz olduğunu nəzər alaraq, aşağıdakı


şəkildə çevirmələr edək:
a 2 − 1 = (a − 1)(a + 1) = (2k + 1 − 1)(2k + 1 + 1) = 2k (2k + 2) =
= 4k (k + 1)
Burada k (k + 1) ardıcıl iki natural ədədin hasili olduğundan,
cüt ədəddir. Deməli, a 2 − 1 fərqi 8 -ə bölünür.

490. 120 = 23 ⋅ 3 ⋅ 5 olduğunu nəzərə alaraq, verilmiş ifadəni


vuruqlara ayıraq:
260
n5 − 5n3 + 4n= n(n 4 − 5n 2 + 4)= n(n 2 − 4)(n 2 − 1)=
= (n − 2)(n − 1)n(n + 1)(n + 2) = n5 − 5n3 + 4n
n5 – 5n3 + 4n çoxhədlisinin 120 -ə bölünməsini isbat etmək
üçün onun 3, 5 və 8 -ə bölündüyünü isbat etmək kifayətdir.
Beş ardıcıl natural ədəddən biri 3 və digəri 5-ə bölünür.
Göstərək ki, bu hasil həm də 8-ə bölünür. Əgər n cüt ədəd
olarsa, ( n − 2, n və n + 2 hasili 8-ə bölünür. Əgər n tək ədəd
olarsa, (n − 1)(n + 1) hasili 8 -ə bölünər. Deməli, verilmiş
çoxhədli 120 -yə bölünür.
491. Bucaqlı bölmədən istifadə edək:

-5
2k 2 + k − 8 5
Onda = 2k + 3 − olar.
k −1 k −1
Bərabərsizliyin sağ tərəfi tam ədəd olduğu üçün 5 ədədi k − 1
fərqinə qalıqsız bölünməlidir. Bunun üçün k − 1 ikihədlisi -1;
1, -5, 5 qiymətlərini ala bilər. Onda k − nın alacağı qiymətlər
6, 2, 0, -4 olar.

x3 + 2 x − 3
492. Tutaq ki, = a, a ∈ Zz .Onda
x2 + 1
x 2 + 2 x − 3= ax 2 + a və ya (a − 1) x 2 − 2 x + (a + 3) =
0 olar.
İki halı nəzərdən keçirək:
1) Tutaq ki a = 1 . Onda sonuncu tənlik xətti olar:
−2 x +=
4 0, = x 2.
261
2) Tutaq ki a ≠ 1. Bu halda kvadrat tənlik alınır. Onun
diskriminantı mənfi olmalıdır:
1
D =1 − (a − 1)(a =3) ≥ 0, 1 −1a–2 +a2a+− a3a–≥3a0,≥0,
4
a 2 + 2a − 4 ≥ 0
Sonuncu bərabərsizliyin tam həllərini tapaq: burada
−1 − 5 ≤ a ≤ −1 + 5 və 5 ≈ 2, 2 olduğundan, tam həllər
−3, −2, −1, 0,1 olar.
1
a = −3 olduqda, −4 x 2 − 2 x =
0 , 2 x 2 + x =0; x1 =0, x2 =− .
2
a= −2, a = −1 və a = 0 hallarına da uyğun şəkildə baxılır.
1 1 −1 ± 5
Beləliklə, x -ın −3, −1, − , 0, , 1, 2, qiymətlərində
2 3 2
verilmiş kəsr ifadə tam qiymətlər alır.
493. a ədədi olaraq on ardıcıl ədəddən 10-a bölünən ədədi
seçək, belə ədəd həmişə var və yeganədir. b və c ədədləri
kimi son rəqəmləri cəmi 10 olan (məsələn, birinin son rəqəmi
3 digərinin 7) ədədləri seçək. Onda a (b + c) hasili 100-ə
bölünər.
494. Bu ədədlərdən 2-cisini n ilə işarə edək. Onda bu üç
ardıcıl ədədin kubları cəmini aşağıdakı şəkildə çevirək:
(n − 1)3 + n3 + (n + 1)3 = n3 − 3n 2 + 3n − 1 + n3 +
+ n3 + 3n 2 + 3n + 1= 3n3 + 6n= 3n(n 2 + 2)
Vuruqlardan biri 3 olduğu üçün bu cəm 3-ə bölünür.
Göstərək ki, bu cəm həm də 9-a bölünür. İki halı nəzərdən
keçirək.

262
1) Tutaq ki, n ədədi 3-ə bölünür. Onda sonuncu hasil də 3-ə
bölünər.
2) Tutaq ki, n ədədi 3-ə bölünmür. n 2 -nın 3-ə
bölünməsindən alınan qalıq 1 olacaq, başqa sözlə
n 2 =3k + 1(k =0,1, 2). Bu halda
n 2 + 2= (3k + 1) 2 + 2= 9k 2 + 6k + 1 + 2= 9k 2 + 6k + 3 olduğundan,
n 2 + 2 cəmi 3-ə bölünər. Deməli, bu halda da 3n(n 2 + 2)
hasili 9-a bölünər.
495. Natural ədədin kvadratının 11-ə bölünməsindən alınan
qalıqları nəzərdən keçirək
a -nın 11-ə bölünməsindən alınan qalıqlar 0 1 2 3 4 5 6 7 8 9 10
a 2 -nın 11-ə bölünməsindən alınan qalıqlar 01495335941

Cədvəli nəzərdən keçirsək, görərik ki, iki natural ədədin


kvadratları cəmi 0 və ya 11 yalnız o vaxt ola bilər ki, bu
ədədlərdən hər birinin 11-ə bölünməsindən alınan qalıq 0
olsun, yəni a və b ədədlərindən hər biri 11-ə bölünsün.
496. Əksini fərz edək, tutaq ki, elə bir n ədədi var ki,
n 2 + 3n + 4 ifadəsi 49 -a bölünür. Onda
n 2 + 3n +=
4 49k , k ∈ Zz .
Bu tənliyi n-ə görə kvadrat tənlik kimi həll edək:
n 2 + 3n + (4 − 49k ) =
0
Bu tənliyin tam həllərinin olması üçün onun diskriminantı
hər hansı tam ədədin kvadratı olmalıdır:
D =−
9 4(4 − 49k ) = a 2 ; 4 ⋅ 49k − 7 =a2
Sonuncu tənliyin sol tərəfi 7-yə bölünməlidir. Onda a ədədi
də 7-yə və deməli a 2 ədədi 49-a bölünməlidir. Beləliklə, son
tənliyin sağ tərəfi 49-a bölündüyü halda sağ tərəfi bölünmür.

263
Burada ziddiyyət alındığı üçün bizim fərziyyəmiz doğru
deyil.
497. Verilmiş cəmlərin fərqini aşağıdakı şəkildə çevirək:
(mn + pq ) − (mq + np ) = m(n − q ) − p (n − q ) = (n − q)(m − p)
Göründüyü kimi bu fərq və mn + pq azalanı m − p ifadəsinə
bölünür və deməli mq + np çıxılanı da m − p ikihədlisinə
bölünməlidir.
498. Cəmi əmələ gətirən toplananların sayı ( p − 1) -dir.
p > 2 sadə ədəd olduğu üçün tək ədəddir. Onda p − 1 cütdür.
Cəmi aşağdakı şəkildə yazaq:
 1  1 1  1 1 
S = 1 + + + + +  + +
 p − 1   2 p − 2   3 p −3
 
 1 1  p p
+ + = + +
( p + 1)  p − 1 2( p − 2)
1 1
 ( p − 1)
2 2 
p p
+ + +
3( p − 3) 1
( p 2 − 1)
2
Bu kəsrləri toplasaq, surətdə p -yə bölünən bir ədəd,
məxrəcdə isə 1 ⋅ 2 ⋅ 3 ⋅ ⋅ ( p − 1) = ( p − 1)! ədədi alınacaq.
( p − 1)! ədədi p ilə qarşılıqlı sadə olan bir ədəd olduğundan,
p -yə ixtisar oluna bilməz.

499. Fərz edək ki, belə bir ab ədədi vardır. Yəni,


= ⋅ba , 1 < k ≤ 9, k ∈ N . Onda
ab kkba
ab + ba =kkba
⋅ba + ba = (k + 1)ba .

264
Digər tərəfdən ab + ba= (10a + b) + (10b + a )= 11(a + b) .
Lakin (k + 1)ba = 11(a + b) bərabərliyi mümkün deyil, çünki
nə k + 1, nə də ba ədədi 11-ə bölünmür.
Deməli, məsələnin şərtini ödəyən ab ədədi yoxdur.

500. 1) Tutaq ki, 0 < a < 1, onda


ax 1 1
lim =
lim a x ⋅ lim =
0⋅ =
0.
x →∞ 1 + a x x →∞ x →∞ 1 + a x
1+ 0
ax 1 1
2) Əgər a = 1 olarsa, onda lim = = .
x →∞ 1 + a x 1+1 2
ax 1 1
3) Əgər a > 1 olarsa, onda lim = = = 1
x →∞ 1 + a x
+1 0 +1
1
ax

501. 1) 0 < a < 1 olsun, onda


a x − a− x a2x −1 0 −1
lim = lim = = −1
x →∞ a x + a − x x →∞ a 2 x + 1 0 +1
2) a = 1 olarsa, onda
a x − a− x 1 −1
lim x = = 0
x →∞ a + a − x 1+1
3) a > 1 olarsa, onda
1
−x 1− 2x
a −a
x
a 1− 0
lim x = lim = = 1
x →∞ a + a − x
1+ 2x 1+ 0
x →∞ 1
a

265
502.
xm −1 ( x − 1)( x m −1 + x m − 2 +  + x + 1)
lim =
lim
x →1 x n − 1 x →1 ( x − 1)( x n −1 + x n − 2 +  + x + 1)
x m −1 + x m − 2 +  + x + 1 m
=
lim
x →1 x n −1 + x n − 2 +  + x + 1 n
n(n − 1)
503. Burada 2n = (1 + 1) n = 1 + n + + +1 =
2
 n2 n  n n2 n2
1+ n +  −  + +1 = 1+ + + +1 >
 2 2 2 2 2
n2 1 2
olduğunu nəzərə alsaq, 2n > ⇒ 0 < n < 2 olar.
2 2 n
Sonuncu bərabərsizliyin hər tərəfini n -ə vursaq,
n 2 n 2
0 < n < ⇒ 0 ≤ lim n ≤ lim alarıq.
2 n n →∞ 2 n →∞ n
2 n
Lakin lim = 0 olduğu üçün lim n = 0 olur.
n →∞ n n →∞ 2

504. n
n − 1= z , z > 0 olsun. Onda
n(n − 1) 2
n =(1 + z ) n =1 + nz +
⋅ z + + zn
2
n(n − 1) 2 2 2 4
Buradan n > z ,z < < ( n > 2) ,
2 n −1 n
2 2
0< z< və ya 0 < n n − 1 < .
n n
Beləliklə, lim n n = 1 .
n →∞

266
x − a + x−a
505. Burada =
x2 − a2
(=x − a + x − a )( x + a )
x −a ( x + a) 2 2

x−a+ x−a( x + a) x−a + x + a


=
x −a ( x + a)
2 2
x+a( x + a)
Beləliklə,
x − a + x+a x−a + x + a
= lim
lim =
x→a
x −a
2 2 x→a
x+a x + a ( )
0+ a + a 1
= =
(
2a a + a ) 2a

506. Burada

1− x − 3 (= 1 − x − 3)( 1 − x + 3) ( 4 − 2 x + x ) 3 3 2

2+ 3 x ( 2 + x ) ( 4 − 2 x + x ) ( 1 − x + 3)
3 3 3 2

(1 − x − 9 ) ( 4 − 2 3 x + 3 x 2 ) 4 − 2 3 x + 3 x2
= = −
(8 + x) ( 1− x + 3 ) 1− x + 3

olduğunu nəzərə alsaq,


1− x − 3 4 − 2 3 x + 3 x2 4+4+4
lim =
lim =
− =
−2 olar.
x →−8 2+ x
3 x →−8 1− x + 3 3+3

267
1− x − 3
Beləliklə, lim = −2
x →−8 2+ 3 x

507. Aşağıdakı şəkildə çevrilmələr edək:


1
1+
n2 + n
3
n2 + n n
=
lim =
lim 3 lim
x →∞ n − 2 (n − 2)3 x →∞ 3 2
x →∞
 2
(n − 2) 1 − 
 n
2
 1 1  2
Burada lim 1 +  = 1, lim = 0, lim 1 −  =
1
x →∞ n − 2
x →∞
 n x →∞
 n
olduğunu nəzərə alsaq,
1
1+
n= 1⋅ 0
lim 3 3= 0 olar.
x →∞  2 1
(n − 2) 1 − 
 n

508. Tutaq ki, 3


( x + 1) 2 − 3 ( x − 1) 2 =
f ( x) , onda

f ( x)
( 3 2 3
( x + 1) − ( x − 1)
=
2 3
)( 4 3 2 2 3
( x + 1) + ( x + 1) ( x − 1) + ( x − 1)
4
)
3 4 3 2 2 3 4
( x + 1) + ( x + 1) ( x − 1) + ( x − 1)

( x + 1)3 − ( x − 1)3
=
3
( x + 1) 4 + 3 ( x + 1) 2 ( x − 1) 2 + 3 ( x − 1) 4
4x
=
3
( x + 1) 4 + 3 ( x + 1)( x − 1) 2 + 3 ( x − 1) 4

268
4
=
3 2 3
 1  1  1  1
3
1 +  ⋅ ( x + 1) + 3 1 +  1 −  ( x − 1) + 3 1 −  ( x − 1)
 x  x  x  x
3
 1
Burada lim 1 +  ( x + 1) =
3 ∞,
x →∞
 x
2 3
 1  1  1
lim 3 1 +  1 −  ( x − 1) =∞, lim 3 1 −  ( x − 1) =∞
x →∞
 x  x x →∞
 x
olduğu üçün, lim f ( x) = 0 alarıq.
x →∞

509. Aşağıdakı şəkildə çevrilmələr aparaq:

( )( )=
3
x2 x3 + 1 − x3 − 1 x3 + 1 + x3 − 1
lim
x →∞
x3 + 1 + x3 − 1
3
x 2 ( x3 + 1 − x3 + 1)
3
2⋅ x2
= lim= lim =
x →∞
x3 + 1 + x3 − 1 x →∞ x3 + 1 + x3 − 1
2 2
= lim = = 1
x →∞ 1 1 1+1
1+ 3 + 1− 3
x x
 3
Deməli, lim  x 2
x →∞

( 
x3 + 1 − x3 − 1  =

)
1.

510. Aşağıdakı şəkildə çevirmələr aparaq:

lim
x →∞
( ( x + a )( x + b) − x = )
269
=
lim
(( x + a )( x + b) − x )(
( x + a )( x + b) + x )
x →∞ ( x + a )( x + b) + x
ab
(a + b) x + ab a+b+
=
lim lim x =
x →∞
x + (a + b) x + ab + x
2 x →∞ a + b ab
1+ + 2 +1
x x
a+b
=
2
a+b
Beləliklə, lim
x →∞
( ( x + a )( x + b) − x = .
2
)

511. Aşağıdakı şəkildə çevirmələr aparaq:


3
1 + x2 −1
=
(
( 3 1 + x 2 − 1) 3 (1 + x 2 ) + 3 1 + x 2 + 1 )
( )
lim lim
x →0 x2 x →0
x 2 3 (1 + x 2 ) 2 + 3 1 + x 2 + 1

( 1 + x − 1)( (1 + x ) + 1 + x + 1)
3 2 3 2 2 3 2

=
x ( (1 + x ) + 1 + x + 1)
lim
x →0 2 3 2 2 3 2

x2
=
( )
lim
x →0 2 3
x (1 + x 2 ) 2 + 3 1 + x 2 + 1
1 1
=
lim
x →0 3
(1 + x ) + 1 + x + 1 3
2 2 3 2

3
1 + x2 −1 1
Deməli, lim =
x →0 x2 3

270
512.

lim
x ( x2 + 1 − x
=
lim
x )(=
x +1 + x)
2

x +1 + x
x →∞ x →∞
x +1 + x
2
2

1 1
= lim=
x →∞ 1 2
1+ 2 +1
x

1 − cos(1 − cos x)
513. Tutaq ki, = f ( x). Onda
x4
 x  x
1 − cos  2sin 2  2sin 2  sin 2 
=f ( x) =  2
=  2
4 4
x x
2
x   sin  sin 2 x  
4
 x x 
4
2sin  sin 2  16  
2
sin    
1  2  
=
2 2 = 2 ⋅ 
: ⋅  
 2 x
2
sin 4 x 8  x   sin 2 x 
 sin 
 2 2  2   2 

x  x
sin sin  sin 2 
Burada lim 2 = 1 və lim  2
=1
x →0 x x →0 2 x
sin
2 2
olduğunu nəzərə alsaq,
2
 x
4
  2 x
1
sin   sin  sin 2   1
lim f ( x) = lim  2 ⋅    = ⋅14 ⋅12 = 1
x →0 x →0 8 x   sin 2 x  8 8
 
 2   
 2 

271
514. Bərabərsizliyi aşağıdakı şəkildə yazaq:
2 1 1
1+ + + ≥ 3+ 2 2 (1)
sin 2α sin α cos α
1 1 1 2 2
Burada + ≥2 = ,
sin α cos α sin α cos α sin 2α
2 1 1 2 2 2
1+ + + ≥ 1+ + ≥
sin 2α sin α cos α sin 2α sin 2α
≥ 1+ 2 + 2 2 = 3 + 2 2
 1  1 
Beləliklə, 1 +  1 +  ≥ 3+ 2 2 .
 sin α   cos α 

tgx − sin x
515. Tutaq ki, = f ( x) . Onda
x3
x
2tgx ⋅ sin 2
tgx(1 − cos x) 2
=f ( x) = =
x3 x3
2
 x
sin 
1 1 sin x  2
= 2⋅ ⋅ ⋅ ⋅ 
4 cos x x  x 
 2 
Beləliklə,
2
 x
1 1 sin x  sin 2 
lim f ( x)= lim ⋅ lim ⋅ lim  =
x →0 2 x →0 cos x x →0 x x →0  x 
 2 
1 1
= ⋅1 ⋅1 ⋅12 =
2 2

272
1 − cos 2 x 1 − cos 2α

sin x − sin α
2 2
2 2
=
516. lim =
lim
x→a x2 − α 2 x→a x2 − α 2

sin( x + α ) sin( x − α ) sin 2α sin 2α


= lim ⋅ lim = = ⋅1
x→a x +α x → a x −α 2α 2α

π 
517. Tutaq ki,  − x  ⋅ tgx = f ( x) . Onda
2 
π  π  π  1
f ( x) =  − x  ctg  − x  = cos  − x  ⋅
2  2  2  sin  π − x   π − x 
   
2  2 

Beləliklə,
π  1
lim= f ( x) lim cos  − x  ⋅ lim
x→
π
x→
π
2  x → π2 sin  π − x   π − x 
2 2    
2  2 
π 
sin  − x 
π  2  =1
lim cos  − x = cos 0= 1 və lim
π
 2  π π
x→
2
x→
2 −x
2
1
olduğundan, lim f ( x) =1 ⋅ =1 alarıq.
π 1
x→
2

16 4!
518. n = 2 olarsa, < bərabərsizliyi doğrudur.
3 (2!) 2
Fərz edək ki, verilmiş bərabərsizlik n = k üçün doğrudur,
yəni

273
4k (2k )!
< (1)
k + 1 (k !) 2
İsbat edək ki, onda verilmiş bərabərsizlik n= k + 1 üçün də
4k +1 (2(k + 1))!
doğrudur, yəni < (2)
k + 2 ((k + 1)!) 2
4k +1 4k ⋅ 4(k + 1)
Burada = olduğunu nəzərə alsaq,
k + 2 (k + 2)(k + 1)
(1) və (2) ifadələrindən
4k +1 (2k + 2)!⋅ 2 (2k + 2)!⋅ 2 (2(k + 1))!
< < =
k + 2 (k !) (2k + 5k + 2) (k !) (k + 1) ⋅ 2 ((k + 1)!) 2
2 2 2 2

alarıq. Onda riyazi induksiya prinsipinə görə istənilən


n > 2 üçün verilmiş bərabərsizlik doğrudur.
519. Aşağıdakı şəkildə çevrilmələr aparaq:
 
tg 2 x  2 sin 2 x 1 1 
lim = lim  ⋅ ⋅ ⋅ 
x → 0 sin 5 x x →0 5
 2 x sin 5 x cos 2 x 

 5x 
Birinci görkəmli limitə görə
sin 2 x sin 5 x 1
lim = 1, lim = 1 və lim =1
x →0 2x x → 0 5x x → 0 cos 2 x
tg 2 x 2 2
olduğu üçün lim = ⋅1 ⋅1 ⋅1 = alarıq.
x → 0 sin 5 x 5 5

520. Verilmiş tənliyi aşağıdakı şəkildə yazaq:


1 − cos 2 x cos 2 x − cos 6 x cos 6 x − cos12 x
+ + + +
2 2 2

274
cos(n 2 − n) x − cos(n 2 + n) x 1 − cos(n 2 + n) x
+ =
1, və ya = 1.
2 2
Buradan cos(n 2 + n) x = −1 və ya (n 2 + n) x = π (2k + 1) və
π (2k + 1)
x= , k = 0, ±1, ± 2, alınır.
n(n + 1)

521. Tutaq ki, x1 və x2 ədədləri x 2 + px + q üçhədlinin tam


kökləridir. Onda p = −( x1 + x2 ), q = x1 x2 .Buradan
30 = p + q = ( x1 − 1)( x2 − 1) − 1 və ya ( x1 − 1)( x2 − 1) =
31 .
Alınmış 31 ədədi sadə ədəd olduğu üçün onu 1 ⋅ 31 və ya
(−1) ⋅ (−31) tam ədədlərin hasili kimi göstərmək olar. Birinci
halda kökləri 2 və 32 olan x 2 − 34 x + 64 üçhədlisi, ikinci
halda kökləri 0 və -30 olan x 2 + 30 x ifadəsini alırıq.
522. Bərabərsizliyi aşağıdakı şəkildə yazmaq olar:
5 x + 13x
> 23 x =
8x .
2
Ədədi və həndəsi orta haqqında teoremə əsasən
5 x + 13x
> 5 x ⋅13x = 65 x > 64 x = 8 x yazmaq olar.
2
Deməli, verilmiş bərabərsizlik doğrudur.

523. n = 3 qiyməti üçün 23 > 7 doğrudur.


Tutaq ki, n = k üçün də verilmiş bərabərsizlik doğrudur,
yəni 2k > 2k + 1
1) İsbat edək ki, onda n= k + 1 üçün də bərabərsizlik
doğrudur, yəni 2k +1 > 2(k + 1) + 1= 2k + 3 və ya
2 k ⋅ 2 > 2k + 3 (2).
275
(1) bərabərsizliyinə görə (2k + 1) ⋅ 2 > 2k + 3 (3) doğrudursa,
onda (2) bərabərsizliyi də doğrudur. (3) bərabərsizliyindən
2k > 1 doğru bərabərsizliyi alındığı üçün riyazi induksiya
prinsipinə görə istənilən n ≥ 3 üçün verilmiş bərabərsizlik
doğrudur.
524. Tənliyin həllinin olması üçün
sin x ≥ 0, cos x ≤ 0 olmalıdır. Çevirmələr aparsaq:

( ) ( − cos x ) ⇒ sin x =
2
=
2
sin x cos 2 x.
Beləliklə, misalın həllinin olması üçün sin x ≥ 0, cos x ≤ 0 və
5 −1
sin 2 x + sin x − 1 =0 olmalıdır. Buradan sin x = .
2
Lakin cos < 0 olduğundan,
5 −1
x = π (2k + 1) − arcsin , k = 0, ± 1, ± 2,
2
π π 1 1
525. Burada <α < olduğu üçün, sin α > və cos α > .
6 3 2 2
1 1 π
Ona görə də < 2 və < 2 . 0 ≤ x ≤ olduğunu
sin α cos α 2
π sin x π π
nəzərə alsaq, 0 ≤ ≤ < və
4sin α 4sin α 2
π cos x π π
0≤ ≤ < .
4 cos α 4 cos α 2
 π sin x   π cos x 
Beləliklə, tg   > 0, tg   > 0.
 4sin α   4 cos α 
x = α , x < α , x > α hallarının hər birini ayrılıqda nəzərdən
keçirək. Əgər x = α olarsa,

276
 π sin α   π cos α  π
tg   + tg  = 2tg = 2 > 1.
 4sin α   4 cos α  4
Əgər x < α olarsa,
 π sin x   π cos x   π cos x 
tg   + tg   ≥ tg  >
 4sin α   4sin α   4 cos α 
 π cos α  π
> tg  = tg = 1
 4 cos α  4
Nəhayət, əgər x > α olarsa,
 π sin x   π cos x   π sin x   π sin α 
tg   + tg   ≥ tg   > tg  =
 4sin α   4 cos α   4sin α   4sin α 
π
= tg= 1 alarıq.
4
526. Aşağıdakı şəkildə çevirmələr edək:
cos(a + x) − cos(a − x) −2sin a sin x
lim = lim =
x →0 x x →0 x
sin x
= −2sin a ⋅ lim =
−2sin a ⋅1 = −2sin a
x →0 x

π π π
527. Tutaq x − =
z , onda x= z + . Bu halda x →
3 3 3
olduqda z → 0 olar. Aşağıdakı şəkildə çevirmələr aparaq:

 π
sin  x − 
=
lim  3
=
lim
sin z
x→
π 1 − 2 cos x z → 0  π
3 1 − 2 cos  z + 
 3

277
z
cos
sin z 2 1 3
= lim = lim = =
z →0 1 − cos z + 3 sin z z →0 sin z + 3 cos z 3 3
2 2

π 1
528. Tutaq ki, sin x = z , onda x → olduqda z → .
6 2
Aşağıdakı şəkildə çevirmələr aparaq:

lim =
2sin 2 x + sin x − 1
=
lim
( z − 1)( z + 1)
x → 2sin x − 3sin x + 1 z → ( 2 z − 1)( z − 1)
π 2 1
6 2

3
z +1
= lim = 2 = −3 .
1 z −1 1
z→
2 −
2
2tgα
529. tg 2α = olduğunu nəzərə alsaq, verilmiş
1 − tg 2α
tənliyi aşağıdakı şəkildə yazmaq olar:
2tg 5 x 2
= və ya
1 − tg 5 x (1 − tg 5 x) cos 5 x
2 2

sin 5 x 1
=
(1 − tg 5 x) cos 5 x (1 − tg 5 x) cos 5 x
2 2

Əgər verilmiş tənliyin həlli varsa, onda o sin 5 x = 1 tənliyinin


də həlli olmalıdır. Lakin bu halda cos 5 x = 0 olduğundan
verilmiş tənliyin mənası olmaz.
530. Verilmiş bərabərsizliyi aşağıdakı şəkildə yazaq:

( ) ( )
2 2
x −1 +1 + x −1 −1 ≥2 (1)

278
Bərabərsizliyin sol tərəfi x ≥ 1 olduqda təyin olunmuşdur.
(1) bərabərsizliyini aşağıdakı şəkildə yazaq:
x −1 +1 + x −1 −1 ≥ 2 (2)

Əgər x − 1 ≥ 1 və ya x ≥ 2 olarsa,
x − 1 + 1 + x − 1 − 1 ≥ 2 və ya 2 x − 1 ≥ 2, x ≥ 2 alarıq.
Əgər x − 1 < 1 və ya 1 ≤ x < 2 olarsa, (2) bərabərsizliyi
2 = 2 bərabərliyinə çevrilir.
Beləliklə, verilmiş bərabərsizliyin həlli x ≥ 1 qiymətlər
çoxluğudur.

531. x − 2 =z və ya x= z + 2 əvəzləməsi edək.


Əgər x → 2 olarsa, onda z → 0 olar.
x2 − 4 z ( z + 4) z ( z + 4)
Beləliklə, =
lim lim = lim =
x→2 π x z →0 π ( z + 2) z →0 πz
cos cos − sin
4 4 4
π πz
z ( z + 4)
4 4
= lim 4 =− lim 4 ⋅ lim( z + 4) =
π z → 0 π z π z →0 sin π z z →0
− sin
4 4
4 16
=− ⋅1 ⋅ 4 =−
π π
x −4
2
16
Deməli, lim = − .
x→2 πx π
cos
4
532. Aşağıdakı şəkildə çevirmələr edək:

279
sin(3π − 3 x)
sin(3π − 3 x)
=− ⋅ 3π − 3 x
sin 3 x 3
=
sin 2 x − sin(2π − 2 x) 2 sin(2π − 2 x)
2π − 2 x
sin(3π − 3 x)
lim
sin 3 x 3 x →π 3π − 3 x
Onda lim =− ⋅
x →π sin 2 x 2 lim sin(2π − 2 x)
x →π 2π − 2 x
sin(3π − 3 x)
lim
sin 3 x 3 x →π 3π − 3 x 3 1 3
Onda lim =− ⋅ =− ⋅ =−
x →π sin 2 x 2 lim sin(2π − 2 x) 2 1 2
x →π 2π − 2 x

533. Sistemi aşağıdakı şəklə salaq:


x + y = 2 − z
 3
 x + y =2 − z
3 3

Sistemin birinci tənliyini kuba yüksəldək:


x3 + 3 x 2 y + 3 xy 2 + y 3 =8 − 12 z + 6 z 2 − z 3
Bu tənlikdən sistemin ikinci tənliyini çıxaq:
3 x 2 y + 3 xy 2 =−
6 12 z + 6 z 2 , xy ( x + y ) = 2( z − 1) 2 ,
xy ( x + y ) = 2(1 − x − y ) 2 ,
xy ( x + y ) = 2(1 − 2( x + y ) + ( x + y ) 2 ),
xy ( x + y ) =2 − 4( x + y ) + 2( x + y ) 2
Sonuncu tənlikdən görünür ki, 2 ədədi ( x + y ) ikihədlisinə
bölünür. Deməli, ( x + y ) ifadəsi ancaq 1, 2, -1 və -2
qiymətlərini ala bilər. Dörd halın hər birini ayrılıqda
nəzərdən keçirsək: (0; 1; 1), (1; 0; 1), (1; 0; 1), (1; 1; 0)
həllərini alarıq.
280
534. Tənliyin hər iki tərəfinin tangensini tapaq:
 1 1  1
tg  arctg + arctg  = tg  arctg  ,
 x y  2
 1  1
tg  arctg  + tg  arctg 
 x  y 1
= ,
 1  1 2
1 − tg  arctg  ⋅ tg  arctg 
 x  y
1 1
+
x y 1 x+ y 1
= , = , 2 x + 2 y = xy − 1, 2 x + 1 = xy − 2 y
1 1 2 xy − 1 2
1− ⋅
x y
2 x + 1 (2 x − 4) + 5 5
y= = = 2+
x−2 x−2 x−2
y -tam ədəd olduğu üçün ( x − 2) ifadəsi 1, 5, -1 və -5
qiymətlərini ala bilər. Dörd halın hər birini ayrılıqda
nəzərdən keçirsək, (3; 7), (7; 3), (1; -3) (-3; 1) cütlərini
alarıq. Yoxlama aparsaq, alınmış cütlərin tənliyin
həlli olduğunu görərik.
535. Tənliyin sol tərəfini vuruqlarına ayıraq:
xy 2 − xy − y 2 + y= y ( xy − x − y + 1)= y ( y − 1)( x − 1) ,
94 ədədini sadə vuruqlarına ayıraq: 94 = 2 ⋅ 47.
Burada y − 1= 1, y= 2 və x −= 1 47,= x 48 ola bilər.
Beləliklə, tənliyin həlli (48;2) cütü olur.

536. Tənlikdən göründüyü kimi 0 ≤ x ≤ 98 ,


0 ≤ y ≤ 98 olmalıdır. Tənliyi =
y 98 − x şəklində yazaraq

281
hər tərəfi kvadrata yüksəltsək:
y = 98 + x − 2 98 x və ya y = 98 + x − 14 2 x alırıq. Onda
=
2 x 4= a 2 , x 2a 2 . Burada a mənfi olmayan müsbət tam
ədəddir. Lakin x ≤ 98, 2a 2 ≤ 98, a 2 ≤ 49 , 0 ≤ a ≤ 7 .
Beləliklə, a = 0,1, 2,3, 4,5, 6, 7 qiymətlərini alır. x -in bunlara
uyğun qiymətləri x = 0, 2,8,18,32,50, 72,98 olduğundan,
verilmiş tənliyi ödəyən həllər cütünü (0; 98), (2; 72), (8; 50),
(18; 32), (32; 18), (50; 8), (72; 2), (98; 0) olur.

537. x − a = (b − a ) sin 2 t əvəzləməsi edək:


∫ ( x − a )(b − x)dx = 2(b − a ) 2 ∫ sin 2 t cos 2 tdt =
(b − a ) 2 (b − a ) 2  1 
= ∫ (1 − cos 4=
t )dt  t − sin 4t=
+c
4 4  4 
(b − a ) 2 x − a 2 x − ( a + b)
arcsin + ( x − a )(b − x) +
4 b−a 4
+ c ( a < x < b)

538. Hissə-hissə inteqrallama düsturundan istifadə edək:


xdx
= u a 2 + x 2 , dv = dx olsun. Onda du = ,
a2 + x2
x2
v = x və ∫ a 2 + x=
2
dx x a 2 + x 2 − ∫ = dx
a2 + x2
( x2 + a2 ) − a2
= x a +x −∫ 2 2
= dx
a2 + x2
= x a 2 + x 2 − ∫ a 2 + x 2 dx + a 2 n x + a 2 + x 2 + c

Buradan

282
x 2 a2
∫ a 2 + x 2 dx
= a + x 2 + n x + a 2 + x 2 + c .
2 2
539. Hissə-hissə inteqrallama düsturunu tətbiq edək:
dx d (ctgx) ctgx cos 2 x cosx
∫ 3 =∫ =− −∫ 3
dx = − 2 −
sin x sin x sin x sin x sin x
dx x
− ∫ 3 + n tg
sin x 2
dx 1 x cos x
Buradan ∫ 3
= n tg − +c.
sin x 2 2 2sin 2 x

 x−a x+a
=
540. Burada cos a cos  −  eyniliyini tətbiq
 2 2 
edək:
 x − a   x + a 
cos   − 
dx 1   2   2 
∫ = ∫ dx =
sin x − sin a 2 cos a x−a x+a
sin cos
2 2
x−a
sin
1 2 +c
= n
cos a x+a
cos
2

541. Burada
1− x
(1 − x) 1 − x 2 =(1 − x) (1 − x)(1 + x) =(1 − x)(1 + x)
1+ x
1− x 2
olduğundan, = t əvəzləməsi edək. Onda
1+ x

283
1− t2 −4tdt 2t 2 2
=x =
, dx , =1 − x ,=
1+ x .
1+ t 2
(1 + t )
2 2
1+ t 2
1+ t2
Beləliklə,
dx 4tdt
∫ = −∫ =
(1 − x) 1 − x 2 2t 2 2
(1 + t ) ⋅
2 2
⋅ ⋅t
1+ t2 1+ t2
dt 1 1+ x
= −∫ 2 = +c = +c
t t 1− x

542. x = 2a sin 2 t əvəzləməsi edək. Onda sadə çevirmələrdən


sonra alarıq:
x 1
∫x dx =
8a 2 ∫ sin 4 tdt =
a 2 (3t − 2sin 2t + sin 4t ) +
2a − x 4
x 3a + x
=+c 3a 2 arcsin − 2 x(2a − x) + c (0 ≤ x < 2a )
2a 2

1
543. x = əvəzləməsi edək. Onda
t
dt t
− d 

dx
=∫ t 2
= −∫
dt
= −∫ 2 =
x 4x −1
2
1 4
− 4−t 2
t
2

t t2
1 1 −  
2
t 1
=− arcsin + c =− arcsin +c
2 2x

544. Sadə çevirmələr aparsaq, aşağıdakıları alarıq:

284
 1  ctg 5 x
∫ ctg 6 xdx = ∫ ctg 4 x  2 − 1 dx = − −
 sin x  5
 1  ctg 5 x ctg 3 x
− ∫ ctg 2 x  2 − 1 dx = − + − ctgx − x + c
 sin x  5 3

545. 2sin 2 α + 3tgα ⋅ ctgα= 2sin 2 α + 3. Məlumdur ki,


2sin 2 α + 3 ifadəsi özünün ən kiçik qiymətini sin α = 0
olduqda alır. Lakin bu halda ctg α təyin olunmadığından,
verilmiş ifadə də təyin olunmamışdır. Eyni zamanda
2sin 2 α + 3 ifadəsi özünün ən kiçik qiymətini sin α = 1 və ya
sin α = −1 olduqda alır. Lakin bu halda da tgα təyin
olunmadığından, verilmiş ifadə təyin olunmur. Beləliklə,
2sin 2 α + 3tgα ⋅ ctgα ifadəsinin ən böyük və ən kiçik qiyməti
yoxdur.

546. Dərəcəni azaltma düsturlarını tətbiq edək. Onda:


y= 5(1 + cos 2 x) − 3sin 2 x + 1 − cos 2 x=
= 5 + 5cos 2 x − 3sin 2 x + 1 − cos 2 x = 6 + 4 cos 2 x − 3sin 2 x
olar. y = a cos x + b sin x + c funksiyasının ən böyük qiyməti
ymax = a 2 + b 2 + c olduğundan, verilmiş funksiyanın ən
böyük qiyməti ymax = 6 + 32 + 42 = 6 + 5 = 11 olur.

547. Toplananlardan hər birini ayrı-ayrılıqda hesablayaq:


 15π    π   π
=
arccos  sin  arccos  sin  2π= +   arccos= sin 
 7    7   7
  π π   5π  5π
= arccos  cos  = −   arccos = cos  .
  2 7   14  14

285
 4
cos  2 arcsin  ifadəsini hesablayaq:
 5
4  π 4
α = arcsin , α ∈  0;  olsun. Onda sin α = və
5  2 5
16 9 3
cos α = 1 − sin 2 α = 1 − = = . Buradan
25 25 5
 4 9 16 7
cos 2α =
cos  2 arcsin  = cos 2 α − sin 2 α = − = − .
 5 25 25 25
 15π   4  5π 7
Beləliklə, arccos  sin  + cos  2 arcsin  = − .
 7   5  14 25

548. (cos1080 + cos 360 ) sin 700 sin 500 sin100 =


= (sin 540 − sin180 ) cos 200 cos 400 cos800.
Burada sin 540 −=
sin180 2sin180 =
cos 360
2(sin180 cos180 ) cos 360 2sin 360 cos 360 sin 720
= = = =
2 cos180 2 cos180 2 cos180
cos180 1
= = 0
2 cos18 2
Digər tərəfdən
8sin 200 cos 200 cos 400 cos800
=
cos 200 cos 400 cos800 =
8sin 200
2(2(2sin 200 cos 200 ) cos 400 cos800
=
8sin 200
2(2sin 400 cos 400 ) cos800 2sin 800 cos800 sin1600
= = = =
8sin 200 8sin 200 8sin 200

286
sin 200 1
= 0
= olduğundan,
8sin 20 8
1 1 1
(sin 540 − sin180 ) cos 200 cos 400 cos800 = ⋅ = .
2 8 16
1
(cos14α + cos12α )
cos α cos13α
549. =2 .
cos 3α + cos 5a cos 3α + cos 5α
Burada 14α =17α − 3α =π − 3α olduğundan,
cos14α = − cos 3α və 12α =17α − 5α =π − 5α və
cos12α = − cos 5α .
Bu qiymətləri alınmış ifadədə nəzərə alsaq,
1 1
(cos14α + cos12α − (cos 3α + cos 5α )
2 1
= 2 = − alarıq.
cos 3α + cos 5α cos 3α + cos 5α 2

550. y =−6sin 2 2 x − 2sin 2 x cos 2 x + 8 − 3 ifadəsini


nəzərdən keçirək və göstərək ki, x -ın istənilən qiymətində
y > 0 . Sadə çevirmələrdən sonra alarıq:
1 − cos 4 x
y − 6⋅ − sin 4 x + 8 − 3 = 3cos 4 x − sin 4 x +
2
=
+5 − 3 10 cos(4 x + ϕ ) + 5 − 3,
3 1
Burada cos ϕ = , sin ϕ = . Nəzərə alsaq ki,
10 10
10 cos(4 x + ϕ ) ifadəsinin minimal qiyməti − 10 -dur.
y > 0 olduğunu göstərmək üçün − 10 + 5 − 3 > 0 olduğunu
göstərmək kifayətdir. Həqiqətən də
10 + 3 < 5 ⇔ 13 + 2 30 < 25 ⇔ 2 30 < 12

287
⇔ 30 < 6 ⇔ 30 < 36. Beləliklə, təyin oblastı x ∈ R .

551. g (2 x − 3) = 4 x 2 − 8 x + 7 olduğundan,
g ′(2 x − 3) ⋅ (2 x − 3)′ = 8 x − 8 ⇒ 2 g ′(2 x − 2) = 8 x − x ⇒
⇒ g ′(2 x − 3) = 4 x − 4
t +3
2x − 3 =t olsun. Onda x = və
2
t +3   t +3− 2 
g ′(t ) = 4  − 1 = 4   = 2(t + 1)
 2   2 
və ya g ′( x=
) 2x + 2

1 f ′( x) 1 1
552. ln f ( x) = ln( x + 1) ⇒ =− 2 ln( x + 1) +
x f ( x) x x( x + 1)
1
 1 ln( x + 1) 
Onda f ′( x) =
( x + 1) x  −  və
 x( x + 1) x2 
1  e
f ′(1) =
2  − ln 2  =
1 − 2 ln 2 =
ln e − ln 4 =
ln
2  4

553. = x f −1 ( x 5 − 2 x 4 + x 3 − 2 x 2 + 4 x − 4) bərabərliyinin hər


iki tərəfindən törəmə alaq:
f −1 ′ ( x5 − 2 x 4 + x3 − 2 x 2 + 4 x − 4)(5 x 4 − 8 x3 + 3 x 2 − 4 x + 4) =
( )
=1

( f )′ ( x
−1 5
− 2 x 4 + x3 − 2 x 2 + 4 x − 4) =4
1
5 x − 8 x + 3x 2 − 4 x + 4
3

x5 − 2 x 4 + x3 − 2 x 2 + 4 x − 4 =4 tənliyini həll edək:


x5 − 2 x 4 + x3 − 2 x 2 + 4 x − 8 =
0,

288
x 4 ( x − 2) + x 2 ( x − 2) + 4( x − 2) =
0,
( x − 2)( x 4 + x 2 + 4) = 0 ⇒ x = 2

Onda ( f −1 )′ (4)
1 1
= =
5 ⋅ 2 − 8 ⋅ 2 + 3 ⋅ 2 − 4 ⋅ 2 + 4 24
4 3 2

= =
554. ln f ( x) 2sin x ln x, (ln f ( x))′ 2sin x ln x ,
f ′( x)  sin x  2sin x  sin x 
=2  cos xnx +  , f ′( x) =2 x  cos xx + 
f ( x)  x   x 
π  π
π  π   π π sin 2 
2sin
2
= f ′  2   cos =
n +
π 
Onda
2 2  2 2
 2 
π  π 2
2
 2
2
=2 ⋅   ⋅  0 +  =2 ⋅ ⋅ =π
2  π 4 π

555. f ( x) − f ( x − 1) = x 2 bərabərliyində arqumentə 1-dən


x -ə qədər ardıcıl natural qiymətlər verərək alınmış tənliklər
sistemini alt-alta toplayaq:
 f (1) − f (0) = 12

 f (2) − f (1) = 22
+
..........................
 f (1) − f ( x − 1) = x2

f ( x) − f (0) = 12 + 2 +  + x 2
x( x + 1)(2 x + 1)
Burada 12 + 22 +  + x 2 = olduğunu nəzərə
6
1
alsaq, f ( x=
) (2 x3 + 3 x 2 + x) olar. Onda
6

289
f ′( x) =
1
6
( 6 x 2 + 6 x + 1) = x 2 + x + və
1
6
1 1 7 1
f ′(1) + f ′(0) = 1 + 1 + + = = 2
6 6 3 3

556. f ′( x) = 2 x ⋅ g (2 x + 1) + ( x 2 + 1) ⋅ g ′(2 x + 1) ⋅ (2 x + 1)′ =


= 2 x ⋅ g (2 x + 1) + 2( x 2 + 1) ⋅ g ′(2 x + 1) olduğu üçün,
f ′(2) = 4 g (5) + 10 g ′(5) = 4 ⋅ 6 + 10 ⋅ 3 = 24 + 30 = 54

ln x
557. y = funksiyası ən böyük qiymətini x = e -də alır.
x
1 ln 3
Deməli, > ⇒ e3 > 3e
e 3
558. Bərabərsizliyin hər tərəfini kvadrata yüksəldək:
1 − cos 2 2 > 2sin 2 ə ya 1 + cos(π − 2 2) > 2sin 2 .
Göstərək ki, cos(π − 2 2) > cos 0,35 > 0,93 .
Həqiqətən də sin 2 0,35 < (0,35) 2 < 0,13 .
Deməli, cos 2 0,35 =
1 − sin 2 0,35 > 0,87;
cos 0,35 > 0,87 > 0,93 .
Beləliklə, 1 + cos(π − 2 2) > 1,93 . Digər tərəfdən,
 2π  2π  
= 2 2sin  +2−  <
3  
2sin
 3 
 2π  2π  2π  π 
< 2  sin +2−  cos = 3 + 2  − 1 <
 3  3  3  3 
< 1, 75 + 0,1 =
1,85

290
g ( x)
559. F ( x) = ∫ f (u )du üçün
a

′( x) g ′( x) ⋅ f ( g ( x)) olduğunu nəzərə alsaq,


F=
x 1
F ( x) =∫ (t + 2) 2 dt üçün
0

( )
1

x +2 2

( x )′ ⋅ ( )
1

F ′( x)= x +2 = 2
olur. Onda
2 2

(=
4 + 2)
1
2
1
F ′(4)
=
2 4 2

ln x 2 ln x 4 x ln x
F ′( x) ( x 2 )′
560.= − x′ ⋅ = −
2 + ln x
2 2
2 + ln x 2 + 4 ln 2 x
2

ln x 4 x ln x ln x
− = − olduğundan,
2 + ln x 2 + 4 ln x 2 + ln 2 x
2

2e 1 2e − 1
F ′(e) = − = .
3 3 3
561. Aşağıdakı şəkildə çevirmələr aparaq:
dx dx
∫=sec dx ∫= ∫
cos x π
=

sin  − x 
 2 
dx
π x 
cos 2  − 
dx  4 2
∫ π x  π x  = ∫ π x 
2sin  −  cos  −  2tg  − 
 4 2  4 2  4 2

291
π x 
=
Burada t tg  −  əvəzləməsi edək. Onda
 4 2
dx
dt = . Bu əvəzləmələri inteqralda nəzərə
2 π x
2 cos  − 
 4 2
dt 1
alsaq, ∫ sec xdx =−∫ t =−n t + c =n t + c =
π x 
= n ctg  −  + c .
 4 2
α 1 + cos α
Burada ctg = düsturunu nəzərə alsaq,
2 sin α
π 
1 + cos  − x 
π x  2=  + c n 1 + sin x +
n ctg  −=  + c n
 4 2 π  cos x
sin  − x 
2 
+c n sec+ tgx + c alarıq.
=
π
π
6
π π
Onda ∫ sec xdx
= n | sec x + tgx
= |06 n sec − tg −=
n1
0 6 6
2 1 1
= n + = n 3 =  n3
3 3 2

562. Əgər n nəfər bir sıra üzrə əyləşərsə, bütün üsulların


sayı n ! olar. Lakin n nəfər dəyirmi masa ətrafında əyləşərsə,
n halda bu adamların bir-birinə nəzərən vəziyyəti eyni olur
və yalnız masa ətrafında dönmələri ilə fərqlənir. Başqa sözlə
n vəziyyətdə hər adamın sağında və solunda eyni insanlar

292
olduğundan, bu vəziyyətlərin hər biri eyni hesab edilir. Ona
n!
görə də n nəfər dəyirmi masa ətrafında = (n − 1)! üsulla
n
əyləşə bilər.
563. İxtiyari iki qızın (iki oğlanın) yanaşı oturması üçün
qızlar cüt (tək), oğlanlar tək (cüt) saylı yerlərdə
əyləşməlidirlər. n adam n ! üsulla yerini dəyişə bildiyindən,
bütün üsulların sayı 2(n !) 2 sayda olar. Lakin 2n halda oğlan
və qızların bir-birinə nəzərən vəziyyəti eyni olub yalnız masa
ətrafında dönmələri ilə fərqləndiyindən, bütün üsulların sayı
2(n !) 2
= (n − 1)!n ! olar.
2n

564. Tutaq ki, A = {a1 , a2 , , an } verilmiş çoxluqdur. Onda


boş çoxluq, n elementli çoxluğun bütün 0 < k < n elementli
çoxluqları və həmçinin verilmiş n elementdən ibarət çoxluq
da daxil olmaqla bütün çoxluqlar A çoxluğunun alt
çoxluqlarıdır. Hər bir P alt çoxluğuna n uzunluqlu bir vektor
kimi baxaq. Belə ki, əgər vektorun k nömrəli koordinatı
verilmiş çoxluğa daxildirsə, 1, əks halda 0 yazılır. Bu cür
kodlaşdırma sistemi, alt çoxluqlarla kodlar arasında bir
qiymətli uyğunluq yaradır. Hər bir koordinatı 2 üsulla
seçmək mümkün olduğundan, bütün belə vektorların sayı 2n
olar. Deməli, n elementdən ibarət bir çoxluğun bütün alt
çoxluqlarının sayı 2n -dir.
565. Burada sin 5 x ≤ 1, − cos 2 x ≤ 1 olduğu üçün
sin 5 x − 2 cos 2 x ≤ 1 + 2 ⋅1 =3 . Göründüyü kimi tənliyin sol
tərəfi 3 ədədini aşmır, sağ tərəfi isə 3-ə bərabərdir. Bərabərlik

293
sin 5 x = 1,
halı yalnız  olduqda mümkündür. x -ın hər iki
cos 2 x = −1
tənliyi ödəyən qiymətlərini tapmaq üçün tənliklərdən birinin
köklərini tapaq və bu köklər içərisində digər tənliyi ödəyən
həlləri seçək. İkinci tənlikdən:
π 5π
cos 2 x = −1, x= + π k , onda 5= x + 5π k ,
2 2
 5π  π 
sin 5 x = sin  + 5π k  = sin  + π k  .
 2  2 
Aydındır ki, yalnız k -nın cüt qiymətləri üçün sin 5 x = 1
π
ödənilir. Deməli, x= + 2π n verilmiş tənliyin köküdür.
2
566. Məlumdur ki, əgər 0 < a < 1 olarsa, onda y = a t
funksiyası t − nin artması ilə azalır. Onda sin 8 x ≤ sin 2 x,
− cos5 x ≤ cos 2 x . Bu bərabərsizlikləri hədbəhəd alt-alta
toplasaq, sin 8 x − cos5 x ≤ sin 2 x + cos 2 x =1 olar. Beləliklə,
verilmiş tənliyin sol tərəfi o vaxt və yalnız o vaxt 1-ə bərabər
olar ki, sin 8 x = sin 2 x, − cos5 x = cos 2 x bərabərlikləri
ödənilmiş olsun. Bu o vaxt ola bilər ki, sin x ifadəsi -1, 0,
1; cos x isə -1, 0 qiymətlərini alsın. Deməli, verilmiş
tənliyin kökləri
π
x1 = + π k , x2 = π + 2π k , k = 0, ±1, ±2, olur.
2
567. Məlumdur ki, a sin x + b cos x =a 2 + b 2 cos( x − ϕ ) .
Buradan xüsusi halda a sin x + b cos x ≤ a 2 + b 2 bərabər-
sizliyi alınır. Burada a və b parametrləri x –dən asılı ola
bilər.

294
Bu bərabərsizliyi birinci iki toplanana tətbiq etsək və
cos 3 x ≤ 1, 2sin 5 x ≤ 2 olduğunu nəzərə alsaq, taparıq ki,
bütün sol tərəf 7-ni aşmır. Burada bərabərlik halı yalnız
=
| cos 3 x | 1,=
sin 5 x 1 olduqda mümkündür, bu isə ola
bilməz. Deməli, verilmiş tənliyin həlli yoxdur.

568. Tənliyin sol tərəfini cos x − a görə kvadrat tənlik kimi


həll edək. Tutaq ki, bu üçhədlinin diskriminanti D-dir. Onda
1
= D 4(cos 4 3 x − cos 2 3 x) olur. D ≥ 0 bərabərsizliyindən
4
cos 2 3 x ≤ 0 və ya cos 2 3 x ≥ 1 . Deməli iki hal mümkündür:
cos 3 x = 0 və cos 3 x = ±1 . Əgər cos 3 x = 0 olarsa, onda
π
cos x = 0 və ya x= + π k , k ∈ Z alınar. Əgər | cos 3 x | =1
2
1 π
olarsa, onda cos x = və ya x = ± + 2π k , k ∈ Z alarıq.
2 3
Hər iki kök tənliyi ödəyir.

569. Tənliyi aşağıdakı şəkildə yazaq:


x
2 arccos x= π − arcsin .
2
Əgər bucaqlar bərabərdirsə, onların triqonometrik
funksiyaları da bərabərdir. Hər tərəfin kosinusunu tapaq:
 x
=
cos(2 arccos x) cos  π − arccos  . Burada
 2
cos(2 arccos= x) 2 x − 1,
2

 x  x x2
cos  π − arcsin  =− cos  arcsin  =− 1−
 2  2 4

295
x2
olduğunu nəzərə alsaq, 2 x 2 − 1 =− 1 − tənliyini alarıq.
4
Tənliyin hər tərəfini kvadrata yüksəltsək, 4 kök alarıq ki
bunlardan yalnız x = 0 tənliyi ödəyir.
Digər ikisi isə kənar kökdür.
570. Tənliyin sol tərəfini qruplaşdıraraq, çevirmə aparsaq:
( x + 1)( x + 4 ) ⋅ ( x + 2 )( x + 3) =
8,
(x 2
+ 5x + 4) ⋅ ( x2 + 5x + 6) =
8.
x2 + 5x + 4 = t əvəz etsək:
t ( t + 2 ) =8 ⇔ t 2 + 2t − 8 =0 ⇒ t1 =−4, t2 =2 .
Bu qiymətləri əvəzləmədə nəzərə alsaq:
1) x 2 + 5 x + 4 =
−4 ⇔ x 2 + 5 x + 8 =
0, bu halda həqiqi
kök yoxdur.
−5 ± 17
2) x 2 + 5 x + 4 = 2 ⇔ x 2 + 5 x + 2 = 0 ⇒ x1,2 = .
2
571. x = 1 verilmiş tənliyin köküdür. Göstərək ki, tənliyin
digər kökü yoxdur. Tənliyin hər iki tərəfini 7 x -ə bölsək,
x x
3 4
  +  = 1 tənliyini alırıq. Tənliyin sol tərəfi monoton
7 7
azalan olduğu üçün hər bir qiymətini yalnız bir dəfə alır.
Deməli, tənliyin yeganə həlli x = 1 -dir.
572 . Verilmiş tənliyi aşağıdakı şəkildə yazaq:
x, f ( x) = 1 + x funksiyasını nəzərdən
1+ 1+ x =
keçirək. Bu funksiya monoton artandır və f ( f ( x)) = x

296
şərti ödənilir. Məlumdur ki, bu halda f ( f ( x)) = x və
f ( x) = x tənlikləri ekvivalentidir. Onda
1+ 5 3+ 5
1 + x = x ⇒ x − x − 1 = 0, x = ,x= alarıq.
2 2
3
 x3 + 1 
1+  
573. Verilmiş tənliyi aşağıdakı şəklə salaq:  2 
= x.
2
Verilmiş tənlik f ( f ( x)) = x şəklindədir, burada
1 + x3
f ( x) = . f ( x) funksiyası monoton artan olduğu üçün
2
f ( f ( x)) = x tənliyi f ( x) = x tənliyi ilə ekvivalentdir. Onda
x3 + 1
= x, x3 − 2 x +=
1 0, ( x − 1)( x 2 + x − 1)
= 0, x=
1 1,
2
−1 + 5 −1 − 5
=x2 = , x3
2 2

574. f (t ) =t 3 + 2t 2 + 2t funksiyasını nəzərdən keçirək.


Burada f ′(t ) = 3t 2 + 4t + 2 > 0, ∀t ∈ R olduğu üçün bu
funksiya artandır.
Digər tərəfdən= y f= ( x), z f=
( y ), x f ( z ) olduğundan,
x = f ( f ( f ( x))). Onda məlum teoremə görə bu tənlik
f ( x) = x və ya x3 + 2 x 2 + 2 x =x tənliyi ilə ekvivalentdir. Bu
tənliyi həll etsək, x( x + 2 x + 1) =
2
0, x( x + 1) 2 =
0, x = 0 və
ya x = −1 alarıq. Onda tənliklər sisteminin həlli, (0; 0; 0),
(-1; -1; -1) olar.

297
575. Tənliyin sol tərəfi 2-ni aşmır. Sağ tərəfi isə ədədi və
həndəsi orta haqqında teoremə əsasən 2 x + 2 x ≥ 2 olur.
Deməli, sağ və sol tərəfin bərabərlik halı yalnız
x = 0 olduqda mümkündür.
576. Məlumdur ki, vektorun skalyar hasili bu vektorların
uzunluqları hasilini aşmır: a1a2 + b1b2 ≤ a12 + b12 ⋅ a22 + b22 .
Burada bərabərlik halı yalnız bu vektorlar kollinear olduqda
a b
mümkündür, yəni bu halda 1 = 1 şərti ödənilməlidir.
a2 b2
Tutaq ki, ( x;1) və ( )
1 + x ; 3 − x vektorları verilmişdir.
Onda
x 1 + x + 1 ⋅ 3 − x ≤ x 2 + 1 ⋅ (1 + x) + (3 − x=
) 2 1 + x2 .
Verilmiş tənlikdən görünür ki, ( x;1) və ( 1+ x; 3 − x )
x 1
vektorları kollineardır, yəni = ,
1+ x 3− x
x 3 − x= x + 1, x3 − 3 x 2 + x + 1= 0,
( x − 1)( x 2 − 2 x − 1) =0, x1 =1, x2 =1 + 2, x3 =1 − 2 .
Sonuncu kök kənar kökdür.
577. Göstərmək olar ki, bu tənliyin həlli yoxdur.
Həqiqətən də tənliyi sadələşdirib, potensiallasaq:
g (cos x − 0,5)(sin x − 0,3) =
−1
1
(cos x − 0,5)(sin x − 0,3) =alarıq.
10

298
Sol tərəfi ədədi və həndəsi orta haqqındakı teoremə görə
 1 
qiymətləndirək  ab ≤ (a + b) 2  :
 4 
 cos x + sin x − 0,8 
2

(cos x − 0,5)(sin x − 0,3) ≤   ≤


 2 
2
 2 − 0,8   1, 42 − 0,8  2
≤   <   = (0,31) 2 < 0,1
 2   2 
Göründüyü kimi sol tərəf sağ tərəfdən kiçikdir.
Deməli verilmiş tənliyin həlli yoxdur.
578. Burada | x |< 1 olmalıdır. Həqiqətən də | x |≥ 1 olduqda
2 x 2 − 1 ≥ 1 və 8 x 4 − 8 x 2 + 1 ≥ 1 . x = cos t , 0 < t < π
əvəzləməsi edək. Onda
2x2 = − 1 2 cos 2 t=− 1 cos 2t , 8 x 4 − 8 x 2 =
+ 1 2(2 x 2 − 1)=
−1
= 2 cos 2= 2t − 1 cos 4t olduğundan,
verilmiş tənlik 8cos t cos 2t cos 4t = 1 şəklinə düşər.
Tənliyin hər iki tərəfini sin t -yə vurub, çevirmələr aparsaq,
9t 7t
=
sin 8t − sin t 0; 2 cos = sin 0 alarıq. Burada 0 < t < π
2 2
2
olduğunu nəzərə alsaq,
= t = π k , k 1, 2,3 və ya
7
π 2
t= + π k, k = 0,1, 2,3 alarıq. Nəhayət x dəyişəninə
9 9
qayıtsaq, tənliyin aşağıdakı kimi kökləri olar:
2π 4π 6π π 1 5π 7π
cos , cos , cos , cos , , cos , cos .
7 7 π 9 2 9 9

299
589. n -in bütün natural qiymətlərində
(n + 1) 2 ≤ n 2 + 3n < (n + 2) 2 bərabərsizliyi doğrudur. Lakin
(n + 1) 2 və (n + 2) 2 ədədləri ardıcıl tam ədədlərin kvadratları
olduğu üçün, bərabərlik halı yalnız (n + 1) 2 = n 2 + 3n olduqda
mümkündür. Buradan n 2 + 3n = n 2 + 2n + 1, n = 1 alınır.

580. Verilmiş cəmi aşağıdakı şəkildə çevirək:


2n + 4k = 2n + 22 k .
n−2k
Tutaq ki, n > 2k . Onda=2n + 22 k a 2 , 22 k (2
= + 1) a 2 ,
burada a natural ədəddir. Sonuncu bərabərlikdə a 2 və 22 k
tam kvadrat olduğu üçün 2n − 2 k + 1 cəmi də tam kvadrat
olmalıdır: 2n − 2 k + 1 = b 2 (b ∈ N ), 2n − 2 k = b 2 − 1 . Məlumdur ki,
əgər b 2 ədədi 3-ə bölünməzsə, onda b 2 − 1 fərqi 3-ə bölünər.
Bu isə mümkün olmadığından, b ədədi 3-ə bölünməlidir.
Onda 2n − 2 k + 1 cəmi 9-a bölünər.
Tutaq ki, n − 2k = 3, n = 2k + 3. Bu qiyməti 2n + 22 k
ifadəsində nəzərə alsaq,
2n + 22 k = 22 k +3 + 22 k =22 k ⋅ (23 + 1) =⋅ 9 22 k = (3 ⋅ 2k ) 2 olar.
Burada k istənilən natural ədəddir. Beləliklə, k -ya sonsuz
sayda qiymət verməklə, 2n + 22 k cəmi üçün sonsuz sayda
tam kvadrat ala bilərik.

581. ax 2 + bx cəminin tam qiymətlər aldığını göstərsək,


məsələni həll etmiş olarıq:

300
x( x − 1)
ax 2 + bx = (ax 2 − ax) + (ax + bx) = 2a ⋅ + (a + b) x.
2
x( x − 1)
Burada 2a, , a + b və x tam qiymətlər aldığı üçün
2
ax 2 + bx ifadəsi də yalnız tam qiymətlər ala bilər.

582. Axtarılan üçrəqəmli ədədi aaa ilə işarə edək. Onda


ədədi silsilədə n həddin cəmi düsturundan istifadə etsək,
n(n + 1)
= aaa= a ⋅111, n(n + 1) = 2 ⋅ a ⋅111 = 2 ⋅ 3 ⋅ 37 a alarıq.
2
Burada n və ya (n + 1) ədədi 37 -yə bölünməli və ya bərabər
olmalıdır.
1) Tutaq ki, məsələn, n + 1 = 2 ⋅ 37 = 74, onda
=n 3a ≤ 27 olduğu üçün bu mümkün deyil.
2) Tutaq ki, n + 1 =37 onda =n 36 = 3a və ya a = 6 .
Bu hal mümkündür.
3) Tutaq ki, n = 37 -dir. Onda 37 ⋅ 38 = 2 ⋅ 3 ⋅ 37 ⋅ a,
3a = 19 alınar. Bu hal da mümkün deyil. Beləliklə, n = 36,
yəni 36 sayda toplanan götürmək lazımdır.
583. Asanlıqla göstərmək olar ki, k və n -in natural
7 k + 15n − 1
qiymətlərində 2 < <4.
3k + 4n
Deməli, bu kəsr ifadə ancaq 3 -ə bərabər ola bilər:
7k + 15n − 1 3n − 1
= 3, 7k + 15n − 1= 9k + 12n, 3n − 2k = 1, k = .
3k + 4n 2
Buradan görünür ki, k -nın natural ədəd olması üçün
n = 2a − 1(a ∈ N ) olmalıdır. Onda

301
3(2a − 1) − 1 6a − 4
=
k = = 3a − 2 .
2 2
Beləliklə, k =3a − 2, n =2a − 1, a ∈ N .

584. sin 11 – sin 10 fərqini nəzərdən keçirək:


sin 11 – sin 10 = 2 sin 0,5 cos 10,5 .
π π
Burada 0 < 0,5 < , 3π < 10,5 < 3π + olduğundan,
2 2
sin 0,5 >0, cos 10,5 < 0.
Deməli, sin 11 – sin 10 < 0 və ya sin 11 < sin 10.

585. a) y = sin x funksiyasının təyin oblastı x ≥ 0 -dır.


Dövri funksiyanın tərifinə görə bu mümkün deyil.
=
b) y cos x + cos 2 x funksiyası x = 0 qiymətində
2-yə bərabər olur. x -in başqa heç bir qiymətində, 2
əmsalı irrasional olduğu üçün funksiya 2 qiymətini almır.
Ona görə də bu funksiya dövri deyil.
c) y = cos x 2 funksiyasının törəməsi dövri deyildir.
Ona görə də funksiyanın özü də dövri funksiya deyildir.

586. Tənliyin sol tərəfini aşağıdakı şəkildə çevirək:


3 4 
3cos x + 4sin x = 5  cos x + sin x  = 5cos( x − ϕ ), burada
5 5 
4
ϕ = arctg -dür. Onda
3
π 
cos( x − ϕ ) − sin=
5 x 0, cos( x − ϕ ) − cos  − 5=
x  0, onda
2 

302
π 
cos( x − ϕ ) − sin=
5 x 0, cos( x − ϕ ) − cos  − 5=
x  0,
2 
π ϕ  π ϕ 
2sin  − − 2 x  ⋅ sin  + = 3 x = 0 .
4 2  2 2 
Burada iki hal mümkündür:
π ϕ  π ϕ π ϕ πk
1) sin  − − 2 x  = 0, − − 2 x = π k , x = − − .
4 2  4 2 8 4 2
Burada (−k ) -nı k ilə əvəz etsək,
π 1 4 πk
x1 =− arctg + , k ∈ Z olar.
8 4 3 2
π ϕ  π ϕ π
2) sin  + − 3 x  = 0, x2 = + + k , k ∈ Z alarıq.
4 2  12 6 3

587.=tgx u= , tgy v əvəzləməsi edək. Onda sin 2x və sin 2y


triqonometrik funksiyalarını u və v ilə ifadə etsək, aşağıdakı
şəkildə tənliklər sistemini alarıq:
10uv= 12 + 12u ,
2


10uv= 6 + 6v .
2

Birinci tənliyin hər iki tərəfini 2-yə bölərək ikinci tənliyi


çıxsaq, 6u 2 + 5uv − 6v 2 = 0 bircins tənliyini alarıq. Sonuncu
u
tənliyin hər iki tərəfini v 2 -na bölüb, t = əvəzləməsi
v
2 3
aparsaq, 6t 2 + 5t − 6 = 0, t1 = , t2 = − alarıq. x və y
3 2
dəyişənlərinə qayıtsaq,
x= ± arctg 2 + π k , y = ± arctg 3 + π n, k , n ∈ Z alarıq.

303
π
588. Alarıq: 2π k ,
(3 x − 9 x 2 + 160 x + 800) =
8
9 x + 160 x + 800 =3 x − 16k , 160 x + 800 =256k 2 − 96kx,
2

256k 2 − 800 8k 2 − 25 8 40 25
x= = = k− − ,
160 + 96k 3k + 5 3 9 9(3k + 5)
25
9k = 24k − 40 −
2k + 5
25
Burada x tam ədəd olduğundan, - tamdır.
3k + 5
Deməli, 25 ədədi (3k + 5) -ə bölünməlidir. Bu o vaxt ola
bilər ki, 3k + 5 ifadəsi ±1, ±5, ±25 ədədlərinə bərabər olsun.
Bütün variantları nəzərdən keçirsək, k = −2 , 0, -10
qiymətlərini alarıq. k = –2 olduqda x = −7, k = 0 olduqda
x = −5 (kənar kök); k = −10 olduqda x = −31 alarıq.

589. Dörd nöqtədən iki diaqonal keçir. İki diaqonal isə bir
nöqtədə kəsişdiyindən n - bucaqlının diaqonallarının kəsişmə
nöqtələrinin sayı,
n(n − 1)(n − 2)(n − 3) n(n − 1)(n − 2)(n − 3)
= Cn4 = olur.
4! 24

590. Tutaq ki, yalnız bir işıqfor vardır. Bir işıqfor 3


vəziyyətdə ola bilər. İkinci işıqforu əlavə edək. 1-ci işıqforun
hər bir vəziyyətinə 2 - cinin 3 vəziyyəti uyğun gələ
bildiyindən bütün vəziyyətlərin sayı 3 ⋅ 3 = 32 –dir. 3-cü
işıqforu əlavə etsək, vəziyyətlərin sayı 3 dəfə artaraq 33 olar.
Yəni hər yeni işıqforun qoşulması vəziyyətlərin sayını 3 dəfə
artırdığından, n işıqforu 3n üsulla yandırmaq olar.

304
591. Verilmiş tənliyi aşağıdakı şəkildə yazmaq olar:
210
cos x(2sin x cos x) =
2 10
, və ya cos 2 x ⋅ sin10 2 x =
1, 024 .
1000
Lakin cos 2 x ≤ 1, sin10 2 x ≤ 1 olduğundan, sonuncu tənliyin
həlli yoxdur.
592. Tutaq ki, C bucağı bu bucaqlardan ən kiçiyidir. Onda
C A B
cos 2 ədədi cos 2 və cos 2 ədədlərindən böyük olar.
2 2 2
A B C
Üçbucaq bərabərsizliyinə görə, cos 2 + cos 2 − cos 2 > 0
2 2 2
olmalıdır. Həqiqətən də
A 2 B C 1 + cos A + cos B − cos C
cos 2 + cos
= − cos 2 =
2 2 2 2
C A+ B A− B
= sin 2 + cos ⋅ cos =
2 2 2
C A+ B A− B  A B C
= sin  cos + cos =  2 cos cos sin > 0.
2 2 2  2 2 2
A B C
Burada , , - iti bucaqlardır. Aydındır ki, məsələ
2 2 2
bərabəryanlı və bərabərtərəfli üçbucaqlar üçün də doğrudur.

593. cos 50 > cos 350 > cos 500 olduğu üçün, verilmiş
üçbucağın böyük tərəfi cos 50 olar. Bu üçbucağın itibucaqlı
olduğunu göstərmək üçün cos 2 50 və cos 2 350 + cos 2 500
cəmini müqayisə etmək lazımdır:
cos 2 50 − (cos 2 350 + cos 2 500 ) =(cos 2 50 − cos 2 350 ) −

305
1
− cos 2 50=
(1 + cos100 − 1 − cos 700 ) − cos 2 50=
0 0

2
= sin 30 sin 40 − sin 2=
0 0
400 sin 400 (sin 300 − sin 400 ) < 0
Burada sin 300 < sin 400 olduğu üçün,
cos 2 5 < cos 2 350 + cos 2 500 . Biz məsələni həll edərkən
tərəflərinin uzunluğu cos 50 , cos 350 , cos 500 olan
üçbucağın varlığını qəbul etmişdik. Lakin sonuncunu
isbat etmək lazımdır. Göstərək ki, cos 50 < cos 350 + cos 500.
Həqiqətən də
cos 2 50 < cos 2 350 + cos 2 500 + 2 cos 350 cos 500 =
= (cos 350 + cos 500 ) 2 və ya cos 50 < cos 350 + cos 500 olur.

α β
594. Verilmiş şərtlər daxilində tg > 0, tg > 0,
2 2
γ
tg > 0 . Ədədi orta və həndəsi orta haqqında teoremə
2
α β γ 1 2α β
əsasən: tg 2 + tg 2 + tg 2 =  tg + tg 2  +
2 2 2 2 2 2

1 β γ  1 γ α α β
+  tg 2 + tg 2  +  tg 2 + tg 2  ≥ tg 2 tg 2 +
2 2 2 2 2 2 2 2

β γ γ α α β β γ
+ tg 2 tg 2 + tg 2 tg 2 tg tg + tg tg +
=
2 2 2 2 2 2 2 2

γ α α β γ β α
tg tg = tg tg + tg  tg + tg =

2 2 2 2 2 2 2

306
β α
tg + tg
α β γ
2 1 − tg α tg =
β2 α β
+tg tg + tg  ⋅  tg tg +
2 2 β
2 1 − tg tg  2 2 α
2 2
2 2
γ α +β  α β α β γ π −γ
+tg tg + 1 − tg tg
=  tg tg + tg tg ×
2 2  2 2 2 2 2 2
 α β α β γ γ α β
× 1 − tg tg  =tg tg + tg ctg 1 − tg tg  =
 2 2 2 2 2 2 2 2
α β α β
= tg tg + 1 − tg tg =1
2 2 2 2
π
Bərabərlik halının alınması üçün α= β= γ= olmalıdır.
3

ϕ ϕ ϕ
1 − tg 2 2tg + 1 − tg 2
595. 1 + ctgϕ =
1+ 2 =2 2 =
ϕ ϕ
2tg 2tg
2 2
ϕ ϕ  ϕ 
2
 
1 −  tg 2 − 2tg + 1 − 1 2 −  tg − 1
=  2
=
2  =  2 
ϕ ϕ
2tg 2tg
2 2
ϕ
ϕ ϕ  ϕ
2
1
= ctg − ctg  tg − 1 ≤ ctg .
2 2 2 2  2
ϕ ϕ 
2
1
burada 0 < ϕ < π olduğu üçün, ctg  tg − 1 ≥ 0 .
2 2 2 

596. Bərabərsizliyin sol tərəfini A ilə işarə edək. Onda

307
1 + cos 2 β 1 + coa 2γ
=A cos 2 α + + = cos 2 α + 1 +
2 2
+ cos( β + γ ) cos( β −=γ ) cos α + cos 2 (π − α ) cos( β − γ )=
2
+1
= cos α − cos( β − γ ) cos α + 1 və ya
2

cos 2 α − cos( β − γ ) cos α + 1 − A =0 .

Alınmış kvadrat tənliyin həqiqi köklərinin olması üçün


diskriminant mənfi olmamalıdır, yəni
cos 2 ( β − γ ) − 4(1 − A) ≥ 0
1
Buradan 4(1 − A) ≤ cos 2 ( β − γ ) ≤ 1 və ya 1 − A ≤ . Beləliklə
4
3 π
A ≥ . Bərabərlik halı α= β= γ= olduqda mümkündür.
4 3

597. γ bucağı kor bucaq olduğundan,


π π π
0 <α + β < və 0 < α < −β < .
2 2 2
π  1
Lakin onda tgα < tg  − β = ctg β = .
2  tg β
1
Beləliklə, tgα < . Bərabərsizliyin hər tərəfini
tg β
tg β > 0 ifadəsinə vursaq, tgα tg β > 1 alarıq.

2
1  1 1
598. 4arctg = 2  arctg + arctg  = 2arctg 5 =
5  5 5 1−
1
25

308
10
5 5 5 120
= 2arctg = arctg + arctg = arctg 12 = arctg
12 12 12 25 119
1−
144
Onda 4arctg 1 − arctg 1= arctg 120 + arctg −  1 = 

5 239 119  239 
120 1

π
= arctg 119 = 239 =
arctg1 .
120 1 4
1+ ⋅
119 239

599. Həqiqətən də
 π  π  π  2 π   2 π 
tg  x 2 +  tg  x 2 − =  ctg  −  x +   tg  x − = 
 6  3 2  6   3
π   π  π  π
= ctg  − x 2  tg  x 2 −  = −ctg  x 2 −  tg  x 2 −  = −1
3   3  3  3

x
600. Kafilik: Əgər tg rasionaldırsa, sin x və cos x -in
2
rasional olması aşkardır:
x x
2tg 1 − tg 2
= sin x = 2 , cos x 2.
x x
1 + tg 2 1 + tg 2
2 2
x
Zərurilik: Əgər sin x və cos x rasional olarsa, tg rasional
2
x sin x
olar: tg = .
2 1 + cos x

309
π
601. Şərtə görə arctg x + arctg y = − arctg z , onda
2
π 
tg (arctgx + arctgy ) =tg  − arctgz  .
2 
x+ y 1
Buradan = ⇔ xz + yz =− 1 xy ⇔ xy + yz + zx =1 .
1 − xy z
Beləliklə, isbat tamam oldu.

602. Şərtə görə arctg x + arctg y = π − arctg z , onda


tg (arctgx + + arctgy
= ) tg (π − arctgz ),
x+ y
buradan =− z ⇔ x + y =− z + xyz ⇔ x + y + z =xyz .
1 − xy
Beləliklə, isbat tamam oldu.

π
603. Qeyd edək ki, arctg1 =, onda arctg 2 + arctg 3 cəmini
4
=
hesablamaq kifayətdir. α arctg
= 2; β arctg 3 qəbul etsək:
tgα + tg β 2+3
tg (α + β ) = = =
−1
1 − tgα ⋅ tg β 1 − 2 ⋅ 3
 π 3π
α , β ∈  0;  olduğundan 0 < α + β < π . Onda α + β =
 2 4
π 3π
olar. Deməli, arctg1 + arctg 2 + arctg 3 = + = π
4 4

cos 700
604. ctg 700 + 4 cos 700 = + 4 cos 700 =
sin 700
cos 700 + 4sin 700 ⋅ cos 700 cos 700 + 2sin1400
= =
sin 700 sin 700
310
cos 700 + 2 cos 500 (cos 700 + cos 500 ) + cos 500
= = =
sin 700 sin 700
2 cos 600 ⋅ cos100 + cos 500 cos100 + cos 500
= =
sin 700 sin 70
2 cos 300 ⋅ cos 200 3 cos 200
= = = 3.
sin 700 cos 200
İsbat tamam oldu.

605. Aydındır ki, x ≠ 0. Onda hər tərəfi 2x -ə bölsək:


1
4 x=
2
+ 3 alarıq.
2x
Əgər x < −1 və ya x > 1 olarsa, onda bu axırıncı tənliyin sol
tərəfi 4 -dən böyük, sağ tərəfi isə 4 -dən kiçik olar. Aydındır
ki, verilmiş tənliyin kökləri [ −1;1] parçasında yerləşir. Tutaq
=
ki, x cos ϕ , ( 0 ≤ ϕ ≤ π ) . Onda verilmiş tənlik aşağıdakı
triqonometrik şəkli alar:
8cos3 ϕ − 6 cos ϕ − 1 =0,
1 1
4 cos3 ϕ − 3cos ϕ = ⇔ cos 3ϕ = ,
2 2
π 2π
buradan ϕ = ± + n, n ∈ Z .
9 3
π 5π 7π
Belə ki, 0 ≤ ϕ ≤ π onda
= ϕ1 = , ϕ2 və ϕ3 = .
9 9 9
π 5π 7π
x cos ϕ ⇒ x=
= 1 cos , x= 2 , x=
3 .
9 9 9
606. Əvvəlcə II şərtə görə

311
2 f (0)= f (0) + f (0) ≥ f (0 + 0)= f (0), buradan
f (0) ≥ 0 ⇒ f (0) = 0.
İndi x ∈ R üçün = = f ( x + (− x)) ≤ f ( x) + f (− x),
0 f (0)
buradan f ( x) ≥ − f (− x). Lakin verilən şərtə görə hər bir
x ∈ R üçün f (− x) ≤ − x, onda f ( x) ≥ − f ( x) ≥ x olar.
I şərti, yəni f ( x) ≤ x -i nəzərə alsaq, hər bir x ∈ R üçün
f ( x) = x alarıq.

607. Əgər x < 1 olarsa, [ x ] ≤ 0 və x 2 − 10 [ x ] + 9 > 0 olur.


Aydındır ki, verilmiş tənliyin kökləri x ≥ 1 şərtini ödəməlidir.
[ x] = k qəbul edək, k ∈ Z , onda k ≥ 1 olmalıdır.
Onda x 2 − 10k + 9 = =
0 və buradan x 10k − 9 .
Tam hissənin xassəsinə görə k ≤ 10k − 9 < k + 1, ( k ≥ 1) .
Bu ikiqat bərabərsizliyi kvadrata yüksəltsək:
k 2 − 10k + 9 ≤ 0,
k 2 ≤ 10k − 9 < k 2 + 2k + 1 və ya  2 alarıq.
k − 8k + 10 > 0
Bu sistemdən 1 ≤ k < 4 − 6 və 4 + 6 < k ≤ 9 .
k ∈ Z və k ≥ 1 olduğundan=
k1 1,=k2 7,=k3 8 və k4 = 9 olur.
Onda x= 10k − 9 ⇒ x=
1 1, x=
2 61, x=
3 71 və x4 = 9 olur.
608. Verilmiş tənliyi 19 ⋅ [ x ] =96 ⋅ { x} kimi yazaq.
96
0 ≤ { x} < 1 olduğundan 0 ≤ 19 [ x ] < 96 ⇔ 0 ≤ [ x ] < ,
19
buradan [ x ] = 0,1, 2,3, 4,5 və uyğun olaraq,

312
19 38 57 76 95
{ x} = 0, , , , , alarıq.
96 96 96 96 96
Onda tənliyin həlləri çoxluğu:
 19 38 57 76 95 
0,1 , 2 , 3 , 4 , 5  olar.
 96 96 96 96 96 

609. Sistemin tənliklərini tərəf-tərəfə toplayıb, [ a ] + {a} =


a
nəzərə alsaq: x + y + z = 3,3 alarıq.
Əgər sistemin I və II tənliyini toplayıb, bu axırıncı tənlikdən
0 və buradan [ y ] = 0 və { x} = 0 alarıq.
çıxsaq: [ y ] + { x} =
Onda { x} = 0 olduqda x ∈ Z və [ y ] = 0 olduqda isə
y ∈ [ 0,1) alarıq.
Bunları nəzərə alsaq sistemin 1 tənliyindən x + { z} =
1,1 və
buradan x = 1 və { z} = 0,1 olar. Nəhayət, sistemin II
tənliyindən y + [ z ] =
2, 2 və buradan isə [ z ] = 2 və y = 0, 2
alarıq. Deməli,
= x 1,=y 0, 2,=z 2,1 olur.

1998 − 19 [ x ]
610. Verilmiş tənlikdən { x} = . 0 ≤ { x} < 1
98
olduğundan
1998 − 19 [ x ]
0≤ < 1 ⇔ 0 ≤ 1998 − 19 [ x ] < 98 ⇔
98
3
⇔ 1900 < 19 ⋅ [ x ] ≤ 1998 ⇔ 100 < [ x ] ≤ 105 .
19
Sonuncu bərabərsizliyi ödəyən 5 tam ədəd vardır.
Deməli, verilmiş tənliyin 5 kökü vardır.

313
611. Tənlikdən görünür ki, x ∈ Z , çünki sol tərəf tam
ədəddir. Onda x3 + 3 x 2 + 2 x = x ( x + 1)( x + 2 ) və x ∈ Z
üçün x ( x + 1)( x + 2 ) hasili 6 -ya tam bölündüyündən
 x3 + 3 x 2 + 2 x + 4   x( x + 1) ( x + 2 ) 2 
 =   = + 
 6   6 3
 x ( x + 1)( x + 2 )  x ( x + 1)( x + 2 )
= = 
 6  6
olar. Onda verilmiş tənliyi aşağıdakı kimi yaza bilərik:
x ( x + 1)( x + 2 )
= x + 1,
6
buradan x1 = −1, x2 =−1 − 7, x3 =−1 + 7 .
x ∈ Z şərtini nəzərə alsaq, verilmiş tənliyin həlli yalnız
x = −1 olur.

612. Sol tərəf tam ədəd olduğundan sağ tərəf: 2 cos 2 x tam
ədəddir. 2 cos=2
x n, ( n ∈ Z ) olsun, onda 0 ≤ 2 cos 2 x ≤ 2
olduğundan n = 0,1, 2 olur.
[tgx ] = 0, 0 ≤ tgx < 1
1) n = 0 olduqda  ⇔  ⇒ x ∈∅,
2 cos x = 0 cos x = 0
2

2) n = 1 olduqda
[tgx ] = 1, 1 ≤ tgx < 2, π
 ⇔ 2 ⇔ tgx =1 ⇔ x = + π k , k ∈ Z ,
2 cos x = 1 tg x = 1
2
4

314
[tgx ] = 2, 2 ≤ tgx < 3
3) n = 2 olduqda  ⇔  ⇔ x ∈∅ .
2 cos x = 2 tgx = 0
2

π
Deməli, verilmiş tənliyin həlli x =+ π k , k ∈ Z olur.
4

613. Aydındır ki, x5 > y 5 ⇔ x > y ⇔ x − y > 0.


Verilən tənliyi aşağıdakı kimi yazaq:
( x − y ) ( x 4 + x3 y + x 2 y 2 + xy 3 + y 4 )= 1997 ( ∗)
1997 sadə ədəd və x − y > 0 olduğundan
x− y =
1 və ya x − y =
1997 ola bilər.
I hal mümkün deyil, çünki 1997 -ni 5-ə böldükdə qalıqda
2 alınır. Sol tərəfi isə
x5 − y 5 = ( y + 1) − y 3 = 5 ( y 4 + 2 y 3 + 2 y 2 + 1) + 1
5

kimi göstərib, 5-ə böldükdə qalıqda 1 alınır.


II hal da mümkün deyil. Əks halda (*)-un sol tərəfindəki
2-ci vuruq 1-ə bərabər olmalıdır. Halbuki,
x 4 + x3 y + x 2 y 2 + xy 3 + y 4 = ( x3 + y 3 ) ( x + y ) + x 2 y 2 =

= ( x + y ) ( x 2 − xy + y 2 ) + x 2 y 2 ≥ x 2 − y 2 > 1
2

Deməli, verilən tənliyin tam həlli ola bilməz.

( x − y) + ( y − z) + ( z − x) = 3 ( x − y )( y − z )( z − x )
3 3 3
614.
eyniliyini tətbiq etsək, verilmiş tənlik
( x − y )( y − z )( z − x ) =
10 şəklinə düşər.

315
x −=
y a, y −=
z b, z −=
x c əvəz etsək:
abc = 10 və a + b + c = 0 alarıq. Birinci bərabərlikdən
çıxır ki, a, b, c ədədləri ±1, ±2, ±5 və ya ±1, ±1, ±10
üçlüklərindən biridir. Lakin bu üçlüklərin heç biri
a+b+c = 0 bərabərliyini ödəmir.
Deməli, verilmiş tənliyin tam həlli yoxdur.

615. Verilmiş tənlik aşağıdakı sistemlə eynigüclüdür:


sin [ x ] ⋅ sin { x} = cos [ x ] ⋅ cos { x} ,

cos [ x ] ⋅ cos { x} ≠ 0.
π
Buradan cos ([ x ] + { x} ) = 0 ⇔ cos x = 0 ⇔ x = + π k, k ∈ Z .
2
İndi cos [ x ] ⋅ cos { x} =
0 tənliyinə baxaq:

π
1) cos [ x ] =0 ⇔ [ x ] = + π n, n ∈ Z ; bu halda ∀n ∈ Z üçün
2
π 
 + π n  ∉ Z . Deməli, cos [ x ] ≠ 0.
2 
π
2) cos { x} =⇔
0 { x} = + π m, m ∈ Z ; bu halda da ∀m ∈ Z üçün
2
π 
 + π m  ∉ [ 0;1) . Deməli, cos { x} ≠ 0 .
2 
π
Onda verilmiş tənliyin həlli x = + π k , k ∈ Z olur.
2

316
[ x] n
616. Tutaq ki,= (n ∈ Z ) və x =n + α , 0 ≤ α < 1.
Onda verilən tənlik x3 − x + α =3 şəklinə düşər.
Digər tərəfdən 0 ≤ α < 1 ⇒ 2 < x3 − x ≤ 3 .
Əgər x ≥ 2 ⇒ x ( x 2 − 1) ≥ 2 ⋅ 3 =6, bu halda x3 − x ≤ 3
bərabərsizliyi ödənmir.
Əgər x < −1 ⇒ x ( x 2 − 1) < 0, bu halda 2 < x3 − x ödənmir.

Əgər −1 ≤ x < 2 ⇒ [ x ] =−1, 0,1. Onda uyğun olaraq:

x3 + 1 = 3 ⇔ x1 = 3
2, x3 = 3 ⇔ x2 = 3
3;
x3 − 1 = 3 ⇔ x3 = 3
4 alarıq.

Lakin  3 2  ≠ −1,  3 3  ≠ 0,  3 4  =1.

Beləliklə, veriliş tənliyin həlli yalnız 3


4 olur.

617. Sistemin tənliklərini tərəf-tərəfə çıxsaq :


x3 − 2 x 2 + 2 x = y 3 − 2 y 2 + 2 y alarıq.
f ( x ) =x3 − 2 x 2 + 2 x funksiyasına baxaq:
bütün x-lər üçün f ′ ( x )= 3 x 2 − 4 x + 2 > 0 olduğundan
f ( x ) =x 3 − 2 x 2 + 2 x funksiyası monoton artandır. Onda
f (=
x ) f ( y ) ⇔=
x y.
II tənlikdə y = x qəbul etsək:
x3 − 4 x 2 + 2 x =0 ⇔ x ( x 2 − 4 x + 2 ) =0 ⇒ x1 =0, x2 =2 + 2,

x3= 2 − 2.

317
Beləliklə, sistemin həlləri çoxluğu
{( 0;0 ) , ( 2 + )(
2; 2 + 2 , 2 − 2; 2 − 2 )} olur.
618. Verilmiş tənliyin kökləri x1 , x2 , x3 , x4 olsun. Onda
Viyet teoreminə görə
 x1 + x2 + x3 + x4 = 0,

 x1 x2 + x1 x3 + x2 x3 + x2 x4 + x3 x4 =
−10,
 x x x x = a.
 1 2 3 4
Şərtə görə x2 − x1 = x3 − x4 = d , d − silsilə fərqidir. Onda
( x1 + x4 ) + ( x2 + x3 ) =
0,

 x1 x2 x3 x4 = a,
 x +x x +x +xx +x x =
( 1 4 )( 2 3 ) 1 4 2 3 −10;

 x1 + x4 = x2 + x3 = 0,  x1 + ( x1 + 3d ) = 0,
  2
−10, ⇒  x1 + ( x1 + d ) =
 x1 x4 + x2 x3 =
2
10,
 2 2  2
 x1 x2 = a,  x1 ( x1 + d ) =
2
a.

Buradan 1) x1 = 3, d = −2 və ya 2) x1 =
−3, d =
2.
Hər iki halda a = 32 ⋅12 = 9 olur.

619. Tutaq ki, A parabola üzərində qeyd olunmuş nöqtə, B


isə çevrə üzərində yerləşən ixtiyari nöqtədir. A və B nöqtələri
və çevrənin C mərkəzi bir düz xətt üzərində yerləşdikdə AB

318
məsafəsi ən kiçik olur. Doğrudan da, bu çevrənin ixtiyarı B1
nöqtəsi üçün CB1 ≤ CA + B1 A və ya BA ≤ B1 A bərabərsizliyi
ödənir. Ona görə parabola üzərində A ( x, x 2 ) nöqtəsinin
yerini dəyişərək, AC məsafəsinin ən kiçik qiymətini tapaq:

( x − 3) + ( x 2 − 3) ,
2
AC = onda=
d AC − 1 olar.
2

İndi y =( x − 3) + ( x 2 − 3) funksiyasının ən kiçik qiymətini


2 2

tapaq: y′= 4 x3 − 10 x − 6= 2 ( x + 1) ( 2 x 2 − 2 x − 3) olduğuna


görə y′ =0 ⇔ 2 ( x + 1) ( 2 x 2 − 2 x − 3) =0, buradan

1+ 7 1− 7
böhran nöqtələri : x1 =
−1, x2 = , x3 = .
2 2
1+ 7
Aydındır ki, x2 = olduqda y ən kiçik qiymət alır.
2
Deməli, axtarılan ən qısa məsafə
2
   1+ 7  
2 2
 1+ 7
=d  − 3  +   3 −1
 − =
 2    2  

( ) +( )
2 2
7 −5 7 −2 43 − 14 7 − 2
= = −1 olur.
4 4 2

=
620. sin 2 α a,=
sin 2 β b=
, sin 2 γ c olsun.
Onda a (1 − b ) + b (1 − c ) + c (1 − a ) ≤ 1,
a + b + c ≤ 1 + ab + bc + ca (∗) bərabərsizliyini isbat etmək

319
kifayətdir.
ca (1 − b) ≤ ac, c (1 − a )(1 − b ) ≤ (1 − a )(1 − b ) olduğundan
ca (1 − b ) + c (1 − a )(1 − b ) ≤ ac + (1 − a )(1 − b ) ,
c (1 − b ) ≤ ac + (1 − a )(1 − b ) ,
c − cb ≤ ac + 1 − a − b + ab olar.
Deməli, ( ∗) bərabərsizliyi, o cümlədən verilən bərabərsizlik
doğrudur.

621. Məlumdur ki, sin α ≤ α .


π π
0 ≤α ≤ ⇒ 0 ≤ cos α ≤ 1 < , onda sin ( cos α ) ≤ cos α olar.
2 2
π
Digər tərəfdən 0 ≤ sin α ≤ α ≤ olduğundan
2
cos α ( sin α ) ≥ cos α olur.
Beləliklə, cos ( sin α ) ≥ cos α > sin ( cos α ) aldıq.

622. Tənliyin hər tərəfini 2-yə vurub, çevirmələr aparsaq:


( sin x − 2sin x + 1) + ( sin y − 2sin y + 1) +
2 2

+ ( sin x + sin y − 2sin x ⋅ sin y ) =


2 2
0,

( sin x − 1) + ( sin y − 1) + ( sin x − sin y ) =


2 2 2
0.

sin x = 1,  π
 x= + 2π n,
  2
=
Buradan sin y 1, ⇒ 
  π
sin x = sin y  y = + 2π k , n, k ∈ Z .
2
320
623. Verilən bərabərliklərin hər tərəfini kvadrata yüksəldib,
tərəf-tərəfə toplasaq:
sin 2 (α + β ) + sin 2 (α − β ) = m 2 + n 2 alarıq.
Buradan
1 − cos ( 2α + 2 β ) 1 + cos ( 2α − 2 β )
sin 2 (α + β ) +=
sin 2 (α − β ) + =
2 2
=
1−
1
2
( cos ( 2α + 2β ) + cos(2α − 2β ) ) =
1 − cos 2α ⋅ cos 2β .

Beləliklə, 1 − cos 2α ⋅ cos 2 β =


m 2 + n 2 aldıq.

624. Həqiqi ədədlər çoxluğunda təyin olunmuş kəsilməz


3x − 1
f ( x) = x funksiyasına baxaq. Arqumentin işarəsini
3 +1
3− x − 1 1 − 3x
dəyişək: f ( − x ) =− x = =− f ( x ) , buradan çıxır ki,
3 + 1 1 + 3x
f (x) tək funksiyadır.
3
Onda ∫ f ( x )dx = 0 olur ki,
−3
log 3 ( 2 x − 7 ) =6 − x alarıq.

Sol tərəf artan, sağ tərəf isə azalan olduğundan bu tənliyin 1-


dən artıq kökü ola bilməz. Onda seçmə ilə x=5 olduğunu
taparıq. Deməli, x = 5 verilmiş tənliyin yeganə köküdür.

1 1 1
625. Məlumdur ki, = − .
x + 5x + 6 x + 2 x + 3
2

Onda inteqralaltı funksiya


1 1 2
+ − =
( x + 2)
2
( x + 3)
2
( x + 2 )( x + 3)
321
1 1 2 2
= + − +
( x + 2) ( x + 3) x+2 x+3
2 2

şəklinə düşər və verilən inteqral:


1 1 1 1
1 1 2 2
∫ ( x + 2)
0
2
dx + ∫
0 ( x + 3)
2
dx − ∫
0
x+2
dx + ∫
0
x+3
dx =

1 1 1 1 1 1 8
=− − − 2 ln( x + 2) + 2 ln( x + 3) =+ 2 ln
x+2 0 x+3 0 0 4 9
olar.

626. e x − 1 =u 2 əvəz edək. Onda


u 2 + 1 = e x ⇒ 2udu = e x dx = (u 2
+ 1) dx, buradan dx =
2udu
u2 +1
1
2u 2
alarıq. Buna görə verilən inteqral: ∫0 u 2 + 1du şəklinə düşər.
Onda
 1   π π
1 1
2u 2
∫0 u 2 + 1du =2∫0 1 − u 2 − 1 du =2 ( u − arctgu ) 0 =2 1 − 4  =−
1
2 .
2

627. Aydındır ki, inteqralaltı funksiyalar biri digərinin


tərsidir. Onda əgər f ( x ) = e x qəbul etsək:
2

1 e

∫ f ( x )dx + ∫ f ( x ) dx
−1
alarıq.
0 1

Buna görə axtarılan nəticə, tərəflərinin uzunluqları e və 1


olan düzbucaqlının sahəsi olacaqdır. Yəni,

322
1 e

∫e dx + ∫ ln x dx = e ⋅1 = e olur.
x2

0 1

628. İnteqralaltı funksiyada arqumentin işarəsini dəyişək:

(
ln − x + 1 + x 2 = ln
1
x + 1+ x
)2
=− ln x + 1 + x 2 ( )
olduğundan y= ln x + 1 + x 2 ( ) funksiyası tək funksiyadır.
İnteqrallama sərhədləri O nöqtəsinə nəzərən simmetrik
olduğundan verilən inteqral sıfıra bərabərdir.

xb − x a
1
629. ∫0 ln x dx = ϕ ( b ) ilə işarə edək. Onda
 1 xb − x a ′ 1
xb ln x 1
ϕ′ (b)  ∫ =
= dx  ∫0 ln x dx b + 1
=
 0 ln x 
1
db
Yəni, ϕ ( b=
) ∫ b + 1 + =6 ln ( b + 1) + C .
0

b=a olduqda ϕ ( b ) = 0 olduğundan C =


− ln(a + 1) alarıq.
b +1
Onda ϕ ( b=
) ln ( b + 1) − ln ( a + 1=) ln olar.
a +1

630. Verilən funksiyanı aşağıdakı kimi çevirək:

f ( x)
=
sin 5 x
=
( sin 5 x + sin 3x ) − ( sin 3x + sin x ) +=
sin x
cos 2 x cos 2 x

323
2sin 4 x 2sin 2 x sin x
= − + =
cos x cos x cos 2 x
sin x
= 8sin x ⋅ cos 2 x − 4sin x + =
cos 2 x
sin x
= 4sin 3 x − 8sin x + .
cos 2 x

Onda f ( x ) funksiyasının ibtidai funksiyalarından biri,


4 1
F ( x) =
− cos 3 x + 8cos x + şəklində olur.
3 cos x

x x
631. ( 0,1] aralığında 0 < sin < doğrudur.
2 2
x x2 x2
Onda sin 2 < , buradan cos x > 1 − > 0.
2 4 2
1 1
− ln ( 2 − x 2 ) 10 =
x 2x
Deməli, ∫0 cos xdx < ∫0 2 − x 2 dx = ln 2.

632. Müsbət funksiyanın istənilən parçada müəyyən inteqralı


müsbət olduğundan
π

∫ (a
0
0 + a1 cos x + a2 cos 2 x + ... + an cos nx )dx =

 a2 an π
 a0 x + a1 sin x + sin 2 x + ... + sin nx  0 = a0π > 0 .
 2 n 

324
1
633. ∫
0
xe x dx = A olsun. Onda Koşi-Bunyakovski

bərabərsizliyinə əsasən

1   1 2x   1   1 1 2x 
( )
2
=A ∫ =
x dx   ∫ e dx  =
2
∫ xdx   2 ∫0 l dx 
0  0  0
e +1
=
1 1 2x
⋅ e
2 2
1
0 =
4
(
1 2
e − 1) =
4
( e − 1) < e − 1

Buradan A < e − 1 aldıq.

sin ( x + y ) =−1
634. sin ( x + y ) − cos ( x − y ) + 2 = 0 ⇒ 
cos ( x − y ) =
1

Onda sin ( x + y ) + cos ( x − y ) =−1 + 1 =0 olur.

635. Üçbucağın daxili bucaqları α , β və γ olsun. Onda


sinuslar teoreminə= görə a 2= R sin α , b 2 R sin β və
c = 2 R sin γ . Kosinuslar teoreminə görə

b 2 + c 2 − a 2 4 R 2 sin 2 β + 4sin 2 γ − 4 R 2 sin 2 α


cos α
= = =
2bc 2 ⋅ 2 R sin β ⋅ 2 R sin γ

sin 2 β + sin 2 γ − sin 2 α


= ∈ Q, rasionaldır.
2sin β ⋅ sin γ

Eyni qayda ilə cos β ∈ Q və cos γ ∈ Q olduğu isbat olunur.

325
636. Katetlər a, b ( a < b ) , hipotenuz isə c olsun.
Onda şərtə görə a + c =2b (*) . Digər tərəfdən a= c ⋅ cos B
və b= c ⋅ sin B . Bunları (*) -da nəzərə alsaq:
c ⋅ cos B + c = 2c ⋅ sin B,
1 + cos B = 2sin B,
2 B B B
=
cos 4sin ⋅ cos
2 2 2
B B
cos ≠ 0 olduğundan, hər tərəfi cos 2 − yə bölsək:
2 2
B 1 1 4
tg = , buradan B = 2arctg və ya B = arctg .
2 2 2 3
3
Onda o biri iti bucaq, A = arctg olar.
4
Qeyd: Bu məsələni başqa üsulla da həll etmək olar.
Silsilə fərqi d olsun, onda a =− b d , c =+ b d . Pifaqor
c 2 onda ( b − d ) + b 2 = ( b + d ) .
teoreminə görə a 2 + b 2 =,
2 2

b 3
Buradan b 2 = 4bd və d = , buna görə a = b.
4 4
a 3 b 4
Aydındır ki, tgA= = və tgB= = ;
b 4 a 3
3 4
onda A = arctg və B = arctg olar.
4 3

637. a və b ( a < b ) katetlər, c isə hipotenuz olsun.


Onda şərtə görə a, b, c həndəsi silsilə əmələ gətirdiyindən

326
b 2 = ac. Digər tərəfdən a = c ⋅ cos B və b = c ⋅ sin B ,
buradan a c = c 2 cos B və b 2 = c 2 sin 2 B. Onda
c 2 sin 2 B = c 2 ⋅ cos B ⇔ cos 2 B + cos B − 1 = 0 , buradan
−1± 5 5 −1
cos B = . cos B > 0 olduğundan cos B = ,
2 2
5 −1 5 −1
B = arccos , o biri iti bucaq isə A = arcsin
2 2
olacaqdır.

638. Dərəcəni azaltma düsturu tətbiq etməklə, aşağıdakı


çevirmələri aparaq:
1 + cos 2 A 1 + cos 2 B
+ + cos 2 C = 1,
2 2
1 1
cos 2 A + cos 2 B + cos 2 ( A + B ) = 0
2 2
Buradan cos ( A + B ) ⋅ cos ( A − B ) + cos 2 ( A + B ) =
0,
cos ( A + B ) ( cos ( A − B ) + cos ( A + B ) ) =
0,
2 cos ( A + B ) ⋅ cos A ⋅ cos B =
0,
cos A =0 ⇒ A =900 , cos B =0 ⇒ B =900 ,
cos ( A + B ) = 0 ⇒ A + B = 900 , yəni, C = 900 olar.
Bu hallarda hər birində üçbucaq düzbucaqlı üçbucaqdır.

639. Koşi-Bunyakovski bərabərsizliyini tətbiq edək:


( sin x ⋅ cos y + sin y ⋅ cos z + sin z ⋅ cos x ) ≤
2

327
≤ ( sin 2 x + sin 2 y + cos 2 x ) ⋅ ( cos 2 y + cos 2 z + sin 2 z ) =

= (1 + sin 2 y ) ⋅ (1 + cos 2 y ) = 2 + sin 2 y ⋅ cos 2 y =

sin 2 2 y 1 9
= 2+ ≤ 2+ = .
4 4 4
3
Buradan sin x ⋅ cos y + sin y ⋅ cos z + sin z ⋅ cos x ≤ alarıq.
2
640. Hər tərəfi 3-ə vurub, 4-cü dərəcədən kök alsaq,
verilmiş bərabərsizliklə eynigüclü bərabərsizlik alarıq:
34 3
3sin x ⋅ cos x ≤ 4 ⇔ 4 3sin 2 x ⋅ cos 6 x ≤ .
2 6

4 4
Koşi bərabərsizliyini tətbiq etməklə, bunu isbat edək:
4
3sin 2 x ⋅ cos 6 x= 4
3sin 2 x ⋅ cos 2 x ⋅ cos 2 x ≤

3sin 2 x + cos 2 x + cos 2 x + cos 2 x 3 ( sin x + cos x ) 3


2 2

≤ = = .
4 4 4
3
Beləliklə, 4 3sin 2 x ⋅ cos 6 x ≤ və buradan da
4
27
sin 2 x ⋅ cos 6 x ≤ alınır.
256

641. Bərabərsizliyin sol tərəfində çevirmə aparsaq:


( tgx1 − ctgx1 ) + ( tgx2 − ctgx2 ) + ... + ( tgxn − ctgxn ) ≤ 0 .
2 2 2

Digər tərəfdən hər bir i = 1, 2,3,..., n üçün ( tgxi − ctgxi ) ≥ 0


2

olduğundan tgxi = ctgxi olmalıdır. Buradan

328
π π
tg 2 xi =1 ⇔ tgxi =1 ⇒ xi = + ki , k i ∈ Z , i = 1, 2,3,..., n.
4 2

642. Sistemin tənliklərini tərəf-tərəfə toplasaq:


−2 x − y + x + 2 y
x+ y+ = 0.
x2 − y 2
x− y
Buradan x+ y− =0,
x2 − y 2
1
x+ y− =0 ⇒ ( x + y ) =⇔
1 x + y =±1
2

x+ y
 x +1
 x − 2 x − 1 = 1,
1) x + y =
1 olduqda  ⇒ x1= 0, x2= 2
1 − x + − x + 2
=−1
 2x −1
Onda y1 = 1, y2 = −1 olar.
2) x + y =−1 olduqda, eyni qayda ilə
x3 = −1 , y3 = 0 və x4 = 1, y4 = −2 alarıq.
Beləliklə, sistemin tam həlləri çoxluğu:
{( 0;1) , ( 2; −1) , ( −1;0 ) , (1; −2 ) } olur.
643. xyz ≠ 0 olduğu aşkardır.
Çevirmə aparsaq: x 2 y 2 + y 2 z 2 + z 2 x 2 =
3 xyz , buradan
x2 y 2 + y 2 z 2 + z 2 x2
=xyz ≥ 1,
3
çünki, x, y, z ∈ Z . Yəni, xyz ≥ 1 .

329
Digər tərəfdən a + b + c ≥ 3 3 abc bərabərsizliyini tətbiq
etsək: 3xyz = x 2 y 2 + y 2 z 2 + z 2 x 2 ≥ 3 3 x 2 y 2 ⋅ y 2 z 2 ⋅ z 2 x 2 =

= 3 3 x 4 y 4 z 4 = 3 xyz 3 xyz
Buradan 3 xyz ≥ 3 xyz 3 xyz , onda xyz > 0 olduğundan
xyz ≤ 1 alarıq.
Deməli, xyz = 1 − dir, buradan x =
±1, y =
±1, z =
±1.
Beləliklə, (=
x, y , z ) {(1,1,1) , (1, −1, −1) , ( −1,1, −1) , ( −1, −1,1)} olur.
644. 99! = (1 ⋅ 99 ) ⋅ ( 2 ⋅ 98 ) ⋅ ( 3 ⋅ 97 ) ... ( 49 ⋅ 51) ⋅ 50 <

 1 + 99   2 + 98   3 + 97   49 + 51 
2 2 2 2

<  ⋅  ⋅  ...   ⋅ 50 =
 2   2   2   2 
= 50

2
⋅ 50

2
  ⋅ 50 = 50 .
...50 2 99

49

Deməli, 99! < 50 . 99

645. Koşi-Bunyakovski bərabərsizliyini tətbiq etməklə ,


bərabərsizliyin sol tərəfini qiymətləndirək:
2
 
 2− 2+ 2 + 2+ 2+ 2  =
 
2
 
=1 ⋅ 2 − 2 + 2 + 1 ⋅ 2 + 2 + 2  ≤
 

(
≤ (12 + 12 ) ⋅ 2 − 2 + 2 + 2 + 2 + 2 = 2 ⋅ 4 = 8, )
buradan 2− 2+ 2 + 2+ 2+ 2 ≤ 2 2.

330
Burada bərabərlik halı ola bilmədiyindən

2 − 2 + 2 + 2 + 2 + 2 < 2 2 olur.

(1 − sin x )
2
646. Belə ki, cos 2 x = 1 − sin 2 x, onda cos 4 x= 2
.
Bunları nəzərə alsaq:
(1 − sin x )
2
2
+ sin10 x =
1,
1 − 2sin 2 x + sin 4 x + sin10 x =
1,
sin 4 x − 2sin 2 x + sin10 x =
0,
sin 2 x ( sin 2 x − 2 + sin 8 x ) =0.

1) sin 2 x = 0 ⇒ sin x = 0 ⇒ x = π K , k ∈ Z ;
sin 2 x ≤ 1,
2) sin x + sin =
2 8
2,  8 nəzərə alsaq, buradan
sin x ≤ 1
sin 2 x = 1, π
 8 ⇒ sin 2 x =1 ⇒ cos 2 x =0 ⇒ x = + π n, n ∈ Z .
sin x = 1 2
π
Beləliklə, x = π k və x = + π n, k , n ∈ Z .
2

 π
647. Qeyd edək ki, y = sin x funksiyası 0;  parçasında
 2
artır. Onda sin 490 > sin 450 ⇒ sin 490 > 2 .
2
 π
y = cos x funksiyası 0,  parçasında azalır. Onda
 2

331
2
cos 90 > cos 450 ⇒ cos 90 > .
2
2 2
Deməli, 2sin 490 ⋅ cos 90 > 2 ⋅ ⋅ =
1 . Buradan sin x > 1
2 2
aldıq, bu isə ola bilməz.

648. Sol tərəfi aşağıdakı kimi çevirək.


x 2 + 2 x ⋅ sin ( xy ) + 1 = x 2 + 2 x ⋅ sin ( xy ) + sin 2 ( xy ) + 1 −

− sin 2 ( xy ) =
( x + sin( xy ) ) + cos 2 ( xy )
2

Beləliklə, verilmiş tənlik:


( x + sin ( xy ) ) + cos 2 ( xy ) =
2
0 şəklinə düşər.

 x + sin ( xy ) =0 sin ( xy ) = −x
Onda  ⇔ ⇒ x =±1 ⇒ x1 =1
 cos ( xy ) = 0 sin ( xy ) = ±1
və x2 = −1 olar.
sin y = −1, π
1) x1 = 1 olduqda  ⇒ y1 =− + 2π k ;
cos y = 0 2
2) x1 = −1 olduqda
sin x =
−1, sin x =
−1, π
 ⇒ ⇒ y2 =− + 2π k .
=
cos y 0= cos y 0 2
π π
Deməli, x1 =
1, y1 =− + 2π k və x2 =−1, y2 =− + 2π k .
2 2

649. Qeyd edək ki, sin x ≤ 1 və cos x ≤ 1 olduğundan


tənliyin sol tərəfi sin 5 x + cos5 x ≤ sin 2 x + cos 2 x =
1,

332
sin 5 x + cos5 x ≤ 1 olar.
Digər tərəfdən sin 4 x ≤ 1 olduğundan, sağ tərəf: 2 − sin 4 x ≥ 1
sin 5 x = sin 2 x,
 π
Onda cos5 x =cos 2 x, ⇒ sin x =1 ⇒ x = + 2π k ,
sin 4 x = 1, 2

k ∈ Z alarıq.
650. Sol tərəfi qiymətləndirək:
x2 + 1 1  1 1 x2 + 1
= ⋅  x +  ≥ ⋅ 2 =1 ⇒ ≥ 1.
2x 2  x 2 2x
Digər tərəfdən sin ( xy ) ≤ 1 . Onda

 x2 + 1  x2 + 1
 =1  = 1,
 2x və ya  2 x
sin ( xy ) = −1 
 sin ( xy ) = 1
π
Bu sistemlərin həlləri: x1 =
−1, y1 = + 2π n və
2
π
x2 =1, y2 = + 2π n, ( n ∈ Z ) olur.
2
651. Koşi bərabərsizliyinə görə
sin 4 + cos 4 y ≥ 2sin 2 x ⋅ cos 2 y olduğundan
4sin x ⋅ cos y ≥ 2 + 2sin 2 x ⋅ cos 2 y .
Buradan sin 2 x ⋅ cos 2 y − 2sin x ⋅ cos y + 1 ≤ 0 ⇔
⇔ ( sin x ⋅ cos y − 1) ≤ 0 ⇒ sin x ⋅ cos y =
2
1

333
Beləliklə, verilmiş sistemdən
sin 4 x + cos 4 y =
2, 1
 ⇔ sin 4 x + 4 = 2.
sin x ⋅ cos y = 1 sin x
Buradan sin 4 x =
1 ⇔ sin x =
±1 , onda
sin x = ±1, sin x =
1,
 ⇒ və ya
sin x ⋅ cos
= =
y 1 cos y 1
 π  π
sin x = −1,  x= + 2π n,  x2 = − + 2π m,
 ⇒ 1
2 və ya  2
cos y = −1  y = π + 2π k ,  y2= π + 2π l
 1
burada k, m, n, l ∈ Z .

652. Koşi bərabərsizliyinə görə


tg 10 x + ctg 10 x ≥ 2 tg 10 x ⋅ ctg 10 x =
2.
Burada bərabərlik halı yalnız tg 10 x = ctg 10 x olduqda
π
mümkündür. Onda x= ± + π n, n ∈ Z olur.
4

653. y=1 qəbul etsək: f ( x + 1) − f ( x) = x + 1 .


x dəyişəninə ardıcıl olaraq, 1, 2, 3, ... , n -1 qiymətlərini
versək:
 f ( 2 ) − f (1) = 2,

 f ( 3) − f ( 2 ) = 3,


 f ( n ) − f ( n − 1) =n

sistemini alarıq.

334
Bu bərabərlikləri hədbəhəd toplasaq:
n(n + 1)
f ( n ) = 1 + 2 + 3 + ... + n = olar.
2
Yoxlama aparsaq, bu ardıcıllığın verilən şərti ödədiyini
görərik.

654. Verilən bərabərliyin sol tərəfini aşağıdakı kimi


çevirək:
1 − cos 2 A + 1 − cos 2 B + 1 − cos 2C
=2
1 + cos 2 A + 1 + cos 2 B + 1 + cos 2C
Buradan 3-cos2 (B+C) –cos2B-cos2C=
=6+2cos2(B+C)+2cos2B+2cos2C,
3cos 2 ( B + C ) + 3cos 2 B + 3cos 2C =
−3,
cos 2 ( B + C ) + cos 2 B + cos 2C =
−1,
2 cos 2 ( B + C ) − 1 + 2 cos( B + C ) ⋅ cos ( B − C ) =−1,
cos ( B + C ) ⋅ ( cos ( B + C ) + cos ( B − C ) ) =
0.
Buradan, ya cos ( B + C ) = 0 ⇒ ∠B + ∠C = 900 ⇒ ∠A = 900 ,
ya da 2 cos B ⋅ cos C =
0 ⇒ cos B =
0
və ya cos C = 0, onda < B = 900 alarıq.
900 və ya < C =

1 π
655. Əgər n = 0 olarsa, cos x = ⇒ x = ± + 2π m, m ∈ Z .
2 3
Əgər n > 0, onda sin x ≠ 0. Buna görə də tənliyin hər tərəfini
2sin x ifadəsinə vursaq:
1
2sin x ⋅ cos x ⋅ cos 2 x ⋅ cos 4 x...cos 2n x = n ⋅ sin x,
2

335
1
sin 2 x ⋅ cos 2 x ⋅ cos 4 x ⋅ ⋅ cos 2n x = sin x.
2n
Ardıcıl olaraq, davam etdirsək:
sin 2n +1 x sin x, sin 2n +=
= 1
x − sin x 0, buradan
2n +1 x + x 2n +1 x − x
2 cos ⋅ sin =
0.
2 2

1) cos
(2 n +1
+ 1) x
2n +1 + 1
=0 ⇒
π
x = +πk ⇒
2 2 2
π 2π k
x =n +1 + n +1 , k ∈ Z ,
2 +1 2 +1
(2n +1 − 1) x 2n +1 − 1 2π m
2) sin =0⇒ x = π m ⇒ x = n +1 ,
2 2 2 −1
2k + 1 m
n +1
∉ Z, n +1
∉ Z , k , m, n,∈ Z , n ≠ −1.
2 +1 2 −1

2tgx 3tgx − tg 3 x
656. Qeyd edək ki, tg 2 x = və tg 3 x = .
1 − tg 2 x 1 − 3tg 2 x
Bunları tənlikdə nəzərə alıb, çevirmə aparsaq:
tgx ( 2tg 4 x − 4tg 2 x + 3) =
0 alarıq.

Buradan tgx = 0 və ya 2tg 4 x − 7tg 2 x + 3 =;


0
1
1) x π k , k ∈ Z ;
= 2) tgx = ± 3 ; 3)tgx = ± .
2
Aydındır ki,
π 1
x= ± + π m, x =± arctg + π n, ( m, n ∈ Z ) x =π k və
3 2

336
π ( 3m ± 1) πl
x= köklərini birləşdirsək: x = ( l ∈ Z ) .(l = 3k
3 3
π ( 3m ± 1)
x π k=
olduqda = , l 3m ± 1 olduqda x = olar).
3
Beləliklə, bütün həlləri belə yazmaq olar:
πl 1
, π k ± arctg (l, k ∈ Z ) .
3 2

ctgα + ctg β sin(α + β ) sin ( a + β )


657. = = ≥
2 2sin α ⋅ sin β cos (α − β ) − cos (α + β )
α +β α +β
sin ( a + β ) 2sin ⋅ cos
≥ = 2 = 2
1 − cos (α + β ) 2sin 2
α + β
2
α +β
cos
2 α +β
= = ctg .
α +β 2
sin
2
ctgα + ctg β α +β
Deməli , ≥ ctg .
2 2

658. Yeni dəyişən daxil edək: y = log 3 x olsun, onda

( 3)
y
x = 3 y , buradan 1 + x =
1+ və

( 3) 2 y (*) şəklinə düşər.


y
verilmiş tənlik 1 + =
Aşkardır ki, y=2 bu tənliyin köküdür. İsbat edək ki, başqa

337
y y
 1   2 
kök yoxdur. (*) tənliyini   +1 =   şəklində
 3  3
1 2
yazaq. < 1 və > 1 olduğundan sol tərəf azalan, sağ
3 3
tərəf isə artandır. Onda bu tənliyin kökü varsa, həmin kök
yeganədir, yəni, y = 2 yeganə kökdür. Beləliklə,
log 3 x= 2 ⇒ x = 9 verilmiş tənliyin yeganə kökü olur.

cos x = 2
t

659. log 2 cos x = t işarə edək. Onda  2 ,


ctg x = 3t
cos 2 x
ctgx > 0 , alarıq. ctg 2 x = olduğundan
1 - cox 2 x
4t
t
3 = ⇔ 3-t = 4-t – 1 , buradan t = -1 aşkar görünür.
1 - 4t
3
Digər tərəfdən 3t – 12t = 4t , ( ) t = 3t + 1 tənliyində sol tərəf
4
azalan, sağ tərəf artan olduğundan həll varsa, həmin həll
yeganədir. Deməli, t = -1 yeganə kökdür.
1 π
Onda cos x = və buradan x = ± + 2πκ , κ ∈ − Z .
2 2
660. Verilmiş tənliyin hər tərəfini 5x -ə bölsək:
x x x
3  4 6
  +   +1 =  alarıq.
5 5 5
x x x
3  4 6
Tutaq ki, f ( x ) =   +   + 1, g ( x ) =   .
5 5 5

338
3 4 6
< 1, < 1 və > 1 olduğundan istənilən x üçün f ( x )
5 5 5
kəsilməz və azalan, g (x) isə kəsilməz və artandır. Onda
f ( x ) = g ( x ) tənliyinin kökü varsa, həmin kök yeganədir.
Bu kökün x = 3 olduğu aşkardır.

661. Sistemin tənliklərini tərəf-tərəfə toplasaq:


x 3 + y 3 + z 3= 3 xyz + 13 .
Axırıncı bərabərliyi sistemdə nəzərə alsaq:
=x3 xyz + 5,
 3
=y xyz − 4, (*)
 3
=z xyz + 12
Bu sistemi tərəf-tərəfə vurub, xyz = u qəbul etsək:
u 3 =( u + 5 )( u − 4 )( u + 12 ) ⇒ 13u 2 − 8u − 240 =0 ⇒
60
⇒ u1 = −4, u2 = .
13
Onda 1) x1 y1 z1 = −4 olduqda (*) sistemindən
x13 =−4 + 5 =1 ⇒ x1 =1 ;
y13 =−4 − 4 =−8 ⇒ y1 =−2;
z13 =−4 + 12 =8 ⇒ z1 =2 alarıq.
60
2) x2 y2 z2 = olduqda (*) sistemindən
13
5 2 6
= x2 3= , y2 3= , z2 3 alarıq.
13 13 13
Deməli, verilmiş sistemin həlləri

339
  5 2 6  
(1; −2; 2 ) ,  3 ; 3 ; 3   çoxluğu olur.
  13 13 13  

 1
662. Tənliyin hər tərəfini  x +  ifadəsinə vursaq:
 2
41
6 x 4 + 2 x3 − x 2 + 2 x + 6 =0
2
tənliyini alarıq. Aydındır ki, x ≠ 0. Onda hər tərəfi x 2 − na
bölsək:
 1   1  41
6  x2 + 2  + 2  x +  − = 0
 x   x 2
1 1
x+ = t əvəz etsək: x 2 + 2 =t 2 − 2 və 12t 2 + 4t − 65 =0,
x x
13 5
t1 = , t2 = − alarıq.
6 2
Bunları əvəzləmədə nəzərə alsaq:
1 13 3 2
1) x + = ⇒ 6 x 2 − 13 x + 6 = 0 ⇒ x1 = , x2 = ;
x 6 2 3
1 5 1
2) x + =− ⇒ 2 x 2 + 5 x + 2 =⇒ 0 x3 =−2, x4 =− .
x 2 2
3 - dərəcəli tənliyin 3-dən artıq həqiqi kökü ola bilməz.
1  1
x4 = − kökü  x +  vuruğunun hesabına alınmışdır, ona
2  2
1
görə də x4 = − kənar kökdür.
2
3 2 
Deməli verilmiş tənliyin kökləri  ; ; −2  çoxluğudur.
2 3 

340
663. 2001 = a olsun, onda a 2 = 2001 və 2002 = a2 + 1 .
Bunları nəzərə alsaq, verilmiş tənlik x9 − ( a 2 + 1) x3 + a =0
şəklinə düşər. Bu tənliyi həll etmək kifayət qədər mürəkkəb
olduğundan həmin çətinlikdən belə qaçmaq olar:
x3a 2 − a + ( x3 − x9 ) =0
a -ya nəzərən kvadrat tənlik kimi həll edək:
1 − 4 x3 ( x3 − x9 ) = (1 2 x6 ) ,
2
D= 1 − 4 x 6 + 4 x12 =−
1 ± (1 − 2 x 6 ) 1 − x6
= a1,2 3
= ⇒ a1 3
=, a2 x 3 .
2x x
Beləliklə, iki tənlik aldıq:
1 − x6
1) = a, ( a > 0 ) olduqda x 6 + ax3 − 1 = 0, D = a 2 + 4 > 0,
x3

onda x3 =
1
2
( )
−a ± a 2 + 4 , buradan x1,2= 3
1
2
(−a ± a 2 + 4 . )
a = 2001 olduğundan x1,2 =3
1
2
(− 2001 ± 2005 . )
2) x3 = a olduqda x = 3 a və ya
= x 3
2001 ⇒
= x3 6
2001
alarıq.
664. Mümkün qiymətlər çoxluğu:
x ≥ a 2 + b2 , x ≥ b2 + c2 , x ≥ a 2 + c2
Tutaq ki, f ( x=) a 2 x − b 2 − c 2 + b 2 x − a 2 − c 2 + c 2 x − a 2 − b 2
f ( x ) funksiyası, ciddi artan üç funksiyanın cəmidir.

( )
Digər tərəfdən f a 2 + b 2 + c 2 = a 2 ⋅ a + b 2 ⋅ b + c 2 ⋅ c = a 3 + b3 + c3 .

341
Aydındır ki, x = a 2 + b 2 + c 2 verilən tənliyin köküdür. f ( x )
təyin oblastında monoton artan olduğundan tənliyin başqa
kökü yoxdur.

665. a 2 + b 2 = ( a + b ) − 2ab eyniliyin tətbiq etməklə,


2

çevirmə aparsaq:
2
 x x  x x
 +  −2 ⋅ =
2,
 x−2 x+2 x−2 x+2
2
 2x2  2 x2
 2  − 2 2 alarıq.
=
 x −4 x −4
2x2
= t qəbul edək. Onda t − t − 2 =0 tənliyini alarıq ki,
2

x2 − 4
t1 = 2 olar. Beləliklə, iki tənlik aldıq:
−1, t2 =
2x2 2 3
1) =−1 ⇒ 3 x 2 =4 ⇒ x1,2 =± ;
x −4
2
3
2 x2
2) = 2 ⇒ 2 x 2 = 2 x 2 − 8 ⇒ x ∈∅
x2 − 4
666. Qeyd edək ki, x < 0 olduqda sol tərəf mənfi, sağ tərəf
isə müsbətdir. Bu halda həll yoxdur, yəni verilmiş tənliyin
mənfi kökləri yoxdur.
1 1 1
Digər tərəfdən x + ≥ 2, x3 + 3 ≥ 2,..., x 2001 + 2001 ≥ 2.
x x x
Bunların hər birində bərabərlik halı yalnız x = 1 olduqda
ödənir. Bütün mötərizələrin sayı 1001-dir, onda verilmiş
tənliyin sağ tərəfi yalnız x =1 olduqda 2002 olur.
Deməli, verilmiş tənliyin həlli x = 1 -dir.

342
667. Aydındır ki, x ≠ 0. Verilmiş tənliyin hər tərəfini x2 -na
bölüb, çevirmə aparsaq:
 2 1   1
 x + 2  − 8  x +  + 17 = 0 alarıq.
 x   x
1 1
x+ = t qəbul edək, onda x 2 + 2 =t 2 − 2 olar.
x x
Bunları yuxarıda nəzərə alsaq:
t 2 − 8t + 15 =
0,
=t1 3,= t2 5.
1 3± 5
1) x + = 3 ⇒ x 2 − 3 x + 1 = 0 ⇒ x1,2 = ;
x 2
1 5 ± 21
2) x+ = 5 ⇒ x 2 − 5 x + 1 = 0 ⇒ x3,4 = .
x 2

668. Parametr daxil edək: 2 = a olsun. Onda


x 3 − ( a + 1) x 2 + a 2 =
0 alarıq.
Bu tənliyi a -ya nəzərən kvadrat tənlik kimi həll edək:

a 2 − ax 2 + x 3 − x 2 =
0,

x 2 ± x 4 − 4 x3 + 4 x 2 x ± x ( x − 2 )
2

= a1,2 = ,
2 2
buradan a1 = x 2 − x, a2 = x. Əvəzləməni nəzərə alsaq:

1+ 1+ 4 2 1− 1+ 4 2
1) x 2 − x − 2 = 0 ⇒ x1 = və x2 = ,
2 2
2) x3 = 2 alarıq.

343
669. Aydındır ki, x ≠ 0. Onda hər tərəfi x 2 − na bölsək:
2
 9 9
x−6−  = x−4− .
 x x
9
Əgər x − 6 − =t əvəz etsək: t = t + 2 alarıq.
2

x
Onda t1 = −1 və t2 = 2 olar.
Bunları əvəzləmədə nəzərə alsaq:
9 5 ± 61
1) x − 6 − =−1 ⇒ x 2 − 5 x − 9 =0 ⇒ x1,2 = ;
x 2
9
2) x - 6 - =2 ⇒ x 2 − 8 x − 9 =0 ⇒ x3 =−1, x4 =9 alarıq.
x

670. Verilən şərtdə x yerinə ( x + π ) yazaq:


1+ f ( x)
1+
1+ f ( x + π ) 1− f ( x) 2 1
f ( x + 2π ) = = = =

1− f ( x + π ) 1 + f ( x ) −2 f ( x ) f ( x)
1−
1− f ( x)
olar. Axırıncı bərabərlikdə x yerinə ( x + 2π ) yazsaq:
1 1
f ( x + 4π ) =
− =
− f ( x ) alırıq.
=
f ( x + 2π ) −
1
f ( x)
Nəhayət, f ( x ) funksiyası, dövrü 4 π olan dövri funksiya
olduğunu isbat etdik.

344
345
N. N. NƏSİBOV, İ. R. HƏSƏNOV

Riyaziyyatdan olimpiada məsələləri


(VIII - XI sinif şagirdləri üçün vəsait)

Юфзф шьяфдфтэи: 21.12.2012. Ащкьфеэ: A5. Цдф тбм лфхэя.


Кшящйкфа юфз ъыгдг. Жцкеш юфз мцкцйш 21,5. Ешкфоэ 200 тъычц.
Ышафкшж №134

346

View publication stats

You might also like